Download as pdf or txt
Download as pdf or txt
You are on page 1of 71

Question ID 84b5125b Question ID 359902ae

Assessment Test Domain Skill Difficulty Assessment Test Domain Skill Difficulty

SAT Reading and Writing Craft and Structure Words in Context SAT Reading and Writing Craft and Structure Words in Context

ID: 84b5125b ID: 359902ae


Artist Marilyn Dingle’s intricate, coiled baskets are ______ sweetgrass and palmetto palm. Following a Gullah technique The following text is adapted from Nathaniel Hawthorne’s 1837 story “Dr. Heidegger’s Experiment.” The main
that originated in West Africa, Dingle skillfully winds a thin palm frond around a bunch of sweetgrass with the help of a character, a physician, is experimenting with rehydrating a dried flower.
“sewing bone” to create the basket’s signature look that no factory can reproduce.
At first [the rose] lay lightly on the surface of the fluid, appearing to imbibe none of its moisture. Soon, however, a
Which choice completes the text with the most logical and precise word or phrase? singular change began to be visible. The crushed and dried petals stirred and assumed a deepening tinge of
crimson, as if the flower were reviving from a deathlike slumber.
A. indicated by
As used in the text, what does the phrase “a singular” most nearly mean?
B. handmade from
A. A lonely
C. represented by
B. A disagreeable
D. collected with
C. An acceptable

ID: 84b5125b Answer D. An extraordinary

Correct Answer: B

Rationale ID: 359902ae Answer


Correct Answer: D
Choice B is the best answer because it most logically completes the text’s discussion of Marilyn Dingle’s
baskets. In this context, to say that Dingle’s baskets are “handmade from” particular plants means that Dingle Rationale
creates baskets herself using those plants but without using machines. The text says that Dingle “skillfully
Choice D is the best answer because as used in the text, “singular” most nearly means extraordinary. The text
winds” parts of palmetto palm plants around sweetgrass plants to make baskets with an appearance that “no
portrays an experiment in which a character rehydrates a dried rose by infusing it with moisture. After
factory can reproduce.” This context suggests that Dingle’s baskets are handmade from sweetgrass and
prolonged contact with the liquid, the rose begins to absorb it, undergoing an exceptional transformation: its
palmetto palm.
color deepens, its previously “crushed and dried” petals shift, and the entire flower revives “from a deathlike
Choice A is incorrect because the text describes how Dingle uses sweetgrass and palmetto palm to create her slumber.” In other words, an extraordinary change is visible in the flower.
baskets, not how her baskets are “indicated by,” or signified by, sweetgrass and palmetto palm. Choice C is
Choice A is incorrect. Although in some contexts “singular” can mean of or relating to an individual or to a
incorrect. Although Dingle’s baskets are described as being made using sweetgrass and palm, there’s nothing
single instance of something, this usage doesn’t imply loneliness or an otherwise unsatisfactory condition of
in the text to suggest that the baskets are “represented by,” or exemplified or portrayed by, sweetgrass and
isolation. Moreover, the text doesn’t attribute such a condition to the rose. Choice B is incorrect. Although
palmetto palm. Instead, the focus of the text is on Dingle’s use of sweetgrass and palmetto palm and the
“singular” has several related meanings, none of them relate to being disagreeable or unpleasant. Moreover, the
impossibility of replicating the appearance of her baskets using machines. Choice D is incorrect because
text doesn’t portray the change undergone by the rose as necessarily disagreeable. Choice C is incorrect
there’s nothing in the text to suggest that Dingle’s baskets are “collected with,” or brought together in a group
because “singular” means extraordinary, not acceptable. The change is portrayed as striking, not barely
with, sweetgrass and palmetto palm. Instead, the text describes how Dingle uses those plants to make her
satisfactory.
baskets.

Question Difficulty: Medium


Question Difficulty: Easy

Question Difficulty: Hard

Question ID ca50de52
Assessment Test Domain Skill Difficulty

SAT Reading and Writing Craft and Structure Text Structure and
Purpose

ID: ca50de52
“How lifelike are they?” Many computer animators prioritize this question as they strive to create ever more realistic
environments and lighting. Generally, while characters in computer-animated films appear highly exaggerated,
environments and lighting are carefully engineered to mimic reality. But some animators, such as Pixar’s Sanjay Patel,
are focused on a different question. Rather than asking first whether the environments and lighting they’re creating are
convincingly lifelike, Patel and others are asking whether these elements reflect their films’ unique stories.

Which choice best describes the function of the underlined question in the text as a whole?

A. It reflects a primary goal that many computer animators have for certain components of the animations they
produce.

B. It represents a concern of computer animators who are more interested in creating unique backgrounds and
lighting effects than realistic ones.

C. It conveys the uncertainty among many computer animators about how to create realistic animations using current
technology.

D. It illustrates a reaction that audiences typically have to the appearance of characters created by computer
animators.

ID: ca50de52 Answer


Correct Answer: A

Rationale

Choice A is the best answer because it most accurately describes the function of the underlined question in
the text as a whole. The text begins with the underlined question, “How lifelike are they?” The text then
explains that many computer animators pose this question about the environments and lighting that they
create for animated films, striving for realistic animation of those components even if the characters
themselves aren’t portrayed in realistic terms. The focus of the text then shifts to describe how some
animators strive to create environments and lighting that reflect the film’s unique stories rather than making
them appear realistic. Therefore, the function of the underlined question is to reflect a primary goal that many
computer animators have for certain components of the animations they produce.

Choice B is incorrect because, as the text makes clear, the underlined question is one posed by computer
animators who wish to create realistic backgrounds and lighting effects, not by those who, instead, wish to
create effects that reflect films’ unique stories and aren’t necessarily realistic; this latter group of animators is
discussed later in the text. Choice C is incorrect. As the text explains, many computer animators strive for
realistic environments and lighting, while others do not; this difference of approach relates to whether these
components should be realistic, not to how realism can be achieved using current technology, and the text
never suggests that animators are uncertain how to achieve it. Choice D is incorrect because the underlined
question pertains to the perspective of computer animators, not the audience, and the text never considers
audience’s reactions to characters in animated films.
Question ID 22a41819 Question ID 82cb7dda
Assessment Test Domain Skill Difficulty Assessment Test Domain Skill Difficulty

SAT Reading and Writing Craft and Structure Words in Context SAT Reading and Writing Craft and Structure Text Structure and
Purpose

ID: 22a41819
ID: 82cb7dda
Rejecting the premise that the literary magazine Ebony and Topaz (1927) should present a unified vision of Black
American identity, editor Charles S. Johnson fostered his contributors’ diverse perspectives by promoting their The field of study called affective neuroscience seeks instinctive, physiological causes for feelings such as pleasure or
authorial autonomy. Johnson’s self-effacement diverged from the editorial stances of W.E.B. Du Bois and Alain Locke, displeasure. Because these sensations are linked to a chemical component (for example, the release of the
whose decisions for their publications were more ______. neurotransmitter dopamine in the brain when one receives or expects a reward), they can be said to have a partly
physiological basis. These processes have been described in mammals, but Jingnan Huang and his colleagues have
Which choice completes the text with the most logical and precise word or phrase? recently observed that some behaviors of honeybees (such as foraging) are also motivated by a dopamine-based
signaling process.
A. proficient

B. dogmatic What choice best describes the main purpose of the text?

A. It describes an experimental method of measuring the strength of physiological responses in humans.


C. ambiguous
B. It illustrates processes by which certain insects can express how they are feeling.
D. unpretentious
C. It summarizes a finding suggesting that some mechanisms in the brains of certain insects resemble mechanisms in
mammalian brains.
ID: 22a41819 Answer
Correct Answer: B D. It presents research showing that certain insects and mammals behave similarly when there is a possibility of a
reward for their actions.
Rationale

Choice B is the best answer. A person who is "dogmatic" believes strongly that their principles and opinions ID: 82cb7dda Answer
are true. Because Du Bois and Locke are implied to have one "unified vision" of Black American identity that
Correct Answer: C
they prioritize over the "diverse perspectives" of different writers, they can be described as dogmatic.
Rationale
Choice A is incorrect. "Proficient" means "skilled." Du Bois and Locke are contrasted with Johnson, but nothing
in the text suggests that Johnson was not skilled at making editorial decisions. Based on the text, the three Choice C is the best answer because it most accurately describes the main purpose of the text, which is to
editors just have different styles; they’re not necessarily more or less skilled. Choice C is incorrect. summarize a finding suggesting that some mechanisms in the brains of certain insects resemble mechanisms
"Ambiguous" means "unclear" or "open to multiple interpretations." However, it’s actually Johnson who in mammalian brains. The text begins by explaining that feelings such as pleasure and displeasure are linked
encouraged multiple interpretations ("diverse perspectives"). Since Du Bois and Locke are said to "diverge" to chemical processes in the brain, such as the release of dopamine when one receives a reward. The text then
from Johnson, we can assume that the views they published were not ambiguous, but instead clear and firm (a indicates that such processes have been seen in mammals but that researchers have recently observed
"unified vision"). Choice D is incorrect. "Unpretentious" means "not trying to impress others with greater skill similar processes involving dopamine in honeybees. Taken together, this information serves to sum up the
or importance than is actually possessed." Du Bois and Locke are contrasted with Johnson, but nothing in the discovery that some mechanisms in the brains of certain insects may resemble mechanisms linked to
text suggests that Johnson is pretentious (trying to impress others). feelings such as pleasure and displeasure in mammals.

Question Difficulty: Hard Choice A is incorrect because the text doesn’t describe any experiments or experimental methods. Instead, the
text describes a phenomenon that has been observed in mammals and then presents the recent observations
of Huang and colleagues that this phenomenon is also seen in honeybees. Choice B is incorrect because
there’s nothing in the text to suggest that certain insects can express how they’re feeling through particular
processes. The text does indicate that certain honeybee behaviors such as foraging are linked to dopamine,
but it doesn’t suggest that these behaviors enable honeybees to communicate feelings or sensations. Choice D
is incorrect because the text presents research showing that certain honeybee behaviors such as foraging are
linked to dopamine and therefore may be motivated by similar mechanisms to those in mammalian brains,
not that honeybees and mammals behave similarly when there is the possibility of reward for their actions.

Question Difficulty: Hard

Question ID e35d481c Question ID 45a109a3


Assessment Test Domain Skill Difficulty Assessment Test Domain Skill Difficulty

SAT Reading and Writing Craft and Structure Words in Context SAT Reading and Writing Craft and Structure Words in Context

ID: e35d481c ID: 45a109a3


Some economic historians ______ that late nineteenth- and early twentieth-century households in the United States The following text is from Bram Stoker’s 1897 novel Dracula. The narrator is being driven in a carriage through a
experienced an economy of scale when it came to food purchases—they assumed that large households spent less on remote region at night.
food per person than did small households. Economist Trevon Logan showed, however, that a close look at the
The baying of the wolves sounded nearer and nearer, as though they were closing round on us from every side. I
available data disproves this supposition.
grew dreadfully afraid, and the horses shared my fear. The driver, however, was not in the least disturbed; he kept
Which choice completes the text with the most logical and precise word or phrase? turning his head to left and right, but I could not see anything through the darkness.

A. surmised As used in the text, what does the word “disturbed” most nearly mean?

B. contrived A. Disorganized

C. questioned B. Alarmed

D. regretted C. Offended

D. Interrupted
ID: e35d481c Answer
Correct Answer: A ID: 45a109a3 Answer
Rationale Correct Answer: B

Choice A is the best answer because it most logically completes the text’s discussion of late nineteenth- and Rationale
early twentieth-century household food purchases. In this context, “surmised” means formed an idea or
Choice B is the best answer because as used in the text, “disturbed” most nearly means alarmed. The text
assumption with little evidence. The text explains that certain economic historians “assumed” that large and
portrays the narrator traveling in a carriage as wolves howl in the surrounding darkness. The text contrasts
small households spent different amounts on food per person, but that another economist found this
the reaction of both the narrator and the horses pulling the carriage with that of the driver of the carriage: the
supposition to be false based on evidence from available data. This context suggests that the economic
narrator and horses are “dreadfully afraid,” but the driver is “not in the least disturbed.” In other words, the
historians made an incorrect assumption without enough consideration of evidence.
driver is not alarmed by the wolves nearby.
Choice B is incorrect. In this context, “contrived” would mean brought about or created through trickery.
Choice A is incorrect. Although in some contexts, “disturbed” can mean disorganized, the text doesn’t portray a
Nothing in the text suggests that the economic historians were deliberately trying to trick people with a claim
character acting in a disorganized manner; instead, the driver continues to drive the carriage, even though the
about food purchasing behaviors in late nineteenth- and early twentieth-century households; the text simply
horses pulling it are alarmed. Choice C is incorrect. Although in some contexts, “disturbed” can mean offended,
suggests that they made an assumption about those behaviors that another historian believes isn’t supported
the text doesn’t portray one character feeling offended, or upset, by another’s actions; instead, it contrasts the
by the available data. Choice C is incorrect because the text indicates that it’s Logan and not the economic
fear felt by the narrator with another character’s lack of fear. Choice D is incorrect. Although in some contexts,
historians who “questioned,” or doubted, the assumption that large and small households in the late
“disturbed” can mean interrupted, the text doesn’t portray an action being interrupted; indeed, the travel
nineteenth and early twentieth centuries spent different amounts on food per person; the economic historians
depicted in the scene continues despite the threat of the wolves outside the carriage.
are the ones who made that assumption to begin with. Choice D is incorrect because nothing in the text
suggests that some economic historians “regretted,” or felt sad or remorseful about, the food purchasing Question Difficulty: Easy
behaviors of late nineteenth- and early twentieth-century households. The text focuses on the idea that the
economic historians made an assumption about those behaviors that may not be supported by available data,
not on the historians’ emotional response to what households did in the past.

Question Difficulty: Hard


Question ID 5e57efec Question ID c966ad55
Assessment Test Domain Skill Difficulty Assessment Test Domain Skill Difficulty

SAT Reading and Writing Craft and Structure Words in Context SAT Reading and Writing Craft and Structure Text Structure and
Purpose

ID: 5e57efec
ID: c966ad55
Economist Marco Castillo and colleagues showed that nuisance costs—the time and effort people must spend to make
donations—reduce charitable giving. Charities can mitigate this effect by compensating donors for nuisance costs, but The following text is from Srimati Svarna Kumari Devi’s 1894 novel The Fatal Garland (translated by A. Christina Albers
those costs, though variable, are largely ______ donation size, so charities that compensate donors will likely favor in 1910). Shakti is walking near a riverbank that she visited frequently during her childhood.
attracting a few large donors over many small donors.
She crossed the woods she knew so well. The trees seemed to extend their branches like welcoming arms. They
greeted her as an old friend. Soon she reached the river-side.
Which choice completes the text with the most logical and precise word or phrase?

A. supplemental to Which choice best describes the function of the underlined portion in the text as a whole?

B. predictive of A. It suggests that Shakti feels uncomfortable near the river.

C. independent of B. It indicates that Shakti has lost her sense of direction in the woods.

D. subsumed in C. It emphasizes Shakti’s sense of belonging in the landscape.

D. It conveys Shakti’s appreciation for her long-term friendships.

ID: 5e57efec Answer


Correct Answer: C ID: c966ad55 Answer
Rationale Correct Answer: C

Rationale
Choice C is the best answer. We’re told that charities that pay donors back for nuisance costs will attract a few
large donors instead of many small donors. This suggests that nuisance costs are not linked to donation size.
Choice C is the best answer because it most accurately describes how the underlined sentence functions in
the text as a whole. The first sentence of the text states that Shakti knows the woods she is walking in well.
Choice A is incorrect. This doesn’t fit the logic of the text. If nuisance costs are supplemental to (meaning in
The next sentence, which is underlined, emphasizes Shakti’s familiarity with, and sense of security within, the
addition to) donation size, that wouldn’t result in charities that compensate donors for those costs attracting a
woods by describing how the tree branches are friendly and “welcoming,” as they are depicted as extending
few large donors over many small donors. Choice B is incorrect. This doesn’t fit the logic of the text. If
open arms to her. The remainder of the text also shows that Shakti is comfortable and content in the woods by
nuisance costs can predict donation size, that wouldn’t necessarily result in charities that compensate donors
describing her as “an old friend” of the trees. Thus, the function of the underlined portion is to emphasize
for those costs attracting a few large donors over many small donors. Choice D is incorrect. This doesn’t fit the
Shakti’s sense of belonging in the wooded landscape that she visits.
logic of the text. If nuisance costs are subsumed in (meaning included in) donation size, that wouldn’t result in
charities that compensate donors for those costs attracting a few large donors over many small donors.
Choice A is incorrect because the text and underlined portion suggest that Shakti is comfortable, not
uncomfortable, in her surroundings: the trees around her are described as welcoming and reassuring.
Question Difficulty: Hard
Moreover, the underlined portion discusses Shakti’s feelings in the forest, not the river, since she hasn’t
reached the river yet. Choice B is incorrect because the text and underlined portion emphasize Shakti’s
familiarity with the woods. The trees are inviting, and she feels like “an old friend” to the woods, so she isn’t
lost or confused there. Choice D is incorrect because the third sentence uses the phrase “as an old friend”
figuratively in reference to Shakti’s sense of familiarity with the landscape, not in reference to her long-
standing friendships with other people, and the text and underlined portion never discuss her feelings about
such friendships.

Question Difficulty: Easy

Question ID 757077f9 Question ID b13378c8


Assessment Test Domain Skill Difficulty Assessment Test Domain Skill Difficulty

SAT Reading and Writing Craft and Structure Words in Context SAT Reading and Writing Craft and Structure Text Structure and
Purpose

ID: 757077f9
ID: b13378c8
During a 2014 archaeological dig in Spain, Vicente Lull and his team uncovered the skeleton of a woman from El Algar,
an Early Bronze Age society, buried with valuable objects signaling a high position of power. This finding may persuade Early in the Great Migration of 1910–1970, which involved the mass migration of Black people from the southern to the
researchers who have argued that Bronze Age societies were ruled by men to ______ that women may have also held northern United States, political activist and Chicago Defender writer Fannie Barrier Williams was instrumental in
leadership roles. helping other Black women establish themselves in the North. Many women hoped for better employment
opportunities in the North because, in the South, they faced much competition for domestic employment and men
Which choice completes the text with the most logical and precise word or phrase? tended to get agricultural work. To aid with this transition, Barrier Williams helped secure job placement in the North
for many women before they even began their journey.
A. waive

B. concede Which choice best states the main purpose of the text?

A. To introduce and illustrate Barrier Williams’s integral role in supporting other Black women as their circumstances
C. refute
changed during part of the Great Migration
D. require
B. To establish that Barrier Williams used her professional connections to arrange employment for other Black women,
including jobs with the Chicago Defender
ID: 757077f9 Answer
C. To demonstrate that the factors that motivated the start of the Great Migration were different for Black women
Correct Answer: B than they were for Black men
Rationale
D. To provide an overview of the employment challenges faced by Black women in the agricultural and domestic
spheres in the southern United States
Choice B is the best answer because it most logically completes the text’s discussion of the significance of the
2014 archaeological finding at El Algar. In this context, “concede” means to admit something is true after first
resisting that admission. The text indicates that some researchers believe “Bronze Age societies were ruled by ID: b13378c8 Answer
men.” But the Bronze Age burial of a woman at El Algar included “valuable objects signaling a high position of
Correct Answer: A
power,” which would raise the possibility that “women may have also held leadership roles.” Thus, the text is
calling into question the notion that only men were leaders in these societies and speculating that people Rationale
holding this view may reconsider their opinion.
Choice A is the best answer because it most accurately describes the text’s purpose, which is to discuss the
Choice A is incorrect because “waive” means to refrain from insisting that something, such as a right or a important role Barrier Williams played in supporting many other Black women as they relocated to the
requirement, be observed; the word isn’t used, however, in contexts where someone acknowledges that an northern United States during the early years of the Great Migration. After introducing Barrier Williams, the
opinion they hold may be invalid, as is the case in the text. Choice C is incorrect. According to the text, the text describes how she helped find jobs for other Black women, who in many cases relocated in search of
finding from the El Algar burial site undermines the view that Bronze Age societies were exclusively ruled by better employment prospects than the South could offer at the time. The text indicates that by doing so, she
men. However, “refute” means to demonstrate that something is false and would not make sense in context. eased these women’s transition as their circumstances changed.
Lull and team’s finding supports the view that women may have also held leadership roles, not that they did
not participate in such roles. Choice D is incorrect because in this context, “require” means to demand or Choice B is incorrect. Although the text mentions Barrier Williams’s work as a political activist and writer for
specify as mandatory. However, it would not make sense for contemporary researchers to demand that Bronze the Chicago Defender, it doesn’t discuss any professional connections she made in these roles or indicate that
Age “women may have also held leadership roles.” she used any such connections in her work to secure employment for other Black women. Choice C is
incorrect. Although the text discusses a factor that caused many women to relocate during the Great
Question Difficulty: Medium Migration, their difficulty finding employment in the South, the text doesn’t indicate that this factor motivated
the start of the Great Migration. Moreover, the text doesn’t discuss the factors that motivated Black men to
migrate. Choice D is incorrect. Although the text mentions the difficult employment prospects for Black
women in the domestic and agricultural sectors in the South during the Great Migration, the text’s main
purpose isn’t to provide an overview of the employment challenges Black women faced in these sectors.
Rather, it provides this information to show that Barrier Williams played a crucial role in supporting many
Black women who relocated to the North by helping them achieve one of their main goals, securing a job.
Question Difficulty: Medium

Question ID 97e5bf55
Assessment Test Domain Skill Difficulty

SAT Reading and Writing Craft and Structure Cross-Text


Connections

ID: 97e5bf55
Text 1
In 1916, H. Dugdale Sykes disputed claims that The Two Noble Kinsmen was coauthored by William Shakespeare and
John Fletcher. Sykes felt Fletcher’s contributions to the play were obvious—Fletcher had a distinct style in his other
plays, so much so that lines with that style were considered sufficient evidence of Fletcher’s authorship. But for the
lines not deemed to be by Fletcher, Sykes felt that their depiction of women indicated that their author was not
Shakespeare but Philip Massinger.
Text 2
Scholars have accepted The Two Noble Kinsmen as coauthored by Shakespeare since the 1970s: it appears in all major
one-volume editions of Shakespeare’s complete works. Though scholars disagree about who wrote what exactly, it is
generally held that on the basis of style, Shakespeare wrote all of the first act and most of the last, while John Fletcher
authored most of the three middle acts.

Based on the texts, both Sykes in Text 1 and the scholars in Text 2 would most likely agree with which statement?

A. John Fletcher’s writing has a unique, readily identifiable style.

B. The women characters in John Fletcher’s plays are similar to the women characters in Philip Massinger’s plays.

C. The Two Noble Kinsmen belongs in one-volume compilations of Shakespeare’s complete plays.

D. Philip Massinger’s style in the first and last acts of The Two Noble Kinsmen is an homage to Shakespeare’s style.

ID: 97e5bf55 Answer


Correct Answer: A

Rationale

Choice A is the best answer. Text 1 states that Sykes felt Fletcher’s contributions to the play were obvious
because he had a distinct style in his other plays. Text 2 states that scholars generally agree “on the basis of
style” that Fletcher wrote most of the three middle acts. Both texts imply that Fletcher’s writing has a unique,
readily identifiable style that can be used to distinguish his work from others.

Choice B is incorrect. While Text 1 refers to the women in Massinger plays, neither text compares the women
of Fletcher’s plays to the women of Massinger’s plays. Text 2 doesn’t mention Massinger at all. Choice C is
incorrect. Text 1 states that Sykes disputed that Shakespeare coauthored the play, and implied that it was
coauthored by Fletcher and Massinger instead. Sykes, therefore, would disagree that The Two Noble Kinsmen
belongs in a Shakespeare compilation. Choice D is incorrect. Text 1 doesn’t suggest that Massinger was
inspired by Shakespeare, and Text 2 doesn’t mention Massinger at all.

Question Difficulty: Hard

Question ID 84ece3f6 Question ID d4a8f7cb


Assessment Test Domain Skill Difficulty Assessment Test Domain Skill Difficulty

SAT Reading and Writing Craft and Structure Words in Context SAT Reading and Writing Craft and Structure Words in Context

ID: 84ece3f6 ID: d4a8f7cb


The following text is adapted from Nathaniel Hawthorne’s 1844 short story “Drowne’s Wooden Image.” Drowne, a Dance choreographer Jawole Willa Jo Zollar aims to give people the opportunity to be ______ her creative process. For
young man, is carving a wooden figure to decorate the front of a ship. example, live performances of her dance HairStories, which debuted in 2001, featured videos of people across the
Day by day, the work assumed greater precision, and settled its irregular and misty outline into distincter grace and United States talking about their hair and audience members sharing pictures of their interesting hairstyles.
beauty. The general design was now obvious to the common eye.
Which choice completes the text with the most logical and precise word or phrase?
As used in the text, what does the word “assumed” most nearly mean?
A. nervous about
A. Acquired
B. completed by
B. Acknowledged
C. delayed by
C. Imitated
D. involved in
D. Speculated

ID: d4a8f7cb Answer


ID: 84ece3f6 Answer Correct Answer: D
Correct Answer: A
Rationale
Rationale
Choice D is the best answer. “Involved in” means “playing an active role in.” This fits the context clues
Choice A is the best answer because as used in the text, “assumed” most nearly means acquired, or came to describing how Zollar includes the audience in her shows by incorporating their stories and pictures.
possess. The text portrays a character named Drowne carving a figure out of wood. At first “irregular and
misty,” or haphazard and indistinct, the figure’s outline gradually showed “distincter grace and beauty” until Choice A is incorrect. “Nervous” means “worried or anxious about.” Nothing in the text suggests that people
the general design of the carved object “was now obvious to the common eye,” or plainly recognizable to would be nervous about Zollar’s creative process. Choice B is incorrect. “Completed by” means “made whole
anyone. In other words, as Drowne continued to carve, the wooden object came to possess, or acquired, greater by.” It doesn’t make sense to say that Zollar’s shows would make her audience complete. Choice C is incorrect.
precision, changing from an indistinct outline or shape into a graceful, beautiful, and clearly recognizable “Delayed by” means “made late by.” Nothing in the text suggests that people would be delayed by Zollar’s
form. creative process.

Choice B is incorrect. Although in some contexts “assumed” can mean acknowledged, or recognized, it doesn’t Question Difficulty: Easy
have that meaning in this context because an inanimate object like the wooden figure can’t acknowledge its
own precision. Choice C is incorrect because there’s nothing in the text to suggest that the wooden figure
merely imitated, or mimicked, precision. Rather, the text suggests that as Drowne carved his wooden figure, it
gradually became more precise. Choice D is incorrect. Although in some contexts “assumed” can mean
speculated, or supposed based on incomplete information, it doesn’t have that meaning in this context
because an inanimate object like the wooden figure can’t speculate about its own precision.

Question Difficulty: Medium


Question ID d4732483 Question ID e818241b
Assessment Test Domain Skill Difficulty Assessment Test Domain Skill Difficulty

SAT Reading and Writing Craft and Structure Text Structure and SAT Reading and Writing Craft and Structure Text Structure and
Purpose Purpose

ID: d4732483 ID: e818241b


Studying late nineteenth- and early twentieth-century artifacts from an agricultural and domestic site in Texas, Astronomers are confident that the star Betelgeuse will eventually consume all the helium in its core and explode in a
archaeologist Ayana O. Flewellen found that Black women employed as farm workers utilized hook-and-eye closures to supernova. They are much less confident, however, about when this will happen, since that depends on internal
fasten their clothes at the waist, giving themselves a silhouette similar to the one that was popular in contemporary characteristics of Betelgeuse that are largely unknown. Astrophysicist Sarafina El-Badry Nance and colleagues recently
fashion and typically achieved through more restrictive garments such as corsets. Flewellen argues that this sartorial investigated whether acoustic waves in the star could be used to determine internal stellar states but concluded that
practice shows that these women balanced hegemonic ideals of femininity with the requirements of their physically this method could not sufficiently reveal Betelgeuse’s internal characteristics to allow its evolutionary state to be firmly
demanding occupation. fixed.

Which choice best states the main purpose of the text? Which choice best describes the function of the second sentence in the overall structure of the text?

A. To describe an unexpected discovery that altered a researcher’s view of how rapidly fashions among Black female A. It describes a serious limitation of the method used by Nance and colleagues.
farmworkers in late nineteenth- and early twentieth-century Texas changed during the period
B. It presents the central finding reported by Nance and colleagues.
B. To discuss research that investigated the ways in which Black female farmworkers in late nineteenth- and early
C. It identifies the problem that Nance and colleagues attempted to solve but did not.
twentieth-century Texas used fashion practices to resist traditional gender ideals
D. It explains how the work of Nance and colleagues was received by others in the field.
C. To evaluate a scholarly work that offers explanations for the impact of urban fashion ideals on Black female
farmworkers in late nineteenth- and early twentieth-century Texas

D. To summarize the findings of a study that explored factors influencing a fashion practice among Black female ID: e818241b Answer
farmworkers in late nineteenth- and early twentieth-century Texas Correct Answer: C

Rationale
ID: d4732483 Answer
Choice C is the best answer because it best describes how the second sentence functions in the text as a
Correct Answer: D whole. The first sentence establishes something astronomers believe with some certainty: that Betelgeuse will
Rationale explode in a supernova. The second sentence then introduces a problem: astronomers aren’t certain when
Betelgeuse will explode because they don’t have enough information about the star’s internal characteristics.
Choice D is the best answer. The text provides an overview of a scholarly work discussing the fashion Finally, the third sentence indicates that researcher Sarafina El-Badry Nance and colleagues investigated a
practices of Black female farmworkers in late 19th- and early 20th-century Texas, and how these practices possible method of obtaining the necessary information about Betelgeuse’s internal characteristics, though
were influenced by both the fashion ideals of the time and the demands of farmwork. they found that the method wouldn’t be sufficient. Thus, the function of the second sentence is to identify the
problem that Nance and colleagues attempted to solve but didn’t.
Choice A is incorrect. The text never discusses the rate of fashion change among Black female farmworkers.
The text also never categorizes Flewellen’s findings as "unexpected." Choice B is incorrect. The text actually Choice A is incorrect because the second sentence introduces the general problem Nance and colleagues
explains that Black female farmworkers were trying to achieve traditional feminine ideals, not resist them. hoped to solve, not a serious limitation of how Nance and colleagues tried to solve it. It is the third sentence
Choice C is incorrect. The text doesn’t evaluate a scholarly work but rather simply describes it. Furthermore, that introduces Nance and colleagues, but no serious limitation of their approach to studying a method of
the text is focused on "agricultural and domestic" fashion, not urban fashion as this choice suggests. determining internal stellar states is described. Choice B is incorrect because the second sentence introduces
the general problem Nance and colleagues hoped to solve, not the central finding they ultimately reported. It is
Question Difficulty: Hard the third sentence that presents Nance and colleagues’ conclusion that a potential method for determining
internal stellar states would be insufficient. Choice D is incorrect because the second sentence doesn’t
indicate how other astronomers or astrophysicists responded to the work done by Nance and colleagues; the
text doesn’t address this information at all.

Question Difficulty: Hard

Question ID 236fee8e Question ID 97ab5669


Assessment Test Domain Skill Difficulty Assessment Test Domain Skill Difficulty

SAT Reading and Writing Craft and Structure Text Structure and SAT Reading and Writing Craft and Structure Words in Context
Purpose

ID: 97ab5669
ID: 236fee8e
Former astronaut Ellen Ochoa says that although she doesn’t have a definite idea of when it might happen, she ______
Archeological excavation of Market Street Chinatown, a nineteenth-century Chinese American community in San Jose, that humans will someday need to be able to live in other environments than those found on Earth. This conjecture
California, provided the first evidence that Asian food products were imported to the United States in the 1800s: bones informs her interest in future research missions to the moon.
from a freshwater fish species native to Southeast Asia. Jinshanzhuang—Hong Kong–based import/export firms—likely
coordinated the fish’s transport from Chinese-operated fisheries in Vietnam and Malaysia to North American markets. Which choice completes the text with the most logical and precise word or phrase?
This route reveals the (often overlooked) multinational dimensions of the trade networks linking Chinese diaspora
A. demands
communities.
B. speculates
Which choice best describes the function of the underlined sentence in the text as a whole?
C. doubts
A. It explains why efforts to determine the country of origin of the items mentioned in the previous sentence remain
inconclusive. D. establishes

B. It provides information that helps support a claim about a discovery’s significance that is presented in the following
sentence. ID: 97ab5669 Answer
C. It traces the steps that were taken to locate and recover the objects that are described in the previous sentence. Correct Answer: B

Rationale
D. It outlines a hypothesis that additional evidence discussed in the following sentence casts some doubt on.

Choice B is the best answer because it most logically completes the text’s discussion of Ochoa’s prediction that
ID: 236fee8e Answer humans will one day need to live in places other than Earth. As used in this context, “speculates” would mean
puts forward an idea without firm evidence. The text states that Ochoa “doesn’t have a definite idea” about
Correct Answer: B
when humans might need to live in other environments and characterizes Ochoa’s prediction as a “conjecture,”
Rationale or a conclusion presented without convincing evidence. This context indicates that Ochoa speculates when
she makes this prediction.
Choice B is the best answer. The underlined sentence provides information about import/export firms,
showing how Chinese communities across the world were connected by trade routes. Choice A is incorrect because saying that Ochoa “demands,” or insists or requires, that humans will one day
need to live in other environments than Earth’s would not make sense in context. The text indicates that she’s
Choice A is incorrect. The underlined sentence never suggests that the countries of origin of the fish are in unsure about the timing but hypothesizes that it will someday happen. Choice C is incorrect because saying
question—in fact, it tells us exactly where they came from. Choice C is incorrect. The passage never describes that Ochoa “doubts,” or questions or disbelieves, that humans will one day need to live in other environments
the steps taken to discover the fish bones described in the previous sentence. Choice D is incorrect. The than Earth’s would not make sense in context. The text indicates that although Ochoa is unsure about the
underlined sentence doesn’t outline a hypothesis but instead provides evidence. And the following sentence timing, she hypothesizes that humans will need to live in places other than Earth and encourages research
agrees with the underlined sentence, so we could eliminate this choice just for saying that the following into future travel to the moon. Choice D is incorrect because saying that Ochoa “establishes,” or proves, that
sentence "casts some doubt on" the underlined one—partly wrong is all wrong. humans will one day need to live in other environments than Earth’s would not make sense in context. Rather
than stating that Ochoa discusses her idea with certainty and supports it with evidence, the text indicates that
Question Difficulty: Medium
Ochoa is unsure about when humans might need to live in other environments.

Question Difficulty: Easy


Question ID 02fd3da7 Question ID 93665100
Assessment Test Domain Skill Difficulty Assessment Test Domain Skill Difficulty

SAT Reading and Writing Craft and Structure Cross-Text SAT Reading and Writing Craft and Structure Words in Context
Connections

ID: 93665100
ID: 02fd3da7
Seminole/Muscogee director Sterlin Harjo ______ television’s tendency to situate Native characters in the distant past:
Text 1 this rejection is evident in his series Reservation Dogs, which revolves around teenagers who dress in contemporary
Public policy researcher Anthony Fowler studied the history of elections in Australia, a country that requires citizens to styles and whose dialogue is laced with current slang.
vote. Fowler argues that requiring citizens to vote leads to a significant increase in voters who would otherwise not
have the time or motivation to vote. Thus, election results in countries that require citizens to vote better reflect the Which choice completes the text with the most logical and precise word or phrase?
preferences of the country as a whole.
A. repudiates

Text 2 B. proclaims
Governments in democratic countries function better when more people vote. However, forcing people to vote may
C. foretells
have negative consequences. Shane P. Singh and Jason Roy studied what happens when a country requires its citizens
to vote. They found that when people feel forced to vote, they tend to spend less time looking for information about D. recants
their choices when voting. As a result, votes from these voters may not reflect their actual preferences.

Based on the texts, how would Singh and Roy (Text 2) most likely respond to the research discussed in Text 1? ID: 93665100 Answer
A. Only countries of a certain population size should implement mandatory voting. Correct Answer: A

B. People who are forced to vote are likely to become politically engaged in other ways, such as volunteering or Rationale
running for office.
Choice A is the best answer because it most logically completes the text’s discussion of Sterlin Harjo’s
C. Requiring people to vote does not necessarily lead to election outcomes that better represent the preferences of approach to representing Native characters on television. As used in this context, “repudiates” means rejects
the country as a whole. or refuses to have anything to do with. The text indicates that television shows tend to depict Native
characters as living long ago, but that Harjo’s series Reservation Dogs focuses on Native teenagers in the
D. Countries that require voting must also make the process of voting easier for their citizens.
present day, representing a “rejection” of the typical approach to depicting Native characters. This context
thus indicates that Harjo repudiates television’s general tendency regarding Native characters.
ID: 02fd3da7 Answer
Choice B is incorrect because the text describes Harjo’s “rejection” of the typical approach to representing
Correct Answer: C Native characters on television, so it wouldn’t make sense to say that Harjo “proclaims,” or declares or affirms,
Rationale television’s general tendency regarding Native characters. Harjo is described as refusing to follow the pattern
of depicting Native characters in the distant past, not as proclaiming that pattern. Choice C is incorrect
Choice C is the best answer. Text 1 claims that mandatory voting results in elections that "better reflect the because the text describes television’s tendency to represent Native characters in the distant past as
preferences of the country." Singh and Roy disagree. They claim that more voters doesn’t equal more quality something that is already occurring, not as something that Harjo “foretells,” or predicts will happen in the
votes—instead, they argue that forced voting may lead to less-informed votes that "may not reflect [voters’] future. The text is focused on Harjo’s “rejection” of this pattern, not on any predictions he may have about it.
actual preferences." Choice D is incorrect because saying that Harjo “recants” something would mean that he withdraws a
previously held belief, and it wouldn’t make sense to say that Harjo recants television’s tendency to represent
Choice A is incorrect. Neither text mentions the population size of countries that require voting, or how that Native characters as living in the past. No beliefs previously held by Harjo are mentioned. Additionally, a
might affect election outcomes. Choice B is incorrect. Neither text discusses the effects of mandatory voting tendency isn’t a belief and thus isn’t something that can be recanted.
on other forms of political engagement. Choice D is incorrect. Neither text discusses the ease or difficulty of
the voting process in countries that require voting. Question Difficulty: Hard

Question Difficulty: Easy

Question ID 06b96bfc Question ID 0f040c50


Assessment Test Domain Skill Difficulty Assessment Test Domain Skill Difficulty

SAT Reading and Writing Craft and Structure Words in Context SAT Reading and Writing Craft and Structure Words in Context

ID: 06b96bfc ID: 0f040c50


A musician and member of the Quechua of Peru, Renata Flores Rivera was eager to promote the Quechua language in The following text is from Yann Martel’s 2001 novel Life of Pi. The narrator’s family owned a zoo when he was a child.
her music, but she was ______ speaking it. She met this challenge by asking her grandmother, a native speaker of
It was a huge zoo, spread over numberless acres, big enough to require a train to explore it, though it seemed to
Quechua, to help her pronounce words in her song lyrics and also by taking classes in the language.
get smaller as I grew older, train included.
Which choice completes the text with the most logical and precise word or phrase? ©2001 by Yann Martel

A. prepared for As used in the text, what does the word “spread” most nearly mean?

B. inexperienced with A. Hidden

C. skilled in B. Discussed

D. excited about C. Extended

D. Coated
ID: 06b96bfc Answer
Correct Answer: B ID: 0f040c50 Answer
Rationale Correct Answer: C

Choice B is the best answer because it most logically completes the text’s discussion of Renata Flores Rivera’s Rationale
use of Quechua in her music. In this context, “inexperienced with” means not accustomed to. The text
Choice C is the best answer because as used in the text, “spread” most nearly means extended. The text states
indicates that Flores Rivera wanted to promote the Quechua language in her music and overcame a challenge
that the zoo is “huge,” that it covers “numberless acres,” and that it is large enough that a train is needed to
by seeking help with pronunciation from her grandmother and by taking language classes. This context
explore it. Thus, the text’s emphasis on the zoo’s size suggests that the zoo extended, or stretched, over a large
conveys the idea that Flores Rivera was not sufficiently familiar with Quechua to use it in her music without
area of land.
help. Thus, she was inexperienced with speaking the language, which she addressed by seeking help.

Choice A is incorrect because if the zoo covers “numberless acres,” then it could not reasonably be described
Choice A is incorrect because describing Flores Rivera as “prepared for”—or ready for—speaking Quechua
as hidden, or concealed from view. Choice B is incorrect because there is nothing in the text to suggest that
wouldn’t make sense in context. The text indicates that speaking Quechua presented a challenge, but if she
the zoo was discussed, or talked about. Rather, the text focuses on the zoo’s large size. Choice D is incorrect.
were ready to speak the language, then there would be no challenge. Choice C is incorrect because describing
Although in some contexts “spread” can mean coated, it doesn’t have that meaning in this context because to
Flores Rivera as “skilled in”—meaning good at or capable of—speaking Quechua wouldn’t make sense in
coat something means to apply a thin layer of a liquid substance, such as oil or paint, to a surface. Therefore, it
context. The text indicates that speaking Quechua presented a challenge, but if she were capable of speaking
would not be accurate to say that the zoo coated the acres on which it sits.
the language, then there would be no challenge. Choice D is incorrect. Flores Rivera was likely “excited
about”—or thrilled or delighted with—speaking Quechua, but this wouldn’t make sense in context. The text
Question Difficulty: Easy
indicates that speaking Quechua presented a challenge, but if she were delighted with speaking the language,
then there would be no challenge.

Question Difficulty: Easy


Question ID e1d5d5df Question ID 9e501aaf
Assessment Test Domain Skill Difficulty Assessment Test Domain Skill Difficulty

SAT Reading and Writing Craft and Structure Words in Context SAT Reading and Writing Craft and Structure Words in Context

ID: e1d5d5df ID: 9e501aaf


According to botanists, a viburnum plant experiencing insect damage may develop erineum—a discolored, felty Research conducted by planetary scientist Katarina Miljkovic suggests that the Moon’s surface may not accurately
growth—on its leaf blades. A ______ viburnum plant, on the other hand, will have leaves with smooth surfaces and ______ early impact events. When the Moon was still forming, its surface was softer, and asteroid or meteoroid impacts
uniformly green coloration. would have left less of an impression; thus, evidence of early impacts may no longer be present.

Which choice completes the text with the most logical and precise word or phrase? Which choice completes the text with the most logical and precise word or phrase?

A. struggling A. reflect

B. beneficial B. receive

C. simple C. evaluate

D. healthy D. mimic

ID: e1d5d5df Answer ID: 9e501aaf Answer


Correct Answer: D Correct Answer: A

Rationale Rationale

Choice D is the best answer because it most logically completes the text’s discussion of damage to viburnum Choice A is the best answer because it most logically completes the text’s discussion of the Moon’s surface. In
plants. In this context, “healthy” would mean not distressed or diseased. The text states that insect damage this context, “reflect” means show or make apparent. The text states that because the surface of the Moon was
may cause viburnum plants to be discolored and have abnormal growths. In the next sentence, the phrase “on softer when the Moon was still forming than it is now, early asteroid and meteoroid impacts “would have left
the other hand” indicates a contrast with the description of plants suffering from damage. Thus, the context less of an impression” and, as a result, evidence of them may no longer exist. This context supports the idea
contrasts the appearance of healthy, undamaged plants with the appearance of damaged plants. that the surface of the Moon may not accurately show signs of early impact events.

Choice A is incorrect because in this context, “struggling” would mean working against difficulties. The text Choice B is incorrect because it wouldn’t make sense to say that the surface of the Moon may not accurately
first describes viburnum plants experiencing damage by insects, and the phrase “on the other hand” then “receive,” or acquire or experience, early impacts from asteroids or meteoroids. The text indicates that the
establishes a contrast with that description. It wouldn’t make sense to contrast struggling viburnum plants impacts have already occurred, and it isn’t clear how the Moon’s surface could be accurate or inaccurate in
with those being damaged by insects, because in both cases the plants would be experiencing difficulties. experiencing them. Choice C is incorrect because it wouldn’t make sense to say that the surface of the Moon
Choice B is incorrect because in this context, “beneficial” would mean producing good or helpful effects. The may not accurately “evaluate,” or determine the significance or condition of, early impacts from asteroids or
text doesn’t discuss how viburnum plants affect other things or suggest that the plants are helpful in some meteoroids, since that would suggest that it’s possible for the Moon’s surface to make a decision of any
way; rather, it focuses on how viburnum plants are affected by certain conditions. Choice C is incorrect kind. Choice D is incorrect. In this context, “mimic” would mean to deliberately simulate or closely imitate
because in this context “simple” would mean plain or uncomplicated. The text doesn’t discuss whether certain something. It wouldn’t make sense to say that the surface of the Moon may not accurately mimic early
viburnum plants are complicated or uncomplicated; rather, it focuses on how viburnum plants are affected by asteroid and meteoroid impacts, since that would suggest that it’s possible for the Moon to deliberately imitate
certain conditions. something.

Question Difficulty: Easy Question Difficulty: Easy

Question ID 9cdcd902 Question ID e459076b


Assessment Test Domain Skill Difficulty Assessment Test Domain Skill Difficulty

SAT Reading and Writing Craft and Structure Words in Context SAT Reading and Writing Craft and Structure Words in Context

ID: 9cdcd902 ID: e459076b


Charles “Teenie” Harris was a photographer for the Pittsburgh Courier from 1936 to 1975. During his career he took The following text is adapted from George Eliot’s 1871–72 novel Middlemarch.
over 70,000 photographs documenting everyday life in Pittsburgh’s Black communities. The Carnegie Museum of Art
[Mr. Brooke] had travelled in his younger years, and was held in this part of the country to have contracted a too
maintains thousands of his photographs, carefully ______ them so that audiences can continue to view them well into
rambling habit of mind. Mr. Brooke’s conclusions were as difficult to predict as the weather.
the future.

As used in the text, what does the word “contracted” most nearly mean?
Which choice completes the text with the most logical and precise word or phrase?
A. Restricted
A. replacing
B. Described
B. inventing
C. Developed
C. preserving
D. Settled
D. counting

ID: e459076b Answer


ID: 9cdcd902 Answer
Correct Answer: C
Correct Answer: C
Rationale
Rationale

Choice C is the best answer because as used in the text, “contracted” most nearly means developed. The text
Choice C is the best answer. "Preserving" means "maintaining" or "keeping in good condition," so preserving
explains that Mr. Brooke has a “too rambling habit of mind,” which the text likens to a disease, saying he is
the photographs means that audiences should be able to view them for a long time.
thought to have contracted it. To contract a disease means to acquire or develop a disease. In other words, the
Choice A is incorrect. "Replacing" means "putting something new in place of" the photographs. Replacing the text indicates that Mr. Brooke is believed to have acquired, or developed, the habit of mind described in the
photos will make it so that audiences can’t view them at all. Choice B is incorrect. "Inventing" means "creating text.
a new idea, process, or thing." The museum can’t invent photographs that already exist. Choice D is incorrect.
Choice A is incorrect. Although “contracted” can mean limited or restricted in some contexts, here Mr. Brooke
Counting the photographs will not help audiences view them well into the future.
is said to draw unpredictable conclusions, suggesting that he exhibits this “too rambling habit of mind,” not
Question Difficulty: Easy that it has been somehow limited or restricted. Choice B is incorrect. Although the text describes Mr. Brooke’s
habit of mind, nothing suggests that those are his descriptions or, indeed, that he described his habit of mind
at all. Choice D is incorrect because settled means calmed or mitigated, but here Mr. Brooke is said to draw
unpredictable conclusions, suggesting that he exhibits this “too rambling habit of mind,” not that it has been
somehow calmed or mitigated.

Question Difficulty: Hard


Question ID 4974b053 Question ID 105ea6de
Assessment Test Domain Skill Difficulty Assessment Test Domain Skill Difficulty

SAT Reading and Writing Craft and Structure Words in Context SAT Reading and Writing Craft and Structure Cross-Text
Connections

ID: 4974b053
ID: 105ea6de
Although science fiction was dominated mostly by white male authors when Octavia Butler, a Black woman, began
writing, she did not view the genre as ______: Butler broke into the field with the publication of several short stories and Text 1
her 1976 novel Patternmaster, and she later became the first science fiction writer to win a prestigious MacArthur Growth in the use of novel nanohybrids—materials created from the conjugation of multiple distinct nanomaterials,
Fellowship. such as iron oxide and gold nanomaterials conjugated for use in magnetic imaging—has outpaced studies of
nanohybrids’ environmental risks. Unfortunately, risk evaluations based on nanohybrids’ constituents are not reliable:
Which choice completes the text with the most logical and precise word or phrase? conjugation may alter constituents’ physiochemical properties such that innocuous nanomaterials form a nanohybrid
that is anything but.
A. legitimate
Text 2
B. impenetrable The potential for enhanced toxicity of nanohybrids relative to the toxicity of constituent nanomaterials has drawn
deserved attention, but the effects of nanomaterial conjugation vary by case. For instance, it was recently shown that a
C. compelling
nanohybrid of silicon dioxide and zinc oxide preserved the desired optical transparency of zinc oxide nanoparticles
D. indecipherable while mitigating the nanoparticles’ potential to damage DNA.

Based on the texts, how would the author of Text 2 most likely respond to the assertion in the underlined portion of
ID: 4974b053 Answer Text 1?

Correct Answer: B A. By concurring that the risk described in Text 1 should be evaluated but emphasizing that the risk is more than offset
by the potential benefits of nanomaterial conjugation
Rationale
B. By arguing that the situation described in Text 1 may not be representative but conceding that the effects of
Choice B is the best answer because it most logically completes the discussion of Octavia Butler’s career. In
nanomaterial conjugation are harder to predict than researchers had expected
this context, “impenetrable” means impossible to enter. The text indicates that the field of science fiction was
dominated by white males when Butler, a Black woman, started writing, but she published several science C. By denying that the circumstance described in Text 1 is likely to occur but acknowledging that many aspects of
fiction short stories and a novel and later won a prestigious award; that is, Butler pursued science fiction nanomaterial conjugation are still poorly understood
writing and had success. This context suggests that Butler didn’t view the genre as impossible to enter.
D. By agreeing that the possibility described in Text 1 is a cause for concern but pointing out that nanomaterial
Choice A is incorrect. In this context, “legitimate” would mean genuinely good or valid. Nothing in the text conjugation does not inevitably produce that result
suggests that Butler didn’t think the science fiction genre was good or valid; in fact, it indicates that she
pursued and made a successful career of publishing work in that field. Choice C is incorrect. In this context,
ID: 105ea6de Answer
“compelling” would mean attracting or demanding attention. The text indicates that Butler chose to write
science fiction, so it wouldn’t make sense to say that she didn’t see the field as drawing her attention. Choice D Correct Answer: D
is incorrect. To say that Butler didn’t consider science fiction “indecipherable,” or impossible to understand, Rationale
would suggest that Butler did understand it. However, the text doesn’t address Butler’s ability to interpret
works in the genre; rather, it focuses on Butler’s successful pursuit of writing science fiction. Choice D is the best answer. The author of Text 2 acknowledges that nanohybrids may be more toxic than their
constituent parts, but also provides an example of a nanohybrid that has reduced toxicity compared to its
Question Difficulty: Medium components: silicon dioxide and zinc oxide together have all the benefits of zinc oxide nanoparticles without
any of the DNA harm zinc oxide has on its own.

Choice A is incorrect. While the author of Text 2 gives an example of a nanohybrid that isn’t as toxic as its
constituent parts, they don’t argue that the benefit outweighs the risk. They merely argue that “the effects of
nanomaterial conjugation vary by case.” Choice B is incorrect. The author of Text 2 states that the effects of
nanomaterial conjugation “vary by case,” and that the attention that their potential toxicity has drawn is
warranted. If the situation in Text 1 weren’t representative, then there would be less attention to the potential
danger of these materials. Furthermore, neither passage suggests that researchers had expected that they

could predict the effects of nanomaterial conjugation. Choice C is incorrect. The author of Text 2 agrees that
the potential toxicity of nanohybrids “has drawn deserved attention,” so they aren’t denying the problem.
Question ID 2903a041
Question Difficulty: Hard
Assessment Test Domain Skill Difficulty

SAT Reading and Writing Craft and Structure Text Structure and
Purpose

ID: 2903a041
Using NASA’s powerful James Webb Space Telescope (JWST), Mercedes López-Morales and colleagues measured the
wavelengths of light traveling through the atmosphere of WASP-39b, an exoplanet, or planet outside our solar system.
Different molecules absorb different wavelengths of light, and the wavelength measurements showed the presence of
carbon dioxide (CO₂) in WASP-39b’s atmosphere. This finding not only offers the first decisive evidence of CO₂ in the
atmosphere of an exoplanet but also illustrates the potential for future scientific breakthroughs held by the JWST.

Which choice best describes the overall structure of the text?

A. It discusses a method used by some researchers, then states why an alternative method is superior to it.

B. It describes how researchers made a scientific discovery, then explains the importance of that discovery.

C. It outlines the steps taken in a scientific study, then presents a hypothesis based on that study.

D. It examines how a group of scientists reached a conclusion, then shows how other scientists have challenged that
conclusion.

ID: 2903a041 Answer


Correct Answer: B

Rationale

Choice B is the best answer. The text begins by describing how the researchers used the JWST to detect CO₂ in
WASP-39b’s atmosphere. Then the text discusses the significance of this finding, both as the first evidence of
CO₂ in an exoplanet’s atmosphere and as an illustration of the JWST’s potential for making new discoveries in
general.

Choice A is incorrect. The text doesn’t compare two different methods, but rather focuses on one study that
used the JWST. Choice C is incorrect. The text doesn’t present a hypothesis, but rather reports on the findings
of a study. Choice D is incorrect. The text doesn’t mention any scientists challenging the conclusion reached
by López-Morales and colleagues.

Question Difficulty: Medium


Question ID 066a3295 Question ID ac9a3a26
Assessment Test Domain Skill Difficulty Assessment Test Domain Skill Difficulty

SAT Reading and Writing Craft and Structure Text Structure and SAT Reading and Writing Craft and Structure Text Structure and
Purpose Purpose

ID: 066a3295 ID: ac9a3a26


Researchers have found a nearly 164,000-year-old molar from a member of the archaic human species known as According to historian Vicki L. Ruiz, Mexican American women made crucial contributions to the labor movement
Denisovans in a cave in Laos, suggesting that Denisovans lived in a wider range of environments than indicated by during World War II. At the time, food processing companies entered into contracts to supply United States armed
earlier evidence. Before the discovery, Denisovans were thought to have lived only at high altitudes in relatively cold forces with canned goods. Increased production quotas conferred greater bargaining power on the companies’
climates in what are now Russia and China, but the discovery of the tooth in Laos suggests that they may have lived at employees, many of whom were Mexican American women: employees insisted on more favorable benefits, and
low altitudes in relatively warm climates in Southeast Asia as well. employers, who were anxious to fulfill the contracts, complied. Thus, labor activism became a platform for Mexican
American women to assert their agency.
Which choice best states the function of the underlined portion in the text as a whole?
Which choice best describes the function of the underlined portion in the text as a whole?
A. It dismisses as untrue the research presented in the previous sentence.
A. It elaborates on a claim about labor relations in a particular industry made earlier in the text.
B. It defines a term used in the description that follows in the rest of the sentence.
B. It offers an example of a trend in the World War II–era economy discussed earlier in the text.
C. It emphasizes the main goal of the research introduced in the previous sentence.
C. It notes a possible exception to the historical narrative of labor activism sketched earlier in the text.
D. It provides context that clarifies the significance of the information that follows in the rest of the sentence.
D. It provides further details about the identities of the workers discussed earlier in the text.

ID: 066a3295 Answer


Correct Answer: D ID: ac9a3a26 Answer
Correct Answer: A
Rationale
Rationale
Choice D is the best answer. The text describes how a new discovery expands our understanding of
Denisovans. The underlined portion describes what we used to believe about Denisovans, which helps the Choice A is the best answer because it best describes how the underlined portion functions in the text as a
reader understand the significance of the discovery of the molar: it suggests that they lived in more places whole. The text says that the increased production quotas of food processing companies during World War II
than we’d previously thought. enabled employees to make better bargains in exchange for their labor. The underlined portion presents an
example of this increased bargaining power: employees requested more favorable benefits, and employers
Choice A is incorrect. The underlined portion doesn’t do this. Instead, it explains what we used to believe complied because they were under pressure to fulfill the demanding terms of their contracts. Thus, the
about Denisovans before the discovery—it doesn’t dismiss the new discovery as false. Choice B is incorrect. underlined portion of the text elaborates on a claim about labor relations in a particular industry (food
The underlined portion doesn’t do this. No term is defined here. Choice C is incorrect. The underlined portion processing) made earlier in the text.
doesn’t do this. The text never tells us what the “goal” of the research was, just what its discovery was.
Choice B is incorrect because there is no indication in the text that the economic factors that influenced food
Question Difficulty: Easy processing also influenced other parts of the economy; thus, the bargaining described in the underlined
portion of the text cannot be called an example of a trend. Choice C is incorrect because the underlined portion
supports the historical narrative of labor activism in food processing that is sketched in the text, instead of
noting an exception to that narrative. Choice D is incorrect because while the underlined portion does discuss
the demands that workers made in exchange for their labor, it does not discuss the identities of the workers.

Question Difficulty: Hard

Question ID 03c9f327 Question ID e3f05561


Assessment Test Domain Skill Difficulty Assessment Test Domain Skill Difficulty

SAT Reading and Writing Craft and Structure Text Structure and SAT Reading and Writing Craft and Structure Words in Context
Purpose

ID: e3f05561
ID: 03c9f327
In the 1970s, video cameras became increasingly affordable for ordinary consumers and gave Ulysses Jenkins and
The following text is from Charlotte Brontë’s 1847 novel Jane Eyre. Jane, the narrator, works as a governess at other artists capabilities that were previously unavailable except to television broadcasters. Jenkins recognized and
Thornfield Hall. took full advantage of this ______ access to powerful technology to create groundbreaking works of video art, such as
I went on with my day’s business tranquilly; but ever and anon vague suggestions kept wandering across my brain of Mass of Images (1978).
reasons why I should quit Thornfield; and I kept involuntarily framing advertisements and pondering conjectures
about new situations: these thoughts I did not think to check; they might germinate and bear fruit if they could. Which choice completes the text with the most logical and precise word or phrase?

A. newfound
Which choice best states the main purpose of the text?

A. To convey a contrast between Jane’s outward calmness and internal restlessness B. delicate

B. To emphasize Jane’s loyalty to the people she works for at Thornfield Hall C. inevitable

C. To demonstrate that Jane finds her situation both challenging and deeply fulfilling D. habitual

D. To describe Jane’s determination to secure employment outside of Thornfield Hall


ID: e3f05561 Answer
Correct Answer: A
ID: 03c9f327 Answer
Rationale
Correct Answer: A

Rationale Choice A is the best answer because it most logically completes the text’s discussion of Ulysses Jenkins’s
video art. As used in this context, “newfound” means recently discovered or established. The text indicates
Choice A is the best answer because it most accurately describes the main purpose of the text, which is to that in the 1970s, video cameras became cheaper and therefore more widely available than they had been in
show that while Jane calmly goes about her daily tasks, she is experiencing internal agitation about possibly the past. The text goes on to say that this development provided Jenkins and other artists with capabilities
seeking a new job. At the start of the text, Jane says, “I went on with my day’s business tranquilly,” indicating that they previously didn’t have. As a result, Jenkins began producing groundbreaking works of video art. This
that she is outwardly calm. This outward calmness is then contrasted with her intense internal restlessness, context supports the idea that Jenkins took advantage of newfound access to video cameras.
as Jane says that thoughts of leaving her job keep running through her mind, that she is “involuntarily
framing advertisements” (meaning that she can’t stop herself from thinking up potential listings for jobs), and Choice B is incorrect because “delicate” means fine in texture or structure or easily broken, neither of which
that she often wonders what new “situations” (or jobs) would be like. would make sense in this context. The text doesn’t focus on describing what video art looks like or whether it’s
breakable. Choice C is incorrect because “inevitable” means impossible to avoid, which wouldn’t make sense
Choice B is incorrect because the text gives no indication of Jane’s feelings, either positive or negative, about in this context. The text doesn’t discuss the likelihood of artists, or anyone else, gaining access to video
the people she works for at Thornfield Hall. And rather than emphasizing that Jane feels particularly loyal to cameras. Choice D is incorrect because “habitual” means doing something regularly or repeatedly. Although
her employers, the text focuses on her constant consideration of leaving her job. Choice C is incorrect because the text does suggest that Jenkins created multiple pieces of video art, its focus is on the fact that video
the text gives no indication that Jane finds her current situation fulfilling, or satisfying. Given that much of the cameras had only just become widely available to artists in the 1970s. Jenkins’s ability to take advantage of
text is focused on Jane’s thoughts about possibly leaving her job for a new one, it might be the case that she video cameras to make art was therefore newfound, not habitual, at the time.
finds her situation challenging, but there is no evidence in the text that Jane also finds that situation
satisfying—she says nothing positive about her current job at all, in fact. Choice D is incorrect because the text Question Difficulty: Easy
describes Jane as wondering about getting a new job, not as determined to definitely do so. Jane keeps
thinking about reasons why she “should” quit her current job (indicating that she hasn’t yet decided to) and
imagining possible new situations she could find, but she says at the end of the text that these thoughts
“might germinate and bear fruit if they could,” meaning that the thoughts haven’t yet led to a decision—that
Jane isn’t yet determined to get a new job somewhere else.

Question Difficulty: Hard


Question ID c4737d6a Question ID 47598085
Assessment Test Domain Skill Difficulty Assessment Test Domain Skill Difficulty

SAT Reading and Writing Craft and Structure Cross-Text SAT Reading and Writing Craft and Structure Text Structure and
Connections Purpose

ID: c4737d6a ID: 47598085


Text 1 Yawn contagion occurs when one individual yawns in response to another’s yawn. Studies of this behavior in primates
Africa’s Sahara region—once a lush ecosystem—began to dry out about 8,000 years ago. A change in Earth’s orbit that have focused on populations in captivity, but biologist Elisabetta Palagi and her colleagues have shown that it can
affected climate has been posited as a cause of desertification, but archaeologist David Wright also attributes the shift occur in wild primate populations as well. In their study, which focused on a wild population of gelada monkeys
to Neolithic peoples. He cites their adoption of pastoralism as a factor in the region drying out: the pastoralists’ (Theropithecus gelada) in Ethiopia, the researchers further reported that yawn contagion most commonly occurred in
livestock depleted vegetation, prompting the events that created the Sahara Desert. males and across different social groups instead of within a single social group.
Text 2
Research by Chris Brierley et al. challenges the idea that Neolithic peoples contributed to the Sahara’s desertification. Which choice best describes the function of the first sentence in the text as a whole?
Using a climate-vegetation model, the team concluded that the end of the region’s humid period occurred 500 years A. It defines a phenomenon that is discussed in the text.
earlier than previously assumed. The timing suggests that Neolithic peoples didn’t exacerbate aridity in the region but,
in fact, may have helped delay environmental changes with practices (e.g., selective grazing) that preserved vegetation. B. It introduces a problem that is examined in the text.

Based on the texts, how would Chris Brierley (Text 2) most likely respond to the discussion in Text 1? C. It makes a claim that is challenged in the text.

A. By pointing out that given the revised timeline for the end of the Sahara’s humid period, the Neolithic peoples’ D. It presents a hypothesis that is evaluated in the text.
mode of subsistence likely didn’t cause the region’s desertification

B. By claiming that pastoralism was only one of many behaviors the Neolithic peoples took part in that may have ID: 47598085 Answer
contributed to the Sahara’s changing climate Correct Answer: A
C. By insisting that pastoralism can have both beneficial and deleterious effects on a region’s vegetation and climate Rationale

D. By asserting that more research needs to be conducted into factors that likely contributed to the desertification of Choice A is the best answer because it most accurately describes how the first sentence functions in the text
the Sahara region as a whole. The first sentence introduces what yawn contagion is, explaining that it occurs when an individual
yawns in response to the yawn of another individual. The text goes on to describe Elisabetta Palagi and her
colleagues’ study of this phenomenon in a wild population of gelada monkeys. According to the text, the study
ID: c4737d6a Answer
showed that wild primate populations experience yawn contagion and that the behavior occurs most
Correct Answer: A commonly in male monkeys and across social groups. Thus, the function of the first sentence is to define the
Rationale phenomenon of yawn contagion that is discussed in the text.

Choice A is the best answer. Brierley et al.’s research directly challenges Wright’s claim that pastoralism Choice B is incorrect. Although the first sentence introduces the text’s discussion of yawn contagion, it doesn’t
turned the Sahara into a desert, suggesting that, in a Sahara that turned arid 500 years earlier than previously present this behavior, or anything else, as a problem. Choice C is incorrect because the first sentence doesn’t
thought, pastoral practices may have actually “preserved vegetation” rather than depleting it. present a claim but instead explains what yawn contagion is. Moreover, the text doesn’t challenge anything;
it’s an informative text that describes the findings of a research study about yawning in wild primate
Choice B is incorrect. Brierley et al.’s research actually disputes the idea that any Neolithic peoples’ behaviors, populations. Choice D is incorrect. Although the text describes a scientific study, and most scientific studies
including pastoralism, could have contributed to the Sahara’s changing climate. In fact, their research implies are guided by a hypothesis, the text doesn’t say what Palagi and her colleagues’ hypothesis was; the text
that the Neolithic peoples’ practices did not “exacerbate aridity” (i.e., make things worse), but may have slowed discusses their findings instead.
environmental changes. Choice C is incorrect. Brierley et al.’s research does not acknowledge that pastoralism
can have deleterious (i.e., negative) effects on a region’s vegetation and climate. It only describes one possible Question Difficulty: Medium
beneficial effect: preserving vegetation through practices like selective grazing. Choice D is incorrect. Brierley
et al.’s research does not call for more research into factors that likely contributed to the desertification of the
Sahara region.

Question Difficulty: Hard

Neither text supports this. Text 1 briefly mentions that soy sauce is “made from fermented soybeans,” but it

Question ID a87c3925 never claims that fermentation is responsible for its flavor in any way. And Text 2 never mentions
fermentation at all.

Assessment Test Domain Skill Difficulty Question Difficulty: Hard

SAT Reading and Writing Craft and Structure Cross-Text


Connections

ID: a87c3925
Text 1
Soy sauce, made from fermented soybeans, is noted for its umami flavor. Umami—one of the five basic tastes along
with sweet, bitter, salty, and sour—was formally classified when its taste receptors were discovered in the 2000s. In
2007, to define the pure umami flavor scientists Rie Ishii and Michael O’Mahony used broths made from shiitake
mushrooms and kombu seaweed, and two panels of Japanese and US judges closely agreed on a description of the
taste.

Text 2
A 2022 experiment by Manon Jünger et al. led to a greater understanding of soy sauce’s flavor profile. The team
initially presented a mixture of compounds with low molecular weights to taste testers who found it was not as salty or
bitter as real soy sauce. Further analysis of soy sauce identified proteins, including dipeptides, that enhanced umami
flavor and also contributed to saltiness. The team then made a mix of 50 chemical compounds that re-created soy
sauce’s flavor.

Based on the texts, if Ishii and O’Mahony (Text 1) and Jünger et al. (Text 2) were aware of the findings of both
experiments, they would most likely agree with which statement?

A. On average, the diets of people in the United States tend to have fewer foods that contain certain dipeptides than
the diets of people in Japan have.

B. Chemical compounds that activate both the umami and salty taste receptors tend to have a higher molecular
weight than those that only activate umami taste receptors.

C. Fermentation introduces proteins responsible for the increase of umami flavor in soy sauce, and those proteins also
increase the perception of saltiness.

D. The broths in the 2007 experiment most likely did not have a substantial amount of the dipeptides that played a
key part in the 2022 experiment.

ID: a87c3925 Answer


Correct Answer: D

Rationale

Choice D is the best answer. Ishii and O’Mahony were trying to isolate the pure umami flavor, while Jünger
was trying to recreate soy sauce, which has a mix of flavors that includes umami. Accordingly, the broths from
Text 1 are not described as having any soy sauce in them—just “shiitake mushrooms and kombu seaweed.” So
they probably don’t have as much of the dipeptides described in Text 2, which were found to be a key part of
soy sauce’s umami-ness and its saltiness.

Choice A is incorrect. Neither text supports this. Neither text gets into the diets of people in the United States,
nor the diets of people in Japan. Choice B is incorrect. Neither text supports this. Text 2 does talk about the
molecular weights of chemical compounds, but there isn’t enough information provided about molecular
weights in Text 1 to make an inference about what the scientists in Text 1 would say. Choice C is incorrect.
Question ID 4a2b2535 Question ID b0f7541b
Assessment Test Domain Skill Difficulty Assessment Test Domain Skill Difficulty

SAT Reading and Writing Craft and Structure Words in Context SAT Reading and Writing Craft and Structure Text Structure and
Purpose

ID: 4a2b2535
ID: b0f7541b
A brief book review cannot fully convey the ______ of Olga Tokarczuk’s novel The Books of Jacob, with its enormous cast
of characters, its complicated, wandering plot, and its page numbers that count backward (beginning at 965 and The following text is adapted from Herman Melville’s 1857 novel The Confidence-Man. Humphry Davy was a prominent
ending at 1). British chemist and inventor.

Years ago, a grave American savant, being in London, observed at an evening party there, a certain coxcombical
Which choice completes the text with the most logical and precise word or phrase?
fellow, as he thought, an absurd ribbon in his lapel, and full of smart [banter], whisking about to the admiration of
A. accuracy as many as were disposed to admire. Great was the savant’s disdain; but, chancing ere long to find himself in a
corner with the jackanapes, got into conversation with him, when he was somewhat ill-prepared for the good
B. inactivity
sense of the jackanapes, but was altogether thrown aback, upon subsequently being [informed that he was] no
C. complexity less a personage than Sir Humphry Davy.

D. restraint Which choice best states the main purpose of the text?

A. It portrays the thoughts of a character who is embarrassed about his own behavior.
ID: 4a2b2535 Answer
B. It presents an account of a misunderstanding.
Correct Answer: C
C. It offers a short history of how a person came to be famous.
Rationale
D. It explains why one character dislikes another.
Choice C is the best answer because it most logically completes the text’s discussion of Olga Tokarczuk’s novel
The Books of Jacob. As used in this context, “complexity” means having many complicated parts that when
taken as a whole are difficult to follow or explain. The text indicates that The Books of Jacob has a large cast of ID: b0f7541b Answer
characters, a complicated and wandering plot (that is, a plot that is difficult to follow), and reverse page Correct Answer: B
numbering. Together, these features make up a novel that’s challenging to read and summarize. This context
Rationale
supports the idea that a brief book review can’t do justice to the novel’s complexity.

Choice B is the best answer. The text tells a story of a first impression that turned out to be wrong: a serious
Choice A is incorrect. Although the word “accuracy,” or being free from error or falsehood, can sometimes be
American savant was dismissive of a goofy-looking, wisecracking guest at a British party, and then was
used to describe a novel, the text doesn’t discuss whether Tokarczuk’s novel has this quality. Instead, the text
shocked to learn that the guest was actually a prominent British chemist and inventor.
describes the novel as having a large cast of characters, a difficult-to-follow plot, and reverse page numbering.
These features suggest complexity, not accuracy. Choice B is incorrect because “inactivity” means being in a
Choice A is incorrect. This is too strong and too narrow. Only at the very end is the savant "thrown aback" by
state of idleness or doing nothing, neither of which would make sense in this context. The text describes
the fact that the man was Sir Humphry Davy—he’s not "embarrassed about his own behavior." Choice C is
Tokarczuk’s novel, and although it’s possible the novel could portray its characters as inactive, it wouldn’t
incorrect. This isn’t the main purpose. The text never provides the history of how Sir Humphry Davy came to
make sense to describe the novel itself as such. Choice D is incorrect because in this context “restraint” would
be famous. Nor does it provide any history for the American savant. Choice D is incorrect. This is too narrow. It
mean holding back or showing self-control, and the text doesn’t indicate that Tokarczuk’s novel has either of
doesn’t include the second half of the text, where the savant gets into a conversation with the man and then
these qualities. In fact, the features of the novel that the text describes, such as a large cast of characters, a
finds out that the man is Sir Humphry Davy.
complicated and wandering plot, and reverse page numbering, suggest excess and complexity, not restraint.
Question Difficulty: Hard
Question Difficulty: Easy

Question ID 8d802289 Question ID 7bf79a90


Assessment Test Domain Skill Difficulty Assessment Test Domain Skill Difficulty

SAT Reading and Writing Craft and Structure Cross-Text SAT Reading and Writing Craft and Structure Cross-Text
Connections Connections

ID: 8d802289 ID: 7bf79a90


Text 1 Text 1
Dance choreographer Alvin Ailey’s deep admiration for jazz music can most clearly be felt in the rhythms and beats his Microbes are tiny organisms in the soil, water, and air all around us. They thrive even in very harsh conditions. That’s
works were set to. Ailey collaborated with some of the greatest jazz legends, like Charles Mingus, Charlie Parker, and why Noah Fierer and colleagues were surprised when soil samples they collected from an extremely cold, dry area in
perhaps his favorite, Duke Ellington. With his choice of music, Ailey helped bring jazz to life for his audiences. Antarctica didn’t seem to contain any life. The finding doesn’t prove that there are no microbes in that area, but the
team says it does suggest that the environment severely restricts microbes’ survival.
Text 2
Jazz is present throughout Ailey’s work, but it’s most visible in Ailey’s approach to choreography. Ailey often Text 2
incorporated improvisation, a signature characteristic of jazz music, in his work. When managing his dance company, Microbes are found in virtually every environment on Earth. So it’s unlikely they would be completely absent from
Ailey rarely forced his dancers to an exact set of specific moves. Instead, he encouraged his dancers to let their own Fierer’s team’s study site, no matter how extreme the environment is. There were probably so few organisms in the
skills and experiences shape their performances, as jazz musicians do. samples that current technology couldn’t detect them. But since a spoonful of typical soil elsewhere might contain
billions of microbes, the presence of so few in the Antarctic soil samples would show how challenging the conditions
Based on the texts, both authors would most likely agree with which statement? are.
A. Dancers who worked with Ailey greatly appreciated his supportive approach as a choreographer.
Based on the texts, Fierer’s team and the author of Text 2 would most likely agree with which statement about
B. Ailey’s work was strongly influenced by jazz. microbes?

C. Audiences were mostly unfamiliar with the jazz music in Ailey’s works. A. Most microbes are better able to survive in environments with extremely dry conditions than in environments with
harsh temperatures.
D. Ailey blended multiple genres of music together when choreographing dance pieces.
B. A much higher number of microbes would probably be found if another sample of soil were taken from the
Antarctic study site.
ID: 8d802289 Answer
C. Microbes are likely difficult to detect in the soil at the Antarctic study site because they tend to be smaller than
Correct Answer: B microbes found in typical soil elsewhere.
Rationale
D. Most microbes are probably unable to withstand the soil conditions at the Antarctic study site.
Choice B is the best answer. Author 1 states that Ailey had a “deep admiration for jazz music” and that he
“helped bring jazz to life for his audiences.” Author 2 states that “Jazz is present throughout Ailey’s work.”
ID: 7bf79a90 Answer
While the authors name different aspects of Ailey’s work as the most influenced by jazz, they agree that jazz
was a strong influence. Correct Answer: D

Rationale
Choice A is incorrect. This isn’t something that either text claims. Neither text mentions how Ailey’s dancers
felt about his approach as a choreographer, so we have no evidence that either author would agree with this. Choice D is the best answer because it presents a statement about microbes with which Fierer’s team (Text 1)
Choice C is incorrect. This isn’t something that either text claims. Neither text mentions how familiar and the author of Text 2 would most likely agree. Text 1 states that microbes usually thrive in very harsh
audiences were with any aspect of Ailey’s works, so we have no evidence that either author would agree with conditions, and so Fierer’s team was surprised when samples collected from an extremely cold and dry area of
this. Choice D is incorrect. This isn’t something that either text claims. Neither text mentions any genre of Antarctica didn’t appear to contain any life. Fierer’s team says that though this doesn’t conclusively prove
music other than jazz, so we have no evidence that either author would agree with this. there are no microbes in the area, it suggests that microbes would have a notably difficult time surviving in the
environment. The author of Text 2 says it’s unlikely that there would be no microbes at all in the Antarctic
Question Difficulty: Easy
study site from which Fierer’s team retrieved soil samples and that there may have been hard-to-detect
microbes in the samples. However, the presence of only a few microbes in the Antarctic samples rather than
the billions found in a typical soil sample (which would presumably be much easier to detect) would illustrate
conditions in the Antarctic soil that make it difficult for microbes to thrive. Since Fierer’s team says that the
seeming absence of microbes in the Antarctic samples suggests an unusually harsh environment and the
author of Text 2 says that even if there are a few undetectable microbes in the samples, the relatively tiny
number of microbes would also suggest an unusually harsh environment, then Fierer’s team and the author of
Text 2 would most likely agree that most microbes are unable to withstand the soil conditions at the Antarctic
Question ID 9ccf463e
study site.
Assessment Test Domain Skill Difficulty
Choice A is incorrect. The samples taken by Fierer’s team were from an area of Antarctica that is described in
part as extremely dry, and these samples didn’t appear to have any life. Therefore, even though these samples SAT Reading and Writing Craft and Structure Words in Context
also came from an extremely cold area, Fierer’s team wouldn’t argue based on the evidence available that
microbes were better able to survive in dry conditions than in areas with harsh temperatures. Moreover, the
ID: 9ccf463e
author of Text 2 says that microbes are found in virtually every environment on Earth but doesn’t compare dry
environments and harsh environments. Choice B is incorrect. Nothing in Text 1 indicates that another The following text is from Nella Larsen’s 1928 novel Quicksand.
collection of samples from the Antarctic study site might yield different results from the samples already
The trees in their spring beauty sent through her restive mind a sharp thrill of pleasure. Seductive, charming, and
taken by Fierer’s team. The author of Text 2 does state that microbes are found in virtually every environment
beckoning as cities were, they had not this easy unhuman loveliness.
on Earth and suggests that new technology may be better able to detect so few microbes in a soil sample, but
the author of Text 2 concludes that the unusual absence of microbes in the Antarctic samples is evidence of As used in the text, what does the word “beckoning” most nearly mean?
the harsh Antarctic environment. Therefore, there is no reason to believe that the author of Text 2 thinks that
A. Demanding
another sample drawn from that same harsh environment would yield a much higher number of microbes.
Choice C is incorrect. The author of Text 2 does speculate that there may have been so few microbes in the B. Signaling
Antarctic samples that current technology couldn’t detect them, but the author doesn’t speculate that this is
due to the size of the microbes. Moreover, nothing that Fierer’s team says suggests that they are speculating C. Inviting
that their samples might have microbes that are smaller than microbes in typical soil samples.
D. Shifting

Question Difficulty: Medium

ID: 9ccf463e Answer


Correct Answer: C

Rationale

Choice C is the best answer because as used in the text, “beckoning” most nearly means “inviting,” or
attractive. The text portrays a woman who is looking at “trees in their spring beauty.” She compares them to
cities, which have their own pleasures even if they do not have the “easy unhuman loveliness” of trees: she
thinks of cities as “seductive” and “charming,” both adjectives that signify something that is enticing, or
attractive. Therefore, cities that are seductive and charming would also be described as inviting people closer
to them.

Choice A is incorrect because there is no indication in this context that cities are “demanding,” or requiring
effort. Choice B is incorrect. Though “signaling,” or communicating something, might be considered a key
feature of the act of “beckoning,” in the context here, “beckoning” suggests that cities have attractive qualities
that naturally draw people to them. Such attractive qualities are not described by the word “signaling” alone.
Therefore, “signaling” is an incorrect answer because it is insufficiently precise. Choice D is incorrect because
there is no reason to think in this context that the cities are “shifting,” or changing shape.

Question Difficulty: Easy

Question Difficulty: Easy

Question ID 835d1ae6
Assessment Test Domain Skill Difficulty

SAT Reading and Writing Craft and Structure Cross-Text


Connections

ID: 835d1ae6
Text 1
Historians studying pre-Inca Peru have looked to ceramic vessels to understand daily life among the Moche people.
These mold-made sculptures present plants, animals, and human faces in precise ways—vessels representing human
faces are so detailed that scholars have interpreted facial markings to represent scars and other skin irregularities.
Some historians have even used these objects to identify potential skin diseases that may have afflicted people at the
time.

Text 2
Art historian and archaeologist Lisa Trever has argued that the interpretation of Moche “portrait” vessels as hyper-
realistic portrayals of identifiable people may inadvertently disregard the creativity of the objects’ creators. Moche
ceramic vessels, Trever argues, are artworks in which sculptors could free their imagination, using realistic objects and
people around them as inspiration to explore more abstract concepts.

Based on the texts, what would Lisa Trever (Text 2) most likely say about the interpretation presented in the underlined
portion of Text 1?

A. Depictions of human faces are significantly more realistic than depictions of plants and other animals are.

B. It is likely that some depictions of human faces with extensive markings are intended to portray the same historical
individual.

C. Some vessels may have been damaged during their excavation and thus provide little insight into Moche culture.

D. Markings on depictions of human faces are not necessarily intended to portray particular details about the physical
appearance of individuals.

ID: 835d1ae6 Answer


Correct Answer: D

Rationale

Choice D is the best answer. Trever thinks that the “hyper-realistic portrayal of identifiable people”
interpretation ignores the sculptors’ imagination and creativity. We can infer that Trevor thinks the facial
markings on the sculptures may not have represented real skin blemishes on real people.

Choice A is incorrect. The text gives us no reason to think that Trever would respond to the underlined
interpretation in this way. Neither text compares the depictions of human faces to the depictions of plants or
animals, so we have no basis to draw this conclusion. Choice B is incorrect. The text gives us no reason to
think that Trever would respond to the underlined interpretation in this way. There’s nothing in either text
about multiple depictions representing the same person, so we have no basis to draw this conclusion. Choice C
is incorrect. The text gives us no reason to think that Trever would respond to the underlined interpretation in
this way. Neither text mentions the state of the vessels (damaged or intact), so we have no basis to draw this
conclusion.
Question ID ca47273b Question ID 69a6d050
Assessment Test Domain Skill Difficulty Assessment Test Domain Skill Difficulty

SAT Reading and Writing Craft and Structure Words in Context SAT Reading and Writing Craft and Structure Words in Context

ID: ca47273b ID: 69a6d050


Biologist Jane Edgeloe and colleagues have located what is believed to be the largest individual plant in the world in In the early 1800s, the Cherokee scholar Sequoyah created the first script, or writing system, for an Indigenous
the Shark Bay area of Australia. The plant is a type of seagrass called Posidonia australis, and it ______ approximately language in the United States. Because it represented the sounds of spoken Cherokee so accurately, his script was easy
200 square kilometers. to learn and thus quickly achieved ______ use: by 1830, over 90 percent of the Cherokee people could read and write it.

Which choice completes the text with the most logical and precise word or phrase? Which choice completes the text with the most logical and precise word or phrase?

A. acknowledges A. widespread

B. produces B. careful

C. spans C. unintended

D. advances D. infrequent

ID: ca47273b Answer ID: 69a6d050 Answer


Correct Answer: C Correct Answer: A

Rationale Rationale

Choice C is the best answer. “Spans” means “extends over a distance of” or “encompasses.” Since we’re talking Choice A is the best answer because it most logically completes the text’s discussion of the writing system
about the world’s largest plant, it makes sense to say that it “spans” about 200 square kilometers. created by Sequoyah. In this context, “widespread” means widely accepted or practiced. The text indicates that
because Sequoyah’s script accurately represented the spoken sounds of the Cherokee language and was easy
Choice A is incorrect. “Acknowledges” means “recognizes” or “admits the truth of.” Either way, it doesn’t make to learn, nearly all Cherokee people were able to read and write it soon after it was created. This context
sense here: a plant can’t “acknowledge” a distance. Choice B is incorrect. “Produces” can mean “makes,” demonstrates that the script was widely used by the Cherokee people.
“causes,” or “presents.” But none of those definitions make sense here: a plant can’t make, cause, or present a
distance. Choice D is incorrect. This doesn’t fit the logic of the text. “Advances” means “moves forward” or Choice B is incorrect. In this context, “careful” would mean exercised with care and attentive concern.
“progresses.” But the plant isn’t necessarily moving forward. Rather, the text suggests that it already covers a Although the work of creating a writing system likely involved great care, the text indicates that the system
distance of 200 square kilometers. was “easy to learn,” which conflicts with the idea that using this system requires a noteworthy amount of care.
Choice C is incorrect because in this context “unintended” means not deliberate. The idea that using
Question Difficulty: Easy Sequoyah’s script was unintentional conflicts directly with the claim that it was easy to learn and used by
“over 90% of the Cherokee people” by 1830. In fact, because one had to learn this system, it’s not clear how one
could use it unintentionally. Choice D is incorrect because in this context “infrequent” means rare or not
occurring often, which conflicts directly with the claim that “over 90% of the Cherokee people” were using
Sequoyah’s script by 1830.

Question Difficulty: Easy

Question ID 81da17d3 Question ID e41dfaab


Assessment Test Domain Skill Difficulty Assessment Test Domain Skill Difficulty

SAT Reading and Writing Craft and Structure Cross-Text SAT Reading and Writing Craft and Structure Words in Context
Connections

ID: e41dfaab
ID: 81da17d3
In 1929 the Atlantic Monthly published several articles based on newly discovered letters allegedly exchanged between
Text 1 President Abraham Lincoln and a woman named Ann Rutledge. Historians were unable to ______ the authenticity of the
Italian painters in the 1500s rarely depicted themselves in their work. Even more rare were self-portrait paintings that letters, however, and quickly dismissed them as a hoax.
portrayed the artist as a painter. At the time, painting was not yet respected as a profession, so painters mostly chose
to emphasize other qualities in their self-portraits, like their intellect or social status. In the city of Bologna, the first Which choice completes the text with the most logical and precise word or phrase?
artist to depict themself painting was a man named Annibale Carracci. A painting of his from around 1585 shows
A. validate
Carracci in front of an easel holding a palette.
B. interpret
Text 2
C. relate
In their self-portraits, Bolognese artists typically avoided referring to the act of painting until the mid-1600s. However,
Lavinia Fontana’s 1577 painting, Self-Portrait at the Keyboard, stands out as the earliest example of such a work by an D. accommodate
artist from Bologna. Although the artist is depicted playing music, in the background, one can spot a painting easel by
a window.
ID: e41dfaab Answer
Based on the texts, how would the author of Text 2 most likely respond to the underlined claim in Text 1?
Correct Answer: A
A. Carracci and Fontana were among the most well-respected painters in Bologna at the time.
Rationale
B. The depiction of Fontana in Self-Portrait at the Keyboard was intended to underscore the artist’s creativity.
Choice A is the best answer because it most logically completes the text’s discussion of letters allegedly
C. Fontana likely inspired the reference to an easel and palette in Carracci’s painting. exchanged between President Lincoln and Rutledge. In this context, “validate” means to confirm that
something is real or correct. According to the text, it was alleged, or claimed, that the newly discovered letters
D. Self-Portrait at the Keyboard was painted earlier than Carracci’s painting and also refers to the artist’s craft.
had been written by Lincoln and Rutledge. The text also indicates that historians ultimately decided the
letters were a hoax, or fraudulent. This context suggests that the historians couldn’t confirm that the letters
ID: 81da17d3 Answer were authentic.

Correct Answer: D Choice B is incorrect. The text focuses on the authenticity of the letters, which were claimed to have been
Rationale written by Lincoln and Rutledge and were then quickly dismissed as fraudulent by historians. Rather than
conveying that the historians simply weren’t able to “interpret,” or explain in an understandable way, the
Choice D is the best answer. The author of Text 2 states that Fontana’s painting, which depicts a painting easel letters’ authenticity, the text suggests that the historians decided the letters lacked authenticity altogether.
in the background, was made in 1577, eight years before Carracci’s painting. Therefore, they might argue that Choice C is incorrect. The text states that the historians quickly dismissed the letters claimed to have been
Caracci was not “the first artist” to depict themself as a painter. written by Lincoln and Rutledge as fraudulent; this suggests that rather than being unable to “relate,” or tell
others about, the letters’ authenticity, the historians were able to share what they’d decided about the letters.
Choice A is incorrect. The texts don’t support this choice. Neither text mentions the reputation or status of Choice D is incorrect because it wouldn’t make sense to suggest that the historians couldn’t “accommodate,” or
either Carracci or Fontana. Choice B is incorrect. The author of Text 2 does not comment on the intention or give consideration to, the authenticity of the letters claimed to have been written by Lincoln and Rutledge; the
meaning of Fontana’s self-depiction in Self-Portrait at the Keyboard. This choice also holds little connection to text states that the historians decided that the letters were fraudulent, which indicates that they did consider
the underlined claim. Choice C is incorrect. The texts don’t support this choice. The author of Text 2 does not whether the letters were authentic.
suggest any inspirational relationship between Fontana and Carracci. The author of Text 2 is concerned with
showing that Fontana’s painting is the earliest example of an artist referring to painting in their self-portrait, Question Difficulty: Easy
but makes no mention of her influence on others.

Question Difficulty: Easy


Question ID e13171c4 Question ID 4d1a9c0d
Assessment Test Domain Skill Difficulty Assessment Test Domain Skill Difficulty

SAT Reading and Writing Craft and Structure Text Structure and SAT Reading and Writing Craft and Structure Words in Context
Purpose

ID: 4d1a9c0d
ID: e13171c4
Following the principles of community-based participatory research, tribal nations and research institutions are equal
Historians Tiya Miles and Roy E. Finkenbine have both documented the assistance Indigenous peoples gave to Black partners in health studies conducted on reservations. A collaboration between the Crow Tribe and Montana State
freedom seekers leaving the South before the US Civil War. Much of the historical evidence of this help comes from University ­______ this model: tribal citizens worked alongside scientists to design the methodology and continue to
Indigenous oral traditions and from autobiographies written by the freedom seekers. One such narrative is Jermain assist in data collection.
Loguen’s autobiography, which tells about how Neshnabé (Potawatomi) villagers offered him food, lodging, and
directions during his 1835 journey from Tennessee to Canada. Which choice completes the text with the most logical and precise word or phrase?

A. circumvents
Which choice best describes the function of the underlined sentence?

A. It provides an example of an autobiography that describes help given by an Indigenous people to a Black freedom B. eclipses
seeker.
C. fabricates
B. It shows why Loguen decided to write in great detail about his experiences traveling from Tennessee to Canada in
D. exemplifies
his autobiography.

C. It argues that autobiographies are particularly important sources of information about geography in the United
ID: 4d1a9c0d Answer
States before the Civil War.
Correct Answer: D
D. It suggests that most historians believe that Neshnabé villagers were more successful in assisting freedom seekers
Rationale
than other people were.

Choice D is the best answer because it most logically completes the text’s discussion of the collaboration
ID: e13171c4 Answer between the Crow Tribe and Montana State University. As used in this context, “exemplifies” means
demonstrates. The text conveys how the Crow Tribe–Montana State University collaboration serves to
Correct Answer: A
illustrate the model of community-based participatory research introduced earlier in the text and expanded
Rationale on later in the text.

Choice A is the best answer. The previous sentence broadly mentions "autobiographies written by the freedom Choice A is incorrect because referring to “circumvents,” or avoids, wouldn’t make sense in context. The text
seekers." This sentence identifies Loguen’s autobiography as a specific example. suggests that the Crow Tribe–Montana State University collaboration serves as an example of the principles of
community-based participatory research, not that the collaboration evades this model. Choice B is incorrect
Choice B is incorrect. The sentence never explains why Loguen chose to write about his experiences. Choice C because referring to “eclipses,” or overshadows, wouldn’t make sense in context. The text describes the Crow
is incorrect. The previous sentence identifies autobiographies as useful sources of historical information Tribe–Montana State University collaboration as an equal partnership, which indicates that it’s an example of
about a specific topic, but not for "information about geography." The underlined sentence provides details of the community-based participatory research model, not that it overshadows the model. Choice C is incorrect
one autobiography as an example of a source of information about that specific topic (interactions between because saying that the collaboration “fabricates,” or creates, the model wouldn’t make sense in context. The
Indigenous people and Black freedom seekers). Choice D is incorrect. The text never discusses other specific text indicates that the Crow Tribe–Montana State University collaboration serves as an example of the model,
people who helped freedom seekers, and therefore can’t make a comparison between the Neshnabé and not that it created the model.
anyone else.
Question Difficulty: Easy
Question Difficulty: Easy

Question ID 5effa190 Question ID f3fac04f


Assessment Test Domain Skill Difficulty Assessment Test Domain Skill Difficulty

SAT Reading and Writing Craft and Structure Words in Context SAT Reading and Writing Craft and Structure Words in Context

ID: 5effa190 ID: f3fac04f


The process of mechanically recycling plastics is often considered ______ because of the environmental impact and the Bioluminescent beetles called fireflies may seem to create flashes of light randomly, but each species of firefly actually
loss of material quality that often occurs. But chemist Takunda Chazovachii has helped develop a cleaner process of has its own special series of repeated flashes and pauses. These unique ______ allow fireflies of the same species to find
chemical recycling that converts superabsorbent polymers from diapers into a desirable reusable adhesive. each other.

Which choice completes the text with the most logical and precise word or phrase? Which choice completes the text with the most logical and precise word or phrase?

A. resilient A. quantities

B. inadequate B. decorations

C. dynamic C. patterns

D. satisfactory D. agreements

ID: 5effa190 Answer ID: f3fac04f Answer


Correct Answer: B Correct Answer: C

Rationale Rationale

Choice B is the best answer because it most logically completes the text’s discussion about recycling plastics. Choice C is the best answer because it most logically completes the text’s discussion of the flashes created by
In this context, “inadequate” means not satisfactory. The text indicates that the mechanical plastic-recycling fireflies. In this context, “patterns” means distinct and predictable sequences. The text indicates that although
process affects the environment and causes “the loss of material quality.” The text contrasts that with the flashes that fireflies produce appear to occur randomly—that is, without any particular sequence or rhythm
Chazovachii’s chemical plastic-recycling process, which is cleaner and produces a desirable product. The —each species actually produces its own special series of flashes and pauses. Indeed, these series of flashes
text’s emphasis on the negative aspects of mechanical recycling suggests that it is inadequate in terms of are so unique that fireflies can use them to find other members of their species. Therefore, this context
environmental impact and the quality of the material the process yields. supports the idea that fireflies produce flashes in distinct and recognizable patterns.

Choice A is incorrect because in this context “resilient” would mean able to withstand difficulty and the text Choice A is incorrect because “quantities” means certain amounts or numbers of something. Although the text
does not characterize the plastic-recycling process as having this quality or describe any difficulties that discusses how different firefly species produce flashes and pauses in unique sequences that help other
these processes might need to overcome. Choice C is incorrect because in this context “dynamic” would mean members of their species to find them, it doesn’t mention the number of flashes that are used in these
constantly changing. Although the text suggests that there have been changes in the field of recycling, as is sequences. Choice B is incorrect because in this context, “decorations” would mean things that make an object
the case with the advent of Chazovachii’s chemical recycling process, there is nothing to suggest that the more beautiful. Although it may be reasonable to say that firefly flashes are beautiful, the text focuses on the
mechanical process itself has changed or is prone to change. Choice D is incorrect because in this context fact that fireflies use these unique sequences of flashes to find other members of their own species, not that
“satisfactory” would mean acceptable but not perfect. The text mentions only shortcomings of the mechanical the flashes make fireflies more beautiful. Choice D is incorrect because in this context, “agreements” would
process (environmental effects and lower material quality), so the text more strongly supports a negative view refer to deals that individuals have discussed and come to a consensus about. Since fireflies aren’t capable of
of this process and provides no evidence that it would be considered satisfactory. making such agreements, it wouldn’t make sense to use this word to refer to the signals they send each other
with their flashes.
Question Difficulty: Easy
Question Difficulty: Easy
because neither text discusses how likely lithium and sodium are to be detected by analyzing wavelengths of

Question ID d6c77ae5 light.

Question Difficulty: Hard


Assessment Test Domain Skill Difficulty

SAT Reading and Writing Craft and Structure Cross-Text


Connections

ID: d6c77ae5

Text 1
Astronomer Mark Holland and colleagues examined four white dwarfs—small, dense remnants of past stars—in order
to determine the composition of exoplanets that used to orbit those stars. Studying wavelengths of light in the white
dwarf atmospheres, the team reported that traces of elements such as lithium and sodium support the presence of
exoplanets with continental crusts similar to Earth’s.

Text 2
Past studies of white dwarf atmospheres have concluded that certain exoplanets had continental crusts. Geologist Keith
Putirka and astronomer Siyi Xu argue that those studies unduly emphasize atmospheric traces of lithium and other
individual elements as signifiers of the types of rock found on Earth. The studies don’t adequately account for different
minerals made up of various ratios of those elements, and the possibility of rock types not found on Earth that contain
those minerals.

Based on the texts, how would Putirka and Xu (Text 2) most likely characterize the conclusion presented in Text 1?

A. As unexpected, because it was widely believed at the time that white dwarf exoplanets lack continental crusts

B. As premature, because researchers have only just begun trying to determine what kinds of crusts white dwarf
exoplanets had

C. As questionable, because it rests on an incomplete consideration of potential sources of the elements detected in
white dwarf atmospheres

D. As puzzling, because it’s unusual to successfully detect lithium and sodium when analyzing wavelengths of light in
white dwarf atmospheres

ID: d6c77ae5 Answer


Correct Answer: C

Rationale

Choice C is the best answer because it reflects how Putirka and Xu (Text 2) would likely characterize the
conclusion presented in Text 1. Text 1 discusses a study by Mark Holland and colleagues in which they
detected traces of lithium and sodium in the atmospheres of four white dwarf stars. The team claims that this
supports the idea that exoplanets with continental crusts like Earth’s once orbited these stars. Text 2
introduces Putirka and Xu, who indicate that sodium and lithium are present in several different minerals and
that some of those minerals might exist in types of rock that are not found on Earth. Therefore, Putirka and Xu
would likely describe the conclusion in Text 1 as questionable because it does not consider that lithium and
sodium are also found in rocks that are not like Earth’s continental crust.

Choice A is incorrect because the texts do not indicate how widely held any of the viewpoints described
are. Choice B is incorrect because neither text discusses how new this area of study is. Choice D is incorrect

Question ID 6d5ddea4 Question ID df46a2ee


Assessment Test Domain Skill Difficulty Assessment Test Domain Skill Difficulty

SAT Reading and Writing Craft and Structure Words in Context SAT Reading and Writing Craft and Structure Text Structure and
Purpose

ID: 6d5ddea4
ID: df46a2ee
According to Potawatomi ecologist Robin Wall Kimmerer, the Indigenous method of harvesting Hierochloe odorata, or
sweetgrass, by snapping the plant off at the root actually ______ wild populations: it may seem counterintuitive, she The following text is from Joseph Conrad’s 1907 novel The Secret Agent: A Simple Tale. Mr. Verloc is navigating the
says, but this method of removal allows new sweetgrass plants to repopulate the space, with an overall increase in London streets on his way to a meeting.
number and vigor. Before reaching Knightsbridge, Mr. Verloc took a turn to the left out of the busy main thoroughfare, uproarious with
the traffic of swaying omnibuses and trotting vans, in the almost silent, swift flow of hansoms [horse-drawn carriages].
Which choice completes the text with the most logical and precise word or phrase? Under his hat, worn with a slight backward tilt, his hair had been carefully brushed into respectful sleekness; for his
business was with an Embassy. And Mr. Verloc, steady like a rock—a soft kind of rock—marched now along a street
A. selects
which could with every propriety be described as private.
B. originates
Which choice best describes the function of the underlined phrase in the text as a whole?
C. conditions
A. It qualifies an earlier description of Mr. Verloc.
D. replenishes
B. It emphasizes an internal struggle Mr. Verloc experiences.

C. It contrasts Mr. Verloc with his surroundings.


ID: 6d5ddea4 Answer
Correct Answer: D D. It reveals a private opinion Mr. Verloc holds.

Rationale
ID: df46a2ee Answer
Choice D is the best answer because it most logically completes the text’s discussion of the Indigenous
method of harvesting Hierochloe odorata. As used in this context, “replenishes” means helps increase the Correct Answer: A
population or helps it recover. The text explains that although snapping off a wild plant at the root might seem Rationale
detrimental to the wild population, it actually helps Hierochloe odorata, increasing both their “number and
vigor.” This context conveys the idea that even though it seems counterintuitive, the Indigenous method of Choice A is the best answer. The underline phrase qualifies (meaning adds limits or conditions to) the
harvesting Hierochloe odorata actually replenishes the wild population. description of Mr. Verloc as “steady like a rock,” adding that he is a “soft” rock.

Choice A is incorrect. Although a harvesting method could be used to select for certain traits in plants, it’s not Choice B is incorrect. In fact, the passage never mentions Mr. Verloc experiencing any internal struggles.
clear what it would mean for a harvesting method to select “wild populations” of plants. Choice B is incorrect Choice C is incorrect. The underlined phrase doesn’t contrast Mr. Verloc with his surroundings, but is instead
because as used in this context, “originates” means creates. The text doesn’t address the origin of Hierochloe modifying the description of him as a rock. Choice D is incorrect. The underlined phrase doesn’t reveal a
odorata, but rather how the Indigenous harvesting method affects it. Choice C is incorrect because in this private opinion Mr. Verloc holds: instead, it further describes his character for the reader.
context, “conditions” means to influence someone or something to behave in a certain way, and the text
Question Difficulty: Medium
doesn’t suggest the new plants that replace the harvested ones differ in any meaningful way, or in any way
that could be the result of conditioning.

Question Difficulty: Easy


Question ID 5a278f24 Question ID 76e4c51d
Assessment Test Domain Skill Difficulty Assessment Test Domain Skill Difficulty

SAT Reading and Writing Craft and Structure Words in Context SAT Reading and Writing Craft and Structure Words in Context

ID: 5a278f24 ID: 76e4c51d


The work of molecular biophysicist Enrique M. De La Cruz is known for ______ traditional boundaries between academic The güiro, a musical instrument traditionally made from a dried and hollowed gourd, is thought to have originated
disciplines. The university laboratory that De La Cruz runs includes engineers, biologists, chemists, and physicists, and with the Taíno people of Puerto Rico. Players use a wooden stick to scrape along ridges cut into the side of the gourd,
the research the lab produces makes use of insights and techniques from all those fields. creating sounds that are highly ______: the sounds produced by güiros can differ based on the distance between the
ridges, the types of strokes the player uses, and the thickness of the gourd.
Which choice completes the text with the most logical and precise word or phrase?
Which choice completes the text with the most logical and precise word or phrase?
A. epitomizing
A. overlooked
B. transcending
B. powerful
C. anticipating
C. routine
D. reinforcing
D. variable

ID: 5a278f24 Answer


Correct Answer: B ID: 76e4c51d Answer
Correct Answer: D
Rationale
Rationale
Choice B is the best answer. Based on the text, we’re looking for a word that means something similar to
"mak[ing] use of insights and techniques from all those fields." "Transcending" means "going beyond," so Choice D is the best answer because it most logically completes the text’s discussion of the sounds made by
"transcending traditional boundaries" would mean crossing into all those various fields of research, which is güiros. In this context, “variable” means able to change. The text begins by explaining that güiros are
exactly the meaning we want. instruments made out of hollowed gourds with ridges cut into their sides and that players scrape the ridges
with wooden sticks to produce sounds. The text goes on to say that güiros’ sounds can change depending on
Choice A is incorrect. This isn’t a logical word choice. Based on the text, we’re looking for a word that means gourd thickness, the distance between ridges, and the types of strokes the player uses, thus supporting the
something similar to "mak[ing] use of insights and techniques from all those fields." To "epitomize" means to idea that the sounds created by these instruments are variable.
"be a perfect example of," so "epitomizing traditional boundaries" would mean the opposite of what we want:
keeping the fields of research separate. Choice C is incorrect. This isn’t a logical word choice. Based on the Choice A is incorrect because “overlooked” means not being seen or noticed, and there is nothing in the text to
text, we’re looking for a word that means something similar to "mak[ing] use of insights and techniques from suggest that the sounds produced by güiros are overlooked or not noticed. Choice B is incorrect because in this
all those fields." "Anticipating" means "expecting" or "waiting for," and would result in a confusing sentence context, “powerful” would mean having a great ability to produce an effect. While it’s possible that the sounds
with an unclear meaning. Choice D is incorrect. This isn’t a logical word choice. Based on the text, we’re produced by güiros have a strong effect on listeners, the text doesn’t discuss this aspect of their sounds.
looking for a word that means something similar to "mak[ing] use of insights and techniques from all those Choice C is incorrect because “routine” means usual and unvarying, and there is nothing in the text to suggest
fields." "Reinforcing traditional boundaries" would mean the opposite: keeping the fields of research separate. that the sounds produced by güiros are unvarying. In fact, the text describes how the sounds produced by
güiros can differ based on several factors.
Question Difficulty: Medium
Question Difficulty: Hard

Question ID a2835734 Question ID ff97fd53


Assessment Test Domain Skill Difficulty Assessment Test Domain Skill Difficulty

SAT Reading and Writing Craft and Structure Words in Context SAT Reading and Writing Craft and Structure Text Structure and
Purpose

ID: a2835734
ID: ff97fd53
Visual artist Gabriela Alemán states that the bold colors of comics, pop art, and Latinx culture have always fascinated
her. This passion for the rich history and colors of her Latinx community translates into the ______ artworks she In 1973, poet Miguel Algarín started inviting other writers who, like him, were Nuyorican—a term for New Yorkers of
produces. Puerto Rican heritage—to gather in his apartment to present their work. The gatherings were so well attended that
Algarín soon had to rent space in a cafe to accommodate them. Thus, the Nuyorican Poets Cafe was born. Moving to a
Which choice completes the text with the most logical and precise word or phrase? permanent location in 1981, the Nuyorican Poets Cafe expanded its original scope beyond the written word, hosting
art exhibitions and musical performances as well. Half a century since its inception, it continues to foster emerging
A. vivid
Nuyorican talent.
B. unknown
Which choice best describes the overall purpose of the text?
C. definite
A. To explain what motivated Algarín to found the Nuyorican Poets Cafe
D. reserved
B. To situate the Nuyorican Poets Cafe within the cultural life of New York as a whole

C. To discuss why the Nuyorican Poets Cafe expanded its scope to include art and music
ID: a2835734 Answer
Correct Answer: A D. To provide an overview of the founding and mission of the Nuyorican Poets Cafe

Rationale
ID: ff97fd53 Answer
Choice A is the best answer. "Vivid" can mean "colorful" or "bright-colored." This definition fits the context
Correct Answer: D
clues about Alemán’s fascination with and passion for bold colors.
Rationale
Choice B is incorrect. This doesn’t fit the logic of the text. Nothing in the text indicates that Alemán’s artworks
are "unknown." Choice C is incorrect. This doesn’t fit the logic of the text. "Definite" means "certain" or Choice D is the best answer. The text presents a brief history of the Nuyorican Poets Cafe, from how it got
"decided." It wouldn’t make sense to describe artwork as "definite." Choice D is incorrect. This doesn’t fit the started in the ’70s, to its expansion in the ’80s, to its ongoing mission today.
logic of the text. "Reserved" can either mean "slow to reveal emotions" or "booked." But the clues suggest that
Alemán’s artworks are boldly colorful—almost the opposite of "reserved." Choice A is incorrect. This isn’t the overall purpose. The text never mentions Algarín’s motivations. Choice B is
incorrect. This isn’t the overall purpose. The text never discusses the cultural life of New York as a whole.
Question Difficulty: Easy Choice C is incorrect. This is too narrow. One sentence mentions that the Nuyorican Poets Cafe expanded its
scope to include art and music, but this is only one point in the broader history of the Nuyorican Poets Cafe,
which is the overall focus of the text.

Question Difficulty: Medium


scope of the relationship between posture and cognition that has been reported in some previous studies,

Question ID c61a7c4a O’Brien and Ahmed’s study is not one of those studies, and there is no suggestion that anyone has
misunderstood O’Brien and Ahmed’s findings. Choice B is incorrect because the text makes no mention of the
methods used in previous studies of the effects of posture on cognition. Although the text does urge caution
Assessment Test Domain Skill Difficulty when discussing posture’s effects on cognition, it does not critique the results of studies that suggested that
posture can affect cognition. Instead, the text suggests that such results should not be exaggerated or taken
SAT Reading and Writing Craft and Structure Text Structure and too broadly. Choice C is incorrect because although the text implies that overstating posture’s effects on
Purpose
cognition would be a problem, nothing in the text suggests that O’Brien and Ahmed share that view or that
they attempted to solve that problem. O’Brien and Ahmed are presented as hypothesizing that posture would
ID: c61a7c4a affect cognition in their study, not as trying to resolve the problem the text describes.

Some studies have suggested that posture can influence cognition, but we should not overstate this phenomenon. A
Question Difficulty: Hard
case in point: In a 2014 study, Megan O’Brien and Alaa Ahmed had subjects stand or sit while making risky simulated
economic decisions. Standing is more physically unstable and cognitively demanding than sitting; accordingly, O’Brien
and Ahmed hypothesized that standing subjects would display more risk aversion during the decision-making tasks
than sitting subjects did, since they would want to avoid further feelings of discomfort and complicated risk
evaluations. But O’Brien and Ahmed actually found no difference in the groups’ performance.

Which choice best states the main purpose of the text?

A. It argues that research findings about the effects of posture on cognition are often misunderstood, as in the case of
O’Brien and Ahmed’s study.

B. It presents the study by O’Brien and Ahmed to critique the methods and results reported in previous studies of the
effects of posture on cognition.

C. It explains a significant problem in the emerging understanding of posture’s effects on cognition and how O’Brien
and Ahmed tried to solve that problem.

D. It discusses the study by O’Brien and Ahmed to illustrate why caution is needed when making claims about the
effects of posture on cognition.

ID: c61a7c4a Answer


Correct Answer: D

Rationale

Choice D is the best answer because it most accurately describes the main purpose of the text. The text notes
that although some studies have suggested that posture may have an effect on cognition, this phenomenon
should not be overstated. In other words, the text begins by urging caution and restraint when discussing the
effects of posture on cognition, implying that even though some studies have shown posture to affect
cognition, we should not assume that posture always affects cognition or does so in a strong way. The text
goes on to discuss O’Brien and Ahmed’s study as a "case in point" (that is, as an example of the point made
previously). According to the text, O’Brien and Ahmed hypothesized that since standing is more cognitively
demanding than sitting, standing subjects in their experiment would respond differently to decision-making
tasks than sitting subjects would, which would show an effect of posture on cognition. What the researchers
actually found, however, was that the standing and sitting subjects performed the same—posture did not affect
cognition. By presenting a circumstance in which posture does not appear to affect cognition, the discussion
of O’Brien and Ahmed’s study shows why it is important not to overstate the phenomenon. The purpose of the
text, therefore, is to discuss O’Brien and Ahmed’s study to illustrate why caution is needed when making
claims about posture’s effects on cognition.

Choice A is incorrect because the text discusses O’Brien and Ahmed’s study as an example of why caution is
needed when discussing posture’s effects on cognition, not as an example of how research findings related to
posture and cognition are often misunderstood. Although the text does warn against misunderstanding the
Question Difficulty: Medium

Question ID acb852e7
Assessment Test Domain Skill Difficulty

SAT Reading and Writing Craft and Structure Text Structure and
Purpose

ID: acb852e7
The following text is from the 1923 poem “Black Finger” by Angelina Weld Grimké, a Black American writer. A cypress is
a type of evergreen tree.
I have just seen a most beautiful thing,
Slim and still,
Against a gold, gold sky,
A straight black cypress,
Sensitive,
Exquisite,
A black finger
Pointing upwards.
Why, beautiful still finger, are you black?
And why are you pointing upwards?

Which choice best describes the overall structure of the text?

A. The speaker assesses a natural phenomenon, then questions the accuracy of her assessment.

B. The speaker describes a distinctive sight in nature, then ponders what meaning to attribute to that sight.

C. The speaker presents an outdoor scene, then considers a human behavior occurring within that scene.

D. The speaker examines her surroundings, then speculates about their influence on her emotional state.

ID: acb852e7 Answer


Correct Answer: B

Rationale

Choice B is the best answer because it most accurately describes the overall structure of the text. First, the
speaker describes observing a “most beautiful” sight: a tree (“black cypress”) standing out from the golden sky
behind it, looking like a person’s finger “pointing upwards” and appearing “sensitive” and “exquisite.” Then the
speaker wonders about the image’s meaning, asking why the finger is black and why it’s pointing upward.
Thus, the text moves from the speaker’s description of a distinctive sight in nature to her pondering about
what meaning to attribute to that sight.

Choice A is incorrect because the speaker assesses a natural sight—a “black cypress” tree standing “against a
gold, gold sky” like a pointed finger—but doesn’t question the accuracy of her own assessment. Although she
wonders why the finger, which is really a tree, is black and why it’s pointing, the speaker doesn’t suggest that
her belief that the tree resembles a finger is wrong. Choice C is incorrect. Although the speaker describes
seeing a “black cypress” tree standing “against a gold, gold sky” like a pointed finger, she wonders about that
natural image (asking why the finger, which is really a tree, is black and why it’s pointing) and doesn’t give any
indication that any people are present in the scene. Choice D is incorrect. Although the speaker examines and
wonders about one thing in her surroundings—a “black cypress” tree standing “against a gold, gold sky” like a
pointed finger—she doesn’t address her own emotional state or consider how it’s affected by her surroundings.
Question ID 98364791 Question ID aa5897b8
Assessment Test Domain Skill Difficulty Assessment Test Domain Skill Difficulty

SAT Reading and Writing Craft and Structure Words in Context SAT Reading and Writing Craft and Structure Text Structure and
Purpose

ID: 98364791
ID: aa5897b8
In studying the use of external stimuli to reduce the itching sensation caused by an allergic histamine response, Louise
Ward and colleagues found that while harmless applications of vibration or warming can provide a temporary In Jane Austen’s novel Mansfield Park, an almost imperceptible smile from potential suitor Henry Crawford causes the
distraction, such ______ stimuli actually offer less relief than a stimulus that seems less benign, like a mild electric shock. protagonist Fanny Price to blush; her embarrassment grows when she suspects that he is aware of it. This moment—in
which Fanny not only infers Henry’s mental state through his gestures, but also infers that he is drawing inferences
Which choice completes the text with the most logical and precise word or phrase? about her mental state—illustrates what literary scholar George Butte calls “deep intersubjectivity,” a technique for
representing interactions between consciousnesses through which Austen’s novels derive much of their social and
A. deceptive
psychological drama.
B. innocuous
Which choice best describes the function of the underlined sentence in the text as a whole?
C. novel
A. It states a claim about Austen’s skill at representing psychological complexity that is reinforced by an example
D. impractical presented in the following sentence.

B. It advances an interpretation of an Austen protagonist who is contrasted with protagonists from other Austen
ID: 98364791 Answer novels cited in the following sentence.

Correct Answer: B C. It describes a recurring theme in Austen’s novels that is the focus of a literary scholar’s analysis summarized in the
Rationale following sentence.

D. It provides a synopsis of an interaction in an Austen novel that illustrates a literary concept discussed in the
Choice B is the best answer because it most logically completes the text’s discussion of Ward and colleagues’
following sentence.
findings. As used in this context, “innocuous” means mild or unharmful. The text describes the vibration and
warming that Ward and colleagues used to alleviate itching as “harmless applications” and goes on to contrast
these applications with another stimulus that actually offers more relief even though it seems to be stronger ID: aa5897b8 Answer
and “less benign.” This context conveys the idea that vibration and warming were innocuous stimuli.
Correct Answer: D
Choice A is incorrect because the text focuses on a distinction between harmless stimuli and those that seem Rationale
to be less benign. Nothing in the text suggests that any of the treatments are “deceptive,” or misleading;
indeed, even the less effective ones are described as offering some relief. Choice C is incorrect because the text Choice D is the best answer. The underlined sentence provides a concrete example to ground readers’
focuses on the amount of relief from itching offered by harmless stimuli and those that seem to be less benign. understanding of the “deep intersubjectivity” described in the next sentence as central to Austen’s work.
The text doesn’t suggest that any of these stimuli are “novel,” or original and new; heat, vibration, and
electricity aren’t new inventions. Choice D is incorrect because it wouldn’t make sense to describe an Choice A is incorrect. There is no evaluation made of Austen’s skill in this sentence, and no examples are
application of vibration or warming as “impractical,” or not suitable for use. The text indicates that these given in the following sentence. This choice essentially flips the paragraph: it’s this first sentence that
harmless applications are useful in that they offer at least some temporary relief. provides an example. Choice B is incorrect. There are no other Austen protagonists mentioned in this passage,
so this couldn’t be the answer. Choice C is incorrect. The underlined sentence doesn’t identify any “recurring
Question Difficulty: Hard theme,” but instead simply describes one interaction from one book. This interaction exemplifies the literary
technique of “deep intersubjectivity” that is introduced in the next sentence.

Question Difficulty: Hard

Question ID a4ca92fd Question ID 8de51658


Assessment Test Domain Skill Difficulty Assessment Test Domain Skill Difficulty

SAT Reading and Writing Craft and Structure Words in Context SAT Reading and Writing Craft and Structure Cross-Text
Connections

ID: a4ca92fd
ID: 8de51658
Beginning in the 1950s, Navajo Nation legislator Annie Dodge Wauneka continuously worked to promote public
health; this ______ effort involved traveling throughout the vast Navajo homeland and writing a medical dictionary for Text 1
speakers of Diné bizaad, the Navajo language. The idea that time moves in only one direction is instinctively understood, yet it puzzles physicists. According to the
second law of thermodynamics, at a macroscopic level some processes of heat transfer are irreversible due to the
Which choice completes the text with the most logical and precise word or phrase? production of entropy—after a transfer we cannot rewind time and place molecules back exactly where they were
before, just as we cannot unbreak dropped eggs. But laws of physics at a microscopic or quantum level hold that those
A. impartial
processes should be reversible.
B. offhand
Text 2
C. persistent
In 2015, physicists Tiago Batalhão et al. performed an experiment in which they confirmed the irreversibility of
D. mandatory thermodynamic processes at a quantum level, producing entropy by applying a rapidly oscillating magnetic field to a
system of carbon-13 atoms in liquid chloroform. But the experiment “does not pinpoint ... what causes [irreversibility]
at the microscopic level,” coauthor Mauro Paternostro said.
ID: a4ca92fd Answer
Based on the texts, what would the author of Text 1 most likely say about the experiment described in Text 2?
Correct Answer: C
A. It would suggest an interesting direction for future research were it not the case that two of the physicists who
Rationale
conducted the experiment disagree on the significance of its findings.
Choice C is the best answer because it most logically completes the text’s discussion of Annie Dodge
B. It provides empirical evidence that the current understanding of an aspect of physics at a microscopic level must be
Wauneka’s work as a Navajo Nation legislator. As used in this context, “persistent” means existing
incomplete.
continuously. The text states that Wauneka “continuously worked to promote public health,” traveling
extensively and authoring a medical dictionary; this indicates that Wauneka’s effort was persistent. C. It is consistent with the current understanding of physics at a microscopic level but not at a macroscopic level.

Choice A is incorrect because describing Wauneka’s effort related to public health as “impartial,” or not partial D. It supports a claim about an isolated system of atoms in a laboratory, but that claim should not be extrapolated to
or biased and treating all things equally, wouldn’t make sense in context. The text suggests that Wauneka’s a general claim about the universe.
continuous work was partial in one way, as she focused specifically on promoting public health throughout
the Navajo homeland and to speakers of the Navajo language. Choice B is incorrect because the text
ID: 8de51658 Answer
emphasizes that Wauneka’s effort to promote public health as a Navajo Nation legislator was continuous and
extensive, involving wide travels and the authoring of a medical dictionary. Because this work clearly involved Correct Answer: B
care and dedication, it wouldn’t make sense to describe it as “offhand,” or casual and informal. Choice D is Rationale
incorrect because nothing in the text suggests that Wauneka’s effort to promote public health was
“mandatory,” or required by law or rule, even though Wauneka was a Navajo Nation legislator. Rather than Choice B is the best answer. Author 1 describes the puzzle that physicists still can’t solve: at a microscopic
suggesting that Wauneka’s effort was required for any reason, the text emphasizes the continuous and level, the “laws of physics” suggest that we should be able to reverse processes that are not reversible at a
extensive nature of her work. macroscopic level (and, maybe, turn back time!). The experiment confirmed that those processes are not
reversible even on the microscopic level, but it didn’t explain why. This supports Author 1’s point that
Question Difficulty: Easy physicists still don’t fully understand how things work at a microscopic level—maybe the laws need to be
revised.

Choice A is incorrect. We can’t infer that the author of Text 1 would respond this way to the experiment. Text 2
does name two of the physicists involved in the experiment, but it never suggests that they disagree on
anything. Choice C is incorrect. This is the opposite of what the experiment suggests. The experiment
confirmed that the macroscopic-level law (“these things can’t be reversed—like time”) was still true on the
microscopic level—meaning it supports the current understanding of physics at a macroscopic level. Choice D
is incorrect. We can’t infer that the author of Text 1 would respond this way to the experiment. Neither text
makes this distinction between laboratory findings and the way the universe works in general.
Question ID e0656211
Question Difficulty: Medium
Assessment Test Domain Skill Difficulty

SAT Reading and Writing Craft and Structure Words in Context

ID: e0656211
In Nature Poem (2017), Kumeyaay poet Tommy Pico portrays his ______ the natural world by honoring the centrality of
nature within his tribe’s traditional beliefs while simultaneously expressing his distaste for being in wilderness settings
himself.

Which choice completes the text with the most logical and precise word or phrase?

A. responsiveness to

B. ambivalence toward

C. renunciation of

D. mastery over

ID: e0656211 Answer


Correct Answer: B

Rationale

Choice B is the best answer because it most logically completes the text’s description of how Pico feels about
the natural world. In this context, to say that Pico portrays his “ambivalence toward” nature would mean that
he portrays his mixed feelings about nature. The text explains that Pico “honors the centrality of nature” and
also makes it clear that he doesn’t enjoy being in nature. This context suggests that Pico feels ambivalence
toward nature.

Choice A is incorrect because saying that Pico portrays his “responsiveness to” nature would mean that he
portrays himself as quick to react to nature, which isn’t supported by the text. Instead, the text focuses on
Pico’s mixed feelings toward nature, describing him as both honoring nature’s role in his tribe’s beliefs and
expressing his personal dislike for being in nature. Choice C is incorrect because saying that Pico portrays his
“renunciation of” nature would mean that he portrays himself as rejecting nature, which isn’t supported by the
text. The text conveys that Pico demonstrates both positive and negative responses toward nature, not that
he’s giving it up completely. Choice D is incorrect because saying that Pico portrays his “mastery over” nature
would mean that he portrays himself as having control over nature, which isn’t supported by the text. The text
focuses on Pico’s mixed feelings about nature; nothing in the text suggests that Pico feels mastery over nature.

Question Difficulty: Medium

Question ID 9421ed62 Question ID 7d84fe2b


Assessment Test Domain Skill Difficulty Assessment Test Domain Skill Difficulty

SAT Reading and Writing Craft and Structure Text Structure and SAT Reading and Writing Craft and Structure Words in Context
Purpose

ID: 7d84fe2b
ID: 9421ed62
Particle physicists like Ayana Holloway Arce and Aida El-Khadra spend much of their time ______ what is invisible to the
In 2007, computer scientist Luis von Ahn was working on converting printed books into a digital format. He found that naked eye: using sophisticated technology, they closely examine the behavior of subatomic particles, the smallest
some words were distorted enough that digital scanners couldn’t recognize them, but most humans could easily read detectable parts of matter.
them. Based on that finding, von Ahn invented a simple security test to keep automated “bots” out of websites. The
first version of the reCAPTCHA test asked users to type one known word and one of the many words scanners couldn’t Which choice completes the text with the most logical and precise word or phrase?
recognize. Correct answers proved the users were humans and added data to the book-digitizing project.
A. selecting

Which choice best states the main purpose of the text? B. inspecting
A. To discuss von Ahn’s invention of reCAPTCHA
C. creating
B. To explain how digital scanners work
D. deciding
C. To call attention to von Ahn’s book-digitizing project

D. To indicate how popular reCAPTCHA is ID: 7d84fe2b Answer


Correct Answer: B

ID: 9421ed62 Answer Rationale

Correct Answer: A
Choice B is the best answer because it most logically completes the text’s discussion of the work of particle
Rationale physicists. In this context, “inspecting” means viewing closely in order to examine. The text indicates that as
particle physicists, Arce and El-Khadra’s work involves using advanced technology to “closely examine”
Choice A is the best answer because it most accurately states the main purpose of the text. After providing a subatomic particles. In other words, they use technology to inspect small parts of matter that can’t be seen by
brief introduction to computer scientist Luis von Ahn, the text focuses on discussing how von Ahn’s the naked eye.
digitization work led to the invention of a digital security test known as reCAPTCHA.
Choice A is incorrect because nothing in the text suggests that Arce and El-Khadra spend time “selecting,” or
Choice B is incorrect because the text doesn’t address how digital scanners work. Choice C is incorrect. choosing, subatomic particles for some purpose; the text simply states that the particle physicists use
Although the text mentions von Ahn’s book-digitizing project, that information is provided as a detail, not as advanced technology to see and study the behavior of those tiny parts of matter. Choice C is incorrect because
the main purpose of the text. Choice D is incorrect because the text doesn’t provide any indication of nothing in the text suggests that Arce and El-Khadra spend time “creating” subatomic particles, or bringing
reCAPTCHA’s popularity; instead, it describes reCAPTCHA’s origin. them into existence; the text simply states that the particle physicists use advanced technology to see and
study the behavior of those tiny parts of matter. Choice D is incorrect. In this context, “deciding” would mean
Question Difficulty: Easy
making a final choice or judgment about something. It wouldn’t make sense to say that particle physicists get
to choose what is and isn’t visible to the naked eye, especially when the text presents it as fact that subatomic
particles are “the smallest detectable parts of matter” and would therefore be invisible. The text focuses on
Arce and El-Khadra’s close observation of those particles, not on any decisions they might make.

Question Difficulty: Easy


Question ID d2eb1df1 Question ID d72b325e
Assessment Test Domain Skill Difficulty Assessment Test Domain Skill Difficulty

SAT Reading and Writing Craft and Structure Words in Context SAT Reading and Writing Craft and Structure Cross-text
Connections

ID: d2eb1df1
ID: d72b325e
In recommending Bao Phi’s collection Sông I Sing, a librarian noted that pieces by the spoken-word poet don’t lose
their ______ nature when printed: the language has the same pleasant musical quality on the page as it does when Text 1
performed by Phi. What factors influence the abundance of species in a given ecological community? Some theorists have argued that
historical diversity is a major driver of how diverse an ecological community eventually becomes: differences in
Which choice completes the text with the most logical and precise word or phrase? community diversity across otherwise similar habitats, in this view, are strongly affected by the number of species living
in those habitats at earlier times.
A. scholarly
Text 2
B. melodic In 2010, a group of researchers including biologist Carla Cáceres created artificial pools in a New York forest. They
stocked some pools with a diverse mix of zooplankton species and others with a single zooplankton species and
C. jarring
allowed the pool communities to develop naturally thereafter. Over the course of four years, Cáceres and colleagues
D. personal periodically measured the species diversity of the pools, finding—contrary to their expectations—that by the end of
the study there was little to no difference in the pools’ species diversity.

ID: d2eb1df1 Answer Based on the texts, how would Cáceres and colleagues (Text 2) most likely describe the view of the theorists presented
in Text 1?
Correct Answer: B
A. It is largely correct, but it requires a minor refinement in light of the research team’s results.
Rationale
B. It is not compelling as a theory regardless of any experimental data collected by the research team.
Choice B is the best answer. Based on the text, we’re looking for a word that means something similar to
“pleasant musical quality.” That’s exactly what “melodic” means. C. It may seem plausible, but it is not supported by the research team’s findings.

Choice A is incorrect. This isn’t a logical word choice. Based on the text, we’re looking for a word that means D. It probably holds true only in conditions like those in the research team’s study.
something similar to “pleasant musical quality.” “Scholarly” would suggest something that is academic or well-
researched, which doesn’t match the meaning we’re looking for. Choice C is incorrect. This isn’t a logical word
ID: d72b325e Answer
choice. Based on the text, we’re looking for a word that means something similar to “pleasant musical quality.”
“Jarring” would suggest the opposite: something unpleasant or discordant. Choice D is incorrect. This isn’t a Correct Answer: C
logical word choice. Based on the text, we’re looking for a word that means something similar to “pleasant Rationale
musical quality.” “Personal” would suggest something that is expressive or intimate, which doesn’t match the
meaning we’re looking for. Choice C is the best answer. This is how Cáceres and co. would most likely describe the view presented in Text
1. The view in Text 1 is that historical diversity affects how diverse an ecological community eventually
Question Difficulty: Easy becomes. But Cáceres and co. did not get this result: they found no difference in eventual diversity between a
zooplankton pool that started out diverse and a zooplankton pool that started out with only a single species.

Choice A is incorrect. Cáceres and co. would probably not describe the view presented in Text 1 this way. The
view in Text 1 is that historical diversity affects how diverse an ecological community eventually becomes.
Cáceres and co’s findings directly undermine this view: they found no difference in eventual diversity between
a zooplankton pool that started out diverse and a zooplankton pool that started out with only a single species.
Choice B is incorrect. Cáceres and co. would probably not describe the view presented in Text 1 this way. Their
experiment was designed to test this hypothesis, and their findings were "contrary to their expectations." In
other words, before the study, they predicted the theory was correct. Choice D is incorrect. Cáceres and co.
would not describe the view presented in Text 1 this way. Their research finding directly undermines the view
presented in Text 1: so it definitely doesn’t hold true in conditions like those in the study.

Question Difficulty: Hard

Question ID cb526866 Question ID 83687083


Assessment Test Domain Skill Difficulty Assessment Test Domain Skill Difficulty

SAT Reading and Writing Craft and Structure Words in Context SAT Reading and Writing Craft and Structure Words in Context

ID: cb526866 ID: 83687083


The recent discovery of a carved wooden figure dating to around 2,000 years ago in a ditch in England was truly Due to their often strange images, highly experimental syntax, and opaque subject matter, many of John Ashbery’s
surprising. Wooden objects ______ survive for so long due to their high susceptibility to rot, but archaeologists suspect poems can be quite difficult to ______ and thus are the object of heated debate among scholars.
layers of sediment in the ditch preserved the figure by creating an oxygen-free environment.
Which choice completes the text with the most logical and precise word or phrase?
Which choice completes the text with the most logical and precise word or phrase?
A. delegate
A. sturdily
B. compose
B. carelessly
C. interpret
C. rarely
D. renounce
D. simply

ID: 83687083 Answer


ID: cb526866 Answer Correct Answer: C
Correct Answer: C
Rationale
Rationale
Choice C is the best answer because it most logically completes the text’s discussion of John Ashbery’s poems.
Choice C is the best answer because it most logically completes the text’s discussion of the discovery of a As used in this context, “interpret” would mean decipher the meaning of. The text indicates that Ashbery’s
carved wooden figure dating to around 2,000 years ago. In this context, “rarely” means infrequently. The text poems have many unusual features, that it’s difficult to tell what exactly the poems’ subject matter is, and that
states that the discovery of the figure was “truly surprising” and notes that wooden objects are highly prone to scholars strongly disagree about the poems. This context conveys the idea that it’s difficult to interpret
rot. This context conveys the idea that wooden objects infrequently survive for as long as the carved figure has Ashbery’s poems.
survived.
Choice A is incorrect because “delegate” means to assign someone as a representative of another person or to
Choice A is incorrect because “sturdily” means strongly, which wouldn’t make sense in context. If wooden entrust something to someone else, neither of which would make sense in context. The text is focused only on
objects in general could strongly survive for long periods of time, then the discovery of a wooden figure that’s the difficulty that readers have interpreting Ashbery’s poems due to their many unusual features; it doesn’t
around 2,000 years old wouldn’t be surprising. Choice B is incorrect because “carelessly” means accidentally. suggest anything about the poems being difficult to delegate. Choice B is incorrect because describing
The text conveys the idea that wooden objects in general don’t survive for very long because they rot, not that Ashbery’s poems as difficult to “compose,” or put together or produce, would make sense only if the text were
wooden objects in general accidentally survive despite this. Choice D is incorrect because the text conveys the about Ashbery’s experience of writing the poems. It could be true that it was difficult for Ashbery to compose
idea that wooden objects in general don’t survive for very long because they rot, not that wooden objects in his poems, but the text doesn’t address this; it instead discusses how readers interpret and engage with the
general “simply,” or merely, survive for long periods of time. If wooden objects in general could merely survive poems. Choice D is incorrect because describing Ashbery’s poems as being difficult to “renounce,” or give up or
for as long as the figure has survived, then the discovery of the figure wouldn’t have been surprising. refuse, wouldn’t make sense in context. The text focuses on the idea that features of Ashbery’s poems are odd
or unclear and have caused heated scholarly debate. This context suggests that the poems are difficult to
Question Difficulty: Easy interpret, not that the poems are difficult to renounce.

Question Difficulty: Easy


Question ID 637d0878 Question ID 39857700
Assessment Test Domain Skill Difficulty Assessment Test Domain Skill Difficulty

SAT Reading and Writing Craft and Structure Words in Context SAT Reading and Writing Craft and Structure Text Structure and
Purpose

ID: 637d0878
ID: 39857700
The Appalachian Trail is a hiking path in the eastern United States. Much of the 2,000 mile trail passes through
wilderness areas. In order to ______ those areas, the United States Congress passed the National Trails System Act in The following text is from Edith Wharton’s 1905 novel The House of Mirth. Lily Bart and a companion are walking
1968, ensuring that the trail would not be sold or commercially developed. through a park.
Lily had no real intimacy with nature, but she had a passion for the appropriate and could be keenly sensitive to a
Which choice completes the text with the most logical and precise word or phrase? scene which was the fitting background of her own sensations. The landscape outspread below her seemed an
enlargement of her present mood, and she found something of herself in its calmness, its breadth, its long free
A. borrow
reaches. On the nearer slopes the sugar-maples wavered like pyres of light; lower down was a massing of grey
B. postpone orchards, and here and there the lingering green of an oak-grove.

C. protect Which choice best describes the function of the underlined sentence in the text as a whole?

D. decorate A. It creates a detailed image of the physical setting of the scene.

B. It establishes that a character is experiencing an internal conflict.


ID: 637d0878 Answer
C. It makes an assertion that the next sentence then expands on.
Correct Answer: C
D. It illustrates an idea that is introduced in the previous sentence.
Rationale

Choice C is the best answer. "Protect" means "preserve" or "keep safe from." By ensuring that the wilderness ID: 39857700 Answer
areas along the Appalachian Trail can’t be sold or developed, the National Trails System Act protects them.
Correct Answer: D
Choice A is incorrect. "Borrow" means "to take something with intent to return it." The text doesn’t say Rationale
anything about taking and returning the wilderness that surrounds the Appalachian Trail. Choice B is
incorrect. "Postpone" means "to put off until later." Nothing in the passage suggests that Congress wants to Choice D is the best answer because it best describes how the underlined sentence functions in the text as a
"postpone" the wilderness areas (and that doesn’t make sense anyway—they can postpone doing something to whole. The first sentence of the text establishes that Lily can be “keenly sensitive to” scenes that serve as a
the wilderness areas, but they can’t postpone the areas themselves). Choice D is incorrect. "Decorate" means “fitting background” for her feelings—that is, she’s very aware of when a setting seems to reflect her mood. The
"to adorn" or "add extra items or pictures to make more attractive." No details in the text suggest that Congress next sentence, which is underlined, then demonstrates this awareness: Lily views the landscape she’s in as a
wishes to make the trail fancier or more attractive. large-scale reflection of her current mood, identifying with elements such as its calmness. Thus, the function
of the underlined sentence is to illustrate an idea introduced in the previous sentence.
Question Difficulty: Easy
Choice A is incorrect because the underlined sentence describes the scene only in very general terms,
referring to its calmness, breadth, and long stretches of land. It’s the next sentence that adds specific details
about colors, light, and various trees nearby. Choice B is incorrect because nothing in the underlined sentence
suggests that Lily is experiencing an internal conflict. In fact, the sentence indicates that Lily thinks the
landscape reflects her own feeling of calmness. Choice C is incorrect because the only assertion in the
underlined sentence is that Lily feels that broad aspects of the landscape, such as its calmness, reflect her
current mood, and that assertion isn’t expanded on in the next sentence. Instead, the next sentence describes
specific details of the scene without connecting them to Lily’s feelings.

Question Difficulty: Hard

Question ID e929fe98 Question ID 2aaee77f


Assessment Test Domain Skill Difficulty Assessment Test Domain Skill Difficulty

SAT Reading and Writing Craft and Structure Text Structure and SAT Reading and Writing Craft and Structure Text Structure and
Purpose Purpose

ID: e929fe98 ID: 2aaee77f


Composer Florence Price won first place for her score Symphony in E Minor at the 1932 Wanamaker Foundation Some bird species don’t raise their own chicks. Instead, adult females lay their eggs in other nests, next to another bird
Awards. The piece was performed the following year by the Chicago Symphony Orchestra, a significant recognition of species’ own eggs. Female cuckoos have been seen quickly laying eggs in the nests of other bird species when those
its quality. Price continued to compose many musical pieces throughout her career, blending traditional Black spirituals birds are out looking for food. After the eggs hatch, the noncuckoo parents will typically raise the cuckoo chicks as if
with classical European Romantic musical traditions. In recent years, Price’s concertos and symphonies have been they were their own offspring, even if the cuckoos look very different from the other chicks.
performed and recorded by several major orchestras, further preserving her work for others to enjoy.
Which choice best describes the function of the underlined sentence in the text as a whole?
Which choice best states the main purpose of the text?
A. It introduces a physical feature of female cuckoos that is described later in the text.
A. To provide examples of Price’s importance as a composer
B. It describes the appearance of the cuckoo nests mentioned earlier in the text.
B. To argue that more major orchestras should perform Price’s compositions
C. It offers a detail about how female cuckoos carry out the behavior discussed in the text.
C. To describe the musical styles that inspired many of Price’s symphonies
D. It explains how other birds react to the female cuckoo behavior discussed in the text.
D. To compare Price’s scores with those of classical European composers

ID: 2aaee77f Answer


ID: e929fe98 Answer Correct Answer: C
Correct Answer: A
Rationale
Rationale
Choice C is the best answer because it best describes how the underlined sentence functions in the text as a
Choice A is the best answer. The text provides an overview of Florence Price’s importance by describing her whole. The first two sentences establish that birds of some species don’t raise their own young; instead, they
success at the 1932 Wanamaker Foundation Awards, her blending of Black spirituals and classical European lay their eggs in the nests of birds of other species. The underlined sentence then states that female cuckoo
Romantic musical traditions, and the recent performances and recordings of her concertos and symphonies birds engage in this behavior, having been observed specifically laying their eggs in other nests while the
by major orchestras. other birds are out finding food. According to the text, the cuckoo chicks are then raised by the other birds.
Thus, the underlined sentence provides a particular detail about how female cuckoos carry out the behavior of
Choice B is incorrect. The text does mention that Price’s compositions have been performed and recorded by laying eggs for other birds to raise.
major orchestras, but it doesn’t argue that more orchestras should do so. Choice C is incorrect. The text does
mention the blending of Black spirituals and classical European Romantic musical traditions, but only briefly, Choice A is incorrect. Rather than mentioning a physical feature of female cuckoos, the underlined sentence
as part of a broader overview of Price’s career. Choice D is incorrect. The text mentions Price’s blending of introduces a specific behavior of female cuckoos: laying eggs in the nests of birds of other species when the
Black spirituals with classical European Romantic musical traditions, but it doesn’t directly compare Price’s other birds are away. The only reference to physical features is the last sentence’s general mention of cuckoo
scores with those of classical European composers. chicks looking different from chicks of other species. Choice B is incorrect because the underlined sentence
refers to the nests of birds other than cuckoos and doesn’t describe how any nests look, cuckoo or otherwise.
Question Difficulty: Easy Instead, the sentence addresses how female cuckoos use other birds’ nests. Choice D is incorrect because the
underlined sentence describes only female cuckoo behavior (laying eggs in the nests of birds of other species
when the other birds are away); it’s the last sentence of the text that addresses the other birds’ reaction,
indicating that those birds usually raise the cuckoo chicks once they’ve hatched.

Question Difficulty: Easy


Question Difficulty: Hard

Question ID 54804e10
Assessment Test Domain Skill Difficulty

SAT Reading and Writing Craft and Structure Words in Context

ID: 54804e10
While scholars believe many Mesoamerican cities influenced each other, direct evidence of such influence is difficult to
ascertain. However, recent excavations in a sector of Tikal (Guatemala) unearthed a citadel that shows ______
Teotihuacán (Mexico) architecture—including a near replica of a famed Teotihuacán temple—providing tangible
evidence of outside influence in portions of Tikal.

Which choice completes the text with the most logical and precise word or phrase?

A. refinements of

B. precursors of

C. commonalities with

D. animosities toward

ID: 54804e10 Answer


Correct Answer: C

Rationale

Choice C is the best answer because it most logically completes the text’s discussion of architectural
influences among Mesoamerican cities. In this context, “commonalities with” means similarities to or shared
attributes with. The text indicates that a recently discovered citadel in Tikal includes a close imitation of a
famous temple in Teotihuacán (another Mesoamerican city) and other evidence of Teotihuacán influence,
which suggests that the citadel possesses features that resemble architectural features found in Teotihuacán.
This context thus indicates that the Tikal citadel shows commonalities with Teotihuacán architecture.

Choice A is incorrect because there’s nothing in the text that suggests that the Tikal citadel shows
“refinements of,” or improvements on, Teotihuacán architecture. Although the text suggests that the
architecture of Teotihuacán influenced the architecture of the Tikal citadel, and although it’s possible that
later architectural designs could make improvements on earlier designs, the text doesn’t discuss whether, in
imitating Teotihuacán architecture, the Tikal citadel’s builders improved on it. Choice B is incorrect because
describing the citadel in Tikal as showing “precursors of” Teotihuacán architecture—or features that preceded
and foreshadowed those of Teotihuacán architecture—would imply the opposite of what the text suggests
about the relationship between the architecture found in Tikal and Teotihuacán. The text claims that the
discovery of similarities between the Tikal citadel and the architecture of Teotihuacán, including a replica of a
temple in Teotihuacán, provides evidence of outside influences on Tikal architecture. If the Tikal citadel was
influenced by Teotihuacán architecture, then the Teotihuacán architecture must predate the citadel, not the
other way around. In this context, therefore, it wouldn’t make sense to say that the Tikal citadel shows
precursors of Teotihuacán architecture. Choice D is incorrect because the text discusses how the citadel in
Tikal indicates the influence of Teotihuacán architecture, which implies that the makers of the Tikal citadel
likely admired aspects of Teotihuacán architecture enough to imitate it. Thus, there’s no reason to think that
the Tikal citadel provides evidence of the Tikal people’s “animosities toward,” or feelings of strong dislike or
hostility toward, Teotihuacán architecture.

Question ID 4480fae9 Question ID 5d2fd27d


Assessment Test Domain Skill Difficulty Assessment Test Domain Skill Difficulty

SAT Reading and Writing Craft and Structure Words in Context SAT Reading and Writing Craft and Structure Words in Context

ID: 4480fae9 ID: 5d2fd27d


The following text is adapted from Sui Sin Far’s 1912 short story “Mrs. Spring Fragrance.” Mr. and Mrs. Spring While we can infer information about climate activity in Earth’s distant past from physical evidence, we of course
Fragrance immigrated to the United States from China. cannot observe past climates directly. To study early Earth’s climate in action, we must ______ that climate using
computer models that represent various climate conditions consistent with the physical evidence.
Mrs. Spring Fragrance was unaware that Mr. Spring Fragrance, tired with the day’s business, had thrown himself
down on the bamboo settee on the veranda, and that although his eyes were engaged in scanning the pages of Which choice completes the text with the most logical and precise word or phrase?
the Chinese World, his ears could not help receiving the words which were borne to him through the open
A. invent
window.
B. simulate
As used in the text, what does the word “receiving” most nearly mean?

A. Denying C. exaggerate

B. Entering D. preserve

C. Carrying
ID: 5d2fd27d Answer
D. Hearing
Correct Answer: B

Rationale
ID: 4480fae9 Answer
Choice B is the best answer because it most logically completes the text’s discussion of climate activity in
Correct Answer: D
Earth’s distant past. In this context, to “simulate” most nearly means to represent a natural process using
Rationale computer models. According to the text, understanding Earth’s early climate is difficult because we cannot
make direct observations of the distant past. Instead, scientists must create computer models that
Choice D is the best answer because as used in the text, “receiving” most nearly means hearing, or perceiving
approximate early climate conditions, based on the physical evidence that those conditions left behind on
sound. The text describes Mr. Spring Fragrance as he reads a newspaper, focusing on his eyes and ears. While
Earth’s surface. This context supports the idea that computer models simulate Earth’s climate in the distant
his eyes look at the newspaper, he is unwillingly distracted by words coming through a nearby open window
past.
(that is, “his ears could not help receiving the words”). The words are being perceived specifically by his ears
and not his eyes, and ears are the organs of sense with which one hears, so therefore Mr. Spring Fragrance is Choice A is incorrect because scientists use existing physical evidence as a basis for developing computer
hearing the words through the open window. models that describe what Earth’s actual past climate might have been like. The models are not being used to
“invent,” or imagine, a completely fictional climate. Choice C is incorrect because the computer models are
Choice A is incorrect because there is no indication in the text that Mr. Spring Fragrance is denying, or
being used in an attempt to describe as accurately as possible what Earth’s past climate might have been like.
refusing the truth of, the words coming to him through the window. He is merely distracted by them. Choice B
They are not attempting to “exaggerate,” or distort, those features. Choice D is incorrect because the computer
is incorrect because it wouldn’t make sense to say that Mr. Spring Fragrance’s ears couldn’t help entering, or
models do not “preserve,” or protect from deterioration, Earth’s early climate; instead, they attempt to
coming into, the words coming to him through the window. Instead, the text indicates that the opposite occurs:
reproduce the characteristics of that climate, based on the remaining physical evidence of that climate.
that is, when Mr. Spring Fragrance hears the words, they enter his ears, but his ears don’t enter the words.
Choice C is incorrect because it doesn’t make sense in this context to think of Mr. Spring Fragrance as Question Difficulty: Easy
carrying the words coming to him through the window, even though the word “carry” can have a number of
different meanings depending on context: for example, he is not lifting them, he is not supporting them, and
he is not accepting blame for them (as one “carries the blame” for a mistake).

Question Difficulty: Easy


Question ID 9aa44886 Question ID e1befb41
Assessment Test Domain Skill Difficulty Assessment Test Domain Skill Difficulty

SAT Reading and Writing Craft and Structure Words in Context SAT Reading and Writing Craft and Structure Cross-Text
Connections

ID: 9aa44886
ID: e1befb41
The following text is from F. Scott Fitzgerald’s 1925 novel The Great Gatsby.

[Jay Gatsby] was balancing himself on the dashboard of his car with that resourcefulness of movement that is so Text 1
peculiarly American—that comes, I suppose, with the absence of lifting work in youth and, even more, with the In a study of the benefits of having free time, Marissa Sharif found that the reported sense of life satisfaction tended to
formless grace of our nervous, sporadic games. This quality was continually breaking through his punctilious plateau when participants had two hours of free time per day and actually began to fall when they had five hours of
manner in the shape of restlessness. free time per day. After further research, Sharif concluded that this dip in life satisfaction mainly occurred when
individuals spent all their free time unproductively, such as by watching TV or playing games.
As used in the text, what does the word “quality” most nearly mean?

A. Standard Text 2
Psychologist James Maddux cautions against suggesting an ideal amount of free time. The human desire for both free
B. Prestige
time and productivity is universal, but Maddux asserts that individuals have unique needs for life satisfaction.
C. Characteristic Furthermore, he points out that there is no objective definition for what constitutes productivity; reading a book might
be considered a productive activity by some, but idleness by others.
D. Accomplishment
Based on the texts, how would Maddux (Text 2) most likely respond to the conclusion Sharif (Text 1) reached after her
further research?
ID: 9aa44886 Answer
A. By acknowledging that free time is more likely to enhance life satisfaction when it is spent productively than when
Correct Answer: C
it is spent unproductively
Rationale
B. By challenging the reasoning in Text 1, as it has not been proved that productivity commonly contributes to
Choice C is the best answer. “This quality” refers to Gatsby’s “resourcefulness of movement,” which is individuals’ life satisfaction
described as a characteristic or trait of his.
C. By warning against making an overly broad assumption, as there is no clear consensus in distinguishing between
Choice A is incorrect. This isn’t what “quality” means in this context. Here, “this quality” refers to Gatsby’s productive and unproductive activities
“resourcefulness of movement,” which is described as a characteristic or trait of his. “Standard” is a synonym D. By claiming that the specific activities named in Text 1 are actually examples of productive activities rather than
for a different definition of “quality”: the degree of excellence of something. Choice B is incorrect. This isn’t unproductive ones
what “quality” means in this context. Here, “this quality” refers to Gatsby’s “resourcefulness of movement,”
which is described as a characteristic or trait of his. “Prestige” would suggest a high status or an admirable
reputation, which doesn’t match that description. Choice D is incorrect. This isn’t what “quality” means in this ID: e1befb41 Answer
context. Here, “this quality” refers to Gatsby’s “resourcefulness of movement,” which is described as a
Correct Answer: C
characteristic or trait of his. “Accomplishment” would suggest an achievement, which doesn’t match that
description. Rationale

Question Difficulty: Medium Choice C is the best answer because it characterizes how Maddux would most likely respond to the conclusion
Sharif reached after her research. Text 1 describes Sharif’s study of the benefits of free time, saying that the
reported sense of satisfaction plateaued at two hours per day and began to decline at five hours per day.
Further research led Sharif to conclude that time spent doing tasks she defines as unproductive, such as
watching TV or playing games, correlated with a drop in life satisfaction. However, in Text 2 Maddux says that
there is no objective definition of what constitutes productive behavior, giving the example that reading a book
might be considered productive by some but unproductive by others. It can be inferred that Maddux would
also assert that whether watching TV or playing games is productive or unproductive is a matter of subjective
judgment. Thus, Maddux would most likely caution against making an overly broad assumption, as there is no
clear consensus in distinguishing between productive and unproductive activities.

Choice A is incorrect because Maddux asserts that individuals have unique needs for life satisfaction: some
may want to spend that time productively, others unproductively, and what counts as productive is subjective.
Therefore, Maddux would likely not consider it universally true that free time is more likely to enhance life
Question ID afd48140
satisfaction when it is spent productively. Choice B is incorrect because the study described in Text 1 concerns
whether free time contributes to life satisfaction, not whether productivity contributes to life satisfaction. The Assessment Test Domain Skill Difficulty
dip in life satisfaction that Sharif claims to observe in Text 1 happens only after five hours, and mainly if the
time is spent unproductively—that is, two hours of free time spent productively might increase life satisfaction SAT Reading and Writing Craft and Structure Words in Context
just as much as two hours spent unproductively. Choice D is incorrect because Maddux holds the opinion that
whether an activity is productive or unproductive is subjective and depends on the individual; therefore, he ID: afd48140
would most likely claim that watching TV or playing games might be productive for some and unproductive
In 1877, 85% of California’s railways were already controlled by the Southern Pacific Railroad. The company further
for others.
solidified its ______ in rail access to the state’s Pacific coast when it completed the Sunset Route in 1883: running from
Question Difficulty: Medium Louisiana to Southern California, the route established the first transcontinental rail line across the southern United
States.

Which choice completes the text with the most logical and precise word or phrase?

A. dominance

B. creativity

C. insignificance

D. neutrality

ID: afd48140 Answer


Correct Answer: A

Rationale

Choice A is the best answer because it most logically and precisely completes the text’s discussion of the
Southern Pacific Railroad. As used in this context, "dominance" means the state of being more successful than
others. The text states that the Southern Pacific Railroad already controlled 85% of California’s railways in 1877,
and that the Sunset Route it constructed in 1883 was the first transcontinental rail line across the southern
United States. Therefore, the Southern Pacific Railroad would be more successful than other railroads in the
area, and the construction of the Sunset Route would further solidify its dominance in rail access to
California’s coast.

Choice B is incorrect. Although the construction of the Sunset Route must have required some creativity, or
use of imagination, the word "creativity" would not also encompass the fact that the Southern Pacific Railroad
already controlled 85% of California’s railways. The text is more focused on the railroad’s control of the field
and overall success rather than its imaginative thought processes. Choice C is incorrect because if the
Southern Pacific Railroad controlled the majority of California’s railways, it would not make sense to refer to its
"insignificance," or the state of being unimportant. Rather, the opposite would be true. Choice D is incorrect
because in context it would not make sense to refer to the Southern Pacific Railroad’s "neutrality," or absence
of strong opinion on a matter, since no opinions are mentioned. Nor would "neutrality" in the sense of
impartiality in a conflict be a logical or precise choice since no conflict is explicitly described. There might be
an implied conflict between the Southern Pacific Railroad and other railroads doing business in California, but
that is not a conflict in which the Southern Pacific Railroad would take an impartial position.

Question Difficulty: Easy


Question ID b4887dae Question ID c68ceeff
Assessment Test Domain Skill Difficulty Assessment Test Domain Skill Difficulty

SAT Reading and Writing Craft and Structure Text Structure and SAT Reading and Writing Craft and Structure Cross-Text
Purpose Connections

ID: b4887dae ID: c68ceeff


Mathematician Claude Shannon is widely regarded as a foundational figure in information theory. His most important Text 1
paper, “A Mathematical Theory of Communication,” published in 1948 when he was employed at Bell Labs, utilized a Today the starchy root cassava is found in many dishes across West Africa, but its rise to popularity was slow.
concept called a “binary digit” (shortened to “bit”) to measure the amount of information in any signal and determine Portuguese traders brought cassava from Brazil to the West African coast in the 1500s. But at this time, people living in
the fastest rate at which information could be transmitted while still being reliably decipherable. Robert Gallagher, one the capitals further inland had little contact with coastal communities. Thus, cassava remained relatively unknown to
of Shannon’s colleagues, said that the bit was “[Shannon’s] discovery, and from it the whole communications most of the region’s inhabitants until the 1800s.
revolution has sprung.”
Text 2
Which choice best describes the overall structure of the text? Cassava’s slow adoption into the diet of West Africans is mainly due to the nature of the crop itself. If not cooked
A. It presents a theoretical concept, illustrates how the name of the concept has changed, and shows how the name properly, cassava can be toxic. Knowledge of how to properly prepare cassava needed to spread before the food could
has entered common usage. grow in popularity. The arrival of formerly enslaved people from Brazil in the 1800s, who brought their knowledge of
cassava and its preparation with them, thus directly fueled the spread of this crop.
B. It introduces a respected researcher, describes an aspect of his work, and suggests why the work is historically
significant. Based on the texts, the author of Text 1 and the author of Text 2 would most likely agree with which statement?

C. It names the company where an important mathematician worked, details the mathematician’s career at the A. Cassava did not become a significant crop in West Africa until long after it was first introduced.
company, and provides an example of the recognition he received there.
B. Several of the most commonly grown crops in West Africa are originally from Brazil.
D. It mentions a paper, offers a summary of the paper’s findings, and presents a researcher’s commentary on the
C. The climate of the West African coast in the 1500s prevented cassava’s spread in the region.
paper.
D. The most commonly used methods to cook cassava today date to the 1500s.

ID: b4887dae Answer


Correct Answer: B
ID: c68ceeff Answer
Correct Answer: A
Rationale
Rationale
Choice B is the best answer. The text starts with a general statement that introduces Shannon, then describes
a specific contribution from one of his papers, then provides a quote that illustrates just how important this Choice A is the best answer. Text 1 states that cassava’s “rise to popularity was slow” in West Africa. Text 2 also
contribution was. describes cassava’s “slow adoption into the diet of West Africans.” While the two texts identify different causes
for this slow adoption, both agree that cassava took a long time to catch on.
Choice A is incorrect. This isn’t the overall structure. The text only mentions very briefly, in parentheses, that
“binary digit” was shortened to “bit.” It doesn’t go into detail about this name change, and it doesn’t discuss any Choice B is incorrect. This isn’t something that either text claims. Cassava is the only crop discussed in the
“common usage” of the name at all. Choice C is incorrect. This isn’t the overall structure. Shannon’s passages, so we have no basis to draw conclusions about what the authors might say about “several” crops.
employment at Bell Labs is only mentioned once, very briefly: the text never goes into detail about his career Choice C is incorrect. This isn’t something that either text claims. Neither text mentions the “climate of the
there, and it never mentions any recognition he received there. Choice D is incorrect. This is too narrow. West African coast,” so we have no evidence that either author would agree with this. Choice D is incorrect.
Overall, the text is about Shannon’s importance in his field, not just this one paper of his. This isn’t something that either text claims. The 1500s were when cassava was brought to West Africa, but
neither text describes how cassava is cooked, nor do they make any claims about when cooking methods were
Question Difficulty: Hard developed.

Question Difficulty: Easy

Question ID daaed806 Question ID 3e6ad72d


Assessment Test Domain Skill Difficulty Assessment Test Domain Skill Difficulty

SAT Reading and Writing Craft and Structure Words in Context SAT Reading and Writing Craft and Structure Text Structure and
Purpose

ID: daaed806
ID: 3e6ad72d
Mônica Lopes-Ferreira and others at Brazil’s Butantan Institute are studying the freshwater stingray species
Potamotrygon rex to determine whether biological characteristics such as the rays’ age and sex have ______ effect on A study by a team including finance professor Madhu Veeraraghavan suggests that exposure to sunshine during the
the toxicity of their venom—that is, to see if differences in these traits are associated with considerable variations in workday can lead to overly optimistic behavior. Using data spanning from 1994 to 2010 for a set of US companies, the
venom potency. team compared over 29,000 annual earnings forecasts to the actual earnings later reported by those companies. The
team found that the greater the exposure to sunshine at work in the two weeks before a manager submitted an
Which choice completes the text with the most logical and precise word or phrase? earnings forecast, the more the manager’s forecast exceeded what the company actually earned that year.

A. a disconcerting
Which choice best states the function of the underlined sentence in the overall structure of the text?
B. an acceptable A. To summarize the results of the team’s analysis

C. an imperceptible B. To present a specific example that illustrates the study’s findings

D. a substantial C. To explain part of the methodology used in the team’s study

D. To call out a challenge the team faced in conducting its analysis


ID: daaed806 Answer
Correct Answer: D
ID: 3e6ad72d Answer
Rationale
Correct Answer: C

Choice D is the best answer because it most logically completes the text’s discussion of the research that Rationale
Lopes-Ferreira and her colleagues are conducting on the stingray species Potamotrygon rex. As used in this
context, “a substantial” effect means an effect that is sizable or noteworthy. The text indicates that the Choice C is the best answer because it best describes how the underlined sentence functions in the text as a
researchers are seeking to determine whether there are “considerable variations” in the potency of stingray whole. The first sentence presents the implications of Veeraraghavan’s team’s study: sunshine exposure
venom that are associated with variation in the stingrays’ age and sex. This context suggests that the during work hours can cause overly optimistic behavior. The underlined sentence then describes the data the
researchers want to find out whether stingray age and sex have a substantial effect on venom toxicity. team consulted and how they were used (comparing predictions about earnings to what the companies
actually earned), and the final sentence presents what the team found in their examination of the data. Thus,
Choice A is incorrect because there’s nothing in the text that suggests that the researchers have been studying the underlined sentence mainly functions to explain part of the methodology used in the team’s study.
whether the stingrays’ age and sex have “a disconcerting,” or an unsettling and disturbing, effect on the
stingrays’ venom. The text indicates that the researchers wish to determine if stingray age and sex cause large Choice A is incorrect because the underlined sentence explains in part how the team conducted their analysis
variations in the toxicity of stingray venom, not if the effect of age and sex is disconcerting. Choice B is of the effect of sunshine but doesn’t address what the team found; a broad summary is instead given in the
incorrect because the text indicates that researchers want to find out whether differences in stingray age and other two sentences. Choice B is incorrect because the underlined sentence doesn’t present any specific
sex produce differences in stingray venom, not that the researchers want to find out whether age and sex have examples from the team’s comparisons of 29,000 earnings predictions to actual earnings; it simply explains in
“an acceptable,” or a satisfactory, effect on venom. The text makes no mention of what would make an effect on part how the team conducted their analysis. Choice D is incorrect because the underlined sentence simply
venom toxicity acceptable and gives no indication that the researchers are interested in that question. Choice explains in part how the team conducted their analysis; the text never mentions any challenges that the team
C is incorrect because it wouldn’t make sense in context for the researchers to be looking for “an encountered in their study.
imperceptible,” or an unnoticeable, effect of age and sex on stingray venom. The text says that the researchers
Question Difficulty: Hard
are trying to determine if there are “considerable variations” in venom toxicity linked to age and sex, not that
the researchers are trying to find effects that they can’t perceive.

Question Difficulty: Easy


Question ID f3c45b4f Question ID bdab32fc
Assessment Test Domain Skill Difficulty Assessment Test Domain Skill Difficulty

SAT Reading and Writing Craft and Structure Cross-Text SAT Reading and Writing Craft and Structure Words in Context
Connections

ID: bdab32fc
ID: f3c45b4f
Cucurbits, a group of plants that includes squash and melons, relied on mastodons to spread their seeds in the Ice Age.
Text 1 When these animals died out, cucurbits faced extinction in turn, having lost their means of seed dispersal. Around this
Fossils of the hominin Australopithecus africanus have been found in the Sterkfontein Caves of South Africa, but time, however, the ancestors of Indigenous peoples in North America began raising cucurbits as crops, thus ______ the
assigning an age to the fossils is challenging because of the unreliability of dating methods in this context. The plants’ survival.
geology of Sterkfontein has caused soil layers from different periods to mix, impeding stratigraphic dating, and dates
cannot be reliably imputed from those of nearby animal bones since the bones may have been relocated by flooding. Which choice completes the text with the most logical and precise word or phrase?
Text 2
A. verifying
Archaeologists used new cosmogenic nuclide dating techniques to reevaluate the ages of A. africanus fossils found in
the Sterkfontein Caves. This technique involves analyzing the cosmogenic nucleotides in the breccia—the matrix of B. multiplying
rock fragments immediately surrounding the fossils. The researchers assert that this approach avoids the potential for
C. comforting
misdating associated with assigning ages based on Sterkfontein’s soil layers or animal bones.
D. ensuring
Based on the texts, how would the researchers in Text 2 most likely respond to the underlined portion in Text 1?

A. They would emphasize the fact that the A. africanus fossils found in the Sterkfontein Caves may have been
corrupted in some way over the years. ID: bdab32fc Answer
Correct Answer: D
B. They would contend that if analyses of surrounding layers and bones in the Sterkfontein Caves were combined,
then the dating of the fossils there would be more accurate. Rationale

C. They would argue that their techniques are better suited than other methods to the unique challenges posed by Choice D is the best answer because it most logically completes the text’s discussion of cucurbits. In this
the Sterkfontein Caves. context, “ensuring” means guaranteeing, or making sure of, the cucurbits’ survival. The text states that
cucurbits faced extinction in the past because their means of seed dispersal disappeared, but the ancestors of
D. They would claim that cosmogenic nuclide dating is reliable in the context of the Sterkfontein Caves because it is
Indigenous peoples in North America began farming cucurbits around that same time, so the crops were no
applied to the fossils directly.
longer threatened. Therefore, the context supports the idea that the ancestors of Indigenous peoples in North
America helped with ensuring the cucurbits’ survival.
ID: f3c45b4f Answer
Choice A is incorrect because in this context verifying means making sure that something is accurate. In the
Correct Answer: C text, the ancestors of Indigenous peoples in North America were ensuring the survival, not the accuracy of, the
Rationale cucurbits. Choice B is incorrect. Although the cucurbit crops themselves were multiplying, or growing in
number, as a result of the work of the ancestors of Indigenous peoples in North America, it wouldn’t make
Choice C is the best answer. Text 2 states that the researchers used cosmogenic nuclide dating to "avoid the sense in context to say that the survival of the plants was multiplying. Choice C is incorrect because according
potential for misdating" caused by the geology of Sterkfontein, which Text 1 describes as "challenging" and to the text, in raising cucurbits as crops, the ancestors of Indigenous peoples in North America were
unreliable. attempting to help the plants grow and survive; they weren’t attempting to comfort, or free the plants from
pain.
Choice A is incorrect. Neither text suggests that the A. africanus fossils have been "corrupted," but only that
traditional dating methods are difficult in Sterkfontein because of floods and soil mixing where the fossils Question Difficulty: Easy
were found. Nothing is implied to have compromised the fossils themselves. Choice B is incorrect. This choice
misreads Text 2. Text 2 agrees that stratigraphy and other methods are prone to error in the context of
Sterkfontein: there’s a "potential for misdating" when evaluating age based on soil layers and bones. Choice D
is incorrect. Text 2 does not state that cosmogenic nuclide dating is applied to the fossils directly but rather to
the breccia that surrounds them.

Question Difficulty: Hard

Question ID e8c26398 Question ID 5336f2e4


Assessment Test Domain Skill Difficulty Assessment Test Domain Skill Difficulty

SAT Reading and Writing Craft and Structure Words in Context SAT Reading and Writing Craft and Structure Text Structure and
Purpose

ID: e8c26398
ID: 5336f2e4
To develop a method for measuring snow depth with laser beams, NASA physicist Yongxiang Hu relied on ______;
identifying broad similarities between two seemingly different phenomena, Hu used information about how ants move The following text is adapted from Zora Neale Hurston’s 1921 short story “John Redding Goes to Sea.” John is a child
inside colonies to calculate how the particles of light that make up laser beams travel through snow. who lives in a town in the woods.
Perhaps ten-year-old John was puzzling to the folk there in the Florida woods for he was an imaginative child and
Which choice completes the text with the most logical and precise word or phrase? fond of day-dreams. The St. John River flowed a scarce three hundred feet from his back door. On its banks at this
point grow numerous palms, luxuriant magnolias and bay trees. On the bosom of the stream float millions of
A. a collaboration
delicately colored hyacinths. [John Redding] loved to wander down to the water’s edge, and, casting in dry twigs,
B. an accessory watch them sail away down stream to Jacksonville, the sea, the wide world and [he] wanted to follow them.

C. a contradiction Which choice best describes the function of the underlined sentence in the text as a whole?

D. an analogy A. It provides an extended description of a location that John likes to visit.

B. It reveals that some residents of John’s town are confused by his behavior.
ID: e8c26398 Answer
C. It illustrates the uniqueness of John’s imagination compared to the imaginations of other children.
Correct Answer: D
D. It suggests that John longs to experience a larger life outside the Florida woods.
Rationale

Choice D is the best answer. The text after the semicolon tells us that Hu "identif[ied] broad similarities ID: 5336f2e4 Answer
between two seemingly different phenomena," comparing ants with light particles. Since an analogy seeks
Correct Answer: D
similarities between seemingly unrelated phenomena, this fits the context perfectly.
Rationale
Choice A is incorrect. "A collaboration" refers to "an act of working with others," but what comes after the
semicolon doesn’t describe collaboration with other researchers. Instead, it shows a comparison between two Choice D is the best answer because it accurately describes how the underlined sentence functions in the text
different (but ultimately similar) scientific phenomena. Choice B is incorrect. "An accessory" can refer to as a whole. The text establishes that John has a strong imagination and then goes on to describe the St. John
"something added to increase attractiveness or usefulness." No accessories are described in this text. Choice C River near John’s home in the Florida woods. The underlined sentence depicts John sending twigs sailing
is incorrect. "A contradiction" means "a set of ideas or things that are opposed to or inconsistent with each down the river while he imagines them reaching “Jacksonville, the sea, the wide world,” where he wishes he
other." The text describes how Hu used the similarity between ant and light particle movement to develop his could follow. This suggests that John longs to expand his life experiences beyond the Florida woods.
method, so a word that refers to difference would not make sense here.
Choice A is incorrect because the second and third sentences of the text provide an extended description of
Question Difficulty: Medium the riverbank where John likes to go, whereas the underlined sentence describes what John does at that
location. Choice B is incorrect because the first sentence of the text suggests that John’s behavior “was
puzzling” to others around him, whereas the underlined sentence concerns the content of John’s imaginings.
Choice C is incorrect because the underlined sentence elaborates on John’s imagination but doesn’t mention
any other children to whom John could be compared.

Question Difficulty: Hard


Question ID 6f5fc289 Question ID b92c13fa
Assessment Test Domain Skill Difficulty Assessment Test Domain Skill Difficulty

SAT Reading and Writing Craft and Structure Text Structure and SAT Reading and Writing Craft and Structure Words in Context
Purpose

ID: b92c13fa
ID: 6f5fc289
According to statistician Nassim Nicholas Taleb, the best way to predict the amount of time a nonperishable entity
The following text is adapted from Charles Dickens’s 1854 novel Hard Times. Coketown is a fictional town in England. (such as a building or a technology) will continue to exist is to examine how long it has survived so far. In this view, an
[Coketown] contained several large streets all very like one another, and many small streets still more like one another, item’s age is the strongest ______ how much longer it will last.
inhabited by people equally like one another, who all went in and out at the same hours, with the same sound upon
the same pavements, to do the same work, and to whom every day was the same as yesterday and tomorrow, and Which choice completes the text with the most logical and precise word or phrase?
every year the counterpart of the last and the next.
A. uncertainty about

Which choice best states the main purpose of the text? B. indicator of
A. To emphasize the uniformity of both the town and the people who live there
C. motivation for
B. To explain the limited work opportunities available to the town’s residents
D. criticism of
C. To reveal how the predictability of the town makes it easy for people lose track of time

D. To argue that the simplicity of life in the town makes it a pleasant place to live ID: b92c13fa Answer
Correct Answer: B

ID: 6f5fc289 Answer Rationale

Correct Answer: A
Choice B is the best answer. “Indicator” means “something that shows or suggests,” which matches Taleb’s
Rationale argument that an item’s age can suggest how much longer it will last.

Choice A is the best answer. The author describes Coketown as having streets that are all very similar and Choice A is incorrect. The passage tells us that examining an item’s age is the best way to predict how much
residents who live similarly and do the same work. This repetition of similarities emphasizes how everything longer it will last. Therefore, according to Taleb’s theory, an item’s age should add more certainty about how
in Coketown is alike. much longer it will last. Choice C is incorrect. A “motivation” is “a reason for doing.” Items don’t have feelings
and motivations, so it wouldn’t make sense to say that their age is a motivation for how much longer they will
Choice B is incorrect. While the text mentions that all the residents “do the same work,” it never explains what last. Choice D is incorrect. “Criticism” can mean “describing faults or problems” or “an analysis of an artistic
that work is or why everyone does it. Besides, the idea that they all do the same work is just one of several work.” Neither of these definitions makes sense here. An item’s age can’t criticize how much longer it will last.
similarities among the townspeople described in the text. Choice C is incorrect. While the last sentence states
that “every day was the same as yesterday and tomorrow, and every year the counterpart of the last and the Question Difficulty: Easy
next,” it never suggests that people actually “lose track of time.” This is also too narrow to be the main idea,
since time is just one of many aspects of Coketown that the text describes as always being the same. Choice D
is incorrect. The text never mentions whether life is simple in Coketown, and the town sounds as though it’s
probably a pretty dull place to live, rather than a pleasant one.

Question Difficulty: Medium

Question ID 7bc05fa2 Question ID f7c02e89


Assessment Test Domain Skill Difficulty Assessment Test Domain Skill Difficulty

SAT Reading and Writing Craft and Structure Words in Context SAT Reading and Writing Craft and Structure Cross-Text
Connections

ID: 7bc05fa2
ID: f7c02e89
Whether the reign of a French monarch such as Hugh Capet or Henry I was historically consequential or relatively
uneventful, its trajectory was shaped by questions of legitimacy and therefore cannot be understood without a
corollary understanding of the factors that allowed the monarch to ______ his right to hold the throne. Text 1
Films and television shows commonly include a long list of credits naming the people involved in a production. Credit
Which choice completes the text with the most logical and precise word or phrase? sequences may not be exciting, but they generally ensure that everyone’s contributions are duly acknowledged.
Because they are highly standardized, film and television credits are also valuable to anyone researching the careers of
A. disengage
pioneering cast and crew members who have worked in the mediums.
B. annotate
Text 2
C. buttress
Video game scholars face a major challenge in the industry’s failure to consistently credit the artists, designers, and
D. reciprocate other contributors involved in making video games. Without a reliable record of which people worked on which games,
questions about the medium’s development can be difficult to answer, and the accomplishments of all but its best-
known innovators can be difficult to trace.
ID: 7bc05fa2 Answer
Based on the texts, how would the author of Text 1 most likely respond to the discussion in Text 2?
Correct Answer: C
A. By recommending that the scholars mentioned in Text 2 consider employing the methods regularly used by film
Rationale
and television researchers
Choice C is the best answer because it most logically completes the text’s discussion of the legitimacy of the
B. By pointing out that credits have a different intended purpose in film and television than in the medium addressed
reigns of French monarchs such as Hugh Capet and Henry I. As used in this context, “buttress” means to
by the scholars mentioned in Text 2
strengthen or defend. The text indicates that regardless of whether a French monarch’s reign was significant
or uneventful, each monarch faced questions about his right to the throne. The text goes on to say that in order C. By suggesting that the scholars mentioned in Text 2 rely more heavily on credits as a source of information than
to understand the path of a French monarch’s reign, it’s important to understand what contributed to the film and television researchers do
monarch’s ability to “hold the throne.” This context suggests that French monarchs such as Hugh Capet and
D. By observing that a widespread practice in film and television largely prevents the kind of problem faced by the
Henry I had to buttress, or defend, their right to be monarch.
scholars mentioned in Text 2
Choice A is incorrect because it wouldn’t make sense in context to discuss factors that enabled a monarch to
“disengage,” or withdraw his right to the French throne. The text focuses on an examination of people who
ID: f7c02e89 Answer
reigned as French monarchs, not on people who didn’t choose to rule. Choice B is incorrect because it wouldn’t
make sense in context to discuss factors that enabled a monarch to “annotate,” or add notes to or explain, his Correct Answer: D
right to the French throne. Nothing in the text suggests that the monarchs were writing notes about their right Rationale
to the throne; instead, faced with questions about the legitimacy of their reign, the monarchs defended their
right. Choice D is incorrect. Saying that a monarch who is faced with questions about the legitimacy of his Choice D is the best answer because it reflects how the author of Text 1 would most likely respond to Text 2
reign was able to “reciprocate” his right to the French throne would mean that he either returned his right to based on the information provided. Text 2 discusses how the inconsistent use of credits to identify the
the throne or that he responded in kind to the challenge. Neither of these meanings would make sense in contributors to video games can pose an obstacle to scholars of the medium, who rely on such credits to
context because the text focuses on people who did reign as French monarchs and defended their right to do answer questions about the medium’s development. Text 1 notes that in film and television, on the other hand,
so. credits are used consistently and are valuable to researchers studying the cast and crew members in these
mediums. Since Text 1 asserts how the consistent use of credits benefits scholars of film and television, it can
Question Difficulty: Hard be inferred that this text’s author would respond to the discussion in Text 2 by observing that the kind of
problem faced by scholars of video games—the inability to know who contributed to a particular production
and how—is, in film and television studies, largely prevented by the widespread practice of credits in these
mediums.
Choice A is incorrect. Although Text 1 discusses a method used by film and television researchers—namely,
relying on credits to research the careers of cast and crew members—the author doesn’t explicitly recommend
that or any other method. Moreover, Text 1 states that films and television shows themselves, not their
Question ID 82c05b34
researchers, regularly use the method of listing credits. Choice B is incorrect. It can be inferred from Text 2
that when video games do feature credits, they have essentially the same function as credits in film and Assessment Test Domain Skill Difficulty
television—namely, to identify the individuals who worked on a particular production. Therefore, it is unlikely
that the author of Text 1 would characterize video game credits as differing in purpose from film and television SAT Reading and Writing Craft and Structure Cross-Text
Connections
credits. Choice C is incorrect because, as Text 2 explains, credits are not consistently used in video games.
Therefore, it is unlikely that the author of Text 1 would argue that scholars of the medium discussed in this
text—video games—rely more heavily on credits than scholars of film and television, two mediums where ID: 82c05b34
credits consistently appear.
Text 1
The live music festival business is growing in event size and genre variety. With so many consumer options, organizers
Question Difficulty: Hard
are finding ways to cement festival attendance as a special experience worth sharing. This phenomenon is linked to the
growing “experiential economy,” where many find it gratifying to purchase lived experiences. To ensure a profitable
event, venues need to consider the overall consumer experience, not just the band lineup.
Text 2
Music festival appearances are becoming a more important part of musicians’ careers. One factor in this shift is the
rising use of streaming services that allow access to huge numbers of songs for a monthly fee, subsequently reducing
sales of full-length albums. With this shift in consumer behavior, musicians are increasingly dependent on revenue
from live performances.

Based on the texts, both authors would most likely agree with which statement?

A. Consumers are more interested in paying subscription fees to stream music than in attending music festivals in
person.

B. Consumers’ growing interest in purchasing experiences is mostly confined to the music industry.

C. Changing consumer behaviors are leading to changes in music-related businesses.

D. The rising consumer demand for live music festivals also generates higher demand for music streaming platforms.

ID: 82c05b34 Answer


Correct Answer: C

Rationale

Choice C is the best answer. Both authors mention how consumer behaviors have shifted, and how this affects
different aspects of the music industry. Text 1 states that consumers enjoy purchasing “lived experiences,” and
that this influences how organizers design music festivals. Text 2 states that consumers are using streaming
services more, and that this reduces album sales and increases the importance of live performances for
musicians.

Choice A is incorrect. Neither text claims that consumers prefer streaming to festivals, or that these are
mutually exclusive options. Text 1 implies that festivals are popular and profitable, and Text 2 never suggests
that streaming services diminish the demand for live music. Choice B is incorrect. This choice misreads Text
1, which identifies music festivals as just one example of a broader trend of purchasing “lived experiences.”
Text 2 doesn’t mention growing interest in purchasing experiences, in the music industry or otherwise. Choice
D is incorrect. Neither text establishes a cause/effect relationship between the demand for festivals and the
demand for streaming platforms. Text 1 does not mention streaming platforms at all, and Text 2 does not imply
that streaming platforms benefit from the popularity of festivals.

Question Difficulty: Medium

Question ID 849bf8d7 Question ID f7d58b53


Assessment Test Domain Skill Difficulty Assessment Test Domain Skill Difficulty

SAT Reading and Writing Craft and Structure Words in Context SAT Reading and Writing Craft and Structure Words in Context

ID: 849bf8d7 ID: f7d58b53


In the mid-nineteenth century, some abolitionist newspapers ______ westward migration in the United States; by Like other tribal nations, the Muscogee (Creek) Nation is self-governing; its National Council generates laws regulating
printing a letter that described the easy fortunes and high salaries miners could make in California during the Gold aspects of community life such as land use and healthcare, while the principal chief and cabinet officials ______ those
Rush, Frederick Douglass’s newspaper North Star was one such publication that inspired readers to relocate. laws by devising policies and administering services in accordance with them.

Which choice completes the text with the most logical and precise word or phrase? Which choice completes the text with the most logical and precise word or phrase?

A. stimulated A. implement

B. assigned B. presume

C. opposed C. improvise

D. disregarded D. mimic

ID: 849bf8d7 Answer ID: f7d58b53 Answer


Correct Answer: A Correct Answer: A

Rationale Rationale

Choice A is the best answer. "Stimulated" means "encouraged interest or increased activity in." Because the Choice A is the best answer because it most logically completes the text’s discussion of self-government
newspapers discussed the benefits of westward migration and "inspired readers to relocate," we can infer that among the Muscogee (Creek) Nation. In this context, “implement” means to carry out or put into effect. The
they encouraged people to move west. text states that the National Council generates laws, while the principal chief and cabinet officials are
responsible for “devising policies and administering services in accordance with” those laws. This context
Choice B is incorrect. "Assigned" means "gave a job or duty." Newspapers do not have the power to assign suggests that the principal chief and cabinet officials implement the laws: they put the laws into effect by
people to move west, although they can encourage it. Choice C is incorrect. "Opposed" means "disapproved of." creating policies and administering services that accord with those laws.
We can tell that this isn’t the case, because the newspapers discussed "the easy fortunes and high salaries
miners could make in California" and "inspired readers to relocate." Choice D is incorrect. "Disregarded" means Choice B is incorrect because “presume” in this context would mean to assume based on incomplete
"ignored." If the newspapers are writing about the west and "inspir[ing] readers to relocate," they can’t be information, and the text does not suggest that the principal chief and cabinet officials either made
ignoring it at the same time. assumptions about the content of the laws or had incomplete information about them. Choice C is incorrect
because in this context “improvise” would mean to create something without preparation, and the text does
Question Difficulty: Easy not suggest that the principal chief and cabinet officials create policies and administer services without
advance preparation. Choice D is incorrect because nothing in the text suggests that the principal chief and
cabinet officials “mimic,” or imitate, the laws generated by the National Council. To mimic laws would mean to
generate new laws that are imitations of existing laws, but the text indicates that the National Council, not the
principal chief and cabinet officials, is responsible for generating laws. Instead of generating laws, the
principal chief and cabinet officials put laws into effect by “devising policies and administering services in
accordance with” the laws.

Question Difficulty: Easy


Question ID 8b46bb51 Question ID 44cc5f75
Assessment Test Domain Skill Difficulty Assessment Test Domain Skill Difficulty

SAT Reading and Writing Craft and Structure Words in Context SAT Reading and Writing Craft and Structure Words in Context

ID: 8b46bb51 ID: 44cc5f75


A journalist and well-respected art critic of nineteenth-century Britain, Lady Elizabeth Rigby Eastlake did not hesitate to Artificially delivering biomolecules to plant cells is an important component of protecting plants from pathogens, but it
publish reviews that went against popular opinion. One of her most divisive works was an essay questioning the idea of is difficult to transmit biomolecules through the layers of the plant cell wall. Markita del Carpio Landry and her
photography as an emerging medium for fine art: in the essay, Eastlake ______ that the value of photographs was colleagues have shown that it may be possible to ______ this problem by transmitting molecules through carbon
informational rather than creative. nanotubes, which can cross cell walls.

Which choice completes the text with the most logical and precise word or phrase? Which choice completes the text with the most logical and precise word or phrase?

A. exposed A. conceptualize

B. asserted B. neglect

C. discovered C. illustrate

D. doubted D. overcome

ID: 8b46bb51 Answer ID: 44cc5f75 Answer


Correct Answer: B Correct Answer: D

Rationale Rationale

Choice B is the best answer. "Asserted" means "stated confidently." Eastlake "did not hesitate to publish Choice D is the best answer because it most logically completes the text’s discussion of delivering
reviews going against popular opinion," so we can assume that she was confident in sharing her opinions. biomolecules to plant cells. In this context, “overcome” means to succeed in dealing with an obstacle. The text
suggests that although it’s difficult to move biomolecules through plant cell walls, Landry and her colleagues
Choice A is incorrect. "Exposed" means "made visible by uncovering" and, when talking about ideas, tends to have shown that carbon nanotubes may be useful, since they can cross cell walls. This context conveys that
be used in relation to uncovering the truth. Eastlake was sharing an opinion, not uncovering a truth. Choice C Landry and her colleagues think it’s possible, using carbon nanotubes, to succeed in dealing with the obstacle
is incorrect. "Discovered" means "found," but Eastlake was writing an opinion essay. She was writing her own of transmitting biomolecules to plant cells.
opinion, not "discovering" a new universal truth. Choice D is incorrect. "Doubted" means "didn’t believe in."
We’re told that Eastlake "questioned" the idea that photography could be fine art. Placing "doubted" in the Choice A is incorrect because it wouldn’t make sense in context to say that Landry and her colleagues have
blank would actually suggest that Eastlake argued that photos were valuable for creativity and not for shown that it may be possible to “conceptualize,” or form an idea of, the difficulty of transmitting biomolecules
information, which is the opposite of what we were told she believes. through the walls of plant cells. The text presents this difficulty as a known problem that Landry and her
colleagues think they may have solved, not as a mysterious occurrence that they have yet to form ideas about.
Question Difficulty: Medium Choice B is incorrect because the text suggests that Landry and her colleagues think it may be possible to
successfully deal with the problem of transmitting biomolecules through the walls of plant cells, not that
Landry and her colleagues think it may be possible to “neglect,” or simply to disregard and ignore the problem.
Choice C is incorrect because it wouldn’t make sense in context to say that Landry and her colleagues have
shown that it may be possible to “illustrate,” or demonstrate, the difficulty of transmitting biomolecules
through the walls of plant cells by using carbon nanotubes. According to the text, carbon nanotubes allow
molecules to be transmitted to plant cells—something that is otherwise difficult to do. The text therefore
presents carbon nanotubes as a way of possibly solving a problem, not as a means of demonstrating the
problem.

Question Difficulty: Easy

Question ID aaa3ee7c Question ID ea971260


Assessment Test Domain Skill Difficulty Assessment Test Domain Skill Difficulty

SAT Reading and Writing Craft and Structure Words in Context SAT Reading and Writing Craft and Structure Text Structure and
Purpose

ID: aaa3ee7c
ID: ea971260
Critics have asserted that fine art and fashion rarely ______ in a world where artists create timeless works for exhibition
and designers periodically produce new styles for the public to buy. Luiseño/Shoshone-Bannock beadwork artist and The following text is adapted from Louisa May Alcott’s 1869 novel An Old-Fashioned Girl. Polly, a teenager, is visiting
designer Jamie Okuma challenges this view: her work can be seen in the Metropolitan Museum of Art and purchased her friend Fanny.
through her online boutique.
Fanny’s friends did not interest Polly much; she was rather afraid of them [because] they seemed so much older
and wiser than herself, even those younger in years. They talked about things of which she knew nothing and
Which choice completes the text with the most logical and precise word or phrase?
when Fanny tried to explain, she didn’t find them interesting; indeed, some of them rather shocked and puzzled
A. prevail her.

B. succumb Which choice best states the main purpose of the text?
C. diverge A. To portray Polly’s reaction to Fanny’s friends

D. intersect B. To identify the topics Polly talks about with Fanny’s friends

C. To explain how Fanny met some of her friends


ID: aaa3ee7c Answer
D. To illustrate how Fanny’s friends feel about Polly
Correct Answer: D

Rationale
ID: ea971260 Answer
Choice D is the best answer because it most logically completes the text’s discussion about the relationship Correct Answer: A
between fine art and fashion. As used in this context, “intersect” means to connect or overlap. The text
Rationale
indicates that Jamie Okuma challenges the position held by critics because her work can be seen at an art
museum and can be bought by the public from her online boutique. The text also presents the critics’ view as
Choice A is the best answer because it most accurately describes the main purpose of the text, which is to
being influenced by a perception that fine artists create works that are “timeless” and meant for exhibition,
show how Polly reacted to some of Fanny’s other friends. The text describes Polly as being frightened of
whereas fashion designers periodically produce new styles that are meant for purchase. This context suggests
Fanny’s friends because they seemed “much older and wiser” to her and elaborates that they “talked about
that the critics believe that fine art and fashion tend not to overlap—in other words, that they rarely intersect.
things of which” Polly was unfamiliar, uninterested, and shocked. Thus, the main purpose of the text is to
describe Polly’s impressions of Fanny’s other friends.
Choice A is incorrect because it wouldn’t make sense in context to say that critics contend that fine art and
fashion rarely “prevail,” or prove to be triumphant or widespread. The text indicates that Okuma is an example
Choice B is incorrect because the text does not provide any of the topics Polly discussed with Fanny’s friends,
of an artist who demonstrates that it’s possible to make fine art that is also available to the public as fashion.
stating only that Polly found the topics unfamiliar, uninteresting, and shocking. Choice C is incorrect because
Choice B is incorrect because it wouldn’t make sense in context to say that fine art and fashion rarely
the text says nothing about how Fanny and her other friends first met. Choice D is incorrect because the focus
“succumb,” or surrender. The text establishes that unlike what critics believe, Okuma creates works that are in
of the text is on Polly’s feelings about Fanny’s other friends, not on the other friends’ feelings about Polly.
art museums and available for the public to purchase, suggesting that critics believe fine art and fashion rarely
overlap, not that they rarely succumb. Choice C is incorrect because saying that critics believe that fine art and Question Difficulty: Easy
fashion rarely “diverge,” or disagree or move in different directions, wouldn’t make sense in context. The text
presents Okuma’s work as both fine art and fashion, thereby undermining what the critics assert. This
suggests that the critics believe that fine art and fashion rarely intersect rather than that the two rarely
diverge.

Question Difficulty: Hard


representation in the US government has differed from the approach Indigenous politicians have taken to

Question ID 975b0602 achieve representation in national governments elsewhere in the Americas.

Question Difficulty: Hard


Assessment Test Domain Skill Difficulty

SAT Reading and Writing Craft and Structure Text Structure and
Purpose

ID: 975b0602
A number of Indigenous politicians have been elected to the United States Congress since 2000 as members of the
country’s two established political parties. In Canada and several Latin American countries, on the other hand,
Indigenous people have formed their own political parties to advance candidates who will advocate for the interests of
their communities. This movement has been particularly successful in Ecuador, where Guadalupe Llori, a member of the
Indigenous party known as Pachakutik, was elected president of the National Assembly in 2021.

Which choice best states the main purpose of the text?

A. To trace the history of an Indigenous political movement and speculate about its future development

B. To argue that Indigenous politicians in the United States should form their own political party

C. To highlight two approaches to achieving political representation for Indigenous people

D. To consider how Indigenous politicians in the United States have influenced Indigenous politicians in Canada and
Latin America

ID: 975b0602 Answer


Correct Answer: C

Rationale

Choice C is the best answer because it most accurately describes the main purpose of the text, which is to
illustrate two approaches that Indigenous politicians have taken to achieve political representation for their
communities. The text begins by explaining that one approach is exemplified by Indigenous politicians in the
United States who, in an effort to ensure that the interests of their communities are represented in
government, joined preexisting political parties and were subsequently elected to Congress. The text goes on
to highlight a second approach adopted by Indigenous leaders in Canada and several Latin American
countries: rather than joining established political parties, many Indigenous politicians in these countries
have instead formed their own parties to promote candidates for office who support causes that are important
to their communities.

Choice A is incorrect because the text’s focus is on the contrasting approaches adopted by different
Indigenous political movements in different countries; thus, it isn’t accurate to say that the text traces the
history of one political movement. Moreover, the text only discusses examples from 2000 to 2021, a relatively
short period of time; therefore, it provides very little in the way of discussion of larger historical developments,
nor does it make any predictions about how these movements might continue to develop in the future. Choice
B is incorrect because the text never urges Indigenous politicians in the US to alter their strategy of striving
for representation through the established political parties, nor does it suggest that this strategy is inferior to
that of Indigenous politicians in Canada and Latin America, who have formed their own parties. In fact, the
text notes that both strategies have resulted in the election of Indigenous politicians to national governments.
Choice D is incorrect because the text never suggests that Indigenous politicians in the US have influenced
those in Canada and Latin America; instead, it stresses how Indigenous politicians’ approach toward achieving

Question ID 2b085bc6 Question ID f8ca5766


Assessment Test Domain Skill Difficulty Assessment Test Domain Skill Difficulty

SAT Reading and Writing Craft and Structure Text Structure and SAT Reading and Writing Craft and Structure Words in Context
Purpose

ID: f8ca5766
ID: 2b085bc6
Nigerian American author Teju Cole’s ______ his two passions—photography and the written word—culminates in his
The following text is adapted from Paul Laurence Dunbar’s 1902 novel The Sport of the Gods. Joe and some of his 2017 book, Blind Spot, which evocatively combines his original photographs from his travels with his poetic prose.
family members have recently moved to New York City.
Which choice completes the text with the most logical and precise word or phrase?
[Joe] was wild with enthusiasm and with a desire to be a part of all that the metropolis meant. In the evening he
saw the young fellows passing by dressed in their spruce clothes, and he wondered with a sort of envy where they A. indifference to
could be going. Back home there had been no place much worth going to, except church and one or two
B. enthusiasm for
people’s houses.
C. concern about
Which choice best states the main purpose of the text?
D. surprise at
A. It illustrates a character’s reaction to a new environment.

B. It explains why a character has traveled to a city.


ID: f8ca5766 Answer
C. It compares a character’s thoughts about an event at two different times of day. Correct Answer: B

D. It presents a character feeling regret over leaving home. Rationale

Choice B is the best answer because it most logically completes the text’s discussion of Cole’s book Blind Spot.
ID: 2b085bc6 Answer In this context, “enthusiasm for” means excitement about. The text explains that Blind Spot consists of
Correct Answer: A original photographs as well as poetic prose—two elements that correspond to Cole’s passions, identified in the
text, for photography and the written word. This context suggests that Cole’s excitement about photography
Rationale
and writing led him to create a book that successfully combines the two mediums.

Choice A is the best answer because it most accurately describes the main purpose of the text. The narrator
Choice A is incorrect because describing Cole as feeling “indifference to” his two passions wouldn’t make
describes how Joe responds to being in “the metropolis”: he’s excited and “wild with enthusiasm.” He also
sense in context. If Cole is indifferent to his passions, that would mean he doesn’t care about photography or
envies the young fellows who walk by because, dressed as they are, they look as if they have somewhere
writing—in which case they wouldn’t be his passions at all. Choice C is incorrect because there’s nothing in
special to go. The text contrasts this new place with the place Joe comes from, where apparently there wasn’t
the text to suggest that Cole feels “concern about,” or uneasiness about, his passions. The text’s use of the word
as much to do. Thus, the main purpose of the text is to illustrate Joe’s reaction to a new environment.
“culminates” indicates that Blind Spot represents a triumphant climax of Cole’s passions, not a work that
results from his sense of discomfort with photography and writing. Choice D is incorrect because there’s
Choice B is incorrect because the text makes no reference to why Joe has moved. The narrator indicates that
nothing in the text to suggest that Cole feels “surprise at,” or astonished by, his passions. The text indicates
Joe is enthusiastic about being in a city, but there’s no explanation provided for the move. Choice C is incorrect
that Cole’s feeling about his passions “culminates” in a book that “evocatively” combines photographs and
because the text makes no reference to how Joe thinks about an event. The narrator describes young men
writing, suggesting that Cole has a long-standing and skillful relationship to his passions, not that he is
passing by in the evening and then recalls places worth going to at home—church and a few people’s houses—
startled by them.
but there’s no explicit comparison made nor is a time of day mentioned for these events back home. Choice D
is incorrect because the text doesn’t support the idea that Joe feels regret over leaving home. Instead, Joe is
Question Difficulty: Easy
described as “wild with enthusiasm” at being in the city. Joe’s home is mentioned, but only to compare it
unfavorably with the city.

Question Difficulty: Medium


Question ID 9671d61d Question ID a06c434d
Assessment Test Domain Skill Difficulty Assessment Test Domain Skill Difficulty

SAT Reading and Writing Craft and Structure Words in Context SAT Reading and Writing Craft and Structure Words in Context

ID: 9671d61d ID: a06c434d


The following text is from Claude McKay’s 1922 poem “Morning Joy.” The speaker is looking out a window and The work of Kiowa painter T.C. Cannon derives its power in part from the tension among his ______ influences: classic
observing a wold, or large area of land. European portraiture, with its realistic treatment of faces; the American pop art movement, with its vivid colors; and
flatstyle, the intertribal painting style that rejects the effect of depth typically achieved through shading and
perspective.
At night the wide and level stretch of wold,
Which at high noon had basked in quiet gold, Which choice completes the text with the most logical and precise word or phrase?
Far as the eye could see was ghostly white;
A. complementary
Dark was the night save for the snow’s weird light.
B. unknown
I drew the shades far down, crept into bed;
Hearing the cold wind moaning overhead C. disparate
Through the sad pines, my soul, catching its pain,
D. interchangeable
Went sorrowing with it across the plain.

As used in the text, what does the word “drew” most nearly mean?
ID: a06c434d Answer
A. Pulled Correct Answer: C
B. Drained Rationale

C. Inspired Choice C is the best answer because it most logically completes the text’s discussion of the artistic styles that
have influenced Cannon’s work. As used in this context, “disparate” means distinct or dissimilar. The text
D. Sketched
indicates that a tension exists among the styles that have influenced Cannon’s work and goes on to describe
how those styles differ: classic European portraiture favors realism, American pop art uses vivid colors, and
ID: 9671d61d Answer intertribal flatstyle rejects the use of shading and perspective to achieve depth. This context suggests that the
styles that have influenced Cannon’s work are disparate.
Correct Answer: A

Rationale Choice A is incorrect because the text indicates that there is a tension among the influences on Cannon’s
artwork, so it wouldn’t make sense to say that the influences are “complementary,” or that they complete one
Choice A is the best answer because as used in the text, “drew” most nearly means pulled. In the text, the another or make up for one another’s deficiencies. Choice B is incorrect because it wouldn’t make sense to
speaker stands at a window, looking out on a landscape at night. The speaker then “drew the shades far down, characterize Cannon’s influences as “unknown,” or not familiar; it’s clear that the influences are known
crept into bed.” That is, the speaker pulled down, or lowered, the shades until they were completely shut before because the text goes on to list them. Choice D is incorrect because the text indicates that there is a tension
going to sleep. among the influences on Cannon’s work, not that they are “interchangeable,” or capable of being used in one
another’s place.
Choice B is incorrect. Although in some contexts, “drew” can refer to draining or removing liquid, especially
water, as when someone draws water from a well, in this context it refers to the speaker pulling down window Question Difficulty: Hard
shades for the night. Choice C is incorrect. In some contexts, “drew” can be used to describe how a person or
thing inspires, or elicits, a response from someone, as when a performer draws applause from an audience. But
in this context it refers to the speaker pulling down window shades for the night. Choice D is incorrect.
Although “drew” has several meanings, including sketched, or illustrated with a pen or pencil, in this context it
refers to the speaker pulling down window shades for the night.

Question Difficulty: Easy

Question ID c8603ed7 Question ID 9c759a09


Assessment Test Domain Skill Difficulty Assessment Test Domain Skill Difficulty

SAT Reading and Writing Craft and Structure Text Structure and SAT Reading and Writing Craft and Structure Text Structure and
Purpose Purpose

ID: c8603ed7 ID: 9c759a09


San Francisco is known for the colorful murals painted on many of its buildings. The densest collection of murals is The following text is from Georgia Douglas Johnson’s 1922 poem “Benediction.”
found on Balmy Alley in the Mission District neighborhood. In the 1970s, Latina artists painted vivid scenes of Go forth, my son,
community life on walls along this block. As the original murals have faded, later generations of artists have painted Winged by my heart’s desire!
new ones over them. As a result, Balmy Alley has become a living showcase of San Francisco’s artistic spirit, with its Great reaches, yet unknown,
murals reflecting changes in the cultural life of the city. Await
For your possession.
Which choice best states the main purpose of the text? I may not, if I would,
A. To compare the Balmy Alley murals to other murals in San Francisco Retrace the way with you,
My pilgrimage is through,
B. To offer an overview of the history and importance of the Balmy Alley murals But life is calling you!

C. To urge people to protect the murals of San Francisco from decay Which choice best states the main purpose of the text?
D. To describe the rise of mural painting in San Francisco beginning in the 1970s A. To express hope that a child will have the same accomplishments as his parent did

B. To suggest that raising a child involves many struggles


ID: c8603ed7 Answer
C. To warn a child that he will face many challenges throughout his life
Correct Answer: B
D. To encourage a child to embrace the experiences life will offer
Rationale

Choice B is the best answer because it best states the main purpose of the text. The text begins by briefly
ID: 9c759a09 Answer
stating that San Francisco is known for its murals, then transitions to a discussion of a single collection of
murals, which is found on Balmy Alley in the city’s Mission District. The text explains that the murals in this Correct Answer: D
area were originally created in the 1970s, then observes that they have changed over time: as some have faded Rationale
through the decades, others have been painted in their place. The text ends by emphasizing that these murals
are significant because they reflect San Francisco’s artistic spirit and cultural life. Therefore, the text provides Choice D is the best answer because it accurately states the text’s main purpose. The poem begins with the
an overview of the history and importance of the Balmy Alley murals. speaker urging a child to “go forth” with her encouragement (“my heart’s desire”). The speaker goes on to
suggest that new experiences (“Great reaches, yet unknown”) lie ahead for the son that “life is calling” him to
Choice A is incorrect. Although it can be inferred from the text that there are murals in other areas of San seek out. Thus, the main purpose is to encourage a child to embrace the experiences available to him in his
Francisco besides Balmy Alley, the text doesn’t specifically discuss murals in other areas or compare the life.
murals of Balmy Alley to those in other areas. Choice C is incorrect. By observing that some of the murals of
Balmy Alley have been replaced due to fading, the text implies that murals can decay, but it never urges Choice A is incorrect because the speaker encourages the child to pursue new experiences (“Great reaches”)
readers to protect this specific collection of murals—or any murals elsewhere in San Francisco, for that matter. without knowing exactly what those experiences will be (“yet unknown”) or suggesting that they should
Indeed, by describing the ever-changing murals of Balmy Alley as “a living showcase of San Francisco’s match the speaker’s own accomplishments. Choice B is incorrect because the speaker focuses on positive
artistic spirit,” the text emphasizes the positive aspects of the fact that the original Balmy Alley murals have possibilities for her son (“Great reaches, yet unknown”) and her enthusiastic encouragement to embrace those
faded and been replaced with new murals. Choice D is incorrect because the text doesn’t describe the rise of possibilities (“life is calling you!”), while there is no mention of raising a child or associated struggles. Choice C
mural painting in San Francisco generally or note when this occurred. The only historical development of the is incorrect because the speaker frames the possibilities for her son in a positive light when she says that
1970s mentioned in the text is the origin of the murals in a specific area of the city: Balmy Alley in the Mission “great reaches, yet unknown” are waiting for him, and this positive outlook for the son is consistent
District. throughout the text.

Question Difficulty: Easy Question Difficulty: Easy


Question ID 5a4b147c Question ID b411eb09
Assessment Test Domain Skill Difficulty Assessment Test Domain Skill Difficulty

SAT Reading and Writing Craft and Structure Cross-Text SAT Reading and Writing Craft and Structure Words in Context
Connections

ID: b411eb09
ID: 5a4b147c
New and interesting research conducted by Suleiman A. Al-Sweedan and Moath Alhaj is inspired by their observation
Text 1 that though there have been many studies of the effect of high altitude on blood chemistry, there is a ______ studies of
On April 26th, 1777, Sybil Ludington rode 40 miles by horse through Putnam County, New York, to gather up local the effect on blood chemistry of living in locations below sea level, such as the California towns of Salton City and
militia. British forces were burning nearby Danbury, Connecticut, and Ludington wanted to rally rebel troops to meet Seeley.
them. Although she was only 16 years old at the time, her brave feat made Ludington one of the heroes of the
American Revolution. Since then, Ludington has been widely celebrated, inspiring postage stamps, statues, and even Which choice completes the text with the most logical and precise word or phrase?
children’s TV series.
A. quarrel about

Text 2 B. paucity of
Historian Paula D. Hunt researched the life and legacy of Sybil Ludington but found no evidence for her famous ride.
C. profusion of
Although many articles and books have been written about Ludington, Hunt believes writers may have been inventing
details about Ludington as they retold her story. Ludington is revered by Americans today, but there simply isn’t a D. verisimilitude in
strong historical record of her heroic ride.

Based on the texts, both authors would most likely agree with which statement? ID: b411eb09 Answer
A. Sybil Ludington was crucial to the outcome of the Revolutionary War. Correct Answer: B

B. Historians have confirmed which route Sybil Ludington took. Rationale

C. Sybil Ludington was likely not a real person. Choice B is the best answer because it most logically and precisely completes the text’s discussion of studies
of altitude’s effect on blood chemistry. In this context, “paucity of” means lack of. In describing the inspiration
D. Many people have come to admire the story of Sybil Ludington’s ride.
behind Al-Sweedan and Alhaj’s research, the text uses the word “though” to suggest a contrasting relationship
between two types of studies: those examining the effect on blood chemistry of living at a high altitude and
those examining the effect on blood chemistry of living in locations below sea level. This contrasting
ID: 5a4b147c Answer
relationship and the text’s use of the word “many” provide context suggesting that there are few, if any,
Correct Answer: D examples of the second type of study, whereas there are numerous examples of the first type.
Rationale
Choice A is incorrect because it wouldn’t make sense in context for there to be a “quarrel about,” or open
Choice D is the best answer. Both authors agree that Ludington’s story has been widely celebrated and revered disagreement about, studies of the effect on blood chemistry of living in locations below sea level. The text’s
by Americans, even if they disagree on its accuracy. Text 1 states that Ludington has inspired postage stamps, use of the words “though” and “many” suggests a contrasting relationship in terms of amount between two
statues, and TV series, and Text 2 states that many articles and books have been written about her. Thus, both types of studies: those examining the effect on blood chemistry of living at a high altitude and those
authors acknowledge the popularity of Ludington’s story. examining the effect on blood chemistry of living in locations below sea level. There’s nothing in the text to
suggest that the contrast between the two types of studies involves the extent to which researchers broadly
Choice A is incorrect. Neither author claims that Ludington had a significant impact on the war. Text 1 doesn’t agree or disagree about the contents of either type. Choice C is incorrect because it wouldn’t make sense in
mention how Ludington’s ride affected the war overall, and Text 2 suggests that Ludington’s ride may have context for there to be a “profusion of,” or great abundance of, studies of the effect on blood chemistry of living
been exaggerated or invented over time. Choice B is incorrect. Neither author claims that Ludington’s route in locations below sea level. The text’s use of the words “though” and “many” suggests a contrasting
has been verified by historians. Text 1 claims she rode 40 miles through Putnam County, but doesn’t cite any relationship in terms of amount between two types of studies: those examining the effect on blood chemistry
sources for this information, while Text 2 suggests the ride may not have even happened. Choice C is of living at a high altitude and those examining the effect on blood chemistry of living in locations below sea
incorrect. This choice misreads text 2. Neither author claims that Ludington was not a real person, only that level. Rather than logically completing this contrast, “profusion of” would indicate that the two types of
her ride may not have happened. Both authors seem to treat Ludington as a genuine historical figure. studies are similar in terms of amount, with many examples existing of both types. Choice D is incorrect
because it wouldn’t make sense in context for there to be a “verisimilitude in,” or appearance of truth in,
Question Difficulty: Easy
studies of the effect on blood chemistry of living in locations below sea level. The text’s use of the words
“though” and “many” suggests a contrasting relationship in terms of amount between two types of studies:

those examining the effect on blood chemistry of living at a high altitude and those examining the effect on
blood chemistry of living in locations below sea level. There’s nothing in the text to suggest that the contrast
between the two types of studies involves the extent to which either type of study presents an appearance of
Question ID 8963273a
truth.
Assessment Test Domain Skill Difficulty
Question Difficulty: Hard
SAT Reading and Writing Craft and Structure Text Structure and
Purpose

ID: 8963273a
Musician Joni Mitchell, who is also a painter, uses images she creates for her album covers to emphasize ideas
expressed in her music. For the cover of her album Turbulent Indigo (1994), Mitchell painted a striking self-portrait that
closely resembles Vincent van Gogh’s Self-Portrait with Bandaged Ear (1889). The image calls attention to the album’s
title song, in which Mitchell sings about the legacy of the postimpressionist painter. In that song, Mitchell also hints
that she feels a strong artistic connection to Van Gogh—an idea that is reinforced by her imagery on the cover.

Which choice best describes the overall structure of the text?

A. It presents a claim about Mitchell, then gives an example supporting that claim.

B. It discusses Van Gogh’s influence on Mitchell, then considers Mitchell’s influence on other artists.

C. It describes a similarity between two artists, then notes a difference between them.

D. It describes the songs on Turbulent Indigo, then explains how they relate to the album’s cover.

ID: 8963273a Answer


Correct Answer: A

Rationale

Choice A is the best answer because it accurately describes the organization of the elements within the text.
The text begins with the claim that Joni Mitchell’s album covers use images she creates in order to emphasize
ideas embedded in her albums. It then goes on to provide an example of how Mitchell’s self-portrait on the
cover of Turbulent Indigo resembles a painting by Van Gogh, which the text indicates helps emphasize the
strong connection Mitchell feels toward Van Gogh, a connection that is also expressed in the album’s title
song.

Choice B is incorrect because there are no references in the text to artists other than Joni Mitchell and Van
Gogh. Choice C is incorrect because there is nothing in the text that calls attention to any similarities or
differences between Joni Mitchell and Van Gogh. Instead, it mentions that Mitchell feels a strong “artistic
connection” to Van Gogh. Choice D is incorrect because the text discusses the cover before referring to any
songs, and it only references one song from the album not all the songs.

Question Difficulty: Medium


Question ID c4900368 Question ID 1ad04ea0
Assessment Test Domain Skill Difficulty Assessment Test Domain Skill Difficulty

SAT Reading and Writing Craft and Structure Text Structure and SAT Reading and Writing Craft and Structure Words in Context
Purpose

ID: 1ad04ea0
ID: c4900368
In habitats with limited nutrients, certain fungus species grow on the roots of trees, engaging in mutually beneficial
The following text is from the 1924 poem “Cycle” by D’Arcy McNickle, who was a citizen of the Confederated Salish and relationships known as ectomycorrhizae: in this symbiotic exchange, the tree provides the fungus with carbon, a
Kootenai Tribes. nutrient necessary for both species, and the fungus ______ by enhancing the tree’s ability to absorb nitrogen, another
There shall be new roads wending, key nutrient, from the soil.
A new beating of the drum—
Men’s eyes shall have fresh seeing, Which choice completes the text with the most logical and precise word or phrase?
Grey lives reprise their span—
A. overreacts
But under the new sun’s being,
Completing what night began, B. reciprocates
There’ll be the same backs bending,
C. retaliates
The same sad feet shall drum—
When this night finds its ending D. deviates
And day shall have come.....

Which choice best states the main purpose of the text? ID: 1ad04ea0 Answer
A. To consider how the repetitiveness inherent in human life can be both rewarding and challenging Correct Answer: B

B. To question whether activities completed at one time of day are more memorable than those completed at another Rationale
time of day
Choice B is the best answer because it most logically completes the text’s discussion of ectomycorrhizae
C. To refute the idea that joy is a more commonly experienced emotion than sadness is relationships. In this context, “reciprocates” means responds in kind or degree. The text indicates that the
relationship between certain fungi and trees in some habitats is “mutually beneficial” and involves a
D. To demonstrate how the experiences of individuals relate to the experiences of their communities
“symbiotic exchange” in which each organism helps the other access an important nutrient. In other words,
each organism provides the same kind of benefit it receives: the tree provides a nutrient (carbon) for the
fungus and the fungus reciprocates by helping the tree to absorb more of another nutrient (nitrogen).
ID: c4900368 Answer
Correct Answer: A Choice A is incorrect because the text emphasizes that the relationship between certain fungi and trees in
Rationale some habitats involves a “symbiotic exchange” in which each organism helps the other access an important
nutrient. Nothing in the text suggests that the fungus “overreacts,” or responds too strongly, by allowing the
Choice A is the best answer because it accurately states the main purpose of the text. The text begins by tree to be better able to absorb a beneficial nutrient. Choice C is incorrect because “retaliates” means responds
discussing the promise of the future, with positive references to renewal such as “new roads,” “new beating of to a harmful action with a similarly harmful action. The text indicates that the relationship between certain
the drum,” and “fresh seeing.” But with the “new sun,” the text continues, there will still be “the same backs fungi and trees in some habitats is “mutually beneficial” and involves a “symbiotic exchange” in which each
bending” and “the same sad feet” drumming, indicating that these difficulties will follow people into this new organism helps the other, not that the relationship is one in which the organisms harm one another. Choice D
day. The poem thus considers both the rewards and challenges associated with the repetitiveness of human is incorrect. In this context, “deviates” would mean departs from an established course or norm. The text
life. explains that the relationship between certain fungi and trees in some habitats involves a “symbiotic
exchange” in which each organism helps the other access an important nutrient. Because the relationship
Choice B is incorrect because the text doesn’t say anything about how memorable activities are, let alone involves benefits for both the fungus and the tree, it wouldn’t make sense to say that the fungus deviates by
compare the memorability of activities completed at different times of the day. Choice C is incorrect. Although helping the tree be better able to absorb a beneficial nutrient.
the text contrasts hope with difficulty, it does not compare the relative frequency of joyful feelings with that of
sad feelings. Choice D is incorrect because the text makes no distinction between the experiences of Question Difficulty: Easy
individuals and the experiences of their communities.

Question Difficulty: Medium

Question ID a68239ed Question ID 479c7e82


Assessment Test Domain Skill Difficulty Assessment Test Domain Skill Difficulty

SAT Reading and Writing Craft and Structure Text Structure and SAT Reading and Writing Craft and Structure Words in Context
Purpose

ID: 479c7e82
ID: a68239ed
Although critics believed that customers would never agree to pay to pick their own produce on farms, such concerns
The following text is adapted from Oscar Wilde’s 1897 nonfiction work De Profundis. didn’t ______ Booker T. Whatley’s efforts to promote the practice. Thanks in part to Whatley’s determined advocacy,
farms that allow visitors to pick their own apples, pumpkins, and other produce can be found throughout the United
People whose desire is solely for self-realisation never know where they are going. They can’t know. In one sense
States.
of the word it is of course necessary to know oneself: that is the first achievement of knowledge. But to recognise
that the soul of a man is unknowable, is the ultimate achievement of wisdom. The final mystery is oneself. When
Which choice completes the text with the most logical and precise word or phrase?
one has weighed the sun in the balance, and measured the steps of the moon, and mapped out the seven
heavens star by star, there still remains oneself. Who can calculate the orbit of his own soul? A. enhance

Which choice best describes the function of the underlined question in the text as a whole? B. hinder

A. It reinforces the text’s skepticism about the possibility of truly achieving self-knowledge. C. misrepresent

B. It speculates that some readers will share the doubts expressed in the text about the value of self-knowledge. D. aggravate

C. It cautions readers that the text’s directions for how to achieve self-knowledge are hard to follow.
ID: 479c7e82 Answer
D. It concedes that the definition of self-knowledge advanced in the text is unpopular.
Correct Answer: B

Rationale
ID: a68239ed Answer
Correct Answer: A Choice B is the best answer because it most logically completes the text’s discussion of Booker T. Whatley. In
this context, “hinder” means hold back or obstruct. The text explains that Whatley encouraged farms to allow
Rationale
customers on site to pick their own produce for a fee. He did so despite critics’ concerns that the customers
would never pay to do so. This context establishes that the critics’ concerns didn’t hinder Whatley’s efforts to
Choice A is the best answer. The text repeatedly claims that true self-knowledge can’t possibly be achieved,
promote the practice.
and this rhetorical question emphasizes that point.

Choice A is incorrect. The text indicates that critics’ skepticism of the idea that customers would pay to pick
Choice B is incorrect. The underlined question doesn’t do this. The text never expresses doubts about the value
their own produce didn’t have some effect on Whatley’s promotion of the practice. The text illustrates this
of self-knowledge—rather, the text expresses doubts about the possibility of achieving self-knowledge. Choice
assertion by describing Whatley’s “determined advocacy” for the practice. This context suggests that critics’
C is incorrect. The underlined question doesn’t do this. The text doesn’t provide directions for how to achieve
concerns didn’t obstruct Whatley’s efforts, not that critics’ concerns didn’t “enhance,” or increase or improve,
self-knowledge—rather, it claims that true self-knowledge is impossible to achieve. Choice D is incorrect. The
Whatley’s efforts. Choice C is incorrect because in this context, “misrepresent” would mean portray
underlined question doesn’t do this. The text doesn’t ever define self-knowledge, and popularity isn’t
inaccurately, and the text includes no information relevant to the issue of how Whatley’s efforts were
mentioned in the text at all.
portrayed by critics of the practice of charging customers to pick their own produce. Choice D is incorrect. The
Question Difficulty: Easy text indicates that critics’ skepticism of the idea that customers would pay to pick their own produce didn’t
have some effect on Whatley’s promotion of the practice. The text illustrates this assertion by describing
Whatley’s “determined advocacy” for the practice. This context suggests that critics’ concerns didn’t obstruct
Whatley’s efforts, not that critics’ concerns didn’t “aggravate,” or irritate or make more severe, Whatley’s efforts.

Question Difficulty: Easy


Question ID 74446089 Question ID c14daa3c
Assessment Test Domain Skill Difficulty Assessment Test Domain Skill Difficulty

SAT Reading and Writing Craft and Structure Text Structure and SAT Reading and Writing Craft and Structure Words in Context
Purpose

ID: c14daa3c
ID: 74446089
Close analysis of the painting Girl with a Flute, long attributed to the seventeenth-century Dutch painter Johannes
For his 1986 album Keyboard Fantasies, Beverly Glenn-Copeland wrote songs grounded in traditional soul and folk Vermeer, has revealed subtle deviations from the artist’s signature techniques. These variations suggest that the work
music, then accompanied them with futuristic synthesizer arrangements featuring ambient sounds and complex may be that of a student under Vermeer’s tutelage—potentially ______ our understanding of Vermeer as a solitary artist.
rhythms. The result was so strange, so unprecedented, that the album attracted little attention when first released. In
recent years, however, a younger generation of musicians has embraced the stylistic experimentation of Keyboard Which choice completes the text with the most logical and precise word or phrase?
Fantasies. Alternative R&B musicians Blood Orange and Moses Sumney, among other contemporary recording artists,
A. negating
cite the album as an influence.
B. prefiguring
Which choice best describes the function of the underlined sentence in the text as a whole?
C. entrenching
A. It urges contemporary musicians to adopt the unique sound of Keyboard Fantasies.
D. substantiating
B. It responds to criticism of Keyboard Fantasies by some younger musicians.

C. It offers examples of younger musicians whose work has been impacted by Keyboard Fantasies.
ID: c14daa3c Answer
D. It contrasts Keyboard Fantasies with the recordings of two younger musicians. Correct Answer: A

Rationale
ID: 74446089 Answer
Choice A is the best answer. "Negating" means "reversing" or "making invalid." Proving that Vermeer worked
Correct Answer: C with students would reverse the view of him as a solitary artist.
Rationale
Choice B is incorrect. "Prefiguring" means "being an early indicator of." There already existed views of Vermeer
Choice C is the best answer because it most accurately describes the function of the underlined sentence in as a solitary painter, so a new painting would not be an early indicator of those views. Rather, a painting
the text as a whole. The text’s subject is Beverly Glenn-Copeland’s 1986 album Keyboard Fantasies, notable for proving that Vermeer had a student would contradict those earlier views. Choice C is incorrect. "Entrenching"
its innovative, experimental arrangements. According to the text, the album was not initially admired, but in means "solidifying." A painting proving that Vermeer had a student would not solidify views of him as solitary,
recent years it has become popular among younger musicians. The underlined portion of the text mentions but would rather contradict those views. Choice D is incorrect. "Substantiating" means "supporting with proof."
two of those musicians, Blood Orange and Moses Sumney, who “cite the album as an influence.” Therefore, the A painting proving that Vermeer had a student would not support views of him as solitary, but would rather
underlined portion of the text offers examples of younger musicians whose work has been impacted by contradict those views.
Keyboard Fantasies.
Question Difficulty: Hard
Choice A is incorrect because even though the underlined sentence states that Blood Orange and Moses
Sumney were influenced by Keyboard Fantasies, it doesn’t say that all other musicians should also embrace
the album’s experimental style. Choice B is incorrect. Although the text states that Keyboard Fantasies was
not admired on its first release, the text doesn’t present any criticism of the album by younger musicians: it
only presents two younger musicians who cite it as an influence. Choice D is incorrect because the underlined
sentence doesn’t mention any differences between Keyboard Fantasies and the work of Blood Orange and
Moses Sumney.

Question Difficulty: Medium

Question ID d3ca5d59 Question ID 0ee67e09


Assessment Test Domain Skill Difficulty Assessment Test Domain Skill Difficulty

SAT Reading and Writing Craft and Structure Words in Context SAT Reading and Writing Craft and Structure Words in Context

ID: d3ca5d59 ID: 0ee67e09


Stephen Hannock’s luminous landscape paintings are appealing to viewers but have elicited little commentary from Anthropologist Kristian J. Carlson and colleagues examined the fossilized clavicle and shoulder bones of a 3.6-million-
contemporary critics, a phenomenon that may be due to the very fact that the paintings seem so ______. Many critics year-old early hominin known as “Little Foot.” They found that these bones were ______ the clavicle and shoulder bones
focus their attention on art that is cryptic or overtly challenging. of modern apes that are frequent climbers, such as gorillas and chimpanzees, suggesting that Little Foot had adapted
to life in the trees.
Which choice completes the text with the most logical and precise word or phrase?
Which choice completes the text with the most logical and precise word or phrase?
A. innovative
A. surpassed by
B. subversive
B. comparable to
C. profound
C. independent of
D. accessible
D. obtained from

ID: d3ca5d59 Answer


Correct Answer: D ID: 0ee67e09 Answer
Correct Answer: B
Rationale
Rationale
Choice D is the best answer. When talking about a thing, “accessible” means “easy to understand.” This sets up
the contrast in the next sentence, which tells us that critics mostly focus on art that is “cryptic or challenging” Choice B is the best answer because it most logically completes the text’s discussion of the fossilized bones of
(meaning not easy to understand). the hominin known as Little Foot. As used in this context, “comparable to” would mean similar to. The text
indicates that the relationship between the fossilized clavicle and shoulder bones of Little Foot and the
Choice A is incorrect. This doesn’t fit the logic of the text. The next sentence tells us that critics focus on art clavicle and shoulder bones of “frequent climbers,” such as chimpanzees and gorillas, suggests that Little Foot
that is mysterious and challenging. If Hannock’s paintings are “innovative” (meaning advanced and original), had adapted to moving around in trees. This context suggests that the relationship between the fossilized
then critics probably would comment on them. Choice B is incorrect. This doesn’t fit the logic of the text. The bones of Little Foot and the bones of chimpanzees and gorillas is one of similarity—the Little Foot fossils are
next sentence tells us that critics focus on art that is mysterious and challenging. If Hannock’s paintings are likely comparable to the modern ape bones.
“subversive” (meaning disruptive and revolutionary), then critics probably would comment on them. Choice C
is incorrect. This doesn’t fit the logic of the text. The next sentence tells us that critics focus on art that is Choice A is incorrect because if the fossilized bones of Little Foot were “surpassed by,” or exceeded by or made
mysterious and challenging. If Hannock’s paintings are “profound” (meaning very deep and insightful), then inferior to, the bones of modern apes that are frequent climbers, it wouldn’t suggest, as the text says, that Little
critics probably would comment on them. Foot was adapted to moving around in trees. If anything, learning that Little Foot’s clavicle and shoulder bones
were surpassed by those of chimpanzees and gorillas would suggest that Little Foot was poorly adapted to
Question Difficulty: Medium climbing. Choice C is incorrect because if Little Foot’s fossilized clavicle and shoulder bones were
“independent of,” or not influenced by or affiliated with, the bones of modern apes that climb often, it wouldn’t
suggest, as the text says, that Little Foot was adapted to moving around in trees. Choice D is incorrect because
the text indicates that Little Foot’s fossilized bones date to 3.6 million years ago, so they couldn’t have been
“obtained from,” or acquired from, the bones of modern apes.

Question Difficulty: Easy


Question ID 760ee1db Question ID c5b1afe5
Assessment Test Domain Skill Difficulty Assessment Test Domain Skill Difficulty

SAT Reading and Writing Craft and Structure Words in Context SAT Reading and Writing Craft and Structure Words in Context

ID: 760ee1db ID: c5b1afe5


Although the playwrights hoped that their play would be ______ when performed live, critics generally agreed that the Bicycle sharing systems allow users to rent a bicycle at one location within a city and return it to any other designated
production and performances had the opposite effect, wearying audiences instead of energizing them. location in that city, which can cause serious problems of bicycle supply and user demand within the city’s system.
Tohru Ikeguchi uses open-source data and statistical modeling to identify when a high number of users making one-
Which choice completes the text with the most logical and precise word or phrase? way trips is likely to leave some locations within the system ______ bicycles and other areas with insufficient supply.
A. multifaceted
Which choice completes the text with the most logical and precise word or phrase?
B. realistic
A. susceptible to
C. rousing
B. contingent on
D. subtle
C. saturated with

D. depleted of
ID: 760ee1db Answer
Correct Answer: C
ID: c5b1afe5 Answer
Rationale
Correct Answer: C
Choice C is the best answer because it most logically completes the text’s discussion of the play’s effect on Rationale
audiences. As used in this context, “rousing” means exciting or energizing. The text indicates that critics
found the play to have “the opposite effect” on audiences of what the playwrights hoped for. The critics, the Choice C is the best answer because it most logically completes the text’s discussion of Ikeguchi’s model of
text says, thought the play was wearying rather than energizing to viewers. This context supports the idea bicycle supply. In this context, “saturated with” means thoroughly or completely supplied with. The text
that the playwrights hoped live performances of the play would be energizing, or rousing, to audiences. explains a problem encountered by some bicycle-sharing programs: users can return bicycles to different
locations than where the users picked up the bicycles to start, which can result in a mismatch between
Choice A is incorrect because there’s no information in the text suggesting that the playwrights hoped live bicycle supply (that is, where the bicycles are currently located) and user demand (that is, the locations where
performances of the play would be “multifaceted,” or be varied or have many aspects. The text indicates that users are hoping to pick up bicycles). The text goes on to explain that Ikeguchi developed a way to identify
critics found the play to be wearying to audiences and that this was the opposite of what the playwrights when this mismatch is likely to occur. This context suggests that Ikeguchi’s method will show when it is likely
hoped for. Multifaceted is not the opposite of wearying, but rousing is, which suggests that the playwrights that some locations have an insufficient supply and other locations, by implicit contrast, are saturated with
hoped the play would be rousing. Choice B is incorrect because there’s no information in the text suggesting bicycles.
that the playwrights hoped live performances of the play would be “realistic,” or lifelike or sensible. The text
says that critics found the play to be wearying to audiences and that this was the opposite of what the Choice A is incorrect because nothing in the text suggests that some locations are “susceptible to,” or sensitive
playwrights hoped for. Realistic is not the opposite of wearying, but rousing is, which suggests that the to or easily influenced by, bicycles. The text describes the phenomenon of bicycles being redistributed away
playwrights hoped the play would be rousing. Choice D is incorrect because there’s no information in the text from locations where users want them, not anything about those locations being influenced by the
suggesting that the playwrights hoped live performances of the play would be subtle, or complex or bicycles. Choice B is incorrect because the text describes situations in which some locations have an
understated. The text says that critics found the play to be wearying to audiences and that this was the insufficient supply of bicycles because the bicycles have been relocated elsewhere, which suggests that the
opposite of what the playwrights hoped for. Subtle is not the opposite of wearying, but rousing is, which other locations have many bicycles, not that the other locations are “contingent on,” or dependent on, the
suggests that the playwrights hoped the play would be rousing. bicycles. Nothing in the text suggests that the locations themselves depend on the bicycles for
anything. Choice D is incorrect because it would not make sense in context to say that some locations are
Question Difficulty: Easy “depleted of,” or empty of, bicycles while others have an insufficient supply. The text describes situations in
which bicycles have been relocated such that there is a mismatch between bicycle supply and user demand—
the bicycles are no longer at the locations where users want to pick them up. This means that some locations
do not have enough bicycles, while other locations must have many bicycles, not be depleted of bicycles.

Question Difficulty: Medium

underlined portion of Text 1 says nothing about how the Chicxulub impact changed the climate, so while Hull’s

Question ID 84dbd633 team might believe that the impact did in fact change the climate, they could not be said to agree with the
claim in Text 1 on this point. Choice C is incorrect because there is no indication in either text that any
scientists assume that the Chicxulub impact caused the Deccan Traps activity, so there is no reason to
Assessment Test Domain Skill Difficulty conclude that Hull’s team would question why the scientists referred to in Text 1 make such an
assumption. Choice D is incorrect because Text 2 describes how Hull’s team found that the climate had
SAT Reading and Writing Craft and Structure Cross-Text recovered from the changes brought about by the Deccan Traps activity before the K-Pg event occurred, which
Connections
suggests that Hull’s team would say that the Deccan Traps activity had a less enduring effect on global
conditions than the scientists referenced in Text 1 believe, not that the effect on global conditions was more
ID: 84dbd633 significant than those scientists claim.

Text 1
Question Difficulty: Hard
The Cretaceous-Paleogene (K-Pg) mass extinction event is usually attributed solely to an asteroid impact near
Chicxulub, Mexico. Some scientists argue that volcanic activity was the true cause, as the K-Pg event occurred relatively
early in a long period of eruption of the Deccan Traps range that initially produced huge amounts of climate-altering
gases. These dissenters note that other mass extinctions have coincided with large volcanic eruptions, while only the K-
Pg event lines up with an asteroid strike.

Text 2
In a 2020 study, Pincelli Hull and her colleagues analyzed ocean core samples and modeled climate changes around
the K-Pg event. The team concluded that Deccan Traps gases did affect global conditions prior to the event, but that
the climate returned to normal well before the extinctions began—extinctions that instead closely align with the
Chicxulub impact.

Based on the texts, how would Hull’s team (Text 2) most likely respond to the argument in the underlined portion of
Text 1?

A. By agreeing that the Chicxulub impact changed the climate and that the Deccan Traps eruption caused the K-Pg
event

B. By declaring that the changes in climate caused by the Deccan Traps eruption weren’t the main cause of the K-Pg
event

C. By questioning why those scientists assume that the Chicxulub impact caused the Deccan Traps eruption

D. By asserting that the Deccan Traps eruption had a more significant effect on global conditions than those scientists
claim

ID: 84dbd633 Answer


Correct Answer: B

Rationale

Choice B is the best answer because it describes how Hull’s team would most likely respond to the argument
in the underlined portion of Text 1, which asserts that volcanic activity in the Deccan Traps range led to
changes in the climate and caused the K-Pg mass extinction event. According to Text 2, although Hull’s team
found that activity in the Deccan Traps did indeed alter the climate before the K-Pg event, the team
determined that the climate had returned to normal before mass extinctions began. This finding and the
observation that the K-Pg extinctions closely align with the Chicxulub asteroid impact suggest that Hull’s
team would likely dispute the claim in the underlined portion of Text 1 and say that the climate changes
caused by the Deccan Traps activity were not the main cause of the extinctions.

Choice A is incorrect because Text 2 describes how Hull’s team found that the climate had recovered from the
changes brought about by the Deccan Traps activity before the K-Pg event occurred, which suggests that Hull’s
team would disagree that the Deccan Traps activity caused the K-Pg event. Additionally, the claim in the
Question ID a2dd51c1 Question ID 9d73c9eb
Assessment Test Domain Skill Difficulty Assessment Test Domain Skill Difficulty

SAT Reading and Writing Craft and Structure Text Structure and SAT Reading and Writing Craft and Structure Words in Context
Purpose

ID: 9d73c9eb
ID: a2dd51c1
Osage Nation citizen Randy Tinker-Smith produced and directed the ballet Wahzhazhe, which vividly chronicles Osage
In most building demolitions, the building materials are destroyed and sent to landfills. City officials in Portland, history and culture. Telling Osage stories through ballet is ______ choice because two of the foremost ballet dancers of
Oregon, wanted to reduce this waste. The officials passed a law requiring demolition companies to deconstruct some the twentieth century were Osage: sisters Maria and Marjorie Tallchief.
buildings instead. Deconstruction involves carefully taking buildings apart piece by piece. Damage to the materials is
avoided so that they can be reused in new constructions. A 2019 study found that 27 percent of materials from Which choice completes the text with the most logical and precise word or phrase?
deconstructions in Portland were able to be reused. The remaining materials were processed for recycling instead of
A. a suitable
going to a landfill.
B. a determined
Which choice best states the main purpose of the text?
C. an arbitrary
A. To explain an effort made by the city of Portland to reduce demolition waste and some results of that effort
D. an unpredictable
B. To show that popular support for measures that reduce demolition waste has increased since 2019

C. To argue that building deconstruction is not as effective as other measures at reducing demolition waste
ID: 9d73c9eb Answer
D. To discuss laws aimed to reduce demolition waste in Portland and compare them to similar laws in other cities Correct Answer: A

Rationale
ID: a2dd51c1 Answer
Choice A is the best answer. “Suitable” means “appropriate for a particular purpose.” Since the text indicates
Correct Answer: A that two of the best ballet dancers of the twentieth century were Osage, we can infer that the author believes
Rationale that ballet is a very suitable artform for telling Osage stories.

Choice A is the best answer. The author first describes a Portland law aimed at reducing demolition waste, and Choice B is incorrect. The text never suggests that Tinker-Smith’s choice was determined. That would imply
then goes on to explain that 27% of materials from building deconstructions were able to be reused and the that Tinker-Smith initially faced some kind of obstacle or opposition, and nothing like that is mentioned in
rest were recycled. the passage. Choice C is incorrect. The text implies the opposite of this. “Arbitrary” means “based on random
choice or whim rather than reason.” But the text does give us a good reason behind the choice to tell Osage
Choice B is incorrect. The text never discusses the popularity of the law. Choice C is incorrect. The text never stories through ballet: two of the best ballet dancers of the twentieth century were Osage. Choice D is
compares building deconstruction to other measures designed to reduce demolition waste. Choice D is incorrect. The text never suggests that Tinker-Smith’s choice was “unpredictable.” Rather, the fact that two of
incorrect. The text never mentions laws in cities other than Portland, Oregon. the best ballet dancers of the twentieth century were Osage makes ballet especially appropriate for telling
Osage stories.
Question Difficulty: Easy
Question Difficulty: Easy

Question ID 81a3a607 Question ID 48e4021d


Assessment Test Domain Skill Difficulty Assessment Test Domain Skill Difficulty

SAT Reading and Writing Craft and Structure Words in Context SAT Reading and Writing Craft and Structure Text Structure and
Purpose

ID: 81a3a607
ID: 48e4021d
In the Indigenous intercropping system known as the Three Sisters, maize, squash, and beans form an ______ web of
relations: maize provides the structure on which the bean vines grow; the squash vines cover the soil, discouraging The following text is from Holly Goldberg Sloan’s 2017 novel Short.
competition from weeds; and the beans aid their two “sisters” by enriching the soil with essential nitrogen.
More than two years ago my parents bought a piano from some people who were moving to Utah. Mom and
Dad gave it to my brothers and me for Christmas. I had to act really happy because it was such a big present, but I
Which choice completes the text with the most logical and precise word or phrase?
pretty much hated the thing from the second it was carried into the hallway upstairs, which is right next to my
A. indecipherable bedroom. The piano glared at me. It was like a songbird in a cage. It wanted to be set free.
©2017 by Holly Goldberg Sloan
B. ornamental

C. obscure Which choice best states the main purpose of the text?

A. It explains why the narrator always wanted a piano close to her bedroom.
D. intricate
B. It establishes how the narrator feels about the piano.

ID: 81a3a607 Answer C. It suggests that the narrator’s brothers are talented piano players.
Correct Answer: D D. It describes the event that led the narrator’s parents to buy a piano.
Rationale

Choice D is the best answer because it most logically completes the text’s discussion of the Three Sisters ID: 48e4021d Answer
intercropping system. As used in this context, “intricate” would mean made up of complexly related elements. Correct Answer: B
The text indicates that in the Three Sisters system, maize, squash, and beans form a “web of relations” in
Rationale
which the crops interact in various ways. The text’s description of these interactions—the bean vines growing
on the maize stalks, the squash vines keeping weeds away, and the beans adding nutrients that the maize and
Choice B is the best answer because it best states the main purpose of the text, which is to establish the
squash use—provides context suggesting that this “web of relations” is intricate.
narrator’s feelings about the piano. The narrator reveals that she “had to act really happy” about the piano even
though she “pretty much hated the thing” as soon as it was placed upstairs near her bedroom. The narrator
Choice A is incorrect because describing the relationship among the crops in the Three Sisters system as
also describes the piano as glaring at her and compares it to a caged bird that wants to be set free. These
“indecipherable,” or impossible to comprehend, would not make sense in context. Although the text presents
details establish the narrator’s feelings about the piano, suggesting that it makes her uneasy.
the relationship as complex, the text’s description of the role that each crop plays makes it clear that the
relationship is well understood, not indecipherable. Choice B is incorrect because the text discusses the
Choice A is incorrect because the text indicates that the narrator hated having the piano upstairs right next to
practical benefits that each plant in the Three Sisters system provides to other members of the system,
her bedroom, not that she wanted a piano to be close to her bedroom. Choice C is incorrect because the only
showing that the relationship among the crops that make up the system is not “ornamental,” or mainly serving
information provided in the text about the narrator’s brothers is that they were given the piano along with the
a decorative purpose. Choice C is incorrect because describing the relationship among the crops in the Three
narrator. Choice D is incorrect because the text does not describe the event that led the narrator’s parents to
Sisters system as “obscure,” or unknown or poorly understood, would not make sense in context. Although the
buy the piano from the people moving to Utah. Instead, the text focuses on the narrator’s feelings about the
text presents the relationship as complex, the text’s description of the role that each crop plays makes it clear
piano after it was given to her and her brothers.
that the relationship is well understood, not obscure.
Question Difficulty: Easy
Question Difficulty: Medium
Question ID 8d579825 Question ID 3566120b
Assessment Test Domain Skill Difficulty Assessment Test Domain Skill Difficulty

SAT Reading and Writing Craft and Structure Words in Context SAT Reading and Writing Craft and Structure Words in Context

ID: 8d579825 ID: 3566120b


The printing of Virginia Woolf’s novels featured a creative ______ between Woolf and her sister Vanessa Bell: a talented The following text is adapted from Oscar Wilde’s 1895 play The Importance of Being Earnest.
painter, Bell worked closely with Woolf to create original cover art for most of the novels. CECILY: Have we got to part?
ALGERNON: I am afraid so. It’s a very painful parting.
Which choice completes the text with the most logical and precise word or phrase? CECILY: It is always painful to part from people whom one has known for a very brief space of time. The absence of
A. rebellion old friends one can endure with equanimity. But even a momentary separation from anyone to whom one has just
been introduced is almost unbearable.
B. partnership
As used in the text, what does the word “endure” most nearly mean?
C. discovery
A. Regret
D. disagreement
B. Persist

ID: 8d579825 Answer C. Tolerate

Correct Answer: B D. Encourage

Rationale
ID: 3566120b Answer
Choice B is the best answer because it most logically completes the text’s discussion of the cover art of
Virginia Woolf’s novels. In this context, “partnership” means collaboration or joint effort. The text states that Correct Answer: C
Woolf’s sister Vanessa Bell was a talented painter and that Bell worked with Woolf to provide the cover art for Rationale
most of Woolf’s novels. Thus, this context suggests that Woolf and Bell pursued a creative partnership in order
to produce several of Woolf’s novels. Choice C is the best answer because as used in the text, “endure” most nearly means tolerate. In the text, Cecily
and Algernon discuss parting, or saying goodbye. Cecily remarks on the deep pain of saying goodbye to people
Choice A is incorrect because there is no indication that Woolf or Bell were undergoing a “rebellion,” or revolt, whom one has only known briefly and then comments on the equanimity, or calm steadiness, one experiences
against anything in particular. Instead, the text focuses on how they worked together in some aspects of the when separated from old friends. The text sets up an ironic contrast: one can easily tolerate, or put up with, the
production of Woolf’s novels. Choice C is incorrect because there is nothing in the text to indicate that Woolf or absence of close friends, but even a very short separation from a new acquaintance is unbearable.
Bell made a “discovery,” or encountered anything new for the first time as they worked together. The text
suggests that Woolf had already written her novels and that Bell then assisted Woolf with the cover art for Choice A is incorrect. Nothing in the text suggests that Cecily associates regret, or a feeling of sorrow, with the
many of them. Choice D is incorrect because the text implies that Woolf and Bell had a positive relationship. absence of old friends. Rather, the text sets up an ironic contrast between the feeling of calm steadiness one
According to the text, the sisters “worked closely” together to produce the cover art for many of Woolf’s novels; feels when separated from old friends and the unbearable pain of being separated from new acquaintances.
the text doesn’t mention whether while working together, Woolf and Bell had a “disagreement,” or conflict. Choice B is incorrect. Although in some contexts “endure” can mean persist, or proceed stubbornly, it doesn’t
have that meaning in this context because what is being endured is the absence of old friends. Whereas one
Question Difficulty: Easy can persist despite the absence of someone else, one can’t persist the absence itself. Choice D is incorrect
because the text doesn’t convey that Cecily encourages, or urges, old friends to be absent. Although it may be
that Cecily prefers new acquaintances to friends she has known for a long time, the text focuses on her
feelings as a result of others’ absences, not on her treatment of others.

Question Difficulty: Hard

Question ID f2c48e47 Question ID bcc91b1e


Assessment Test Domain Skill Difficulty Assessment Test Domain Skill Difficulty

SAT Reading and Writing Craft and Structure Text Structure and SAT Reading and Writing Craft and Structure Words in Context
Purpose

ID: bcc91b1e
ID: f2c48e47
The spacecraft OSIRIS-REx briefly made contact with the asteroid 101955 Bennu in 2020. NASA scientist Daniella
The following text is from Charlotte Perkins Gilman’s 1910 poem “The Earth’s Entail.” DellaGiustina reports that despite facing the unexpected obstacle of a surface mostly covered in boulders, OSIRIS-REx
No matter how we cultivate the land, successfully ______ a sample of the surface, gathering pieces of it to bring back to Earth.
Taming the forest and the prairie free;
No matter how we irrigate the sand, Which choice completes the text with the most logical and precise word or phrase?
Making the desert blossom at command,
A. attached
We must always leave the borders of the sea;
The immeasureable reaches B. collected
Of the windy wave-wet beaches,
C. followed
The million-mile-long margin of the sea.
D. replaced
Which choice best describes the overall structure of the text?

A. The speaker argues against interfering with nature and then gives evidence supporting this interference.
ID: bcc91b1e Answer
B. The speaker presents an account of efforts to dominate nature and then cautions that such efforts are only
Correct Answer: B
temporary.
Rationale
C. The speaker provides examples of an admirable way of approaching nature and then challenges that approach.
Choice B is the best answer because it most logically completes the text’s discussion of the OSIRIS-REx
D. The speaker describes attempts to control nature and then offers a reminder that not all nature is controllable.
spacecraft’s contact with the asteroid 101955 Bennu. In this context, “collected” means acquired and took away.
The text indicates that although the boulders on the asteroid’s surface caused some unforeseen problems,
OSIRIS-REx was able to gather a sample to return to Earth. This context suggests that OSIRIS-REx successfully
ID: f2c48e47 Answer
collected a sample of 101955 Bennu.
Correct Answer: D

Rationale Choice A is incorrect because in this context “attached” means connected or affixed. The text indicates that
OSIRIS-REx gathered pieces of 101955 Bennu to bring to Earth; it doesn’t suggest that the spacecraft attached
Choice D is the best answer. This best describes the overall structure of the text. In the first half of the text, the anything to the asteroid. Choice C is incorrect because in this context “followed” means tracked or traveled
speaker describes our attempts to control nature: cultivating, taming, and irrigating different kinds of land. In behind and the text discusses OSIRIS-REx’s brief encounter with 101955 Bennu during which the spacecraft
the second half, the speaker states that we can never tame the sea or the beach. gathered a sample to bring to Earth. The text doesn’t suggest that the spacecraft tracked the sample, and it’s
not clear what it would mean for the spacecraft to travel behind the sample it collected. Choice D is incorrect
Choice A is incorrect. This doesn’t describe the overall structure of the text. The speaker never argues that we because in this context “replaced” means put back or returned. The text indicates that OSIRIS-REx gathered
should not interfere with nature. Rather, the speaker says that we are able to tame many different kinds of pieces of 101955 Bennu to bring to Earth but doesn’t suggest that anything was returned to the asteroid.
land, but we are unable to tame the sea or beaches. Choice B is incorrect. This doesn’t describe the overall
structure of the text. The speaker never describes our cultivation, taming, and irrigation of land as “temporary.” Question Difficulty: Easy
Rather, the speaker says that we are able to tame many different kinds of land, but we are unable to tame the
sea or beaches. Choice C is incorrect. This doesn’t describe the overall structure of the text. The speaker never
describes our cultivation, taming, and irrigation of land as an “admirable” approach to nature.” Rather, the
speaker says that we are able to tame many different kinds of land, but we are unable to tame the sea or
beaches.

Question Difficulty: Medium


Question ID 9645f55e Question ID f1c9d2c1
Assessment Test Domain Skill Difficulty Assessment Test Domain Skill Difficulty

SAT Reading and Writing Craft and Structure Cross-Text SAT Reading and Writing Craft and Structure Cross-Text
Connections Connections

ID: 9645f55e ID: f1c9d2c1


Text 1 Text 1
For decades, bluegrass musicians have debated whether their genre should exclude influences from mainstream Stage lighting theorist Adolphe Appia was perhaps the first to argue that light must be considered alongside all the
genres such as rock. Many insist that bluegrass is defined by its adherence to the folk music of the US South, out of various elements of a stage to create a single, unified performance. Researcher Kelly Bremner, however, has noted that
which bluegrass emerged. Such “purists,” as they are known, regard the recordings of Bill Monroe, which established Appia lacked technical expertise in the use of light in the theater. As a result of Appia’s inexperience, Bremner argues,
the bluegrass sound in the 1940s, as a standard against which the genre should still be measured. Appia’s theory of light called for lighting practices that weren’t possible until after the advent of electricity around
1881.
Text 2
Bluegrass isn’t simply an extension of folk traditions into the era of recorded music. In reality, Bill Monroe created the Text 2
bluegrass sound in the 1940s by combining Southern folk music with commercial genres that had arisen only a few Adolphe Appia was not an amateur in the practice of lighting. Instead, it is precisely his exposure to lighting techniques
decades before, such as jazz and the blues. Since bluegrass has always been a mixed genre, contemporary bluegrass at the time that contributed to his theory on the importance of light. When working as an apprentice for a lighting
musicians should not be forbidden from incorporating into it influences from rock and other mainstream genres. specialist in his youth, Appia observed the use of portable lighting devices that could be operated by hand. This
experience developed his understanding of what was possible in the coordination of elements on the stage.
Based on the texts, how would the author of Text 2 most likely regard the perspective of bluegrass purists, as described
in Text 1? Based on the texts, how would the author of Text 2 most likely respond to the claim about Appia’s level of technical
expertise made by Bremner in Text 1?
A. As inconsistent, since bluegrass purists themselves enjoy other musical genres
A. Many lighting technicians dismissed Appia’s ideas about light on the stage.
B. As unrealistic, since bluegrass purists have no way of enforcing their musical preferences
B. Appia likely gained a level of technical expertise during his time as an apprentice.
C. As shortsighted, because bluegrass could enlarge its audience by including influences from mainstream genres
C. Theater practitioners who worked with Appia greatly admired his work.
D. As illogical, because the purists overlook crucial aspects of how the bluegrass sound first originated.
D. Appia was unfamiliar with the use of music and sound in theater.

ID: 9645f55e Answer


Correct Answer: D ID: f1c9d2c1 Answer
Correct Answer: B
Rationale
Rationale
Choice D is the best answer. According to Author 1, the “purists” argue that bluegrass should stick to its folk
music roots. But Author 2 points out that this isn’t how bluegrass emerged: it actually got its sound from a mix Choice B is the best answer. The author of Text 2 directly contradicts Bremner’s claim that Appia lacked
of folk, jazz, and blues. technical expertise by stating that Appia was “not an amateur in the practice of lighting.” His experience as a
lighting specialist’s apprentice would have, the author of Text 2 argues, allowed Appia to “[develop] his
Choice A is incorrect. There’s nothing in Text 2 about what other genres the purists enjoy, so this inference understanding of what was possible” with the elements of theatrical design.
about Author 2’s views isn’t supported. Choice B is incorrect. There’s nothing in Text 2 about whether or how
purists can enforce their preferences, so this inference about Author 2’s views isn’t supported. Choice C is Choice A is incorrect. Neither text describes how other lighting technicians responded to Appia’s ideas.
incorrect. There’s nothing in Text 2 about the size of bluegrass’s audience, so this inference about Author 2’s Furthermore, this claim isn’t relevant to Bremner’s evaluation of Appia’s technical expertise. Choice C is
views isn’t supported. incorrect. Neither text mentions anything about the opinions of theater practitioners who worked with Appia,
so this answer choice does not relate to the claim about Appia’s level of technical expertise made by Bremner
Question Difficulty: Easy in Text 1. Choice D is incorrect. Neither text mentions anything about Appia’s familiarity with or ignorance of
the use of music and sound in theater. Both focus on his expertise (or lack thereof) in lighting.

Question Difficulty: Medium

Question ID 31d0bd9a Question ID dc043599


Assessment Test Domain Skill Difficulty Assessment Test Domain Skill Difficulty

SAT Reading and Writing Craft and Structure Words in Context SAT Reading and Writing Craft and Structure Cross-Text
Connections

ID: 31d0bd9a
ID: dc043599
The parasitic dodder plant increases its reproductive success by flowering at the same time as the host plant it has
latched onto. In 2020, Jianqiang Wu and his colleagues determined that the tiny dodder achieves this ______ with its Text 1
host by absorbing and utilizing a protein the host produces when it is about to flower. Most scientists agree that the moon was likely formed after a collision between Earth and a large planet named Theia.
This collision likely created a huge debris field, made up of material from both Earth and Theia. Based on models of this
Which choice completes the text with the most logical and precise word or phrase? event, scientists believe that the moon was formed from this debris over the course of thousands of years.

A. synchronization
Text 2
B. hibernation Researchers from NASA’s Ames Research Center used a computer to model how the moon could have formed.
Although simulations of the moon’s formation have been done in the past, the team from NASA ran simulations that
C. prediction
were much more detailed. They found that the formation of the moon was likely not a slow process that took many
D. moderation years. Instead, it’s probable that the moon’s formation happened immediately after impact, taking just a few hours.

Which choice best describes a difference in how the author of Text 1 and the author of Text 2 view the evidence for the
ID: 31d0bd9a Answer formation of the moon?

Correct Answer: A A. The author of Text 1 argues that the formation of the moon occurred much earlier than the author of Text 2 argues.

Rationale B. The author of Text 1 suggests there is more evidence confirming the existence of Theia than the author of Text 2
suggests.
Choice A is the best answer because it most logically completes the text’s discussion of a relationship between
the dodder plant and its host plant. As used in this context, “synchronization” means the act of things C. The author of Text 1 claims that the moon’s surface is more similar to Earth’s surface than the author of Text 2
happening at the same time. The text indicates that the dodder and its host plant flower in unison and that claims.
this synchronization occurs because the dodder makes use of a protein produced by the host shortly before
D. The author of Text 1 believes that the moon formed more slowly than the author of Text 2 believes.
flowering.

Choice B is incorrect because referring to “hibernation,” or the state of being dormant or inactive, wouldn’t
ID: dc043599 Answer
make sense in context. The text focuses on something the dodder plant actively engages in—making use of a
protein and producing flowers. Choice C is incorrect because stating that the dodder plant and its host engage Correct Answer: D
together in “prediction,” or the act of declaring or indicating something in advance, wouldn’t make sense in Rationale
context. Rather than indicating that the dodder plant and its host plant make a prediction about flowering
activity, the text suggests that the host produces a protein as part of its regular flowering process and that the Choice D is the best answer. Text 1 states that the moon was formed from the debris “over the course of
dodder then absorbs and uses that protein to flower at the same time. Choice D is incorrect because referring thousands of years,” while Text 2 states that the moon’s formation happened “immediately after impact, taking
to “moderation,” or the act of causing something to become less intense or extreme, wouldn’t make sense in just a few hours.” This shows a clear difference in how the authors view the evidence for the speed of the
context. Although the text states that the dodder plant absorbs and uses a protein made by its host plant, it moon’s formation.
doesn’t suggest that the dodder lessens the host plant’s flowering activity; the two plants simply flower in
unison. Choice A is incorrect. While Text 2 suggests that the moon formed over “just a few hours” and Text 1 says it
took “thousands of years,” neither one mentions when that formation occurred. Choice B is incorrect. While
Question Difficulty: Easy Theia isn’t mentioned in Text 2, neither text describes or disputes evidence of Theia’s existence. Choice C is
incorrect. Neither text makes any claims about the similarity or difference between the moon’s surface and
Earth’s surface.

Question Difficulty: Easy


Question ID 47955354 Question ID a3761c7e
Assessment Test Domain Skill Difficulty Assessment Test Domain Skill Difficulty

SAT Reading and Writing Craft and Structure Words in Context SAT Reading and Writing Craft and Structure Words in Context

ID: 47955354 ID: a3761c7e


Sumerian civilization (which lasted from around 3300 to 2000 BCE) ______ many concepts that persist into present-day Physicist Joseph Weber performed ______ work in gravitational wave research in the 1960s and 1970s, conducting key
civilizations: for example, the first description of the seven-day week appears in the Sumerian Epic of Gilgamesh. experiments that scientists later used as the basis for their own investigations that led to the first verified detection of a
gravitational wave in 2015.
Which choice completes the text with the most logical and precise word or phrase?
Which choice completes the text with the most logical and precise word or phrase?
A. transformed
A. foundational
B. introduced
B. supplementary
C. inherited
C. repetitive
D. overlooked
D. ineffective

ID: 47955354 Answer


Correct Answer: B
ID: a3761c7e Answer
Correct Answer: A
Rationale
Rationale
Choice B is the best answer because it most logically completes the text’s discussion of the contributions of
the Sumerian civilization. In this context, “introduced” means brought into practice or use. The text states that Choice A is the best answer because it most logically completes the text’s discussion of physicist Joseph
the first reference to a seven-day week appears in the Sumerian Epic of Gilgamesh. The text presents this Webster’s research on gravitational waves. In this context “foundational” means the basis on which something
information about the seven-day week as an example of a concept introduced by the Sumerian civilization else develops. The text indicates that Webster’s experiments in the 1960s and 1970s were earlier than, and
that persists into present-day civilizations. “key” to, the work of later scientists in the field; thus, Webster’s work was foundational to the later scientists’
experiments and the eventual detection of a gravitational wave in 2015.
Choice A is incorrect because nothing in the text suggests that the Sumerian civilization “transformed,” or
changed the nature of, concepts that persist into present-day civilizations. Instead, the text’s presentation of a Choice B is incorrect because the text does not suggest that Webster’s work was supplementary, or an
Sumerian literary work that contains the first description of the seven-day week is an example of the additional element of an existing, larger project, but rather it was “the basis for” later experiments. Choice C is
phenomenon described in the first half of the sentence, suggesting that the Sumerians invented many incorrect because the text does not assert that Webster’s work was repetitive, or involved doing something the
concepts that still persist. Choice C is incorrect because the information that a Sumerian literary work same way many times. Rather, the text indicates that Webster’s work formed the basis for later investigations
includes the first description of the seven-day week suggests that Sumerian civilization may have originated in the field of gravitational wave research. Choice D is incorrect because the text does not state that Webster’s
the seven-day week and other concepts that persist into present-day civilizations, not that it “inherited” the work was ineffective, or failed to produce the desired outcome. Rather, the text strongly implies that Webster’s
concepts, or received them from an ancestral figure or culture. Choice D is incorrect because the information work was productive and extremely important to later work in the field of gravitational wave research.
that Sumerian civilization produced the first description of the seven-day week is presented as an example of
the phenomenon described in the first half of the sentence, suggesting that Sumerian civilization originated Question Difficulty: Easy
this and other concepts that still persist, not that the Sumerians “overlooked,” or failed to notice or consider,
such concepts.

Question Difficulty: Easy

Question ID 34d7bb25 Question ID c0e1b70a


Assessment Test Domain Skill Difficulty Assessment Test Domain Skill Difficulty

SAT Reading and Writing Craft and Structure Text Structure and SAT Reading and Writing Craft and Structure Text Structure and
Purpose Purpose

ID: 34d7bb25 ID: c0e1b70a


According to Indian economist and sociologist Radhakamal Mukerjee (1889–1968), the Eurocentric concepts that The following text is adapted from Etsu Inagaki Sugimoto’s 1925 memoir A Daughter of the Samurai. As a young
informed early twentieth-century social scientific methods—for example, the idea that all social relations are reducible woman, Sugimoto moved from feudal Japan to the United States.
to struggles between individuals—had little relevance for India. Making the social sciences more responsive to Indians’
The standards of my own and my adopted country differed so widely in some ways, and my love for both lands
needs, Mukerjee argued, required constructing analytical categories informed by India’s cultural and ecological
was so sincere, that sometimes I had an odd feeling of standing upon a cloud in space, and gazing with
circumstances. Mukerjee thus proposed the communalist “Indian village” as the ideal model on which to base Indian
measuring eyes upon two separate worlds. At first I was continually trying to explain, by Japanese standards, all
economic and social policy.
the queer things that came every day before my surprised eyes; for no one seemed to know the origin or
Which choice best describes the overall structure of the text? significance of even the most familiar customs, nor why they existed and were followed.

A. The text recounts Mukerjee’s early training in the social scientific disciplines and then lists social policies whose Which choice best describes the main purpose of the text?
implementation Mukerjee oversaw.
A. To convey the narrator’s experience of observing and making sense of differences between two cultures she
B. The text mentions some of Mukerjee’s economic theories and then traces their impact on other Indian social embraces
scientists of the twentieth century.
B. To establish the narrator’s hope of forming connections with new companions by sharing customs she learned as a
C. The text presents Mukerjee’s critique of the social sciences and then provides an example of his attempts to child
address issues he identified in his critique.
C. To reveal the narrator’s recognition that she is hesitant to ask questions about certain aspects of a culture she is
D. The text explains an influential economic theory and then demonstrates how that theory was more important to newly encountering
Mukerjee’s work than other social scientists have acknowledged.
D. To emphasize the narrator’s wonder at discovering that the physical distance between two countries is greater than
she had expected
ID: 34d7bb25 Answer
Correct Answer: C ID: c0e1b70a Answer
Rationale Correct Answer: A

Choice C is the best answer. The text does indeed present Mukerjee’s critique of the social sciences—that they Rationale
were too Eurocentric—and then provides an example of how he attempted to address the issues he identified:
Choice A is the best answer because it most accurately describes the main purpose of the text. The narrator
by suggesting a social science model based on the Indian village.
asserts that she loves both her “own” country (Japan) and her “adopted country” (the United States) even
Choice A is incorrect. The text doesn’t discuss Mukerjee’s early training or his oversight of the implementation though the two countries differ “widely.” She also indicates that, at first, she would try to explain unfamiliar
of social policies at all. Choice B is incorrect. The text never discusses any other Indian social scientists. experiences that she had in the United States using the standards ingrained in her from growing up in Japan.
Choice D is incorrect. The text never mentions other social scientists’ responses to Mukerjee’s work. Thus, the main purpose of the text is to convey the narrator’s experience of observing and making sense of the
differences between two cultures she embraces.
Question Difficulty: Hard
Choice B is incorrect because the text makes no reference to possible companions. Although the text does
indicate that the narrator sometimes used the cultural framework she acquired growing up in Japan to explain
some experiences she’s had, there is no suggestion that this was in service of making friends. And although
“no one seemed to know” strongly implies that the narrator has interacted with other people in the United
States, there is no indication that these conversations involved her discussing Japanese customs. Choice C is
incorrect because nothing in the text suggests that the narrator was hesitant to ask questions. In fact, the
narrator indicates that “no one seemed to know the origin” of various customs, which provides evidence that,
rather than being hesitant, she sought information from several people. Choice D is incorrect because the text
makes no reference to the physical distance between Japan and the United States. Although the narrator
indicates that the two countries differ “widely” and likens them to “two separate worlds,” these descriptions
relate to cultural aspects of the countries and the narrator’s feelings about the two countries, not the physical
distance between them.
Question ID f631132b
Question Difficulty: Medium Assessment Test Domain Skill Difficulty

SAT Reading and Writing Craft and Structure Text Structure and
Purpose

ID: f631132b
In the Here and Now Storybook (1921), educator Lucy Sprague Mitchell advanced the then controversial idea that
books for very young children should imitate how they use language, since toddlers, who cannot yet grasp narrative or
abstract ideas, seek reassurance in verbal repetition and naming. The most enduring example of this idea is Margaret
Wise Brown’s 1947 picture book Goodnight Moon, in which a young rabbit names the objects in his room as he drifts
off to sleep. Scholars note that the book’s emphasis on repetition, rhythm, and nonsense rhyme speaks directly to
Mitchell’s influence.

Which choice best describes the overall structure of the text?

A. The text outlines a debate between two authors of children’s literature and then traces how that debate shaped
theories on early childhood education.

B. The text summarizes an argument about how children’s literature should be evaluated and then discusses a
contrasting view on that subject.

C. The text lists the literary characteristics that are common to many classics of children’s literature and then indicates
the narrative subjects that are most appropriate for young children.

D. The text presents a philosophy about what material is most suitable for children’s literature and then describes a
book influenced by that philosophy.

ID: f631132b Answer


Correct Answer: D

Rationale

Choice D is the best answer. The text starts by introducing Mitchell’s philosophy about using simple, repetitive
language in books for young children. Then it describes a book influenced by that philosophy, Goodnight
Moon.

Choice A is incorrect. Although two authors are mentioned in the text, they both agree about the type of
language that should be contained in books for young children. Choice B is incorrect. The text never discusses
the evaluation of children’s literature. It does provide one view of how children’s books should be written, but
never introduces a competing view. Choice C is incorrect. The text doesn’t mention “many classics of
children’s literature.” Instead, it describes an educational theory and identifies one example of a famous
children’s book that was influenced by that theory.

Question Difficulty: Medium

Question ID 80ebb189 Question ID 570970cd


Assessment Test Domain Skill Difficulty Assessment Test Domain Skill Difficulty

SAT Reading and Writing Craft and Structure Words in Context SAT Reading and Writing Craft and Structure Text Structure and
Purpose

ID: 80ebb189
ID: 570970cd
As an architect in Los Angeles in the 1950s, Helen Liu Fong became known for avoiding ______ designs in her buildings.
Instead of using standard shapes and colors, she typically explored innovative forms and daring hues. The following text is adapted from Indian Boyhood, a 1902 memoir by Ohiyesa (Charles A. Eastman), a Santee Dakota
writer. In the text, Ohiyesa recalls how the women in his tribe harvested maple syrup during his childhood.
Which choice completes the text with the most logical and precise word or phrase? Now the women began to test the trees—moving leisurely among them, axe in hand, and striking a single quick
blow, to see if the sap would appear. The trees, like people, have their individual characters; some were ready to
A. creative
yield up their life-blood, while others were more reluctant. Now one of the birchen basins was set under each tree,
B. bold and a hardwood chip driven deep into the cut which the axe had made. From the corners of this chip—at first drop
by drop, then more freely—the sap trickled into the little dishes.
C. traditional
Which choice best describes the function of the underlined sentence in the text as a whole?
D. understandable
A. It portrays the range of personality traits displayed by the women as they work.

ID: 80ebb189 Answer B. It foregrounds the beneficial relationship between humans and maple trees.

Correct Answer: C C. It demonstrates how human behavior can be influenced by the natural environment.
Rationale
D. It elaborates on an aspect of the maple trees that the women evaluate.

Choice C is the best answer because it most logically completes the text’s discussion of Helen Liu Fong’s
architectural designs. In this context, “traditional” means conventional. The text states that rather than use
ID: 570970cd Answer
“standard shapes and colors,” Fong pursued “innovative” and “daring” design choices in her work. Fong’s style
is depicted as inventive, so it therefore makes sense in this context that she avoided mainstream, traditional Correct Answer: D
designs in her buildings. Rationale

Choice A is incorrect because the text indicates that Fong’s work is innovative and experimental. Thus, Fong’s Choice D is the best answer because it best describes the function of the underlined sentence in the text’s
design choices could reasonably be considered creative, or original. She likely would have pursued creative overall portrayal of how the women in Ohiyesa’s tribe harvested maple syrup. The text states that the women
designs, not avoided them. Choice B is incorrect because the text indicates that Fong used “daring” hues in her used an axe to strike the maple trees in order to find out which ones would produce sap. The underlined
designs. Thus, Fong likely would have pursued bold, or brave and vivid design choices; she wouldn’t have sentence compares the trees to people, with the sap described as the trees’ “life-blood.” Some of the trees are
avoided them. Choice D is incorrect because the text doesn’t address whether Fong’s designs are ready to give out their sap, while others are unwilling to do so. Using personification, the sentence provides
understandable, or reasonable or expected. The text focuses on certain characteristics of Fong’s designs, not greater detail about the aspect of the maple trees—their potential to give sap—that the women are evaluating.
on how people received or understood them.
Choice A is incorrect because the personalities of the women are not discussed in the text. Although the
Question Difficulty: Easy underlined sentence does mention “individual characters,” this reference is not to the women in the text but
rather to the maple trees, which the sentence compares to people with individual character traits. Choice B is
incorrect because the underlined sentence focuses on the trees’ willingness or refusal to yield sap, not on the
beneficial relationship between the women and the trees. Additionally, although the text does suggest that the
women and their tribe benefit from the maple trees since the trees allow the women to harvest syrup, there is
nothing in the text to suggest that the trees benefit from this relationship in turn. Choice C is incorrect
because the underlined sentence is comparing maple trees to humans, not addressing the influence of the
natural environment on how the actual humans in the text, the women, behave.

Question Difficulty: Hard


Question ID e7d37666 Question ID eae66bf9
Assessment Test Domain Skill Difficulty Assessment Test Domain Skill Difficulty

SAT Reading and Writing Craft and Structure Words in Context SAT Reading and Writing Craft and Structure Cross-Text
Connections

ID: e7d37666
ID: eae66bf9
It is by no means ______ to recognize the influence of Dutch painter Hieronymus Bosch on Ali Banisadr’s paintings;
indeed, Banisadr himself cites Bosch as an inspiration. However, some scholars have suggested that the ancient Text 1
Mesopotamian poem Epic of Gilgamesh may have had a far greater impact on Banisadr’s work. In 2021, a team led by Amir Siraj hypothesized that the Chicxulub impactor—the object that struck the Yucatán
Peninsula sixty-six million years ago, precipitating the mass extinction of the dinosaurs—was likely a member of the
Which choice completes the text with the most logical and precise word or phrase? class of long-period comets. As evidence, Siraj cited the carbonaceous chondritic composition of samples from the
Chicxulub impact crater as well as of samples obtained from long-period comet Wild 2 in 2006.
A. substantial

B. satisfying Text 2
Although long-period comets contain carbonaceous chondrites, asteroids are similarly rich in these materials.
C. unimportant
Furthermore, some asteroids are rich in iridium, as Natalia Artemieva points out, whereas long-period comets are not.
D. appropriate Given the prevalence of iridium at the crater and, more broadly, in geological layers deposited worldwide following the
impact, Artemieva argues that an asteroid is a more plausible candidate for the Chicxulub impactor.

ID: e7d37666 Answer Based on the texts, how would Artemieva likely respond to Siraj’s hypothesis, as presented in Text 1?

Correct Answer: C A. By insisting that it overestimates how representative Wild 2 is of long-period comets as a class

Rationale B. By arguing that it does not account for the amount of iridium found in geological layers dating to the Chicxulub
impact
Choice C is the best answer because it most logically completes the sentence about the influences on
Banisadr’s work. In context, “It is by no means” followed by “unimportant” conveys how it is relevant to C. By praising it for connecting the composition of Chicxulub crater samples to the composition of certain asteroids
recognize Bosch’s influence on Banisadr. The text points out that the artist himself cites Bosch as an
D. By concurring that carbonaceous chondrites are prevalent in soil samples from sites distant from the Chicxulub
inspiration, and then goes on to claim that The Epic of Gilgamesh has had a more significant influence than
crater
Bosch.

Choice A is incorrect because “substantial,” which means weighty or meaningful, incorrectly suggests that it
ID: eae66bf9 Answer
wouldn’t be meaningful to acknowledge Bosch’s influence on Banisadr. The phrase “indeed, Banisadr himself
cites Bosch as an inspiration” doesn’t support this suggestion. Choice B is incorrect because “satisfying,” Correct Answer: B
which means pleasing, incorrectly suggests that it wouldn’t be pleasing to acknowledge Bosch’s influence on Rationale
Banisadr. The phrase “indeed, Banisadr himself cites Bosch as an inspiration” doesn’t support this
suggestion. Choice D is incorrect because “appropriate,” which means suitable, incorrectly suggests that it Choice B is the best answer. Siraj’s hypothesis is that the Chicxulub impactor was a long-period comet. But
wouldn’t be proper to acknowledge Bosch’s influence on Banisadr. The phrase “indeed, Banisadr himself cites Artemieva points to the iridium found in the crater and in “geological layers that were deposited worldwide
Bosch as an inspiration” doesn’t support this suggestion. after the impact” as evidence that it was actually an asteroid, not a long-period comet.

Question Difficulty: Hard Choice A is incorrect. We can’t infer that this is how Artemieva would respond to Siraj’s hypothesis. Text 2
never discusses whether Wild 2 is representative of long-period comets in general. Rather, Text 2 presents
Artemieva’s argument that the Chicxulub impactor was an asteroid, not a long-term comet. Choice C is
incorrect. We can’t infer that this is how Artemieva would respond to Siraj’s hypothesis. Siraj’s hypothesis
doesn’t make this connection: rather, Siraj hypothesizes that the Chicxulub impactor was a long-term comet.
Choice D is incorrect. We can’t infer that this is how Artemieva would respond to Siraj’s hypothesis. “Soil
samples from sites distant from the Chicxulub crater” is too vague. Only soil samples from sites that are
connected to the impact in some way are involved in either hypothesis.

Question Difficulty: Medium

Question ID a60b0004 Question ID 03080769


Assessment Test Domain Skill Difficulty Assessment Test Domain Skill Difficulty

SAT Reading and Writing Craft and Structure Words in Context SAT Reading and Writing Craft and Structure Cross-Text
Connections

ID: a60b0004
ID: 03080769
Scholarly discussions of gender in Shakespeare’s comedies often celebrate the rebellion of the playwright’s characters
against the rigid expectations ______ by Elizabethan society. Most of the comedies end in marriage, with characters Text 1
returning to their socially dictated gender roles after previously defying them, but there are some notable exceptions. Philosopher G.E. Moore’s most influential work entails the concept of common sense. He asserts that there are certain
beliefs that all people, including philosophers, know instinctively to be true, whether or not they profess otherwise:
Which choice completes the text with the most logical and precise word or phrase? among them, that they have bodies, or that they exist in a world with other objects that have three dimensions.
Moore’s careful work on common sense may seem obvious but was in fact groundbreaking.
A. interjected

B. committed Text 2
External world skepticism is a philosophical stance supposing that we cannot be sure of the existence of anything
C. illustrated
outside our own minds. During a lecture, G.E. Moore once offered a proof refuting this stance by holding out his hands
D. prescribed and saying, “Here is one hand, and here is another.” Many philosophers reflexively reject this proof (Annalisa Coliva
called it “an obviously annoying failure”) but have found it a challenge to articulate exactly why the proof fails.

ID: a60b0004 Answer Based on the texts, how would the author of Text 1 most likely respond to proponents of the philosophical stance
outlined in Text 2?
Correct Answer: D
A. By agreeing with those proponents that Moore’s treatment of positions that contradict his own is fundamentally
Rationale
unserious
Choice D is the best answer because it most logically completes the text’s discussion of gender roles in
B. By suggesting that an instinctive distaste for Moore’s position is preventing external world skeptics from
Shakespeare’s comedies. As used in this context, “prescribed” would mean laid down as rules. The text
constructing a sufficiently rigorous refutation of Moore
indicates that the characters in the comedies often defy gender roles that are “socially dictated” (even if most
characters do return to those roles eventually) and that scholars have been very interested in these acts of C. By arguing that if it is valid to assert that some facts are true based on instinct, it is also valid to assert that some
defiance. This context indicates that what the characters are rebelling against are standards of behavior proofs are inadequate based on instinct
prescribed by the society of the time.
D. By pointing out that Moore would assert that external world skepticism is at odds with other beliefs those
Choice A is incorrect because saying that expectations about gender were “interjected,” or suddenly inserted proponents must unavoidably hold
between other things, wouldn’t make sense in context. There’s no suggestion in the text that the issue of
gender roles was inserted between other things or was an interruption in a larger discussion. Choice B is
ID: 03080769 Answer
incorrect because the text indicates that Shakespeare depicts characters rebelling against expectations about
gender that have been “socially dictated,” not expectations that society has “committed,” or carried out, Correct Answer: D
entrusted, or promised. Choice C is incorrect because the text indicates that Shakespeare depicts characters Rationale
rebelling against expectations about gender that have been “socially dictated,” not expectations that have been
“illustrated,” or clarified with examples. Although it’s possible for expectations about gender roles to be Choice D is the best answer. According to the author of Text 1, Moore’s definition of common sense—things we
illustrated, there’s nothing in the text to indicate that characters in Shakespeare’s comedies rebel against instinctively know are true—includes the belief that we all “exist in a world with other objects.” The author of
illustrations of gender expectations. Text 1 describes this notion as both “obvious” and “groundbreaking.” So it’s safe to infer that the author would
observe that Moore would respond to external world skeptics by arguing that since everyone instinctively
Question Difficulty: Hard knows that things exist outside of their own minds, then external world skepticism must be wrong.

Choice A is incorrect. We can’t infer that the author of Text 1 would respond this way to external world
skeptics. If anything, the author of Text 1 seems to agree with Moore. Choice B is incorrect. We can’t infer that
the author of Text 1 would respond this way to external world skeptics. The author of Text 1 never mentions
external world skeptics directly, let alone why they have a hard time refuting Moore’s position. Choice C is
incorrect. We can’t infer that the author of Text 1 would respond this way to external world skeptics. Text 1’s
presentation of Moore’s concept of common sense only includes the idea that some facts are true based on
instinct—it doesn’t mention the idea that some proofs are inadequate based on instinct.
Question ID b0ea8c28
Question Difficulty: Medium
Assessment Test Domain Skill Difficulty

SAT Reading and Writing Craft and Structure Words in Context

ID: b0ea8c28
Sueño de Familia is an exhibition of drawings, paintings, and ceramics that explores the artistic heritage of US-based
artist Yolanda González. The exhibition ______ five generations, featuring works by González’s great-grandfather,
grandmother, mother, and niece as well as González herself.

Which choice completes the text with the most logical and precise word or phrase?

A. borrows

B. spans

C. judges

D. neglects

ID: b0ea8c28 Answer


Correct Answer: B

Rationale

Choice B is the best answer because it most logically completes the text’s discussion of the Sueño de Familia
art exhibition. In this context, “spans” means extends across or covers. The text states that the exhibition
explores González’s artistic heritage and features artwork by her great-grandfather, grandmother, mother, and
niece. This context conveys the idea that the exhibition spans, or extends across, five generations of
González’s family.

Choice A is incorrect because it wouldn’t make sense to say that the exhibition “borrows,” or acquires, five
generations of González’s family. The text indicates that the exhibition features artwork by family members
from five generations, not that the five generations themselves have been acquired for inclusion in the
exhibition. Choice C is incorrect because the text indicates that the purpose of the exhibition is to highlight
artwork, not to “judge,” or give an opinion on, five generations of the artist’s family. Choice D is incorrect
because the text doesn’t suggest that the exhibition “neglects,” or gives little attention to, five generations of
González’s family. On the contrary, the text indicates that the exhibition is dedicated to exploring González’s
artistic heritage and therefore designed to bring attention to her family members and their artwork.

Question Difficulty: Easy

determination of the waves. The underlined portion addresses only the speaker’s view of the waves and

Question ID 749f3334 doesn’t suggest what her own thoughts might be.

Question Difficulty: Medium


Assessment Test Domain Skill Difficulty

SAT Reading and Writing Craft and Structure Text Structure and
Purpose

ID: 749f3334
The following text is from Charlotte Forten Grimké’s 1888 poem “At Newport.”
Oh, deep delight to watch the gladsome waves
Exultant leap upon the rugged rocks;
Ever repulsed, yet ever rushing on—
Filled with a life that will not know defeat;
To see the glorious hues of sky and sea.
The distant snowy sails, glide spirit like,
Into an unknown world, to feel the sweet
Enchantment of the sea thrill all the soul,
Clearing the clouded brain, making the heart
Leap joyous as it own bright, singing waves!

Which choice best describes the function of the underlined portion in the text as a whole?

A. It portrays the surroundings as an imposing and intimidating scene.

B. It characterizes the sea’s waves as a relentless and enduring force.

C. It conveys the speaker’s ambivalence about the natural world.

D. It draws a contrast between the sea’s waves and the speaker’s thoughts.

ID: 749f3334 Answer


Correct Answer: B

Rationale

Choice B is the best answer because it most accurately describes how the underlined portion functions in the
text as a whole. The text presents the speaker’s experience of viewing the sea. In the underlined portion, the
speaker focuses on the idea that the waves hitting rocks on the shore are a relentless and enduring force: they
are constantly pushed back (“ever repulsed”) but always return (“ever rushing on”), as though they have an
energy that can’t be overcome (“a life that will not know defeat”).

Choice A is incorrect. Although the underlined portion characterizes the waves as a relentless force (always
“repulsed” but still “rushing on” and never being defeated), the speaker doesn’t suggest that the surroundings
are intimidating. Instead, the speaker presents the scene in a positive way, describing the “deep delight” of the
“gladsome,” or cheerful, waves and feeling “the heart / Leap joyous” while viewing the sea. Choice C is incorrect
because the underlined portion doesn’t suggest that the speaker is ambivalent, or has mixed feelings about,
the natural world. Instead, it presents a single view of one part of the immediate surroundings: the speaker
characterizes the sea’s waves as an unstoppable force, since they are constantly pushed back but always
return (“ever repulsed, yet ever rushing on”). Choice D is incorrect. Although the text later suggests the
speaker’s view of her own thoughts by referring to a “clouded brain” and a heart that leaps joyously, this
reference neither occurs within the underlined portion nor establishes a clear contrast with the relentless
Question ID fce80a36 Question ID e4e2aeb3
Assessment Test Domain Skill Difficulty Assessment Test Domain Skill Difficulty

SAT Reading and Writing Craft and Structure Words in Context SAT Reading and Writing Craft and Structure Cross-Text
Connections

ID: fce80a36
ID: e4e2aeb3
In 2008 a complete set of ancient pessoi (glass game pieces) was uncovered from beneath a paving stone in modern-
day Israel. Due to their small size, pessoi were easily misplaced, making a whole set a rare find. This has led some Text 1
experts to suggest that the set may have been buried intentionally; however, without clear evidence, archaeologists are Like the work of Ralph Ellison before her, Toni Morrison’s novels feature scenes in which characters deliver sermons of
left to ______ what happened. such length and verbal dexterity that for a time, the text exchanges the formal parameters of fiction for those of oral
literature. Given the many other echoes of Ellison in Morrison’s novels, both in structure and prose style, these scenes
Which choice completes the text with the most logical and precise word or phrase? suggest Ellison’s direct influence on Morrison.
Text 2
A. speculate about
In their destabilizing effect on literary form, the sermons in Morrison’s works recall those in Ellison’s. Yet literature by
B. dismiss Black Americans abounds in moments where interpolated speech erodes the division between oral and written forms
that literature in English has traditionally observed. Morrison’s use of the sermon is attributable not only to the
C. expand on
influence of Ellison but also to a community-wide strategy of resistance to externally imposed literary conventions.
D. catalog
Based on the texts, how would the author of Text 2 most likely characterize the underlined claim in Text 1?

A. As failing to consider Ellison’s and Morrison’s equivalent uses of the sermon within the wider cultural context in
ID: fce80a36 Answer which they wrote
Correct Answer: A
B. As misunderstanding the function of sermons in novels by Black American writers other than Ellison and Morrison
Rationale
C. As disregarding points of structural and stylistic divergence between the works of Ellison and those of Morrison
Choice A is the best answer. "Speculate" means "to form a theory or guess without any clear evidence." This
D. As being indebted to the tradition of resisting literary conventions that privilege written forms, such as novels, over
makes sense because, due to the lack of "clear evidence," the archaeologists can only guess how the pessoi set
sermons and other oral forms
might have come to be there.

Choice B is incorrect. "Dismiss" can mean "send away" or "treat as unworthy of consideration." The text
ID: e4e2aeb3 Answer
implies that the archaeologists are trying to figure out the truth—they wouldn’t "dismiss" what really
happened. Choice C is incorrect. "Expand on" means "give more details about," but there aren’t any details to Correct Answer: A
give. Without any "clear evidence," the archaeologists can’t give any more details. Choice D is incorrect. Rationale
"Catalog" means "carefully record" or "make a list of." However, there’s no "clear evidence," so there’s no real
information to "catalog." Choice A is the best answer. The author of Text 2 argues that Morrison’s use of the sermon is not only
influenced by Ellison, but also by a “community-wide strategy of resistance” to literary conventions practiced
Question Difficulty: Medium by Black American authors. Ellison, Text 2 alleges, is only one of many influences on Morrison.

Choice B is incorrect. Neither text specifically mentions sermons in works by authors other than Morrison or
Ellison, only a tendency towards eroding “the division between oral and written forms” among Black American
writers. Choice C is incorrect. Both texts describe similarities between the works of Ellison and Morrison, and
neither points out instances of divergence. Text 2 simply suggests that Morrison was influenced by more than
just Ellison. Choice D is incorrect. While Text 2 does discuss Morrison’s resistance to certain literary
conventions, it’s unclear what it would mean for the underlined claim to be “indebted” to that tradition. This
choice recycles language from the text, but not in a way that makes any coherent point.

Question Difficulty: Hard

Question ID 0d402146 Question ID e5da61f1


Assessment Test Domain Skill Difficulty Assessment Test Domain Skill Difficulty

SAT Reading and Writing Craft and Structure Words in Context SAT Reading and Writing Craft and Structure Text Structure and
Purpose

ID: 0d402146
ID: e5da61f1
US traffic signals didn’t always contain the familiar three lights (red, yellow, and green). Traffic lights only ______ red and
green lights until the three-light traffic signal was developed in the 1920s. The following text is adapted from Charles Chesnutt’s 1899 short story “Mars Jeems’s Nightmare.” The narrator and his
wife have recently moved to the southern United States, and Julius is their carriage driver.
Which choice completes the text with the most logical and precise word or phrase?
Julius [was] very useful when we moved to our new residence. He had a thorough knowledge of the
A. avoided neighborhood, was familiar with the roads and the watercourses, knew the qualities of the various soils and what
they would produce, and where the best hunting and fishing were to be had. He was a marvelous hand in the
B. featured
management of horses and dogs.
C. appreciated
Which choice best states the main purpose of the text?
D. disregarded
A. To compare the narrator’s reaction to a new home with his wife’s reaction

B. To give an example of Julius’s knowledge about soil


ID: 0d402146 Answer
Correct Answer: B C. To show that the narrator and Julius often hunt and fish together

Rationale D. To explain different ways in which Julius was helpful

Choice B is the best answer because it most logically completes the text’s discussion of traffic signals. As used
in this context, “featured” means had as a characteristic or part. The text indicates that although US traffic ID: e5da61f1 Answer
signals have lights of three different colors (red, yellow, and green), this wasn’t the case until the 1920s, when Correct Answer: D
the three-light signal was first developed. Before then, the text suggests, traffic signals had fewer lights (as
Rationale
indicated by the word “only” in the second sentence). This context supports the idea that before the 1920s,
traffic signals featured only red and green lights.
Choice D is the best answer because it most accurately describes the main purpose of the text, which is to
explain different ways in which Julius was helpful. The text begins with the narrator stating that Julius was
Choice A is incorrect because “avoided” means kept away from someone or something or prevented something
very helpful to him and his wife when they moved to their new residence. The narrator then provides a list of
from occurring, neither of which would make sense in context. The text doesn’t discuss keeping away from
examples to illustrate Julius’s helpfulness. For instance, the narrator states that Julius was familiar with the
someone or something but instead focuses on what US traffic signals look like. The text states that they didn’t
neighborhood’s roads, which suggests that he was helpful in navigating them, and that Julius helped manage
have lights of three different colors until the three-light signal was developed in the 1920s. Choice C is
the horses and dogs. The text’s many examples of Julius’s usefulness reinforce just how helpful he was and in
incorrect because “appreciated” means admired or increased in value, neither of which would make sense in
how many different ways.
context. The text focuses solely on the fact that US traffic signals have contained lights of three colors only
since the three-light signal was developed in the 1920s. The text doesn’t mention how this characteristic or
Choice A is incorrect because the text doesn’t portray either the narrator’s or his wife’s reaction to their new
traffic signals in general are perceived or what their value is. Moreover, the blank portion of the text describes
home. Rather, the text focuses on how Julius was useful to the narrator and his wife in their new home. Choice
“traffic signals,” which are lifeless objects and therefore unable to admire or increase the value of
B is incorrect. Although the text states that Julius was knowledgeable about the soil, this is one of several
something. Choice D is incorrect because “disregarded” means ignored or treated something as unworthy of
supporting details that illustrate how helpful Julius was. Moreover, the text merely states that Julius was
notice, neither of which would make sense in context. The text doesn’t discuss how people react to traffic
knowledgeable about soil; it doesn’t provide an example of that knowledge. Choice C is incorrect because
signals; rather, it addresses the idea that US traffic signals have contained lights of three colors only since the
there’s nothing in the text to suggest the frequency with which the narrator and Julius hunted and fished
three-light signal was developed in the 1920s. Moreover, the blank portion of the text describes “traffic signals,”
together. In fact, it’s unclear from the text whether the narrator and Julius hunted and fished together at all.
which are lifeless objects and therefore unable to ignore or treat something in a particular manner.
The text merely indicates that Julius knew the best places to hunt and fish—a detail that supports the text’s
main purpose by conveying Julius’s usefulness.
Question Difficulty: Easy

Question Difficulty: Easy


Woolf’s other novels, but instead points out that it engages the same themes as other novels by Woolf that are

Question ID 6a1dc7c5 considered classics. Choice D is incorrect because there’s nothing in Text 1 or Text 2 to suggest that readers
have generally ignored Orlando because of its reputation.

Assessment Test Domain Skill Difficulty Question Difficulty: Hard

SAT Reading and Writing Craft and Structure Cross-Text


Connections

ID: 6a1dc7c5
Text 1
Virginia Woolf’s 1928 novel Orlando is an oddity within her body of work. Her other major novels consist mainly of
scenes of everyday life and describe their characters’ interior states in great detail, whereas Orlando propels itself
through a series of fantastical events and considers its characters’ psychology more superficially. Woolf herself
sometimes regarded the novel as a minor work, even admitting once that she “began it as a joke.”

Text 2
Like Woolf’s other great novels, Orlando portrays how people’s memories inform their experience of the present. Like
those works, it examines how people navigate social interactions shaped by gender and social class. Though it is lighter
in tone—more entertaining, even—this literary “joke” nonetheless engages seriously with the themes that motivated
the four or five other novels by Woolf that have achieved the status of literary classics.

Based on the texts, how would the author of Text 2 most likely respond to the assessment of Orlando presented in Text
1?

A. By conceding that Woolf’s talents were best suited to serious novels but asserting that the humor in Orlando is
often effective

B. By agreeing that Orlando is less impressive than certain other novels by Woolf but arguing that it should still be
regarded as a classic

C. By acknowledging that Orlando clearly differs from Woolf’s other major novels but insisting on its centrality to her
body of work nonetheless

D. By concurring that the reputation of Orlando as a minor work has led readers to overlook this novel but
maintaining that the reputation is unearned

ID: 6a1dc7c5 Answer


Correct Answer: C

Rationale

Choice C is the best answer because it reflects how the author of Text 2 would most likely respond to the
assessment of Orlando in Text 1. Both authors agree that Orlando is unusual for Woolf: Text 1 states that the
novel examines its characters’ psychologies more superficially than Woolf’s other novels do, and Text 2
describes it as being lighter in tone. However, while Text 1 calls Orlando an “oddity” and mentions that Woolf
“began it as a joke,” Text 2 asserts that Orlando engages the same themes as Woolf’s other great novels. Hence,
the author of Text 2 would most likely accept that Orlando differs from Woolf’s other novels but would also
insist on its importance in the context of Woolf’s work as a writer.

Choice A is incorrect. Text 2 does suggest that the humor in Orlando is effective. However, there’s nothing in
Text 2 to suggest that the author would agree that Woolf’s talents were best suited to serious novels. Rather, the
author of Text 2 compares Orlando favorably to other novels by Woolf that are implied to be darker in tone.
Choice B is incorrect because the author of Text 2 does not indicate that Orlando is less impressive than

Question ID 5dce6cab Question ID f99847ed


Assessment Test Domain Skill Difficulty Assessment Test Domain Skill Difficulty

SAT Reading and Writing Craft and Structure Words in Context SAT Reading and Writing Craft and Structure Words in Context

ID: 5dce6cab ID: f99847ed


Given that the conditions in binary star systems should make planetary formation nearly impossible, it’s not surprising For her 2021 art installation Anthem, Wu Tsang joined forces with singer and composer Beverly Glenn-Copeland to
that the existence of planets in such systems has lacked ______ explanation. Roman Rafikov and Kedron Silsbee shed produce a piece that critics found truly ______: they praised Tsang for creatively transforming a museum rotunda into a
light on the subject when they used modeling to determine a complex set of factors that could support planets’ dynamic exhibit by projecting filmed images of Glenn-Copeland onto a massive 84-foot curtain and filling the space
development. with the sounds of his and other voices singing.

Which choice completes the text with the most logical and precise word or phrase? Which choice completes the text with the most logical and precise word or phrase?

A. a discernible A. restrained

B. a straightforward B. inventive

C. an inconclusive C. inexplicable

D. an unbiased D. mystifying

ID: 5dce6cab Answer ID: f99847ed Answer


Correct Answer: B Correct Answer: B

Rationale Rationale

Choice B is the best answer because it most logically completes the text’s description of efforts to explain the Choice B is the best answer because it most logically completes the text’s discussion of the art installation
existence of planets in binary star systems. As used in this context, describing an explanation as “a Anthem. In this context, “inventive” means characterized by invention and creativity. The text explains that
straightforward” one would mean that the explanation is direct and uncomplicated. The text asserts that since critics’ responses to the installation involved praise for Tsang’s creative transformation of a space into a
it should be “nearly impossible” for planets to form in binary star systems, it’s “not surprising” that there isn’t a dynamic exhibit with huge images and lots of sound. This context conveys that the critics found the piece
straightforward explanation for the existence of planets in such systems; the fact that one potential approach particularly creative.
involves “complex” factors offers further contextual support for this idea.
Choice A is incorrect because the text indicates that critics praised the installation for being dynamic and
Choice A is incorrect because it would not make sense in context to say that there isn’t “a discernible” including huge images and lots of sound, and it wouldn’t make sense to describe such an exhibit as
explanation—meaning an explanation capable of being perceived—for the existence of planets in binary star “restrained,” or limited and not extravagant or showy. Choice C is incorrect because it wouldn’t make sense to
systems. The text discusses just such an explanation offered by Roman Rafikov and Kedron Silsbee, which say that critics found the installation “inexplicable,” or incapable of being explained or interpreted, since the
indicates that their explanation can be discerned. Choice C is incorrect because the text emphasizes how critics were able to explain their praise for the installation’s transformation of a space with huge images and
difficult it is to explain the existence of planets in binary star systems, suggesting that the situation isn’t lots of sound. Choice D is incorrect because the text focuses on the idea that critics praised Tsang for
marked by the lack of “an inconclusive” explanation—an explanation that does not resolve the issue—but creatively transforming a space into a dynamic exhibit, not that they found the installation “mystifying,” or
rather that if any explanations have been offered, they’ve likely been inconclusive ones. Choice D is incorrect bewildering and hard to understand. Nothing in the text suggests that the critics couldn’t understand the
because nothing in the text suggests that there is a lack of “an unbiased,” or impartial and unprejudiced, piece.
explanation for the existence of planets in binary star systems. The text indicates that it’s difficult to explain
the existence of planets in such systems and it describes one attempt to do so, but there is no evidence that Question Difficulty: Hard
explanations from Roman Rafikov and Kedron Silsbee or others are biased.

Question Difficulty: Hard


Srivastava’s team’s findings with enthusiasm; it describes the discovery of EGR in both three-banded panther

Question ID 5e101c70 worms and humans as promising and exciting. It would be illogical for the author of Text 2 to say that because
most others treat the discovery with enthusiasm, Text 1’s enthusiastic characterization of the discovery is
unexpected. Choice D is incorrect because Text 1 isn’t at all dismissive of Srivastava’s team’s findings; instead,
Assessment Test Domain Skill Difficulty Text 1 is optimistic about the EGR discovery, characterizing it as promising and exciting. There’s nothing in
Text 2 to suggest that the author of Text 2 would say that Text 1’s praise for the discovery is dismissive, or
SAT Reading and Writing Craft and Structure Cross-Text disdainful.
Connections
Question Difficulty: Hard
ID: 5e101c70

Text 1
Most animals can regenerate some parts of their bodies, such as skin. But when a three-banded panther worm is cut
into three pieces, each piece grows into a new worm. Researchers are investigating this feat partly to learn more about
humans’ comparatively limited abilities to regenerate, and they’re making exciting progress. An especially promising
discovery is that both humans and panther worms have a gene for early growth response (EGR) linked to regeneration.

Text 2
When Mansi Srivastava and her team reported that panther worms, like humans, possess a gene for EGR, it caused
excitement. However, as the team pointed out, the gene likely functions very differently in humans than it does in
panther worms. Srivastava has likened EGR to a switch that activates other genes involved in regeneration in panther
worms, but how this switch operates in humans remains unclear.

Based on the texts, what would the author of Text 2 most likely say about Text 1’s characterization of the discovery
involving EGR?

A. It is reasonable given that Srivastava and her team have identified how EGR functions in both humans and panther
worms.

B. It is overly optimistic given additional observations from Srivastava and her team.

C. It is unexpected given that Srivastava and her team’s findings were generally met with enthusiasm.

D. It is unfairly dismissive given the progress that Srivastava and her team have reported.

ID: 5e101c70 Answer


Correct Answer: B

Rationale

Choice B is the best answer because it reflects how the author of Text 2 would most likely respond to Text 1
based on the information provided. Text 1 discusses the discovery of a regeneration-linked gene, EGR, in both
three-banded panther worms (which are capable of full regeneration) and humans (who have relatively limited
regeneration abilities). Text 1 characterizes this discovery as “especially promising” and a sign of “exciting
progress” in understanding human regeneration. The author of Text 2, on the other hand, focuses on the fact
that the team that reported the EGR finding pointed out that while EGR’s function in humans isn’t yet known,
it’s likely very different from its function in panther worms. Therefore, the author of Text 2 would most likely
say that Text 1’s enthusiasm about the EGR discovery is overly optimistic given Srivastava’s team’s
observations about EGR in humans.

Choice A is incorrect because the author of Text 2 explains that Srivastava and her team explicitly reported
that they haven’t yet identified how EGR functions in humans; therefore, the author of Text 2 wouldn’t say that
Text 1’s excitement is reasonable for the stated reason. Instead, the author of Text 2 would likely characterize
Text 1’s excitement as premature and overly optimistic. Choice C is incorrect because Text 1 does treat

Question ID 12d81fc1 Question ID e4f312c5


Assessment Test Domain Skill Difficulty Assessment Test Domain Skill Difficulty

SAT Reading and Writing Craft and Structure Cross-Text SAT Reading and Writing Craft and Structure Words in Context
Connections

ID: e4f312c5
ID: 12d81fc1
While most animals are incapable of passing somatic mutations—genetic alterations that arise in an organism’s
Text 1 nonreproductive cells—on to their offspring, elkhorn coral (Acropora palmata) presents an intriguing ______: in a 2022
Because literacy in Nahuatl script, the writing system of the Aztec Empire, was lost after Spain invaded central Mexico study, researchers found that elkhorn coral produced offspring that inherited somatic mutations from a parent.
in the 1500s, it is unclear exactly how meaning was encoded in the script’s symbols. Although many scholars had
assumed that the symbols signified entire words, linguist Alfonso Lacadena theorized in 2008 that they signified units Which choice completes the text with the most logical and precise word or phrase?
of language smaller than words: individual syllables.
A. hypothesis

Text 2 B. affinity
The growing consensus among scholars of Nahuatl script is that many of its symbols could signify either words or
C. anomaly
syllables, depending on syntax and content at any given site within a text. For example, the symbol signifying the word
huipil (blouse) in some contexts could signify the syllable “pil” in others, as in the place name “Chipiltepec.” Thus, for D. corroboration
the Aztecs, reading required a determination of how such symbols functioned each time they appeared in a text.

Based on the texts, how would the author of Text 2 most likely characterize Lacadena’s theory, as described in Text 1? ID: e4f312c5 Answer
A. By praising the theory for recognizing that the script’s symbols could represent entire words Correct Answer: C

B. By arguing that the theory is overly influenced by the work of earlier scholars Rationale

C. By approving of the theory’s emphasis on how the script changed over time Choice C is the best answer. An "anomaly" is something that deviates from norms or expectations. In this case,
the elkhorn coral is an anomaly because it can pass on somatic mutations, whereas most other animals can’t.
D. By cautioning that the theory overlooks certain important aspects of how the script functioned
Choice A is incorrect. A "hypothesis" is "a theory about something," but no theories are provided about elkhorn
coral in this text, just facts. Choice B is incorrect. "Affinity" represents "an inclination or liking toward
ID: 12d81fc1 Answer
something." As genetic mutations tend to occur without any conscious effort, you can’t really have an
Correct Answer: D inclination toward passing on somatic mutations. Choice D is incorrect. "Corroboration" means "evidence to
Rationale support or prove something." Because elkhorn coral do the opposite of what most animals do, they do not
provide corroboration of the theory that somatic mutations can’t be passed onto offspring. Rather, they show
Choice D is the best answer. Lacadena’s theory is that Nahuatl script symbols signified syllables, but the the opposite.
consensus described in Text 2 is that they can signify either symbols or full words, depending on the context.
So the author of Text 2 would likely consider Lacadena’s theory too simplistic: it’s missing the importance of Question Difficulty: Hard
the context in determining the meaning of a symbol.

Choice A is incorrect. This conflicts with Text 1’s description of Lacadena’s theory. Lacadena’s theory is that
Nahuatl script symbols signified syllables. Choice B is incorrect. This conflicts with Text 1’s description of
Lacadena’s theory. Text 1 states that Lacadena’s theory differed from what earlier scholars believed. Choice C is
incorrect. We can’t infer that this is how the author of Text 2 would characterize Lacadena’s theory. Neither
text mentions how or even if the script changed over time.

Question Difficulty: Medium


Question ID a4f50d30 Question ID fa7a89f1
Assessment Test Domain Skill Difficulty Assessment Test Domain Skill Difficulty

SAT Reading and Writing Craft and Structure Words in Context SAT Reading and Writing Craft and Structure Words in Context

ID: a4f50d30 ID: fa7a89f1


Scientists previously thought that all electric eels belong to a single species, but a team of researchers led by zoologist Studying how workload affects productivity, Maryam Kouchaki and colleagues found that people who chose to do
C. David de Santana proved this idea wrong by ______ that there are in fact three distinct species of electric eels. relatively easy tasks first were less ______ compared to those who did hard tasks first. Finishing easy tasks gave
participants a sense of accomplishment, but those who tackled hard tasks first actually became more skilled and
Which choice completes the text with the most logical and precise word or phrase? productive workers over time.
A. pretending
Which choice completes the text with the most logical and precise word or phrase?
B. complaining
A. secretive
C. requiring
B. efficient
D. demonstrating
C. outgoing

D. unsympathetic
ID: a4f50d30 Answer
Correct Answer: D
ID: fa7a89f1 Answer
Rationale
Correct Answer: B
Choice D is the best answer. “Demonstrating” means “showing,” and the text describes how de Santana Rationale
disproved a previous belief in only one species of electric eel by showing that three electric eel species
actually exist. Choice B is the best answer because it most logically completes the text’s discussion about Kouchaki and
colleagues’ research into how workload affects productivity. In context, “efficient” means effective or well
Choice A is incorrect. Pretending that there are three species of electric eel might be a fun game for marine organized. The text indicates that, according to Kouchaki and colleagues’ research, people who worked on
biologists, but it wouldn’t prove the existence of more than one species. Choice B is incorrect. Complaining hard tasks first were “more skilled and productive” than those who did easy tasks first. This context conveys
won’t prove anything about eels, so de Santana could not have proved wrong the idea of only one species of the idea that despite their sense of accomplishment, the people who chose to do the easy tasks first were less
electric eel by complaining about it. Choice C is incorrect. “Requiring” means “needing.” It wouldn’t make sense efficient or productive than those who tackled hard tasks first.
to say that de Santana “needed” there to be three distinct species of electric eel.
Choice A is incorrect because there’s nothing in the text to suggest that workers who do easy tasks first are
Question Difficulty: Easy less “secretive,” or uncommunicative or silent, than those who do hard tasks first. Rather, the text suggests that
people are less skillful or efficient if they tackle easy tasks before the hard ones. Choice C is incorrect because
“outgoing” means openly friendly, which wouldn’t make sense in this context. The text focuses on Kouchaki
and colleagues’ research in which people who worked on hard tasks first were “more skilled and productive”
than those who did easy tasks first and were therefore less efficient. Choice D is incorrect because there’s
nothing in the text to suggest that workers who do easy tasks first are less “unsympathetic,” or insensitive or
unkind, than those who do hard tasks first. Rather, the text suggests that people are less skillful or efficient if
they tackle easy tasks before the hard ones.

Question Difficulty: Easy

Question ID e37b9e34 Question ID faa5696c


Assessment Test Domain Skill Difficulty Assessment Test Domain Skill Difficulty

SAT Reading and Writing Craft and Structure Words in Context SAT Reading and Writing Craft and Structure Words in Context

ID: e37b9e34 ID: faa5696c


Some researchers believe that the genes that enable groundhogs and certain other mammals to hibernate through the Arturo A. Schomburg was dedicated to preserving books, art, and other materials from peoples of African descent
winter by slowing their breathing and heart rates and lowering their body temperature may be ______ in humans: around the world. To get these items, Schomburg ______ friends and colleagues, whom he asked to bring back rare and
present yet having essentially no effect on our bodily processes. valuable objects from their international travels. Now, Schomburg’s collection is a valuable resource for scholars of
Black history and culture.
Which choice completes the text with the most logical and precise word or phrase?
Which choice completes the text with the most logical and precise word or phrase?
A. decisive
A. admired
B. lacking
B. disagreed with
C. variable
C. warned
D. dormant
D. depended on

ID: e37b9e34 Answer


Correct Answer: D
ID: faa5696c Answer
Correct Answer: D
Rationale
Rationale
Choice D is the best answer because it logically completes the text’s discussion about genes related to
hibernation. In this context, “dormant” means inactive. The text explains that the same genes that enable Choice D is the best answer because it most logically completes the text’s discussion of Arturo A. Schomburg’s
certain nonhuman mammal species to hibernate during the winter by altering their bodily processes are also collection. In this context, the phrase “depended on” means relied on. The text explains that Schomburg,
found in our species but have “essentially no effect” on humans’ bodily processes. In other words, these genes wanting to preserve “books, art, and other materials” created by peoples of African descent from around the
don’t function in humans. world, built a large collection of these objects. The text also states that in order to obtain these items,
Schomburg asked friends and colleagues who were traveling internationally to bring them back for him. This
Choice A is incorrect because in this context, “decisive” means has the power to affect the outcome of implies that he wouldn’t have been able to create his collection without help. Thus, the context supports the
something, but the text states that genes related to hibernation are instead inactive in humans—that is, the idea that Schomburg relied on, or depended on, friends and colleagues to build his collection.
genes don’t affect humans’ bodily processes, although they are present in their bodies. Choice B is incorrect
because in this context, “lacking” means missing, but the text states that the genes are present in humans, Choice A is incorrect because “admired” means respected and approved of. Although it’s reasonable to expect
though inactive. Choice C is incorrect because “variable” means characterized by the potential to change, but that Schomburg respected his friends and colleagues, the text doesn’t discuss any such aspect of their
the text indicates that these genes don’t change in their effect on humans’ bodily processes; instead, the genes relationships. The text focuses instead on what Schomburg asked his friends and colleagues to do to help
are consistently inactive in humans. build his collection. Choice B is incorrect because “disagreed with” means held an opposing belief, and nothing
in the text suggests that Schomburg and his friends or colleagues held opposing views on any subject. In fact,
Question Difficulty: Medium the text strongly implies that Schomburg’s friends and colleagues supported him in his goal of preserving
objects of importance for Black history and culture by bringing him such objects from around the world.
Choice C is incorrect because “warned” means informed someone in advance about something dangerous,
which wouldn’t make sense in this context. Nothing in the text suggests that Schomburg thought his friends
and colleagues were in any danger, let alone that he warned them of danger.

Question Difficulty: Easy


Text 2 that Focarelli and Panetta believe that the newspaper industry is different from any other industry

Question ID f0ae0da3 when it comes to the effects of mergers. Although their own research was about consumer banking, Text 2
suggests that they view their conclusions as applicable to mergers in general.

Assessment Test Domain Skill Difficulty Question Difficulty: Medium

SAT Reading and Writing Craft and Structure Cross-text


Connections

ID: f0ae0da3
Text 1
When companies in the same industry propose merging with one another, they often claim that the merger will benefit
consumers by increasing efficiency and therefore lowering prices. Economist Ying Fan investigated this notion in the
context of the United States newspaper market. She modeled a hypothetical merger of Minneapolis-area newspapers
and found that subscription prices would rise following a merger.
Text 2
Economists Dario Focarelli and Fabio Panetta have argued that research on the effect of mergers on prices has focused
excessively on short-term effects, which tend to be adverse for consumers. Using the case of consumer banking in Italy,
they show that over the long term (several years, in their study), the efficiency gains realized by merged companies do
result in economic benefits for consumers.

Based on the texts, how would Focarelli and Panetta (Text 2) most likely respond to Fan’s findings (Text 1)?

A. They would recommend that Fan compare the near-term effect of a merger on subscription prices in the
Minneapolis area with the effect of a merger in another newspaper market.

B. They would argue that over the long term the expenses incurred by the merged newspaper company will also
increase.

C. They would encourage Fan to investigate whether the projected effect on subscription prices persists over an
extended period.

D. They would claim that mergers have a different effect on consumer prices in the newspaper industry than in most
other industries.

ID: f0ae0da3 Answer


Correct Answer: C

Rationale

Choice C is the best answer because, based on the information presented in the texts, it represents how
Focarelli and Panetta would most likely respond to Fan’s findings. Text 1 indicates that Fan found that a
newspaper merger would result in a rise in subscription prices. This rise wouldn’t benefit customers, who
would have to pay more for news after a merger. Text 2 presents Focarelli and Panetta’s argument that merger
research tends to focus too much on what happens immediately after the merger. Text 2 goes on to describe
their finding that mergers can be economically beneficial for consumers over the long term. This suggests that
Focarelli and Panetta would encourage Fan to investigate the long-term effect of the hypothetical newspaper
merger on subscription prices.

Choice A is incorrect because Text 2 doesn’t indicate that Focarelli and Panetta connect the effects of mergers
to specific locations. Instead, Focarelli and Panetta focus on the length of time over which the effects of
mergers should be evaluated. Choice B is incorrect because Text 2 indicates that Focarelli and Panetta found
that merged companies experience "efficiency gains" over the long term, meaning that their expenses go down
relative to their output, not that their expenses increase. Choice D is incorrect because there’s no indication in

Question ID 54c6128b Question ID 2c50ed1a


Assessment Test Domain Skill Difficulty Assessment Test Domain Skill Difficulty

SAT Reading and Writing Craft and Structure Text Structure and SAT Reading and Writing Craft and Structure Cross-Text
Purpose Connections

ID: 54c6128b ID: 2c50ed1a


When ancient oak planks were unearthed during subway construction in Rome, Mauro Bernabei and his team Text 1
examined the growth rings in the wood to determine where these planks came from. By comparing the growth rings Literary scholars have struggled with the vastness of Nigerian writer Wole Soyinka’s collective works of drama
on the planks to records of similar rings in oaks from Europe, the team could trace the wood to the Jura region of (spanning over 20 plays in total). It is best, however, to understand Soyinka’s body of work as a dramatist
France, hundreds of kilometers from Rome. Because timber could only have been transported from distant Jura to chronologically. Soyinka’s progression as a playwright can be considered to fall into three periods, with each one
Rome by boat, the team’s findings suggest the complexity of Roman trade routes. representing a particular thematic and stylistic cohesion: the 1960s, the two decades between 1970 and 1990, and
lastly, from roughly 1990 onwards.
Which choice best describes the function of the underlined sentence in the text as a whole?

A. It presents a conclusion about Roman trade routes based on the team’s findings. Text 2
It is tempting to impose a linear sense of order on the expanse of Wole Soyinka’s body of work as a dramatist.
B. It questions how the team was able to conclude that the planks were used to build a boat. However, critics who have considered Soyinka’s plays to fit neatly into three phases overlook potential commonalities
in Soyinka’s work that span across these phases. Additionally, this view may discount significant differences in the
C. It explains why the planks were made from oak rather than a different kind of wood.
styles and content of plays written around the same time.
D. It describes common methods used in Roman subway construction.
Which choice best describes a difference in how the author of Text 1 and the author of Text 2 view the study of
Soyinka’s works of drama?
ID: 54c6128b Answer
A. While the author of Text 1 believes that thinking about Soyinka’s works of theater in phases is useful, the author of
Correct Answer: A Text 2 views such an approach as limiting.
Rationale
B. Although the author of Text 1 claims that Soyinka’s style as a dramatist has evolved over time, the author of Text 2
argues that Soyinka’s style has remained consistent throughout his career.
Choice A is the best answer because it most accurately describes how the underlined sentence functions in
the text as a whole. The first sentence explains that Bernabei and his team studied growth rings to obtain C. The author of Text 1 considers Soyinka’s plays to showcase his strongest writing, whereas the author of Text 2
information about the ancient oak planks found during a construction project in Rome. The next sentence believes that Soyinka’s poetry is where he is most skilled.
presents what the researchers learned: the wood from the planks came from France’s Jura region, which is far
from Rome. The underlined sentence then presents the implications of the findings about the planks: the wood D. The author of Text 1 argues that Soyinka’s early plays were his most politically charged, whereas the author of Text
must have been brought to Rome by boat, a difficult task that suggests Roman trade routes were complex. 2 claims that Soyinka’s most recent plays are the most politicized.
Thus, the underlined sentence mainly functions to present a conclusion about Roman trade routes based on
the team’s findings.
ID: 2c50ed1a Answer
Choice B is incorrect because the text doesn’t suggest that the team thought the ancient planks were used in Correct Answer: A
the construction of a boat, nor does the underlined sentence question that conclusion. Instead, the text states
Rationale
that the wood could only have been transported from Jura to Rome in a boat. Choice C is incorrect because the
underlined sentence simply offers a conclusion drawn from the team’s findings about the likely place of origin Choice A is the best answer. The author of Text 1 states that Soyinka’s progression as a playwright can be
of the ancient planks; the text never mentions why oak was chosen for the planks instead of other wood. considered to fall into three periods, implying that this is a helpful way to understand his works. The author of
Choice D is incorrect because neither the underlined sentence nor the text as a whole addresses any methods Text 2, on the other hand, challenges this view and says that it overlooks potential commonalities and
that Romans used in constructing subways. Instead, the underlined sentence offers a conclusion drawn from differences in Soyinka’s work across what Text 1 calls distinctive stylistic phases.
the team’s findings about the likely place of origin of the ancient wooden planks discovered.
Choice B is incorrect. This choice overstates the central claim of Text 2. The author of Text 2 argues against
Question Difficulty: Medium the chronological progression supported in Text 1, but does not go so far as to say that Soyinka’s style
remained consistent. In fact, Text 2 points out “significant differences in styles and content” among Soyinka’s
plays. Choice C is incorrect. Neither of the texts mention Soyinka’s poetry, nor do they rank his dramatic
writing relative to his other work. Choice D is incorrect. Neither text discusses the political aspects of
Soyinka’s plays, nor do they make any claims about whether they have changed over time.
Question Difficulty: Easy

Question ID 0ed94d4c
Assessment Test Domain Skill Difficulty

SAT Reading and Writing Craft and Structure Text Structure and
Purpose

ID: 0ed94d4c
Jackie Ormes’s Torchy Brown in Dixie to Harlem (1937–38) was the first comic strip by a Black woman to appear in a
widely read newspaper. The strip tells the story of Torchy, a young woman who leaves Mississippi to become a
performer in New York City. Torchy’s story reflects the experience of the Great Migration (1910–1970), when millions of
Black Americans left the South in search of opportunities in other parts of the United States. Torchy Brown thus shows
how Ormes used comics to comment humorously on issues affecting Black Americans, which she continued to do
throughout her career.

Which choice best states the main purpose of the text?

A. To show how Ormes’s Torchy Brown inspired other Black women to write comic strips in the 1930s

B. To illustrate how the subjects Ormes addressed in her comic strips changed over the course of her career

C. To give an example of how Ormes presented the experiences of Black Americans in her comic strips

D. To claim that several characters in Torchy Brown were based on people that Ormes knew personally

ID: 0ed94d4c Answer


Correct Answer: C

Rationale

Choice C is the best answer. The text describes how Torchy Brown depicted the experiences of a young Black
woman experiencing America’s Great Migration. It further states that Ormes continued to use comics
throughout her career to humorously comment on important issues impacting Black Americans.

Choice A is incorrect. The text never mentions other Black women comic strip writers. Choice B is incorrect.
The text never mentions any changes in the subjects Ormes addressed. Choice D is incorrect. The text never
mentions the inspiration for characters in Torchy Brown.

Question Difficulty: Easy

Question ID 79fe7550 Question ID 0a04cac5


Assessment Test Domain Skill Difficulty Assessment Test Domain Skill Difficulty

SAT Reading and Writing Craft and Structure Words in Context SAT Reading and Writing Craft and Structure Text Structure and
Purpose

ID: 79fe7550
ID: 0a04cac5
Researcher Haesung Jung led a 2020 study showing that individual acts of kindness can ______ prosocial behavior
across a larger group. Jung and her team found that bystanders who witness a helpful act become more likely to offer The following text is adapted from Jane Austen’s 1814 novel Mansfield Park. The speaker, Tom, is considering staging a
help to someone else, and in doing so, can inspire still others to act. play at home with a group of his friends and family.
We mean nothing but a little amusement among ourselves, just to vary the scene, and exercise our powers in
Which choice completes the text with the most logical and precise word or phrase? something new. We want no audience, no publicity. We may be trusted, I think, in choosing some play most perfectly
unexceptionable; and I can conceive no greater harm or danger to any of us in conversing in the elegant written
A. require
language of some respectable author than in chattering in words of our own.
B. remember
Which choice best states the main purpose of the text?
C. foster
A. To offer Tom’s assurance that the play will be inoffensive and involve only a small number of people
D. discourage
B. To clarify that the play will not be performed in the manner Tom had originally intended

C. To elaborate on the idea that the people around Tom lack the skills to successfully stage a play
ID: 79fe7550 Answer
Correct Answer: C D. To assert that Tom believes the group performing the play will be able to successfully promote it

Rationale
ID: 0a04cac5 Answer
Choice C is the best answer because it most logically completes the text’s discussion of Jung and her team’s
study of acts of kindness. In this context, “foster” means encourage or promote the development of. The text Correct Answer: A
indicates that Jung and her team found that seeing a helpful (or prosocial) act makes a bystander more likely Rationale
to help someone else, which can in turn inspire additional people to help others. That is, the team showed that
single acts of kindness can foster additional prosocial acts across a group. Choice A is the best answer because it most accurately portrays the main purpose of the text. At the beginning
of the text, Tom asserts that he and the other people staging the play are doing so only for “a little amusement
Choice A is incorrect because nothing in the text suggests that Jung and her team found that single acts of among ourselves” and aren’t interested in attracting an audience or any attention with the production. Then,
kindness “require,” or depend on or make obligatory, broader prosocial (or helpful) behavior across a group. Tom promises that the play they chose is modest and appropriate, and he further reasons that using the well-
There’s no suggestion in the text that individual acts of kindness can only occur if other prosocial acts have written prose of “some respectable author” is better than using their own words. Overall, the main purpose of
already occurred, and the text indicates only that an act of kindness can inspire additional helpful acts, not the text is to convey Tom’s promise that the play will be inoffensive and involve only a few people.
that it necessarily will do so. Choice B is incorrect because the text focuses on a possible direct effect of
individual acts of kindness, or single helpful actions, and it wouldn’t make sense to suggest that actions can Choice B is incorrect because the text doesn’t indicate that Tom had earlier intentions for the play’s
“remember,” or hold a memory of, something. Choice D is incorrect because the text doesn’t indicate that Jung performance or that anything has changed since the group first decided to stage a play. Instead, the text
and her team found that single acts of kindness can “discourage,” or hinder, prosocial (or helpful) behavior focuses on how harmless the entire endeavor will be. Choice C is incorrect. Although Tom mentions that using
across a group. On the contrary, the text states that Jung and her team found that seeing a helpful act makes a the words of a “respectable author” will be better than using their own words, he never addresses the idea that
bystander more likely to help someone else, which can in turn inspire even more people to help others. the people around him generally aren’t skilled enough to stage a play. Choice D is incorrect because in the text
Tom specifically says that they “want no audience, no publicity,” which indicates that they don’t plan on
Question Difficulty: Hard promoting the play at all.

Question Difficulty: Hard


Question ID 1fa751f1 Question ID 49bbe4d7
Assessment Test Domain Skill Difficulty Assessment Test Domain Skill Difficulty

SAT Reading and Writing Craft and Structure Words in Context SAT Reading and Writing Craft and Structure Words in Context

ID: 1fa751f1 ID: 49bbe4d7


Handedness, a preferential use of either the right or left hand, typically is easy to observe in humans. Because this trait For painter Jacob Lawrence, being ______ was an important part of the artistic process. Because he paid close attention
is present but less ______ in many other animals, animal-behavior researchers often employ tasks specially designed to to all the details of his Harlem neighborhood, Lawrence’s artwork captured nuances in the beauty and vitality of the
reveal individual animals’ preferences for a certain hand or paw. Black experience during the Harlem Renaissance and the Great Migration.

Which choice completes the text with the most logical and precise word or phrase? Which choice completes the text with the most logical and precise word or phrase?

A. recognizable A. skeptical

B. intriguing B. observant

C. significant C. critical

D. useful D. confident

ID: 1fa751f1 Answer ID: 49bbe4d7 Answer


Correct Answer: A Correct Answer: B

Rationale Rationale

Choice A is the best answer because it most logically completes the text’s discussion about handedness in Choice B is the best answer because it most logically completes the text’s discussion of Jacob Lawrence’s
animals. As used in this context, “recognizable” means apparent or identifiable. The text indicates that artistic process. In this context, “observant” means watchful and perceptive. The text emphasizes that the
handedness is “easy to observe in humans,” but that animal-behavior researchers use special tasks to “close attention” Lawrence paid to “all the details” of his neighborhood allowed him to reflect subtle elements
determine handedness in other animals. This context and the use of “less” before the blank indicate that of “the beauty and vitality of the Black experience” in his artwork. This context indicates that being observant
compared with handedness in humans, handedness in other animals is less recognizable. of his surroundings was an important part of Lawrence’s work as an artist.

Choice B is incorrect because there’s nothing in the text to suggest that handedness is less “intriguing,” or Choice A is incorrect because the text gives no indication that Lawrence was “skeptical,” or had an attitude of
fascinating, in nonhuman animals than it is in humans. The text focuses on how easy it is to observe doubt in general or about particular things, let alone that skepticism was important to him as an artist. Rather
handedness in humans as compared with other animals; the text doesn’t suggest that handedness is more than indicating that he was skeptical, the text focuses on how Lawrence paid careful attention to everything
fascinating in humans. Choice C is incorrect because there’s nothing in the text to suggest that handedness is around him and reflected his observations in his artwork. Choice C is incorrect because the text gives no
less “significant,” or important or meaningful, in nonhuman animals than it is in humans. The text focuses on indication that Lawrence was “critical,” which in this context would mean inclined to criticize harshly or
how easy it is to observe handedness in humans as compared with other animals; the text doesn’t suggest that unfairly. Rather than indicating that Lawrence found fault in things, the text suggests that he paid careful
handedness is more significant in humans. Choice D is incorrect because “useful,” or functional or helpful, attention to everything around him and that his artwork reflects this careful attention. Choice D is incorrect
wouldn’t make sense in context. The text focuses on the ease with which researchers can determine whether because the text doesn’t suggest that Lawrence was “confident,” or self-assured. Rather than addressing how
an animal or person is right- or left-handed, not on how useful handedness in nonhuman animals is compared Lawrence felt about himself and how that feeling affected his artistic process, the text emphasizes the careful
with handedness in humans. attention Lawrence paid to everything around him—attention that allowed him to capture subtle elements of a
particular place and time in his artwork.
Question Difficulty: Medium
Question Difficulty: Easy

Question ID f6d1f735 Question ID fcc328c6


Assessment Test Domain Skill Difficulty Assessment Test Domain Skill Difficulty

SAT Reading and Writing Craft and Structure Words in Context SAT Reading and Writing Craft and Structure Text Structure and
Purpose

ID: f6d1f735
ID: fcc328c6
Researchers have struggled to pinpoint specific causes for hiccups, which happen when a person’s diaphragm contracts
______. However, neuroscientist Kimberley Whitehead has found that these uncontrollable contractions may play an Streams and rivers carry soil and rocks from one location to another. But there is another way for these geological
important role in helping infants regulate their breathing. materials to move. Scientists call this process “aeolian transport.” In aeolian transport, winds move small particles of
soil or rock over potentially great distances. Geologist Melisa Diaz and her team studied dust in Antarctica to find out if
Which choice completes the text with the most logical and precise word or phrase? it was moved by aeolian transport. They discovered that the dust matched geological material in Australia. Aeolian
transport had carried it from one continent to another, across thousands of miles of open ocean.
A. involuntarily

B. beneficially Which choice best describes the function of the underlined portion in the text as a whole?

A. It presents Melisa Diaz’s remarks about difficulties that her team encountered.
C. strenuously
B. It introduces a scientific term that is used in the discussion that follows.
D. smoothly
C. It emphasizes the surprising nature of the findings that are presented.

ID: f6d1f735 Answer D. It explains the difference between two kinds of geological material.
Correct Answer: A

Rationale ID: fcc328c6 Answer

Choice A is the best answer because it most logically completes the text’s discussion of diaphragm Correct Answer: B
contractions and hiccups. In this context, “involuntarily” means done without any control, or by reflex. The Rationale
text explains that when a person’s diaphragm repeatedly contracts and results in hiccups (which may be
beneficial for infants), those muscle contractions are “uncontrollable.” This context indicates that the Choice B is the best answer because it most accurately describes the function of the underlined portion in the
diaphragm contractions occur without the person’s control. text as a whole. The first two sentences introduce the idea that soil and rocks can be moved from one location
to another by something other than rivers and streams. The underlined sentence then states that the term
Choice B is incorrect because it wouldn’t support the logical relationship established in the text’s discussion of used by scientists to refer to this process is “aeolian transport.” The discussion that follows explains what
diaphragm contractions and hiccups. The text indicates that although specific causes for hiccups haven’t aeolian transport is (the movement of small geological materials over potentially great distances by the wind)
been identified, it may be the case that the muscle contractions that occur have an important purpose in and describes an example of a study that found that dust particles had been moved by aeolian transport from
infants. It wouldn’t make sense to say that even though the contractions occur “beneficially,” or with a good or Australia to Antarctica. Thus, the underlined portion introduces a scientific term that is used in the discussion
helpful effect, they might play a positive role in infants’ breathing regulation. Choice C is incorrect because the that follows.
text indicates that the diaphragm contractions that result in hiccups are “uncontrollable.” Because those
muscle contractions are described as happening automatically and without the person’s control, it wouldn’t Choice A is incorrect because the underlined portion of the text doesn’t present any remarks from Melisa Diaz,
make sense to describe them as occurring “strenuously,” or in a way that requires great effort or energy. Choice nor does it suggest that Diaz and her team encountered difficulties in their study. Instead, the phrase in
D is incorrect because the text doesn’t describe the quality of the diaphragm contractions that result in quotation marks in the underlined portion simply presents a term that is used by scientists to refer to a
hiccups beyond stating that they are “uncontrollable.” Nothing in the text indicates that those muscle specific process discussed in the text. Choice C is incorrect because nothing in either the underlined portion
contractions occur “smoothly,” or evenly and continuously. or the text as a whole suggests that the findings of the study presented in the text were surprising or
unexpected. In fact, the text suggests that Diaz and her team wanted to see if aeolian transport could explain
Question Difficulty: Easy the appearance of certain geological materials in Antarctica, and their findings did indeed confirm the
involvement of aeolian transport. Choice D is incorrect. Although the first sentence of the text mentions soil
and rocks, which are two different kinds of geological material, the underlined portion doesn’t refer to these
materials, nor does it explain the difference between them. Rather, the underlined portion introduces the
scientific term “aeolian transport,” which is used in the discussion that follows.

Question Difficulty: Easy


Question ID 84a7fbca Question ID aa7ae735
Assessment Test Domain Skill Difficulty Assessment Test Domain Skill Difficulty

SAT Reading and Writing Craft and Structure Words in Context SAT Reading and Writing Craft and Structure Words in Context

ID: 84a7fbca ID: aa7ae735


When Mexican-American archaeologist Zelia Maria Magdalena Nuttall published her 1886 research paper on The following text is adapted from Mohsin Hamid’s 2017 novel Exit West. Saeed lives with his mother and father.
sculptures found at the ancient Indigenous city of Teotihuacan in present-day Mexico, other researchers readily ______ On cloudless nights after a daytime rain, Saeed’s father would sometimes bring out the telescope, and the family
her work as groundbreaking; this recognition stemmed from her convincing demonstration that the sculptures were would sip green tea on their balcony, enjoying a breeze, and take turns to look up at objects whose light, often, had
much older than had previously been thought. been emitted before any of these three viewers had been born—light from other centuries, only now reaching Earth.
©2017 by Mohsin Hamid
Which choice completes the text with the most logical and precise word or phrase?
As used in the text, what does the word “reaching” most nearly mean?
A. acknowledged
A. Arriving at
B. ensured
B. Consulting with
C. denied
C. Running to
D. underestimated
D. Clinging to

ID: 84a7fbca Answer


Correct Answer: A
ID: aa7ae735 Answer
Correct Answer: A
Rationale
Rationale
Choice A is the best answer because it most logically completes the text’s discussion of Nuttall’s 1886 research
paper. In this context, “acknowledged” means recognized as having a certain status. The text indicates that Choice A is the best answer. The word "reaching" in this text means "to arrive" at Earth. Before, the light had
other researchers recognized Nuttall’s work as groundbreaking because of its “convincing demonstration” been traveling across space; now, it had arrived at Earth.
related to the age of the ancient sculptures. In other words, the researchers recognized the groundbreaking
status of Nuttall’s work. Choice B is incorrect. "Consulting with" means "seeking advice or information from someone." Light is not
alive, so it can’t consult with Earth. Choice C is incorrect. It’s confusing to say that starlight is "only now
Choice B is incorrect because in this context, “ensured” would mean to have guaranteed or made sure running to Earth," both because light doesn’t literally "run" and because the text is describing the moment the
something was the case. The text states that other researchers gave Nuttall’s work recognition after it was light touches Earth, not the period of time when it’s traveling to Earth. Choice D is incorrect. This might be
published, but there’s no indication that they contributed to the work or had any involvement that would have tempting, as "clinging to" has a connotation of "sticking to." It wouldn’t make sense to say that the light was
allowed them to make sure the work would be groundbreaking. Choice C is incorrect because the text doesn’t "only now" clinging to Earth.
suggest that other researchers “denied,” or refused to admit or accept, that Nuttall’s work was groundbreaking;
on the contrary, it indicates that researchers praised the work, recognizing it as groundbreaking due to its Question Difficulty: Easy
“convincing demonstration” related to the age of the ancient sculptures. Choice D is incorrect because the text
doesn’t suggest that other researchers “underestimated,” or undervalued, Nuttall’s work; on the contrary, it
indicates that researchers praised the work, recognizing it as groundbreaking due to its “convincing
demonstration” related to the age of the ancient sculptures.

Question Difficulty: Easy

Question ID 428cd2c1 Question ID 62a18353


Assessment Test Domain Skill Difficulty Assessment Test Domain Skill Difficulty

SAT Reading and Writing Craft and Structure Words in Context SAT Reading and Writing Craft and Structure Words in Context

ID: 428cd2c1 ID: 62a18353


Some people have speculated that two helmets with attached horns discovered in Denmark in 1942 belonged to The following text is adapted from Zora Neale Hurston’s 1921 short story “John Redding Goes to Sea.” John wants to
Vikings, but scholars have long been skeptical. Archaeologist Helle Vandkilde and colleagues recently provided travel far beyond the village where he lives near his mother, Matty.
radiocarbon dates for the helmets, and their findings ______ scholars’ skepticism: the helmets date to the Nordic Bronze
[John] had on several occasions attempted to reconcile his mother to the notion, but found it a difficult task.
Age, centuries before the Vikings existed.
Matty always took refuge in self-pity and tears. Her son’s desires were incomprehensible to her, that was all.
Which choice completes the text with the most logical and precise word or phrase?
As used in the text, what does the phrase “reconcile his mother to” most nearly mean?
A. anticipate
A. Get his mother to accept
B. inspect
B. Get his mother to apologize for
C. reveal
C. Get his mother to match
D. justify
D. Get his mother to reunite with

ID: 428cd2c1 Answer


ID: 62a18353 Answer
Correct Answer: D
Correct Answer: A
Rationale
Rationale
Choice D is the best answer because it most logically completes the text’s discussion of the helmets found in
Choice A is the best answer. The expression “reconcile to” means “to cause (a person) to accept something
Denmark. In this context, “justify” means confirm or give reasons for. The text indicates that scholars have
difficult or disagreeable.” The text suggests that John wants his mother to accept his desire to travel, even
long been skeptical about the supposed Viking origin of two helmets found in Denmark. The radiocarbon
though she doesn’t like that idea.
dating of the helmets conducted by Vandkilde and colleagues demonstrates that the helmets date from the
Nordic Bronze Age, making the helmets too old to have belonged to Vikings. This context supports the idea Choice B is incorrect. This doesn’t make sense. John doesn’t want his mother to apologize for his own desire to
that the scholars’ skepticism is justified. travel—he wants her to accept his desire to travel. Choice C is incorrect. The text doesn’t suggest that John
wants his mother to match his desire to travel. Rather, he wants her to accept his desire to travel even though
Choice A is incorrect because “anticipate” means expect or come before, neither of which would make sense in
she doesn’t like it. Choice D is incorrect. This is tempting, because it seems to pick up on the idea of people
context. The text indicates that scholars have long been skeptical of the idea that the helmets belonged to
“reconciling” after a fight, but it actually doesn’t make sense. The text never suggests that John’s mother was
Vikings. Because the skepticism has existed for a long time, Vandkilde and colleagues’ research couldn’t be
“united with” the idea of him traveling in the past—if anything, it seems like she’s always been against it.
said to anticipate it. Instead, the radiocarbon dating results justify or confirm the scholar’s skepticism. Choice
Besides, it would be strange to say that a person “reunites with” a notion.
B is incorrect because “inspect” means examine or review, which wouldn’t make sense in this context.
Research findings are inanimate and therefore unable to inspect scholars’ skepticism. The text focuses on the Question Difficulty: Medium
origin of two helmets, which some people believe belonged to Vikings. Vandkilde and colleagues found that
the helmets are from the Nordic Bronze Age and therefore much older than the Vikings. This context suggests
that the researchers’ findings justify, not inspect, the scholars’ skepticism. Choice C is incorrect because
“reveal” means uncover or report, which wouldn’t make sense in context. The text focuses on the origin of two
helmets, which some people believe belonged to Vikings. Vandkilde and colleagues tested the helmets and
found that they date to the Nordic Bronze Age and therefore are much older than the Vikings. This context
suggests that the researchers’ findings confirm or justify the scholars’ skepticism, which was already known
so didn’t need to be revealed.

Question Difficulty: Easy


Question ID 4eee64fa Question ID c2c26e20
Assessment Test Domain Skill Difficulty Assessment Test Domain Skill Difficulty

SAT Reading and Writing Craft and Structure Text Structure and SAT Reading and Writing Craft and Structure Words in Context
Purpose

ID: c2c26e20
ID: 4eee64fa
The following text is adapted from Sadakichi Hartmann’s 1894 short story “Magnolia Blossoms.” The narrator is
Space scientists Anna-Lisa Paul, Stephen M. Elardo, and Robert Ferl planted seeds of Arabidopsis thaliana in samples of standing on the deck of a boat.
lunar regolith—the surface material of the Moon—and, serving as a control group, in terrestrial soil. They found that
while all the seeds germinated, the roots of the regolith-grown plants were stunted compared with those in the control What a night it was! My soul had left its body to lose itself in the wild unrestrained beauty around me—from
group. Moreover, unlike the plants in the control group, the regolith-grown plants exhibited red pigmentation, where it came—and only left a trembling suggestion of its existence within me. The other passengers moved
reduced leaf size, and inhibited growth rates—indicators of stress that were corroborated by postharvest molecular around me like shadows, and again and again my eyes drank in all the glory and wealth of that night.
analysis.
As used in the text, what does the word “suggestion” most nearly mean?
Which choice best states the main purpose of the text? A. Trace
A. It describes an experiment that addressed an unresolved question about the extent to which lunar regolith
B. Opinion
resembles terrestrial soils.
C. Dispute
B. It compares two distinct methods of assessing indicators of stress in plants grown in a simulated lunar
environment. D. Command

C. It presents evidence in support of the hypothesis that seed germination in lunar habitats is an unattainable goal.
ID: c2c26e20 Answer
D. It discusses the findings of a study that evaluated the effects of exposing a plant species to lunar soil conditions.
Correct Answer: A

Rationale
ID: 4eee64fa Answer
Correct Answer: D Choice A is the best answer because as used in the text, “suggestion” most nearly means trace. The text
portrays the narrator standing on the deck of a boat, admiring the view of nature afforded by this position: “My
Rationale
soul had left its body to lose itself in the wild unrestrained beauty around me,” says the narrator, “and only left
Choice D is the best answer. The text describes an experiment wherein space scientists compared plant a trembling suggestion of its existence within me.” This intense response to beauty is such that the narrator’s
growth in terrestrial and lunar soil conditions. It then discusses the findings of the study, including the fact soul seems to disengage from its body, leaving behind only a barely detectible indication of its presence there.
that all the seeds germinated but that the plants grown in lunar soil exhibited signs of stress. In other words, the narrator senses only a trace of soul left in the body.

Choice A is incorrect. The text doesn’t address this question, and never describes any specific characteristics Choice B is incorrect. Although in some contexts “suggestion” can refer to an implied or indirectly expressed
of either soil. It merely describes the outcome of an experiment that exposed a plant species to lunar soil opinion, the text doesn’t portray the narrator expressing an opinion; instead, the narrator is explaining an
conditions. Choice B is incorrect. The text never compares methods of assessing indicators of stress—instead, experience of intense emotion. Choice C is incorrect. While “suggestion” might be used in some contexts to
it simply mentions several stress indicators observed in the study (red pigmentation, reduced leaf size, and refer to the tactful expression of a differing viewpoint, it doesn’t refer to the dispute or difference of opinion
inhibited growth rates). Choice C is incorrect. The text doesn’t present any evidence that we could never itself. Moreover, the text doesn’t portray a dispute between characters with differing viewpoints. Choice D is
achieve seed germination in lunar habitats, and in fact states that the seeds in the lunar soil did germinate. incorrect. Although in some contexts, “suggestion” might be used to refer to a politely worded command, the
text doesn’t portray a scenario in which someone receives such a command.
Question Difficulty: Hard
Question Difficulty: Easy

Question ID a70cbc53 Question ID 3d658a5a


Assessment Test Domain Skill Difficulty Assessment Test Domain Skill Difficulty

SAT Reading and Writing Craft and Structure Text Structure and SAT Reading and Writing Craft and Structure Words in Context
Purpose

ID: 3d658a5a
ID: a70cbc53
Some foraging models predict that the distance bees travel when foraging will decline as floral density increases, but
Raymond Antrobus, an accomplished poet and writer of prose, recently released his debut spoken word poetry album, biologists Shalene Jha and Claire Kremen showed that bees’ behavior is inconsistent with this prediction if flowers in
The First Time I Wore Hearing Aids, in collaboration with producer Ian Brennan. The album contains both dense patches are ______: bees will forage beyond patches of low species richness to acquire multiple resource types.
autobiographical and reflective pieces combining Antrobus’s spoken words with Brennan’s fragmented audio elements
and pieces of music to convey how people who are deaf may experience sound, both its presence and absence. Some Which choice completes the text with the most logical and precise word or phrase?
critics suggest that the album questions the function of sound in the world, highlighting that the experience of sound
A. depleted
is multifaceted.
B. homogeneous
Which choice best describes the overall structure of the text?
C. immature
A. It introduces a collaborative spoken word poetry project, details the approach taken to produce the work, and then
provides an example of critique the album received upon release. D. dispersed

B. It mentions a collection of spoken word poems, distinguishes one poem as being an exemplar on the album, and
then offers a summary of the subject matter of the whole collection. ID: 3d658a5a Answer
C. It summarizes the efforts to produce a collection of spoken word poems, presents biographies of two people who Correct Answer: B
worked on the album, and speculates about the meaning behind the poetry. Rationale

D. It connects two artists to the same spoken word poetry project, explains the extent of their collaboration on each
Choice B is the best answer because it most logically completes the text’s discussion of Jha and Kremen’s
poem, and then provides an overview of the technique used to produce the work.
finding about bees’ foraging behavior. In this context, “homogeneous” means uniform or of the same kind. The
text indicates that some models predict that the distance that bees travel when they’re foraging declines as
ID: a70cbc53 Answer the density of flowers increases. The text goes on to say, however, that Jha and Kremen identified a
circumstance in which bees don’t behave this way. Specifically, if bees encounter “patches of low species
Correct Answer: A
richness”—that is, patches in which the flowers are largely from the same species—they’ll travel beyond those
Rationale patches to get varied food resources. This context thus suggests that bees don’t behave as some models
predict if the dense patches of flowers the bees encounter are homogeneous.
Choice A is the best answer. The text first introduces the album as being a collaboration between Antrobus and
Brennan, then describes the approach taken to produce it, then mentions how critics have said that it calls Choice A is incorrect because the text indicates that Jha and Kremen found that bees will behave differently
into question the function of sound. than some models predict if the bees encounter flower patches that are not rich in species, not if the flowers
are “depleted,” or emptied or reduced in quality or quantity. Although it could be true that bees are likely to
Choice B is incorrect. While the text does mention a collection of spoken word poems, it doesn’t single out one leave depleted patches in search of more resources, the text doesn’t indicate that Jha and Kremen investigated
poem as being particularly noteworthy. Additionally, the text doesn’t simply summarize the subject matter—it that possibility. Choice C is incorrect because there’s no information in the text suggesting that bees will not
goes into detail about the content and production of the album. Choice C is incorrect. The text doesn’t provide behave as some models predict if flowers in patches are “immature,” or not fully developed. Instead, the text
biographical information about the two artists, and the text doesn’t speculate about the meaning behind the indicates that Jha and Kremen found that bees will behave contrary to some models’ predictions if the flower
poetry—instead, it relays what some critics have said about the album. Choice D is incorrect. The text doesn’t patches are not rich in species. Choice D is incorrect because the text indicates that bees’ behavior will be
provide just an overview of the production techniques used but instead goes into more detail about the content inconsistent with the predictions of some models if the flower patches that the bees encounter are of low
and audio elements of the album, as well as critical response to the album. species richness, not if the flowers are in patches that are “dispersed,” or widely scattered. Although the text
does describe bees as leaving patches that are not rich in species to forage elsewhere, there’s no suggestion
Question Difficulty: Hard
that Jha and Kremen found that the distance between dense flower patches affects whether the bees behave
as some models predict.

Question Difficulty: Hard


accurately describe the difference. Text 1 actually says that Tuchman "fails to address" the influence of events

Question ID 17bf10de in Eastern Europe, while Text 2 says that Tuchman’s thesis was that European powers (not Eastern European
leaders) were committed to military plans.

Assessment Test Domain Skill Difficulty Question Difficulty: Hard

SAT Reading and Writing Craft and Structure Cross-Text


Connections

ID: 17bf10de
Text 1
Despite its beautiful prose, The Guns of August, Barbara Tuchman’s 1962 analysis of the start of World War I, has certain
weaknesses as a work of history. It fails to address events in Eastern Europe just before the outbreak of hostilities,
thereby giving the impression that Germany was the war’s principal instigator. Had Tuchman consulted secondary
works available to her by scholars such as Luigi Albertini, she would not have neglected the influence of events in
Eastern Europe on Germany’s actions.

Text 2
Barbara Tuchman’s The Guns of August is an engrossing if dated introduction to World War I. Tuchman’s analysis of
primary documents is laudable, but her main thesis that European powers committed themselves to a catastrophic
outcome by refusing to deviate from military plans developed prior to the conflict is implausibly reductive.

Which choice best describes a difference in how the authors of Text 1 and Text 2 view Barbara Tuchman’s The Guns of
August?

A. The author of Text 1 argues that Tuchman should have relied more on the work of other historians, while the author
of Text 2 implies that Tuchman’s most interesting claims result from her original research.

B. The author of Text 1 believes that the scope of Tuchman’s research led her to an incorrect interpretation, while the
author of Text 2 believes that Tuchman’s central argument is overly simplistic.

C. The author of Text 1 asserts that the writing style of The Guns of August makes it worthwhile to read despite any
perceived deficiency in Tuchman’s research, while the author of Text 2 focuses exclusively on the weakness of
Tuchman’s interpretation of events.

D. The author of Text 1 claims that Tuchman would agree that World War I was largely due to events in Eastern
Europe, while the author of Text 2 maintains that Tuchman would say that Eastern European leaders were not
committed to military plans in the same way that other leaders were.

ID: 17bf10de Answer


Correct Answer: B

Rationale

Choice B is the best answer. Both texts are critical of The Guns of August, but for different reasons: the author
of Text 1 argues that Tuchman missed an important factor leading up to the war because she didn’t consult
secondary sources, and the author of Text 2 argues that Tuchman’s main thesis is "reductive," which is a close
synonym for "overly simplistic."

Choice A is incorrect. This doesn’t accurately describe the difference. This choice’s summary of Text 1 is
accurate, but Text 2 never says that Tuchman’s most interesting claims result from her original research.
Choice C is incorrect. This doesn’t accurately describe the difference. Text 1 never says that The Guns of
August is worthwhile to read despite its research weaknesses. Text 2 does call out a weakness of Tuchman’s
interpretation of events, but it also praises her analysis of primary sources. Choice D is incorrect. This doesn’t

Question ID cd2ce51f Question ID b4c6cff6


Assessment Test Domain Skill Difficulty Assessment Test Domain Skill Difficulty

SAT Reading and Writing Craft and Structure Words in Context SAT Reading and Writing Craft and Structure Words in Context

ID: cd2ce51f ID: b4c6cff6


Like the 1945 play it reimagines—Federico García Lorca’s The House of Bernarda Alba—Marcus Gardley’s 2014 play The The following text is adapted from Karel Čapek’s 1920 play R.U.R. (Rossum’s Universal Robots), translated by Paul Selver
House That Will Not Stand prominently features women. In both plays, the all-female cast ______ an array of female and Nigel Playfair in 1923. Fabry and Busman are telling Miss Glory why their company manufactures robots.
characters, including a strong mother and several daughters dealing with individual struggles. FABRY: One Robot can replace two and a half workmen. The human machine, Miss Glory, was terribly imperfect. It
had to be removed sooner or later.
Which choice completes the text with the most logical and precise word or phrase? BUSMAN: It was too expensive.
A. engulfs FABRY: It was not effective. It no longer answers the requirements of modern engineering. Nature has no idea of
keeping pace with modern labor.
B. encourages
As used in the text, what does the word “answers” most nearly mean?
C. comprises
A. Explains
D. provokes
B. Rebuts

C. Defends
ID: cd2ce51f Answer
Correct Answer: C D. Fulfills

Rationale
ID: b4c6cff6 Answer
Choice C is the best answer because it most logically completes the text’s discussion of Gardley’s play. In this
context, “comprises” means constitutes or makes up the totality of, and the text indicates that The House That Correct Answer: D
Will Not Stand had an “all-female cast” that stands in some relationship to “an array of female characters” in Rationale
the play. Because all cast members are female, the characters must be played by these female cast members;
therefore the cast constitutes, or comprises, the collection of characters. Choice D is the best answer because as used in the text, “answers” most nearly means fulfills. In the text, Fabry
and Busman claim that the robots manufactured by their company are more efficient than human workers,
Choice A is incorrect. In this context, “engulfs” would mean encloses or overwhelms, and although it is fairly which they refer to as “the human machine.” Fabry observes that the human machine “no longer answers the
common to describe an actor as embodying (or personifying realistically) a character, there is nothing in the requirements of modern engineering.” That is, human workers are incapable of meeting the rigorous needs of
text to suggest that the cast members enclosed or overwhelmed the characters they played. Choice B is modern, industrialized workplaces.
incorrect because in this context, “encourages” would mean inspires with courage or hope. Although the text
does mention “a strong mother and several daughters dealing with individual struggles,” which might suggest Choice A is incorrect. Although in some contexts “answers” can mean explains, it doesn’t have that meaning
that there are moments of encouragement among the characters during the play, there is nothing to suggest in this context because the topic under discussion is human beings’ inability to perform labor efficiently, not
that the cast members encouraged the characters they portrayed. Choice D is incorrect because, in this their inability to engage in discussion or explanation. Choice B is incorrect. Although in some contexts
context, “provokes” would mean instigates or incites to anger. Nothing in the text addresses provocation or “answers” can mean rebuts, or proves a claim or argument to be false, it wouldn’t make sense to speak of
what it might mean for actors to provoke the characters they are playing. proving requirements to be false; requirements might or might not be reasonable, but they can’t be verified as
truthful or untruthful, as claims or accusations can. Choice C is incorrect. Although in some contexts,
Question Difficulty: Medium “answers” can mean defends against criticism, or justifies, it doesn’t have that meaning in this context
because the opinion that Fabry expresses is that human workers can no longer fulfill the requirements of
modern workplaces, not that they have ceased to justify those requirements or to defend them against
criticism; indeed, there is no suggestion in the text that workers ever defended those requirements.

Question Difficulty: Medium


Question ID d8d1ecaa Question ID d0198544
Assessment Test Domain Skill Difficulty Assessment Test Domain Skill Difficulty

SAT Reading and Writing Craft and Structure Words in Context SAT Reading and Writing Craft and Structure Cross-Text
Connections

ID: d8d1ecaa
ID: d0198544
Business researcher Melanie Brucks and colleagues found that remote video conference meetings may be less
conducive to brainstorming than in-person meetings are. The researchers suspect that video meeting participants are
focused on staring at the speaker on the screen and don’t allow their eyes or mind to wander as much, which may Text 1
ultimately ______ creativity. In 2007, a team led by Alice Storey analyzed a chicken bone found in El Arenal, Chile, dating it to 1321–1407 CE—over
a century before Europeans invaded the region, bringing their own chickens. Storey also found that the El Arenal
Which choice completes the text with the most logical and precise word or phrase? chicken shared a unique genetic mutation with the ancient chicken breeds of the Polynesian Islands in the Pacific. Thus,
Polynesian peoples, not later Europeans, probably first introduced chickens to South America.
A. recommend

B. criticize Text 2
An Australian research team weakened the case for a Polynesian origin for the El Arenal chicken by confirming that the
C. impede
mutation identified by Storey has occurred in breeds from around the world. More recently, though, a team led by
D. construct Agusto Luzuriaga-Neira found that South American chicken breeds and Polynesian breeds share other genetic markers
that European breeds lack. Thus, the preponderance of evidence now favors a Polynesian origin.

ID: d8d1ecaa Answer Based on the texts, how would the author of Text 2 most likely respond to the underlined claim in Text 1?

Correct Answer: C A. By broadly agreeing with the claim but objecting that the timeline it presupposes conflicts with the findings of the
genetic analysis conducted by Storey’s team
Rationale
B. By faulting the claim for implying that domestic animals couldn’t have been transferred from South America to the
Choice C is the best answer. The first sentence tells us that video meetings are “less conducive to” (meaning
Polynesian Islands as well
less good for) brainstorming. This suggests that the video meeting participants’ focus is bad for their
creativity. “Impede” means “delay” or “prevent,” which works perfectly in this context. C. By critiquing the claim for being based on an assumption that before the European invasion of South America, the
chickens of Europe were genetically uniform
Choice A is incorrect. This choice is too positive to fit the context. The first sentence tells us that video
meetings are “less conducive to” (meaning less good for) brainstorming. This suggests that the video meeting D. By noting that while the claim is persuasive, the findings of Luzuriaga-Neira’s team provide stronger evidence for it
participants’ focus is bad for their creativity. Choice B is incorrect. This choice doesn’t make sense. The than the findings of the genetic analysis conducted by Storey do
participants’ intense focus on the screen is the subject of the missing verb. It wouldn’t make sense to say that
their over-focusing “criticizes” their creativity. Choice D is incorrect. “Construct” means “build” or “make,”
ID: d0198544 Answer
which is too positive to fit the context. The first sentence tells us that video meetings are “less conducive to”
(meaning less good for) brainstorming. This suggests that the video meeting participants’ focus is bad for their Correct Answer: D
creativity. Rationale

Question Difficulty: Medium Choice D is the best answer because it accurately describes how the author of Text 2 would most likely
respond to the underlined claim in Text 1. Text 1 indicates that Storey found a genetic mutation in South
American chickens from before the European invasion and in Polynesian chickens, which implies that
chickens were first brought to South America by Polynesian people. Text 2 explains that the genetic mutation
Storey found is in chickens from all over the world, thus undercutting the mutation as evidence of a
Polynesian origin. However, Text 2 goes on to say “[m]ore recently” Luzuriaga-Neira and colleagues found
multiple genetic markers shared by South American and Polynesian chickens but “that European breeds lack,”
which strongly suggests a Polynesian origin for the South American chickens. This indicates that the author
of Text 2 believes Luzuriaga-Neira’s evidence for a Polynesian origin is compelling while Storey’s evidence has
been undermined. Thus, the author of Text 2 would most likely agree with the underlined statement and
believes Luzuriaga-Neira and colleagues’ evidence for the statement is stronger than Storey’s evidence is.

Choice A is incorrect because both texts indicate that chickens were introduced to South America before the
arrival of Europeans. Text 1 states that the El Arenal chicken bone dates from “1321–1407 CE—over a century
before Europeans invaded the region” and concludes that these chickens were likely brought to South America
Question ID a2be625e
by Polynesians. While Text 2 is not as explicit about the time period as Text 1 is, nothing in Text 2 undermines
the timing of events ascribed to Storey’s account in Text 1. Choice B is incorrect because both texts agree that Assessment Test Domain Skill Difficulty
chickens were first brought to South America by Polynesian peoples (the underlined claim), and nothing in
Text 2 suggests that this claim is in any way deficient because the possibility that animals could have been SAT Reading and Writing Craft and Structure Text Structure and
Purpose
transferred from South America to Polynesia was not explicitly addressed. Choice C is incorrect because the
criticism that Text 2 raises about the ideas in Text 1 is specifically about whether the single genetic mutation
cited by Storey in fact supports the idea of a Polynesian origin for South American chickens. There is nothing ID: a2be625e
in Text 2 to suggest that the underlined sentence (Storey’s conclusion) is deficient because it is based on an
The following text is from Sarah Orne Jewett’s 1899 short story “Martha’s Lady.” Martha is employed by Miss Pyne as a
assumption about the genetic uniformity of European chickens.
maid.
Miss Pyne sat by the window watching, in her best dress, looking stately and calm; she seldom went out now, and it
Question Difficulty: Hard
was almost time for the carriage. Martha was just coming in from the garden with the strawberries, and with more
flowers in her apron. It was a bright cool evening in June, the golden robins sang in the elms, and the sun was going
down behind the apple-trees at the foot of the garden. The beautiful old house stood wide open to the long-
expected guest.

Which choice best states the main purpose of the text?

A. To convey the worries brought about by a new guest

B. To describe how the characters have changed over time

C. To contrast the activity indoors with the stillness outside

D. To depict the setting as the characters await a visitor’s arrival

ID: a2be625e Answer


Correct Answer: D

Rationale

Choice D is the best answer because it most accurately reflects the main purpose of the text. The text portrays
Miss Pyne as awaiting the arrival of a carriage while Martha brings strawberries and flowers from the garden
into the house. The text also describes the surroundings of the scene, stating that Miss Pyne looks “stately and
calm,” the evening is bright and cool, and birds are singing in the garden as the sun sets. Then the last
sentence states that the house was “wide open to the long-expected guest,” which strongly suggests that Miss
Pyne’s anticipation and Martha’s activities were in preparation for the guest who is expected to arrive in the
carriage. Thus, the text depicts the setting and conveys what these characters are doing as they await the
arrival of their visitor.

Choice A is incorrect because there is nothing in the text to indicate that the characters feel any worry about
the guest’s arrival. The text indicates that the guest was “long-expected,“ but characterizing Miss Pyne as
“stately and calm” conflicts with the idea that the characters are worried about the guest. Choice B is incorrect
because the text describes a moment in time when two characters are awaiting the arrival of a visitor rather
than an extended period over which characters could be seen changing. Choice C is incorrect. Although the
text describes the activity indoors (Miss Pyne sitting calmly), it describes a higher level of activity, not
stillness, outside (Martha bringing fruit and flowers and birds singing).

Question Difficulty: Medium


Question ID ab56a107 Question ID f773a56b
Assessment Test Domain Skill Difficulty Assessment Test Domain Skill Difficulty

SAT Reading and Writing Craft and Structure Cross-Text SAT Reading and Writing Craft and Structure Words in Context
Connections

ID: f773a56b
ID: ab56a107
As Mexico’s first president from an Indigenous community, Benito Juarez became one of the most ______ figures in his
country’s history: among the many significant accomplishments of his long tenure in office (1858–1872), Juarez
Text 1 consolidated the authority of the national government and advanced the rights of Indigenous peoples.
Digital art, the use of digital technology to create or display images, isn’t really art at all. It doesn’t require as much skill
as creating physical art. “Painting” with a tablet and stylus is much easier than using paint and a brush: the technology Which choice completes the text with the most logical and precise word or phrase?
is doing most of the work.
A. unpredictable

Text 2 B. important
The painting programs used to create digital art involve more than just pressing a few buttons. In addition to knowing
C. secretive
the fundamentals of art, digital artists need to be familiar with sophisticated software. Many artists will start by drawing
an image on paper before transforming the piece to a digital format, where they can apply a variety of colors and D. ordinary
techniques that would otherwise require many different traditional tools.

Based on the texts, how would the author of Text 2 most likely respond to the claims of the author of Text 1? ID: f773a56b Answer
A. By arguing that a piece of art created digitally can still be displayed traditionally Correct Answer: B

B. By explaining that it’s actually much harder to use a tablet and stylus to create art than to use paint and a brush Rationale

C. By insisting that digital art requires artistic abilities and skill even if it employs less traditional tools Choice B is the best answer because it most logically completes the text’s discussion of Juarez. In this context,
“important” means marked by significant work or consequence. The text indicates that Juarez, who was the
D. By admitting that most digital artists don’t think fundamental drawing skills are important
first president of Mexico from an Indigenous community, became a certain kind of figure in Mexico’s history. It
then supports that claim by describing some of the “many significant accomplishments” from Juarez’s long
ID: ab56a107 Answer tenure in office. This context conveys that Juarez is a significant and consequential figure in Mexico’s history.

Correct Answer: C Choice A is incorrect because the text focuses on Juarez’s role as the first president of Mexico from an
Rationale Indigenous community and on his many major accomplishments during his lengthy time in office; nothing in
the text suggests that Juarez was “unpredictable,” or tended to behave in ways that couldn’t be predicted.
Choice C is the best answer because it reflects how the author of Text 2 would respond to the claims in Text 1. Choice C is incorrect because nothing in the text suggests that Juarez was a particularly “secretive” figure, or
Both texts address skills needed to produce digital art. Text 1 claims that digital art doesn’t require the same that he tended to keep things private or hidden from others. Instead, the text focuses on things that are known
amount of skill as creating physical art and that “the technology is doing most of the work.” Text 2 states that about Juarez: that he was the first president of Mexico from an Indigenous community, that he had a lengthy
digital art requires “knowing the fundamentals of art” and that many digital artists begin their work on paper tenure, and that his many major accomplishments included consolidating the national government’s authority
and then transfer it to a digital format using “sophisticated software” and “a variety of colors and techniques.” and advancing Indigenous rights. Choice D is incorrect because the text focuses on the idea that Juarez, who
Therefore, the author of Text 2 would most likely insist that digital art requires artistic abilities even if it was the first president of Mexico from an Indigenous community, had many major accomplishments during
employs less traditional tools. his lengthy time in office. Rather than suggesting that Juarez was an “ordinary,” or common and typical, figure
in Mexico’s history, this context conveys that Juarez was instead a notable figure.
Choice A is incorrect because neither text discusses nondigital means of displaying art. Choice B is incorrect
because the author of Text 2 doesn’t address whether it’s harder to use a tablet and stylus than it is to use paint Question Difficulty: Easy
and a brush. Text 2 does argue that digital art requires skills that aren’t part of the traditional methods for
producing art, but the text doesn’t address relative difficulty. Choice D is incorrect because the author of Text 2
states that digital artists still need to know “the fundamentals of art” and that many digital artists begin their
work by drafting on paper before transferring the work to a digital format.

Question Difficulty: Medium

Choice B is incorrect because Text 2 indicates that the fossils initially looked avian, so the paleontologists
described in Text 2 wouldn’t be confused by the researchers in Text 1 initially thinking that O. khaungraae
Question ID f653b273 might be related to birds. The paleontologists would find that initial thought understandable, not confusing.
Choice C is incorrect because Text 1 never mentions lizards, so it wouldn’t make sense for the paleontologists
Assessment Test Domain Skill Difficulty in Text 2 to say that the researchers in Text 1 mistakenly assumed that O. khaungraae must be a lizard. Choice
D is incorrect. Although the paleontologists in Text 2 might agree that the initial thought of the researchers in
SAT Reading and Writing Craft and Structure Cross-Text Text 1 was reasonable, nothing in Text 2 suggests that the two skulls were shaped differently.
Connections
Question Difficulty: Medium
ID: f653b273

Text 1
A tiny, unusual fossil in a piece of 99-million-year-old amber is of the extinct species Oculudentavis khaungraae. The O.
khaungraae fossil consists of a rounded skull with a thin snout and a large eye socket. Because these features look like
they are avian, or related to birds, researchers initially thought that the fossil might be the smallest avian dinosaur ever
found.

Text 2
Paleontologists were excited to discover a second small fossil that is similar to the strange O. khaungraae fossil but has
part of the lower body along with a birdlike skull. Detailed studies of both fossils revealed several traits that are found
in lizards but not in dinosaurs or birds. Therefore, paleontologists think the two creatures were probably unusual
lizards, even though the skulls looked avian at first.

Based on the texts, what would the paleontologists in Text 2 most likely say about the researchers’ initial thought in
Text 1?

A. It is understandable because the fossil does look like it could be related to birds, even though O. khaungraae is
probably a lizard.

B. It is confusing because it isn’t clear what caused the researchers to think that O. khaungraae might be related to
birds.

C. It is flawed because the researchers mistakenly assumed that O. khaungraae must be a lizard.

D. It is reasonable because the O. khaungraae skull is about the same size as the skull of the second fossil but is
shaped differently.

ID: f653b273 Answer


Correct Answer: A

Rationale

Choice A is the best answer because it reflects what the paleontologists in Text 2 would most likely say about
what the researchers in Text 1 initially thought. Text 1 focuses on the discovery of a strange fossil consisting of
the skull of the extinct species Oculudentavis khaungraae. According to Text 1, the fossil has features that
appear to be avian, or related to birds, which led researchers to initially think that the fossil might be a very
small avian dinosaur. Text 2 begins by noting the discovery of a second fossil similar to the one discussed in
Text 1, then explains that based on detailed studies of both fossils, paleontologists think that the two creatures
were probably unusual lizards, even though the skulls appeared avian at first. This suggests that the
paleontologists in Text 2 recognize that the fossils do indeed look like they could be related to birds. For this
reason, the paleontologists in Text 2 would most likely say that the initial thought of the researchers in Text 1—
that the fossil was avian—is understandable, even if the fossil is probably not avian but rather is from a lizard.
Question ID 5e732e67 Question ID 14b7dced
Assessment Test Domain Skill Difficulty Assessment Test Domain Skill Difficulty

SAT Reading and Writing Craft and Structure Text Structure and SAT Reading and Writing Craft and Structure Text Structure and
Purpose Purpose

ID: 5e732e67 ID: 14b7dced


Many films from the early 1900s have been lost. These losses include several films by the first wave of Black women The following text is from Walt Whitman’s 1860 poem “Calamus 24.”
filmmakers. We know about these lost movies only from small pieces of evidence. For example, an advertisement for I HEAR it is charged against me that I seek to destroy institutions;
Jennie Louise Touissant Welcome’s documentary Doing Their Bit still exists. There’s a reference in a magazine to Tressie But really I am neither for nor against institutions
Souders’s film A Woman’s Error. And Maria P. Williams’s The Flames of Wrath is mentioned in a letter and a newspaper (What indeed have I in common with them?—Or what with the destruction of them?),
article, and one image from the movie was discovered in the 1990s. Only I will establish in the Mannahatta [Manhattan] and in every city of These States, inland and seaboard,
And in the fields and woods, and above every keel [ship] little or large, that dents the water,
Which choice best describes the overall structure of the text? Without edifices, or rules, or trustees, or any argument,
A. The text identifies a complex problem, then presents examples of unsuccessful attempts to solve that problem. The institution of the dear love of comrades.

B. The text summarizes a debate among researchers, then gives reasons for supporting one side in that debate. Which choice best describes the overall structure of the text?

C. The text describes a general situation, then illustrates that situation with specific examples. A. The speaker questions an increasingly prevalent attitude, then summarizes his worldview.

D. The text discusses several notable individuals, then explains commonly overlooked differences between those B. The speaker regrets his isolation from others, then predicts a profound change in society.
individuals.
C. The speaker concedes his personal shortcomings, then boasts of his many achievements.

D. The speaker addresses a criticism leveled against him, then announces a grand ambition of his.
ID: 5e732e67 Answer
Correct Answer: C
ID: 14b7dced Answer
Rationale
Correct Answer: D
Choice C is the best answer. The first three sentences describe the general situation: these early films have Rationale
been lost, and we only know about them from small pieces of evidence. The rest of the text offers specific
examples of the small pieces of evidence. Choice D is the best answer because it best describes the overall structure of the text. The speaker begins by
stating that he has heard that others are accusing him of seeking to destroy institutions. The speaker then
Choice A is incorrect. This isn’t the overall structure. The fact that we only know about these lost early films addresses this criticism by stating that he is “neither for nor against institutions.” Instead, the speaker states
from small pieces of evidence isn’t presented as a “complex problem”—that’s too extreme. And the examples that his ultimate goal is to instill “the institution of the dear love of comrades” everywhere in the country.
presented are not “unsuccessful attempts” to solve it. If anything, the examples represent a success, because Therefore, the overall structure of the text is best described as an address of criticism followed by an
we discovered that these films existed in the first place. Choice B is incorrect. This isn’t the overall structure. announcement of a grand ambition.
There’s no “debate” presented in the text, so there’s no “side” for the text to support. Choice D is incorrect. This
isn’t the overall structure. The text doesn’t discuss any “differences” between the filmmakers. Choice A is incorrect. While the speaker does address an opinion of him that he believes to be untrue, he
doesn’t indicate that this attitude has become increasingly prevalent. The speaker also concludes by
Question Difficulty: Easy explaining his goal for the future rather than his current worldview. Choice B is incorrect because the text
doesn’t portray the speaker as isolated or regretful, and the speaker gestures toward a hope for societal change
but doesn’t offer an explicit prediction that it will happen. Choice C is incorrect because the speaker addresses
a criticism of him that he believes to be false; he doesn’t admit any personal shortcomings. Moreover, the
speaker concludes by stating a goal he has rather than showcasing his achievements.

Question Difficulty: Medium

Question ID 3118ca93 Question ID 21d95d1d


Assessment Test Domain Skill Difficulty Assessment Test Domain Skill Difficulty

SAT Reading and Writing Craft and Structure Words in Context SAT Reading and Writing Craft and Structure Words in Context

ID: 3118ca93 ID: 21d95d1d


The fashion resale market, in which consumers purchase secondhand clothing from stores and online sellers, generated Ofelia Zepeda’s contributions to the field of linguistics are ______: her many accomplishments include working as a
nearly $30 billion globally in 2019. Expecting to see continued growth, some analysts ______ that revenues will more linguistics professor and bilingual poet, authoring the first Tohono O’odham grammar book, and co-founding the
than double by 2028. American Indian Language Development Institute.

Which choice completes the text with the most logical and precise word or phrase? Which choice completes the text with the most logical and precise word or phrase?

A. produced A. pragmatic

B. denied B. controversial

C. worried C. extensive

D. predicted D. universal

ID: 3118ca93 Answer ID: 21d95d1d Answer


Correct Answer: D Correct Answer: C

Rationale Rationale

Choice D is the best answer because it most logically completes the text’s discussion of the fashion resale Choice C is the best answer because it most logically completes the text’s discussion of how Ofelia Zepeda has
market’s continued growth. As used in this context, “predicted” means forecast, or indicated that something contributed to the field of linguistics. As used in this context, “extensive” means having a wide or considerable
would happen in the future. The text indicates that the fashion resale market made a lot of money in 2019 and extent. The text indicates that Zepeda’s many accomplishments in linguistics are varied, including teaching
that some analysts expected the market to continue to grow. This context suggests that the analysts believed linguistics, writing poetry in more than one language, creating a grammar book, and co-founding a language
that the fashion resale market was going to make more money than it had already made, with the analysts institute. This context supports the idea that Zepeda’s contributions to the field are extensive.
indicating that revenues would more than double by 2028.
Choice A is incorrect because the sentence presents Zepeda’s accomplishments as examples to support the
Choice A is incorrect because it wouldn’t make sense in context to say that some analysts “produced,” or claim made in the first part of the sentence. It wouldn’t make sense to say that achievements as a professor,
manufactured or brought about, the increase in future revenues of the fashion resale market. The analysts poet and author, and co-founder of a language institute demonstrate that Zepeda’s contributions in her field
themselves couldn’t have brought about the future revenue growth, since, as the text suggests, they were are “pragmatic,” or related to practical matters and not involving intellectual or artistic matters. Choice B is
merely in the position of drawing conclusions about future fashion resale market revenue based on 2019 incorrect because the sentence presents Zepeda’s accomplishments as a professor, poet and author, and co-
revenue. Choice B is incorrect because the text indicates that some analysts expected the fashion resale founder of a language institute as examples to support the claim made in the first part of the sentence. There’s
market to continue to grow in the future, not that they “denied,” or rejected, this notion. Nothing in the text no reason to believe that the positive achievements listed demonstrate that Zepeda’s contributions in her field
supports the idea that these analysts thought the revenues wouldn’t grow. Choice C is incorrect because the are “controversial,” or have caused disputes and opposing viewpoints. Choice D is incorrect because in this
text indicates that some analysts expected the fashion resale market to continue to grow in the future, not that context, “universal” would mean including or covering everything in a group. The sentence presents Zepeda’s
they “worried,” or felt concerned, that revenue would significantly increase by 2028. Nothing in the text accomplishments as examples to support the claim made in the first part of the sentence, and it wouldn’t
suggests that the analysts felt concerned about the increase; rather, the text suggests that the increase would make sense to say that these specific achievements—particularly as the author of a grammar book specific to
represent a favorable outcome, since it would mean that the fashion resale market grew to generate even more the Tohono O’odham language—demonstrate that Zepeda’s contributions relate to everything in the field of
revenue. linguistics.

Question Difficulty: Easy Question Difficulty: Easy


Question ID 56ec23a0 Question ID a318c1ef
Assessment Test Domain Skill Difficulty Assessment Test Domain Skill Difficulty

SAT Reading and Writing Craft and Structure Text Structure and SAT Reading and Writing Craft and Structure Words in Context
Purpose

ID: a318c1ef
ID: 56ec23a0
The Cambrian explosion gets its name from the sudden appearance and rapid diversification of animal remains in the
Hiroshi Senju is known worldwide for his paintings of waterfalls. These paintings are large and tend not to show the fossil record about 541 million years ago, during the Cambrian period. Some scientists argue that this ______ change in
entire waterfall. Instead, Senju focuses on just the point where the falling water reaches the pool below, keeping the the fossil record might be because of a shift in many organisms to body types that were more likely to be preserved.
top of the waterfall out of view. While Senju’s paintings are rooted in art movements originating in the United States,
the artist uses traditional Japanese techniques and materials that make his work instantly recognizable. Which choice completes the text with the most logical and precise word or phrase?

A. catastrophic
Which choice best describes the overall structure of the text?

A. It introduces an artist and then explains some common characteristics of well-known paintings by that artist. B. elusive

B. It explains a specific painting technique and then provides examples of artists who use the technique. C. abrupt

C. It describes a famous painting and then compares it to a lesser-known painting from the same time period. D. imminent

D. It gives an opinion on an artist and then suggests multiple reasons why the artist’s work has been largely
overlooked. ID: a318c1ef Answer
Correct Answer: C

ID: 56ec23a0 Answer Rationale

Correct Answer: A
Choice C is the best answer because it most logically and precisely completes the text’s discussion of the
Rationale fossil record from the Cambrian period. In this context, “abrupt” means sudden. The text explains that the
fossil record reflects the unexpected appearance and rapid diversification, or increase in variety, of animal
Choice A is the best answer. The first sentence introduces Senju as a famous artist, while the next three remains during the Cambrian period. This context establishes that these remains’ entry into the fossil record
sentences describe the defining features of his art, such as it only showing part of the waterfall and its origins was sudden.
in US art movements and Japanese techniques.
Choice A is incorrect. Although the word “explosion” appears in the name of the event marked by the fossil
Choice B is incorrect. The text doesn’t provide examples of any other artists who use Senju’s techniques. record change, the text never suggests that the change was “catastrophic,” or disastrous. In context,
Choice C is incorrect. The text doesn’t describe any single famous painting or make comparisons between “explosion” refers to the rapid diversification, or the swift increase in variety, of animal remains in the fossil
paintings. Choice D is incorrect. The text doesn’t provide an opinion on Senju (just facts), nor does it suggest record—a phenomenon that the text presents in a relatively neutral manner, without commenting on whether
that his art has been overlooked—in fact, it states that he is “known worldwide.” it was negative or positive. Choice B is incorrect because the text never suggests that the change toward
greater diversification is “elusive,” or difficult to locate, in the fossil record. Rather, the text notes that the
Question Difficulty: Easy
change occurred about 541 million years ago, suggesting that scientists have indeed been able to locate it.
Choice D is incorrect because it wouldn’t make sense in context to describe the change in the fossil record as
“imminent,” or about to occur, since the text indicates that the change already occurred millions of years ago.

Question Difficulty: Easy

Question ID bce627d9 Question ID e56b66e5


Assessment Test Domain Skill Difficulty Assessment Test Domain Skill Difficulty

SAT Reading and Writing Craft and Structure Words in Context SAT Reading and Writing Craft and Structure Words in Context

ID: bce627d9 ID: e56b66e5


Mineralogical differences are detectable in samples collected from two locations on the near-Earth asteroid Ryugu, but Set in a world where science fiction tropes exist as everyday realities, Charles Yu’s 2010 novel How to Live Safely in a
such differences may not indicate substantial compositional variations in the asteroid. Cosmochemist Kazuhide Science Fictional Universe traces a time traveler’s quest to find his father. Because the journey at the novel’s center is so
Nagashima and colleagues note that at the small scale of the samples, the distribution of minerals is unlikely to be ______, with the protagonist ricocheting chaotically across time, the reader often wonders whether the pair will ever be
______. reunited.

Which choice completes the text with the most logical and precise word or phrase? Which choice completes the text with the most logical and precise word or phrase?

A. neglected A. haphazard

B. redundant B. premeditated

C. ongoing C. inspirational

D. uniform D. fruitless

ID: bce627d9 Answer ID: e56b66e5 Answer


Correct Answer: D Correct Answer: A

Rationale Rationale

Choice D is the best answer. The text tells us that the samples are too "small scale" to reflect the composition Choice A is the best answer because it most logically completes the text’s discussion of Yu’s novel. In this
of the asteroid, which probably doesn’t show the same variation on a large scale. This suggests that the context, “haphazard” means marked by a lack of plan or order. The text indicates that the quest featured in the
mineral composition of the samples are unlikely to be exactly the same from sample to sample. novel, which involves the protagonist bouncing across time, is chaotic and causes the reader to often wonder
what will happen. This context suggests that the protagonist’s journey seems to be marked by a lack of order.
Choice A is incorrect. "Neglected" means "suffering a lack of proper care" or "abandoned," which doesn’t work
here. The text never suggests that the distribution of minerals in the samples would be neglected, so this Choice B is incorrect because the text indicates that the journey featured in Yu’s novel involves a character
statement doesn’t logically follow. Choice B is incorrect. "Redundant" means "not or no longer useful or “ricocheting chaotically,” or bouncing in a disordered way, across time and causes the reader to often wonder
needed," which is too strong. The text doesn’t suggest that the variation between the samples isn’t a useful what will happen. It wouldn’t make sense to say that a chaotic journey seems “premeditated,” or characterized
finding at all—just that we can’t assume that the large-scale composition of the asteroid will show the same by forethought and planning. Choice C is incorrect because the text doesn’t give any indication that readers
variation. But the composition of the samples might be useful for something else. Choice C is incorrect. regard the journey in Yu’s novel as “inspirational,” or as causing extraordinarily creative or brilliant thoughts
"Ongoing" means "still in progress," which doesn’t make sense: the distribution of minerals in a sample can’t or actions; instead, the text focuses on the idea that the protagonist’s journey is chaotic, or disordered, and
be "ongoing." doesn’t give readers a clear sense of what will happen. Choice D is incorrect. Rather than suggesting that the
journey featured in Yu’s novel is “fruitless,” or has an unsuccessful outcome, the text focuses on the idea that
Question Difficulty: Medium while reading about the protagonist’s chaotic movements across time, readers are often unsure of what will
happen—that is, they don’t know whether the protagonist will be successful in finding his father.

Question Difficulty: Medium


Question ID f83f0aab Question ID d5235d39
Assessment Test Domain Skill Difficulty Assessment Test Domain Skill Difficulty

SAT Reading and Writing Craft and Structure Words in Context SAT Reading and Writing Craft and Structure Words in Context

ID: f83f0aab ID: d5235d39


Some scientists have suggested that mammals in the Mesozoic era were not a very ______ group, but paleontologist The Mule Bone, a 1930 play written by Zora Neale Hurston and Langston Hughes, is perhaps the best-known of the few
Zhe-Xi Luo’s research suggests that early mammals living in the shadow of dinosaurs weren’t all ground-dwelling examples of ______ in literature. Most writers prefer working alone, and given that working together cost Hurston and
insectivores. Fossils of various plant-eating mammals have been found in China, including species like Vilevolodon Hughes their friendship, it is not hard to see why.
diplomylos, which Luo says could glide like a flying squirrel.
Which choice completes the text with the most logical and precise word or phrase?
Which choice completes the text with the most logical and precise word or phrase?
A. characterization
A. predatory
B. interpretation
B. obscure
C. collaboration
C. diverse
D. commercialization
D. localized

ID: d5235d39 Answer


ID: f83f0aab Answer Correct Answer: C
Correct Answer: C
Rationale
Rationale
Choice C is the best answer because it logically and precisely completes the text’s discussion of The Mule
Choice C is the best answer because it most logically completes the text’s discussion of the kinds of mammals Bone, a play that Zora Neale Hurston and Langston Hughes wrote together. In this context, “collaboration”
alive during the Mesozoic era. As used in this context, “diverse” means to have a significant amount of variety. means working together with someone to write a literary work. The text indicates that most writers prefer to
The text indicates that some scientists have suggested that Mesozoic mammals can’t be characterized in a work alone and that working together destroyed the friendship between Hurston and Hughes. This establishes
certain way, then contrasts the view put forward by those scientists with Luo’s research, which shows that that The Mule Bone is a relatively rare example of collaboration in literature.
Mesozoic mammals “weren’t all ground-dwelling insectivores” and instead were “various.” This context
suggests that some scientists have viewed Mesozoic mammals as being all alike, or not a very diverse group. Choice A is incorrect because in this context, “characterization” would mean a literary work’s portrayal of
characters’ psychological experiences and motivations, but the text doesn’t discuss characterization in The
Choice A is incorrect because it wouldn’t make sense to say that some scientists have suggested that Mule Bone specifically or in collaborative works more generally. Choice B is incorrect because in this context,
Mesozoic mammals weren’t very “predatory,” or that they didn’t prey on other animals, since the text “interpretation” would mean the explanation of a literary work’s meaning or significance, but the text doesn’t
establishes a contrast between what some scientists have suggested and Luo’s research showing that discuss how readers or critics have interpreted The Mule Bone; instead, the text discusses how the play was
Mesozoic mammals “weren’t all ground-dwelling insectivores.” This context suggests that some scientists written collaboratively and how the writing process affected the two authors. Choice D is incorrect because in
have regarded Mesozoic mammals as all being insectivores, or animals that prey on insects, not that some this context, “commercialization” would mean writing a literary work in such a way as to ensure its
scientists have suggested that Mesozoic mammals didn’t prey on other animals. Choice B is incorrect because commercial appeal, but the text never discusses commercial appeal as a factor in the writing of The Mule Bone
it wouldn’t make sense to say that some scientists have suggested that Mesozoic mammals weren’t very specifically or the writing of collaborative works more generally.
“obscure,” or concealed or not well known, since the text establishes a contrast between what some scientists
have suggested and Luo’s research showing that Mesozoic mammals were a varied group. There’s no contrast Question Difficulty: Easy
between saying that the mammals weren’t concealed or well known and the mammals being varied. Choice D
is incorrect because it wouldn’t make sense to say that some scientists have suggested that Mesozoic
mammals weren’t very “localized,” or confined to a particular area, since the text establishes a contrast
between what some scientists have suggested and Luo’s research showing that Mesozoic mammals were a
varied group. There’s no contrast between saying that the mammals weren’t localized and the mammals being
varied. Although the text mentions mammal fossils found in China, nothing in the discussion of Luo’s research
addresses the limits of Mesozoic mammal habitats.

Question Difficulty: Hard

Question ID 1fbf276a Question ID 1c6b1fa0


Assessment Test Domain Skill Difficulty Assessment Test Domain Skill Difficulty

SAT Reading and Writing Craft and Structure Words in Context SAT Reading and Writing Craft and Structure Text Structure and
Purpose

ID: 1fbf276a
ID: 1c6b1fa0
Interruptions in the supply chain for microchips used in personal electronics have challenged an economist’s assertion
that retailers can expect robust growth in sales of those devices in the coming months. The delays are unlikely to ______ In 1801, a Blackfoot chief named Ac Ko Mok Ki drew a finely detailed map of the Upper Missouri region. This work
her projection entirely but will almost certainly extend its time frame. demonstrates a vast amount of topographic knowledge, as the map features specific names of mountains and rivers, as
well as the first-known sketch of the drainage network of the Missouri River. The map is especially notable because Ac
Which choice completes the text with the most logical and precise word or phrase? Ko Mok Ki also included details about the numerous tribes that lived in the area.

A. dispute
Which choice best describes the function of the underlined sentence in the text as a whole?
B. withdraw A. It emphasizes Ac Ko Mok Ki’s desire to represent other tribes on the map.

C. underscore B. It explains how Ac Ko Mok Ki developed an interest in mapmaking.

D. invalidate C. It identifies some reasons why the map is impressive.

D. It details how the map was used for hunting and trading purposes.
ID: 1fbf276a Answer
Correct Answer: D
ID: 1c6b1fa0 Answer
Rationale
Correct Answer: C

Choice D is the best answer because it most logically completes the text’s discussion of the economist’s claim Rationale
about sales of personal electronic devices. In this context, “invalidate” most nearly means nullify or make
invalid. The text indicates that interruptions in the supply of microchips for personal electronics “have Choice C is the best answer because it best describes how the underlined sentence functions in the text as a
challenged” the economist’s claim that sales of personal electronics will show strong growth in the coming whole. The text presents information about a map drawn by Blackfoot chief Ac Ko Mok Ki in 1801. The
months. The text goes on to clarify the effect of the delays on the economist’s projection, stating that the underlined sentence states that the map "demonstrates a vast amount of topographic knowledge" and
delays are very likely to extend the time frame over which the projected growth in sales will occur. This mentions that it features the "specific names of mountains and rivers" and includes the first-known sketch of
context suggests that the delays are unlikely to invalidate the economist’s projection entirely—the delays will the Missouri River’s drainage network. These are all characteristics that indicate that the map was executed
probably alter the time frame of the projection, not nullify it or make it invalid. with remarkable skill. Thus, the underlined sentence identifies some reasons why the map is impressive.

Choice A is incorrect because saying that the delays are unlikely to “dispute,” or argue against, the economist’s Choice A is incorrect. Though the sentence after the underlined sentence in the text mentions that Ac Ko Mok
projection wouldn’t make sense. Since the delays are an inanimate circumstance, they couldn’t argue against a Ki included information about other tribes on his map, the underlined sentence itself does not address this
prediction about the sales of personal electronics. Choice B is incorrect because saying that the delays are topic. Choice B is incorrect because nothing in the underlined sentence indicates how Ac Ko Mok Ki became
unlikely to “withdraw,” or remove from consideration, the economist’s projection wouldn’t make sense. interested in mapmaking, only that his mapmaking skills are impressive. Choice D is incorrect because
Although the economist could withdraw her projection because of the delays, the delays themselves couldn’t though the underlined sentence describes several features of the map, it does not specifically describe how
withdraw her projection since they’re an inanimate circumstance and thus can’t choose to remove something the map was used.
from consideration. Choice C is incorrect because there’s nothing in the text to suggest that the delays will
Question Difficulty: Easy
“underscore,” or emphasize, the economist’s projection. Instead, the text suggests that the delays are likely to
extend the time frame of the economist’s projection but not to undermine the projection entirely.

Question Difficulty: Medium


Question ID f4166aae Question ID 9b2fbb2e
Assessment Test Domain Skill Difficulty Assessment Test Domain Skill Difficulty

SAT Reading and Writing Craft and Structure Words in Context SAT Reading and Writing Craft and Structure Words in Context

ID: f4166aae ID: 9b2fbb2e


In addition to being an accomplished psychologist himself, Francis Cecil Sumner was a ______ increasing the Logically, a damaged fossil should provide less information than an intact one, but for paleontologist Brigitte
opportunity for Black students to study psychology, helping to found the psychology department at Howard Schoenemann, a broken area on a fossilized trilobite (a crustacean-like creature) ______ fresh insight, allowing her to
University, a historically Black university, in 1930. view the inner structure of the organism’s eye.

Which choice completes the text with the most logical and precise word or phrase? Which choice completes the text with the most logical and precise word or phrase?

A. proponent of A. resolved

B. supplement to B. adjusted

C. beneficiary of C. offered

D. distraction for D. directed

ID: f4166aae Answer ID: 9b2fbb2e Answer


Correct Answer: A Correct Answer: C

Rationale Rationale

Choice A is the best answer because it most logically completes the text’s discussion of Francis Cecil Sumner. Choice C is the best answer because it most logically completes the text’s discussion about how, for
As used in this context, “proponent of” means supporter of. The text says that Sumner helped to found the Schoenemann, a damaged trilobite fossil was informative. In context, “offered” means provided or gave. The
psychology department at historically Black Howard University in 1930. This is evidence that Sumner text suggests that although it may seem counterintuitive, a broken fossilized trilobite allowed Schoenemann
supported increasing the opportunity for Black students to study psychology. to observe more details of the trilobite’s eye than an intact fossilized trilobite would. This context conveys the
idea that a damaged fossil offered, or provided, fresh insight into the structure of a trilobite’s eye.
Choice B is incorrect because the phrase “supplement to,” or addition to, wouldn’t make sense in context. The
text discusses Sumner’s efforts to increase the number of Black psychology students, but it doesn’t make Choice A is incorrect because “resolved” means determined or figured out, which wouldn’t make sense in this
sense to describe him as an addition to his efforts. Choice C is incorrect because Sumner was already an context. The text focuses on how Schoenemann was able to get more information from a broken fossil than an
accomplished psychologist himself when he helped to found the Howard University psychology department. intact one. Although the damaged fossil may have allowed her to determine certain information, as an
While Black students were the beneficiaries of his efforts—that is, they received help because of his efforts—it inanimate object the fossil isn’t capable of resolving anything. Choice B is incorrect because saying that the
wouldn’t make sense in this context to describe Sumner as a “beneficiary of” opportunities, because he was broken part of a trilobite fossil “adjusted,” or changed or modified, fresh insight wouldn’t make sense in this
the one doing the helping. Choice D is incorrect because founding a psychology department at Howard context. The text focuses on how Schoenemann was able to get more information from a broken fossil than an
University wouldn’t be a “distraction for” Sumner’s aim to increase the opportunity for Black students to study intact one. This context suggests that the fossil offered fresh insight, or understanding, not that it adjusted
psychology—that is, it wouldn’t be something that draws Sumner’s attention away from that goal, but rather fresh insight. Choice D is incorrect because “directed” means managed or instructed, which wouldn’t make
the opposite. sense in this context. The text focuses on how Schoenemann was able to get more information from a broken
fossil than an intact one. This context suggests that the fossil offered fresh insight, or understanding, not that
Question Difficulty: Medium it directed fresh insight.

Question Difficulty: Easy

Question ID 1782cdd7 Question ID 6d44060a


Assessment Test Domain Skill Difficulty Assessment Test Domain Skill Difficulty

SAT Reading and Writing Craft and Structure Text Structure and SAT Reading and Writing Craft and Structure Text Structure and
Purpose Purpose

ID: 1782cdd7 ID: 6d44060a


In many agricultural environments, the banks of streams are kept forested to protect water quality, but it’s been Works of moral philosophy, such as Plato’s Republic or Aristotle’s Nicomachean Ethics, are partly concerned with how
unclear what effects these forests may have on stream biodiversity. To investigate the issue, biologist Xingli Giam and to live a morally good life. But philosopher Jonathan Barnes argues that works that present a method of living such a
colleagues studied an Indonesian oil palm plantation, comparing the species richness of forested streams with that of life without also supplying a motive are inherently useful only to those already wishing to be morally good—those with
nonforested streams. Giam and colleagues found that species richness was significantly higher in forested streams, a no desire for moral goodness will not choose to follow their rules. However, some works of moral philosophy attempt
finding the researchers attribute to the role leaf litter plays in sheltering fish from predators and providing food to describe what constitutes a morally good life while also proposing reasons for living one.
resources.
Which choice best describes the overall structure of the text?
Which choice best states the main purpose of the text?
A. It provides a characterization about a field of thought by noting two works in it and then details a way in which
A. It discusses research intended to settle a debate about how agricultural yields can be increased without negative some works in that field are more comprehensive than others.
effects on water quality.
B. It mentions two renowned works and then claims that despite their popularity it is impossible for these works to
B. It explains the differences between stream-protection strategies used in oil palm plantations and stream-protection serve the purpose their authors intended.
strategies used in other kinds of agricultural environments.
C. It summarizes the history of a field of thought by discussing two works and then proposes a topic of further
C. It describes findings that challenge a previously held view about how fish that inhabit streams in agricultural research for specialists in that field.
environments attempt to avoid predators.
D. It describes two influential works and then explains why one is more widely read than the other.
D. It presents a study that addresses an unresolved question about the presence of forests along streams in
agricultural environments.
ID: 6d44060a Answer
Correct Answer: A
ID: 1782cdd7 Answer
Rationale
Correct Answer: D

Rationale Choice A is the best answer. The text starts by stating what moral philosophy is concerned with and naming
two examples of works in the field. Then it describes a shortcoming of some works in that field (they say how
Choice D is the best answer. The author first describes an unresolved question: what effect do bank forests but not why), and finally it states that other works try to avoid that shortcoming (by including both how and
have on stream biodiversity? Then the author presents a study that answers the question: bank forests why to live a morally good life).
increase stream biodiversity.
Choice B is incorrect. This is too extreme. The text never mentions whether the two works are popular or not,
Choice A is incorrect. This isn’t the main purpose of the text. The text never mentions agricultural yields. and it never argues that these works don’t serve their intended purpose of describing how to live a morally
Choice B is incorrect. This isn’t the main purpose of the text. The text never mentions other kinds of good life. Rather, the text claims that works of moral philosophy that don’t include both how and why to be
agricultural environments. Choice C is incorrect. This isn’t the main purpose of the text. The text never moral are not useful to readers who don’t already want to be moral. Choice C is incorrect. This isn’t the overall
mentions any previously held view about how fish in these streams try to avoid predators. structure. The text never discusses the history of moral philosophy at all, and it doesn’t propose any topic for
further research. Choice D is incorrect. This isn’t the overall structure. The text never discusses which of the
Question Difficulty: Easy two works is more widely read.

Question Difficulty: Medium


Question ID 590f0ad2 Question ID 19688783
Assessment Test Domain Skill Difficulty Assessment Test Domain Skill Difficulty

SAT Reading and Writing Craft and Structure Text Structure and SAT Reading and Writing Craft and Structure Text Structure and
Purpose Purpose

ID: 590f0ad2 ID: 19688783


Industrial activity is often assumed to be a threat to wildlife, but that isn’t always so. Consider the silver-studded blue The following text is from Lucy Maud Montgomery’s 1908 novel Anne of Green Gables. Anne, an eleven-year-old girl,
butterfly (Plebejus argus): as forest growth has reduced grasslands in northern Germany, many of these butterflies have has come to live on a farm with a woman named Marilla in Nova Scotia, Canada.
left meadow habitats and are now thriving in active limestone quarries. In a survey of multiple active quarries and
Anne reveled in the world of color about her.
patches of maintained grassland, an ecologist found silver-studded blue butterflies in 100% of the quarries but only
“Oh, Marilla,” she exclaimed one Saturday morning, coming dancing in with her arms full of gorgeous boughs,
57% of the grassland patches. Moreover, butterfly populations in the quarries were four times larger than those in the
“I’m so glad I live in a world where there are Octobers. It would be terrible if we just skipped from September to
meadows.
November, wouldn’t it? Look at these maple branches. Don’t they give you a thrill—several thrills? I’m going to
Which choice best describes the function of the underlined portion in the text as a whole? decorate my room with them.”
“Messy things,” said Marilla, whose aesthetic sense was not noticeably developed. “You clutter up your room
A. It challenges a common assumption about the species under investigation in the research referred to in the text.
entirely too much with out-of-doors stuff, Anne. Bedrooms were made to sleep in.”
B. It introduces discussion of a specific example that supports the general claim made in the previous sentence.
Which choice best states the main purpose of the text?
C. It suggests that a certain species should be included in additional studies like the one mentioned later in the text. A. To demonstrate that Anne has a newly developed appreciation of nature
D. It provides a definition for an unfamiliar term that is central to the main argument in the text. B. To describe an argument that Anne and Marilla often have

C. To emphasize Marilla’s disapproval of how Anne has decorated her room


ID: 590f0ad2 Answer
D. To show that Anne and Marilla have very different personalities
Correct Answer: B

Rationale
ID: 19688783 Answer
Choice B is the best answer because it most accurately describes how the underlined portion functions in the
Correct Answer: D
text as a whole. The first sentence presents the general claim that industrial activity is not always a threat to
wildlife. The underlined portion of the sentence that follows suggests that the silver-studded blue butterfly is Rationale
an example of wildlife thriving in areas of industrial activity: active limestone quarries. Thus, the function of
Choice D is the best answer because it most accurately describes the main purpose of the text. The text begins
the underlined portion is to introduce a specific example in support of the general claim in the previous
by noting that Anne “reveled in the world of color about her”—that is, she takes great delight in colorful things.
sentence.
It then relates a scene when she enthusiastically enters the house with autumn foliage and announces that
Choice A is incorrect. Although the first sentence indicates that “industrial activity is often assumed” to harm she will decorate her room with it. The focus of the text then shifts to Marilla, who has an undeveloped
wildlife, in the case of the silver-studded blue butterfly the text mentions neither an assumption about this “aesthetic sense,” or appreciation of beauty, as can be seen when she dismisses the maple leaves as “messy
species nor any challenge to such an assumption. Choice C is incorrect because the text mentions only one things” and criticizes Anne for cluttering her room with objects from outside. This episode thus illustrates that
study: the “survey.” Additional studies are not mentioned in the text. Choice D is incorrect because neither the Anne and Marilla differ in their appreciation of beauty and, more generally, in their basic character: Anne is
underlined portion nor any other portion of the text provides a definition for any of the terms used in the text’s exuberant and joyful, while Marilla is stern and critical. Therefore, the purpose of the text is to show that Anne
argument. and Marilla have very different personalities.

Question Difficulty: Medium Choice A is incorrect because the text presents Anne’s appreciation of nature as a basic personality trait, not
as a newfound enthusiasm, and never indicates how recently she developed that appreciation. Choice B is
incorrect. Although the text portrays Anne and Marilla as having different personalities and attitudes toward
natural beauty and home decoration, it doesn’t show them engaging in an argument about this difference or
suggest that they often argue about it. Choice C is incorrect. Although the text does indicate that Marilla
disapproves of how Anne plans to decorate her room, Marilla’s disapproval is a supporting detail that serves to
develop her personality, which the text as a whole contrasts with Anne’s personality.

Question Difficulty: Medium

Question ID d7dccee7
Assessment Test Domain Skill Difficulty

SAT Reading and Writing Craft and Structure Words in Context

ID: d7dccee7
In a 2019 study, Jeremy Gunawardena and colleagues found that the single-celled protozoan Stentor roeseli not only
uses strategies to escape irritating stimuli but also switches strategies when one fails. This evidence of protozoans
sophisticatedly “changing their minds” demonstrates that single-celled organisms may not be limited to ______
behaviors.

Which choice completes the text with the most logical and precise word or phrase?

A. aggressive

B. rudimentary

C. evolving

D. advantageous

ID: d7dccee7 Answer


Correct Answer: B

Rationale

Choice B is the best answer because it most logically completes the text’s discussion of single-celled organism
behavior. As used in this context, “rudimentary” means basic or unsophisticated. According to the text, a study
of the single-celled protozoan Stentor roeseli showed that the organisms can switch strategies for escaping
certain stimuli, “sophisticatedly ‘changing their minds’” and using new strategies should other strategies fail.
This context suggests that single-celled organisms may not be limited to behaviors that are basic or
rudimentary, since the study showed that single-celled protozoans can respond complexly to irritating
stimuli.

Choice A is incorrect because the text doesn’t suggest that single-celled organisms may not be limited to
behavior that is “aggressive,” or threatening. Rather, the text suggests that single-celled organisms may not be
limited to behaviors that are basic, since the study of Stentor roeseli showed that single-celled protozoans can
respond complexly to irritating stimuli. Choice C is incorrect because the text doesn’t suggest that single-
celled organisms may not be limited to behavior that is “evolving,” or advancing. Rather, the text suggests that
single-celled organisms may not be limited to behaviors that are basic, since the study of Stentor roeseli
showed that single-celled protozoans can respond complexly to irritating stimuli. Choice D is incorrect
because the text doesn’t suggest that single-celled organisms may not be limited to behavior that is
“advantageous,” or helpful. Rather, the text suggests that single-celled organisms may not be limited to
behaviors that are basic, since the study of Stentor roeseli showed that single-celled protozoans can respond
complexly to irritating stimuli.

Question Difficulty: Medium


Question ID d69bc408 Question ID 22105871
Assessment Test Domain Skill Difficulty Assessment Test Domain Skill Difficulty

SAT Reading and Writing Craft and Structure Text Structure and SAT Reading and Writing Craft and Structure Cross-Text
Purpose Connections

ID: d69bc408 ID: 22105871


The following text is adapted from Aphra Behn’s 1689 novel The Lucky Mistake. Atlante and Rinaldo are neighbors who Text 1
have been secretly exchanging letters through Charlot, Atlante’s sister. In a study of insect behavior, Samadi Galpayage and colleagues presented bumblebees with small wooden balls and
[Atlante] gave this letter to Charlot; who immediately ran into the balcony with it, where she still found Rinaldo in a observed many of the bees clinging to, rolling, and dragging the objects. The researchers provided no external rewards
melancholy posture, leaning his head on his hand: She showed him the letter, but was afraid to toss it to him, for fear (such as food) to encourage these interactions. The bees simply appeared to be playing—and for no other reason than
it might fall to the ground; so he ran and fetched a long cane, which he cleft at one end, and held it while she put because they were having fun.
the letter into the cleft, and stayed not to hear what he said to it. But never was man so transported with joy, as he Text 2
was at the reading of this letter; it gives him new wounds; for to the generous, nothing obliges love so much as love. Insects do not have cortexes or other brain areas associated with emotions in humans. Still, Galpayage and her team
have shown that bumblebees may engage in play, possibly experiencing some kind of positive emotional state. Other
Which choice best describes the overall structure of the text? studies have suggested that bees experience negative emotional states (for example, stress), but as Galpayage and her
A. It describes the delivery of a letter, and then portrays a character’s happiness at reading that letter. team have acknowledged, emotions in insects, if they do indeed exist, are likely very rudimentary.

B. It establishes that a character is desperate to receive a letter, and then explains why another character has not yet Based on the texts, how would the author of Text 2 most likely respond to the underlined portion of Text 1?
written that letter.
A. By objecting that the bees were actually experiencing a negative feeling akin to stress rather than a positive feeling
C. It presents a character’s concerns about delivering a letter, and then details the contents of that letter.
B. By arguing that some insects other than bumblebees may be capable of experiencing complex emotional states
D. It reveals the inspiration behind a character’s letter, and then emphasizes the excitement that another character
C. By pointing out that even humans sometimes struggle to have fun while engaging in play
feels upon receiving that letter.
D. By noting that if the bees were truly playing, any positive feelings they may have experienced were probably quite
basic
ID: d69bc408 Answer
Correct Answer: A
ID: 22105871 Answer
Rationale
Correct Answer: D
Choice A is the best answer because it most accurately describes the overall structure of the text. The narrator Rationale
begins by explaining how Charlot carefully delivers Atlante’s letter to Rinaldo, and then relates that Rinaldo
feels “transported with joy” after reading the letter. Therefore, the overall structure of the text is best described Choice D is the best answer. The author of Text 2 agrees with the author of Text 1 that bumblebees may engage
as a description of the delivery of a letter followed by the portrayal of a character’s happiness after reading the in play and possibly experience some kind of positive emotional state. However, the author of Text 2 also
letter. qualifies this claim by stating that emotions in insects, if they do exist, are “likely very rudimentary.”

Choice B is incorrect because the text indicates that the letter has been written; there’s no explanation why Choice A is incorrect. While Text 2 states that other studies might indicate “negative emotional states” in bees,
another character hasn’t written one. In addition, the text’s description of Rinaldo “in a melancholy posture” it does not contradict the findings from the Galpayage study—that the bees might have been having fun.
suggests that he’s sad and thoughtful, not that he’s desperate to receive the letter. Choice C is incorrect. Choice B is incorrect. The author of Text 2 does not mention or imply that any insects, including bumblebees,
Although the text states that Charlot won’t toss the letter to Rinaldo because she doesn’t want it to fall, the text are capable of experiencing complex emotional states. The author of Text 2 states that if insects do feel
doesn’t refer to the contents of the letter. Instead, the text describes how happy Rinaldo feels after reading emotions, those emotions are “likely very rudimentary.” Choice C is incorrect. The author of Text 2 does not
it. Choice D is incorrect. Although the text does describe Rinaldo’s reaction to the letter, the text doesn’t begin compare or contrast the behavior or emotions of insects and humans, and neither does the author of Text 1.
by discussing Atlante’s inspiration for writing the letter. Instead, the text begins by discussing the delivery of
the letter. Question Difficulty: Easy

Question Difficulty: Medium

Question ID 02e49a0c Question ID 27d9bb69


Assessment Test Domain Skill Difficulty Assessment Test Domain Skill Difficulty

SAT Reading and Writing Craft and Structure Text Structure and SAT Reading and Writing Craft and Structure Cross-Text
Purpose Connections

ID: 02e49a0c ID: 27d9bb69


Genetic studies have led researchers to suggest that turtles are most closely related to the group that includes modern Text 1
crocodiles. But studies of fossils have suggested instead that turtles are most closely related to other groups, such as Many studies in psychology have shown that people seek out information even when they know in advance that they
the one that contains modern snakes. However, many of the fossil studies have relied on incomplete data sets. For a have no immediate use for it and that they won’t directly benefit from it. Such findings support the consensus view
2022 investigation, biologist Tiago R. Simões and colleagues examined more than 1,000 reptile fossils collected among researchers of curiosity: namely, that curiosity is not instrumental but instead represents a drive to acquire
worldwide. From this large data set, they found clear agreement with the results of the genetic studies. information for its own sake.
Text 2
Which choice best describes the function of the underlined sentence? While acknowledging that acquiring information is a powerful motivator, Rachit Dubey and colleagues ran an
A. It offers an overview of the tools scientists use to examine fossils. experiment to test whether emphasizing the usefulness of scientific information could increase curiosity about it. They
found that when research involving rats and fruit flies was presented as having medical applications for humans,
B. It describes a limitation of some studies about the origin of turtles. participants expressed greater interest in learning about it than when the research was not presented as useful.

C. It summarizes previous research on the evolution of crocodiles.


Based on the texts, how would Dubey and colleagues (Text 2) most likely respond to the consensus view discussed in
D. It criticizes a widely held belief about genetic studies of reptiles. Text 1?

A. By suggesting that curiosity may not be exclusively motivated by the desire to merely acquire information

ID: 02e49a0c Answer B. By conceding that people may seek out information that serves no immediate purpose only because they think
Correct Answer: B they can use it later

Rationale C. By pointing out that it is challenging to determine when information-seeking serves no goal beyond acquiring
information
Choice B is the best answer. The sentence mentions that some fossil studies have relied on incomplete data
sets, suggesting that these studies are limited in what they can tell us about turtles’ origins. D. By disputing the idea that curiosity can help explain apparently purposeless information-seeking behaviors

Choice A is incorrect. While the sentence mentions the incompleteness of the data sets studied, it doesn’t
mention any tools or techniques used to examine fossils. Choice C is incorrect. The sentence doesn’t mention ID: 27d9bb69 Answer
anything about the evolution of crocodiles. Choice D is incorrect. This sentence doesn’t directly mention or Correct Answer: A
criticize any "widely held belief," and it focuses on a limitation of fossil studies of reptiles, not genetic studies.
Rationale
Question Difficulty: Easy
Choice A is the best answer. The researchers in Text 2 recognize that acquiring information is a powerful
motivator, but showed that this motivation can still be affected by other factors, like whether or not the
information is expected to be useful or not. This suggests that other desires may play a part in driving people
to acquire information.

Choice B is incorrect. The consensus view in Text 1 is that people acquire information regardless of whether
they think they can use it later. Dubey and colleagues acknowledge this fact (so they don’t claim people seek
out information “only” because it might be useful later). Choice C is incorrect. This choice misreads the results
of Dubey and colleagues’ study in Text 2. Neither text discusses the difficulty of determining the motivation for
information-seeking. Choice D is incorrect. This choice contradicts Text 2, which starts with Dubey and
colleagues “acknowledging that acquiring information is a powerful motivator” (i.e., agreeing that curiosity
explains the seeking of apparently purposeless information). The research in Text 2 simply suggests that more
than just curiosity can motivate information-seeking behavior when the information has a purpose.

Question Difficulty: Medium


Wengrow’s view that humans have flexibly shifted among various social structures, not on the importance of

Question ID c885c38b particular structures relative to others. Choice C is incorrect because Text 2 doesn’t include any information
suggesting that Graeber and Wengrow believe that hierarchies didn’t emerge until after the rise of agriculture.
In fact, Text 2 indicates that Graeber and Wengrow cite evidence suggesting that some hunter-gatherer groups
Assessment Test Domain Skill Difficulty formed social structures with hierarchical elements (“communities that included esteemed individuals”)
50,000 years ago, long before the rise of agriculture, which Text 1 says occurred around 12,000 years ago. Choice
SAT Reading and Writing Craft and Structure Cross-Text D is incorrect because there’s no information in Text 2 suggesting that Graeber and Wengrow would challenge
Connections
the assumption that groupings of hunter-gatherers were among the earliest forms of social structure.
Although Text 1 does indicate that hunter-gatherer groups are assumed to be the earliest human social system,
ID: c885c38b Text 2 says only that Graeber and Wengrow believe that some hunter-gatherer groups made use of different
social structures at different times. Text 2 doesn’t imply that Graeber and Wengrow doubt that hunter-gatherer
groups preceded most other social structures.
Text 1
Conventional wisdom long held that human social systems evolved in stages, beginning with hunter-gatherers forming
Question Difficulty: Hard
small bands of members with roughly equal status. The shift to agriculture about 12,000 years ago sparked population
growth that led to the emergence of groups with hierarchical structures: associations of clans first, then chiefdoms, and
finally, bureaucratic states.

Text 2
In a 2021 book, anthropologist David Graeber and archaeologist David Wengrow maintain that humans have always
been socially flexible, alternately forming systems based on hierarchy and collective ones with decentralized leadership.
The authors point to evidence that as far back as 50,000 years ago some hunter-gatherers adjusted their social
structures seasonally, at times dispersing in small groups but also assembling into communities that included
esteemed individuals.

Based on the texts, how would Graeber and Wengrow (Text 2) most likely respond to the “conventional wisdom”
presented in Text 1?

A. By conceding the importance of hierarchical systems but asserting the greater significance of decentralized
collective societies

B. By disputing the idea that developments in social structures have followed a linear progression through distinct
stages

C. By acknowledging that hierarchical roles likely weren’t a part of social systems before the rise of agriculture

D. By challenging the assumption that groupings of hunter-gatherers were among the earliest forms of social
structure

ID: c885c38b Answer


Correct Answer: B

Rationale

Choice B is the best answer because it describes the most likely way that Graeber and Wengrow (Text 2) would
respond to the “conventional wisdom” presented in Text 1. According to Text 1, the conventional wisdom about
human social systems is that they developed through stages, beginning with hunter-gatherer bands, then
moving to clan associations, then chiefdoms, and finally arriving at states with bureaucratic structures. Text 2
indicates that Graeber and Wengrow believe that human social systems have been flexible, shifting between
different types of structures, including both hierarchical and collective systems, and that these shifts may
have even occurred seasonally. This suggests that Graeber and Wengrow would dispute the idea that
developments in social structures have followed a linear progression through distinct stages.

Choice A is incorrect because nothing in Text 2 suggests that Graeber and Wengrow believe that decentralized
collective societies are more significant than hierarchical systems. Text 2 is focused on Graeber and
Question Difficulty: Hard

Question ID de2c2f57
Assessment Test Domain Skill Difficulty

SAT Reading and Writing Craft and Structure Cross-Text


Connections

ID: de2c2f57
Text 1
The fossil record suggests that mammoths went extinct around 11 thousand years (kyr) ago. In a 2021 study of
environmental DNA (eDNA)—genetic material shed into the environment by organisms—in the Arctic, Yucheng Wang
and colleagues found mammoth eDNA in sedimentary layers formed millennia later, around 4 kyr ago. To account for
this discrepancy, Joshua H. Miller and Carl Simpson proposed that arctic temperatures could preserve a mammoth
carcass on the surface, allowing it to leach DNA into the environment, for several thousand years.
Text 2
Wang and colleagues concede that eDNA contains DNA from both living organisms and carcasses, but for DNA to
leach from remains over several millennia requires that the remains be perpetually on the surface. Scavengers and
weathering in the Arctic, however, are likely to break down surface remains well before a thousand years have passed.

Which choice best describes how Text 1 and Text 2 relate to each other?

A. Text 1 discusses two approaches to studying mammoth extinction without advocating for either, whereas Text 2
advocates for one approach over the other.

B. Text 1 presents findings by Wang and colleagues and gives another research team’s attempt to explain those
findings, whereas Text 2 provides additional detail that calls that explanation into question.

C. Text 1 describes Wang and colleagues’ study and a critique of their methodology, whereas Text 2 offers additional
details showing that methodology to be sound.

D. Text 1 argues that new research has undermined the standard view of when mammoths went extinct, whereas Text
2 suggests a way to reconcile the standard view with that new research.

ID: de2c2f57 Answer


Correct Answer: B

Rationale

Choice B is the best answer. Text 1 introduces Wang and colleagues’ study and its surprising results, and then
mentions Miller and Simpson’s hypothesis as a possible way to explain them. Text 2, however, challenges
Miller and Simpson’s hypothesis by pointing out the difficulties of preserving mammoth carcasses on the
surface for thousands of years: “scavengers and weathering” are the additional details that complicate the
Miller/Simpson hypothesis.

Choice A is incorrect. Neither text compares two different approaches for studying mammoth extinction. Text
1 describes one study and one hypothesis pertaining to it. Text 2 critiques that hypothesis. Choice C is
incorrect. Text 1 does not describe a critique of Wang and colleagues’ methodology, but rather an interpretation
of their results by Miller and Simpson. Text 2 does not offer additional details showing that methodology to be
sound, but rather casts doubt on the Miller/Simpson explanation. Choice D is incorrect. Both components
mentioned here (the new “undermining” research and the theory for reconciling this discovery) are contained
in Text 1. Text 2 then shows how the attempt to reconcile the standard view and new research is flawed, and
still fails to explain the discrepancy.
Question ID 3f753a8e Question ID 82b7c3b2
Assessment Test Domain Skill Difficulty Assessment Test Domain Skill Difficulty

SAT Reading and Writing Craft and Structure Words in Context SAT Reading and Writing Craft and Structure Words in Context

ID: 3f753a8e ID: 82b7c3b2


Investigating whether shared false visual memories—specific but inaccurate and widely held recollections of images The following text is from Booth Tarkington’s 1921 novel Alice Adams.
such as product logos—are caused by people’s previous ______ incorrect renditions of the images, researchers Deepasri
Mrs. Adams had always been fond of vases, she said, and every year her husband’s Christmas present to her was a
Prasad and Wilma Bainbridge found that, in fact, such memories are often not explained by familiarity with erroneous
vase of one sort or another—whatever the clerk showed him, marked at about twelve or fourteen dollars.
versions of the images.
As used in the text, what does the word “marked” most nearly mean?
Which choice completes the text with the most logical and precise word or phrase?
A. Staged
A. compliance with
B. Priced
B. exposure to
C. Stained
C. criteria for
D. Watched
D. forfeiture of

ID: 82b7c3b2 Answer


ID: 3f753a8e Answer
Correct Answer: B
Correct Answer: B
Rationale
Rationale
Choice B is the best answer. The text suggests that Mrs. Adam’s typical Christmas present from her husband
Choice B is the best answer. "Exposure to" means "having contact with." It makes sense that Prasad and
was a vase that cost, or was "priced at," about twelve or fourteen dollars.
Bainbridge were investigating whether seeing false versions of images was a cause of false visual memories.
Notice how "exposure to incorrect renditions" matches the idea of "familiarity with erroneous versions," which Choice A is incorrect. This isn’t the meaning of "marked" as used here. It wouldn’t make sense to say that a
appears later in the sentence. vase was "staged at twelve or fourteen dollars." Choice C is incorrect. This isn’t the meaning of "marked" as
used here. It wouldn’t make sense to say that a vase was "stained at twelve or fourteen dollars." Choice D is
Choice A is incorrect. "Compliance with" means "going along with a command or directive." False versions of
incorrect. This isn’t the meaning of "marked" as used here. It wouldn’t make sense to say that a vase was
images can’t give commands or directives, so this doesn’t apply. Choice C is incorrect. "Criteria" means
"watched at twelve or fourteen dollars."
"standards by which to judge something." It’s not clear how people would come to have standards for the
wrong version of an image in the first place, let alone how those standards would cause them to falsely Question Difficulty: Easy
remember the correct version. In other words, this choice would result in a confusing, unclear sentence.
Choice D is incorrect. "Forfeiture of" means "a giving up of something." It wouldn’t make sense to say that false
memories of an image might be caused by giving up the wrong version of the image.

Question Difficulty: Hard

Question Difficulty: Easy

Question ID 159ef46d
Assessment Test Domain Skill Difficulty

SAT Reading and Writing Craft and Structure Cross-Text


Connections

ID: 159ef46d
Text 1
Although food writing is one of the most widely read genres in the United States, literary scholars have long neglected
it. And within this genre, cookbooks attract the least scholarly attention of all, regardless of how well written they may
be. This is especially true of works dedicated to regional US cuisines, whose complexity and historical significance are
often overlooked.

Text 2
With her 1976 cookbook The Taste of Country Cooking, Edna Lewis popularized the refined Southern cooking she had
grown up with in Freetown, an all-Black community in Virginia. She also set a new standard for cookbook writing: the
recipes and memoir passages interspersing them are written in prose more elegant than that of most novels. Yet
despite its inarguable value as a piece of writing, Lewis’s masterpiece has received almost no attention from literary
scholars.

Based on the two texts, how would the author of Text 1 most likely regard the situation presented in the underlined
sentence in Text 2?

A. As typical, because scholars are dismissive of literary works that achieve popularity with the general public

B. As unsurprising, because scholars tend to overlook the literary value of food writing in general and of regional
cookbooks in particular

C. As justifiable, because Lewis incorporated memoir into The Taste of Country Cooking, thus undermining its status as
a cookbook

D. As inevitable, because The Taste of Country Cooking was marketed to readers of food writing and not to readers of
other genres

ID: 159ef46d Answer


Correct Answer: B

Rationale

Choice B is the best answer. Text 1 states that literary scholars ignore regional cookbooks most of all, even
when they have historical significance and are well written. So the author of Text 1 wouldn’t be surprised that
scholars ignored Edna Lewis’s cookbook.

Choice A is incorrect. We can’t infer that this is how the author of Text 1 would regard the situation. Text 1
never suggests that scholars are dismissive of popular works in general. Instead, Text 1 says that scholars
ignore food writing specifically, despite its popularity—and despite the fact that it can be historically
significant and complex. Choice C is incorrect. We can’t infer that this is how the author of Text 1 would regard
the situation. Text 1 never suggests that elements of other genres should be kept out of cookbooks. Choice D is
incorrect. We can’t infer that this is how the author of Text 1 would regard the situation. Text 1 never discusses
how food writing is or should be marketed.
Question ID e1e89221 Question ID b11bb2a3
Assessment Test Domain Skill Difficulty Assessment Test Domain Skill Difficulty

SAT Reading and Writing Craft and Structure Words in Context SAT Reading and Writing Craft and Structure Words in Context

ID: e1e89221 ID: b11bb2a3


The following text is from Frances Hodgson Burnett’s 1911 novel The Secret Garden. Mary, a young girl, is outside The following text is adapted from Amy Lowell’s 1912 poem “Summer.”
trying her new jump rope.
It is summer, glorious, deep-toned summer,
The sun was shining and a little wind was blowing—not a rough wind, but one which came in delightful little The very crown of nature’s changing year
gusts and brought a fresh scent of newly turned earth with it. She skipped round the fountain garden, and up one When all her surging life is at its full.
walk and down another. To me alone it is a time of pause,
A void and silent space between two worlds,
As used in the text, what does the word “rough” most nearly mean? When inspiration lags, and feeling sleeps,
A. Harsh Gathering strength for efforts yet to come.

B. Scratchy As used in the text, what does the phrase “a void” most nearly mean?

C. Basic A. A useless

D. Vague B. An empty

C. A forgotten

ID: e1e89221 Answer D. An incomplete


Correct Answer: A

Rationale ID: b11bb2a3 Answer


Choice A is the best answer because as used in the text, "rough" most nearly means harsh, or forceful and Correct Answer: B
unpleasant. The text describes Mary’s surroundings as she plays: the sun is out and there’s "a little wind." To Rationale
further illustrate the wind, the narrator contrasts the word "rough" with a description of the wind blowing in
"delightful little gusts," suggesting that the wind is not unpleasant or harsh at all. Choice B is the best answer because as used in the text, a span of time is described as "a void" space, which
most nearly means an empty or vacant one. In the text, the speaker describes summertime in counterintuitive
Choice B is incorrect. Although in some contexts "rough" objects, or objects with irregular surfaces, can terms: although nature’s "surging life is at its full" during the season, the speaker feels summer to be "a time of
sometimes be scratchy, rough doesn’t mean scratchy in this context. The text explains that the wind is not pause, / A void and silent space between two worlds." The speaker says further that during summer, "feeling
rough but rather gentle, "delightful little gusts," which suggests that the use of "rough" here is referring to the sleeps / Gathering strength" for future efforts. Thus, the speaker regards summer as an empty stretch of time,
degree of force of the wind. Choice C is incorrect because there’s nothing in the text to suggest that the wind to be followed by a period of greater activity.
wasn’t "basic," or simple and uncomplicated. Instead, the text describes the wind as blowing not roughly or
harshly but in "delightful little gusts." Choice D is incorrect because the word "vague" means not clearly Choice A is incorrect. Although the text does present summer as a time of inactivity, it doesn’t characterize
expressed or seen. Nothing in the text indicates that the wind was barely noticeable to Mary as she played that inactivity as useless, or as having no purpose; in fact, the speaker regards summer as a time when
outside, but rather the text states that it was delightful. "feeling" gathers "strength for efforts yet to come." Choice C is incorrect. Although the text characterizes
summer as a time "when inspiration lags, and feeling sleeps," it doesn’t discuss the season’s relationship to the
Question Difficulty: Easy speaker’s memory or suggest that summer can easily be forgotten. Choice D is incorrect. In some contexts,
"void" can mean devoid of, or lacking, a particular element, and such a lack could be conceived of as
incompleteness. However, the text doesn’t portray summer as not being complete or whole; instead, it
characterizes vacancy or inactivity as being an essential quality of the season, as experienced by the speaker.

Question Difficulty: Easy

Question ID 97360a00 Question ID 9c35759f


Assessment Test Domain Skill Difficulty Assessment Test Domain Skill Difficulty

SAT Reading and Writing Craft and Structure Text Structure and SAT Reading and Writing Craft and Structure Words in Context
Purpose

ID: 9c35759f
ID: 97360a00
Novelist N. K. Jemisin declines to ______ the conventions of the science fiction genre in which she writes, and she has
The following text is adapted from Gwendolyn Bennett’s 1926 poem “Street Lamps in Early Spring.” suggested that her readers appreciate her work precisely because of this willingness to thwart expectations and avoid
Night wears a garment formulaic plots and themes.
All velvet soft, all violet blue...
And over her face she draws a veil Which choice completes the text with the most logical and precise word or phrase?
As shimmering fine as floating dew...
A. question
And here and there
In the black of her hair B. react to
The subtle hands of Night
C. perceive
Move slowly with their gem-starred light.
D. conform to
Which choice best describes the overall structure of the text?

A. It presents alternating descriptions of night in a rural area and in a city.


ID: 9c35759f Answer
B. It sketches an image of nightfall, then an image of sunrise.
Correct Answer: D
C. It makes an extended comparison of night to a human being. Rationale

D. It portrays how night changes from one season of the year to the next.
Choice D is the best answer because it most logically completes the text’s discussion of Jemisin’s writing. In
this context, “conform to” means to act in accordance with something. The text suggests that in her science
fiction writing, Jemisin’s willingness to go against expectations and not use plots and themes that seem to
ID: 97360a00 Answer
follow a formula reflects how she treats the standard practices of the genre. This context conveys that Jemisin
Correct Answer: C chooses not to act in accordance with those conventions.
Rationale
Choice A is incorrect. In this context, “question” would mean doubt or object to. The text indicates that Jemisin
Choice C is the best answer because it most accurately describes the overall structure of the text. Throughout is willing to go against expectations and not use formulaic plots and themes in her science fiction writing,
the text, the speaker characterizes nighttime as if it were a person who wears clothing (“a garment” that is suggesting that she may actually object to those conventions of the genre, not that she chooses not to question
“velvet soft” and “violet blue”) and a veil “over her face” and who moves her hands “slowly with their gem- them. Choice B is incorrect because the text indicates that in her science fiction writing, Jemisin is willing to
starred light” through her dark hair. Thus, the text is structured as an extended comparison of night to a go against expectations and not use formulaic plots and themes. Rather than suggesting that Jemisin chooses
human being. not to “react to,” or act in response to, the standard practices of the genre, this context suggests that she is
acting in response to such conventions by deliberately avoiding them. Choice C is incorrect. In this context,
Choice A is incorrect because the text never mentions any particular location; instead, it focuses on “perceive” would mean become aware of or understand. The text indicates that in her science fiction writing,
presenting a single description of night as a person with certain clothing and features. Choice B is incorrect Jemisin is willing to go against expectations and not use formulaic plots and themes. This context conveys
because the text doesn’t make any reference to the sun or sunrise; instead, it focuses on presenting a single that Jemisin is aware of and deliberately avoids those conventions of the genre, not that she chooses not to be
image of night as a person with certain clothing and features. Choice D is incorrect. Rather than describing aware of them.
how nighttime changes seasonally (or in any other way), the text presents a single image of night as a person
with certain clothing and features. Question Difficulty: Medium

Question Difficulty: Medium


Question ID 6a1194e8 Question ID 4fa7e50e
Assessment Test Domain Skill Difficulty Assessment Test Domain Skill Difficulty

SAT Reading and Writing Craft and Structure Words in Context SAT Reading and Writing Craft and Structure Words in Context

ID: 6a1194e8 ID: 4fa7e50e


Rydra Wong, the protagonist of Samuel R. Delany’s 1966 novel Babel-17, is a poet, an occupation which, in Delany’s According to a US tax policy expert, state taxes are ______ other factors when considering an interstate move. Even
work, is not ______: nearly a dozen of the characters that populate his novels are poets or writers. significant differences in state taxation have almost no effect on most people’s decisions, while differences in
employment opportunities, housing availability, and climate are strong influences.
Which choice completes the text with the most logical and precise word or phrase?
Which choice completes the text with the most logical and precise word or phrase?
A. infallible
A. consistent with
B. atypical
B. representative of
C. lucrative
C. overshadowed by
D. tedious
D. irrelevant to

ID: 6a1194e8 Answer


Correct Answer: B
ID: 4fa7e50e Answer
Correct Answer: C
Rationale
Rationale
Choice B is the best answer because it most logically completes the text’s discussion of Samuel R. Delany’s
character Rydra Wong. As used in this context, “atypical” would mean unrepresentative or not common. The Choice C is the best answer because it most logically completes the text’s discussion of the factors that
text indicates that Wong is one of “nearly a dozen” characters in Delany’s novels who are poets or writers. This influence peoples’ decisions to move to a different state. As used in this context, “overshadowed by” means to
context conveys that being a poet isn’t an atypical occupation for a character in one of Delany’s works. be surpassed by or caused to seem less important than other factors affecting a move. The text indicates that,
according to a US tax policy expert, when people think about an interstate move, state taxes have little effect
Choice A is incorrect because “infallible” means to be accurate or without fault, which wouldn’t make sense in on their decisions, while employment opportunities, housing availability, and climate have a very strong
context. The text focuses on the fact that Delany has written many characters who are poets and writers. This effect. This context suggests that people consider these other factors to be more important than state taxes.
context suggests that the occupation isn’t atypical for Delany, not that the occupation isn’t infallible, or
problematic. Choice C is incorrect because “lucrative” means to be profitable, which wouldn’t make sense in Choice A is incorrect because the text indicates that state taxes aren’t as important a consideration as other
context. If writing poet characters weren’t profitable, it wouldn’t be logical to explain this by citing that Delany factors when people are thinking of moving to another state. The context doesn’t suggest that state taxes are
gave many of his characters the same occupation. Choice D is incorrect because “tedious” means to be boring, “consistent with,” or in agreement with these other factors. Choice B is incorrect because it wouldn’t make
which wouldn’t make sense in context. The text focuses on the fact that Delany has written many characters sense in context to say that state taxes are “representative of,” or typical of, other factors. Taxes aren’t an
who are poets and writers. This context suggests that the occupation isn’t atypical for Delany, not that the example of employment opportunities, housing availability, and climate, which are the other factors listed in
occupation isn’t tedious. the text. Choice D is incorrect because it wouldn’t make sense in context to say that state taxes are “irrelevant
to,” or unconnected or unimportant to other factors. State taxes are irrelevant to peoples’ decisions, not to other
Question Difficulty: Medium factors. In other words, although the text suggests that state taxes may be irrelevant to people considering a
move to another state, the other factors mentioned in the text, such as employment opportunities, are unable
to have an opinion about state taxes. Furthermore, the text indicates that significant differences in state taxes
have almost no effect on peoples’ choices to move, but they aren’t completely unimportant.

Question Difficulty: Hard

Question ID aa7fc89b Question ID aad56f2b


Assessment Test Domain Skill Difficulty Assessment Test Domain Skill Difficulty

SAT Reading and Writing Craft and Structure Text Structure and SAT Reading and Writing Craft and Structure Words in Context
Purpose

ID: aad56f2b
ID: aa7fc89b
As a young photographer in the 1950s, William Klein ______ the conventions of photography by creating images that
The following text is adapted from Susan Glaspell’s 1912 short story “‘Out There.’” An elderly shop owner is looking at were high contrast and included blurred and distorted elements—features generally seen as flaws. So unorthodox was
a picture that he recently acquired and hopes to sell. Klein’s work that he had difficulty finding a publisher for his now-iconic 1956 photo book Life is Good & Good for You in
It did seem that the picture failed to fit in with the rest of the shop. A persuasive young fellow who claimed he was New York.
closing out his stock let the old man have it for what he called a song. It was only a little out-of-the-way store which
subsisted chiefly on the framing of pictures. The old man looked around at his views of the city, his pictures of cats Which choice completes the text with the most logical and precise word or phrase?
and dogs, his flaming bits of landscape. “Don’t belong in here,” he fumed.
A. reviewed
And yet the old man was secretly proud of his acquisition. There was a hidden dignity in his scowling as he shuffled
about pondering the least ridiculous place for the picture. B. defied

Which choice best states the main purpose of the text? C. respected

A. To reveal the shop owner’s conflicted feelings about the new picture D. prevented

B. To convey the shop owner’s resentment of the person he got the new picture from
ID: aad56f2b Answer
C. To describe the items that the shop owner most highly prizes
Correct Answer: B
D. To explain differences between the new picture and other pictures in the shop
Rationale

ID: aa7fc89b Answer Choice B is the best answer. "Defied" means "resisted" or "deliberately disobeyed," which matches the way
Klein broke from the conventions of photography in his time by including features that were generally seen as
Correct Answer: A
flaws.
Rationale
Choice A is incorrect. "Reviewed" means "analyzed" or "evaluated," but that doesn’t really fit the context here.
Choice A is the best answer because it most accurately describes the main purpose of the text. The text begins Klein isn’t directly providing any thoughtful examination of the conventions of photography: he’s just breaking
by stating that the new picture “failed to fit in” with the other items that the shop owner has. The text goes on all the rules. Choice C is incorrect. "Respected" can mean "admired" or "followed" (as in the case of
to illustrate that point by describing the other pictures the shop owner has, indicating that the shop owner is conventions). It’s clear that Klein didn’t respect conventions, given his use of photographic features that were
fuming because he doesn’t think the new picture belongs in the store. In the second paragraph, however, the generally considered "flaws." Choice D is incorrect. "Prevented" means "stopped," and Klein did not stop the
text indicates that the shop owner is “secretly proud of his acquisition.” The main purpose of the text is thus to conventions—they still existed. Instead, he made images that were seen as "flawed" under those conventions.
reveal the shop owner’s conflicted feelings about the new picture.
Question Difficulty: Easy
Choice B is incorrect because the text doesn’t suggest that the shop owner resents the young man who sold
him the new picture; in fact, the text gives no indication of the owner’s feelings about the young man at all.
Choice C is incorrect. Although the text indicates that the new picture is different from the other items in the
shop, there’s no suggestion that the shop owner prizes either the new picture or the pictures of the city, pets,
and landscapes more than he prizes any other items. Choice D is incorrect because the text doesn’t describe
what the new picture looks like; rather, the text identifies some of the other kinds of images that the shop
owner has and states that they’re different from the new picture without explaining how they’re different.

Question Difficulty: Medium


Question ID 48555763 Question ID e7247766
Assessment Test Domain Skill Difficulty Assessment Test Domain Skill Difficulty

SAT Reading and Writing Craft and Structure Text Structure and SAT Reading and Writing Craft and Structure Text Structure and
Purpose Purpose

ID: 48555763 ID: e7247766


The following text is from Herman Melville’s 1854 novel The Lightning-rod Man. Horizontal gene transfer occurs when an organism of one species acquires genetic material from an organism of
The stranger still stood in the exact middle of the cottage, where he had first planted himself. His singularity impelled a another species through nonreproductive means. The genetic material can then be transferred “vertically” in the
closer scrutiny. A lean, gloomy figure. Hair dark and lank, mattedly streaked over his brow. His sunken pitfalls of eyes second species—that is, through reproductive inheritance. Scientist Atma Ivancevic and her team have hypothesized
were ringed by indigo halos, and played with an innocuous sort of lightning: the gleam without the bolt. The whole infection by invertebrate parasites as a mechanism of horizontal gene transfer between vertebrate species: while
man was dripping. He stood in a puddle on the bare oak floor: his strange walking-stick vertically resting at his side. feeding, a parasite could acquire a gene from one host, then relocate to a host from a different vertebrate species and
transfer the gene to it in turn.
Which choice best states the function of the underlined sentence in the overall structure of the text?
Which choice best describes the function of the underlined portion in the text as a whole?
A. It elaborates on the previous sentence’s description of the character.
A. It explains why parasites are less susceptible to horizontal gene transfer than their hosts are.
B. It introduces the setting that is described in the sentences that follow.
B. It clarifies why some genes are more likely to be transferred horizontally than others are.
C. It establishes a contrast with the description in the previous sentence.
C. It contrasts how horizontal gene transfer occurs among vertebrates with how it occurs among invertebrates.
D. It sets up the character description presented in the sentences that follow.
D. It describes a means by which horizontal gene transfer might occur among vertebrates.

ID: 48555763 Answer


Correct Answer: D ID: e7247766 Answer
Correct Answer: D
Rationale
Rationale
Choice D is the best answer. This best states the function of the underlined sentence. The sentence basically
says: “He stood out, so I looked more closely at him.” Then the rest of the text describes him in detail. Choice D is the best answer. The text defines horizontal gene transfer and then gives one possibility for how it
happens in vertebrates (via infection by parasites). The underlined part describes how that mechanism could
Choice A is incorrect. This doesn’t state the function of the underlined sentence. The previous sentence work.
basically says: “He was still standing in the middle of the cottage”—it doesn’t include any description of the
character himself. Choice B is incorrect. This doesn’t state the function of the underlined sentence. The Choice A is incorrect. The underlined portion doesn’t do this. Parasites are only described as the mechanism
following sentences describe the character, not the setting. Choice C is incorrect. This doesn’t state the that does the transferring, not the species that gives or receives the genes. Choice B is incorrect. The
function of the underlined sentence. The underlined sentence basically says: “He stood out, so I looked more underlined portion doesn’t do this. The text never discusses which genes are more likely to be transferred.
closely at him.” The previous sentence basically says: “He was still standing in the middle of the cottage.” Choice C is incorrect. The underlined portion doesn’t do this. The text never discusses how horizontal gene
There’s no contrast between these two sentences. transfer occurs among invertebrates.

Question Difficulty: Medium Question Difficulty: Medium

Question ID ae2b3112 Question ID e7b709fc


Assessment Test Domain Skill Difficulty Assessment Test Domain Skill Difficulty

SAT Reading and Writing Craft and Structure Text Structure and SAT Reading and Writing Craft and Structure Words in Context
Purpose

ID: e7b709fc
ID: ae2b3112
Archaeologists studying an ancient amphitheater in Switzerland believe that it dates back to the fourth century CE.
By combining Indigenous and classical music, Cree composer and cellist Cris Derksen creates works that reflect the Their discoveries of a coin made between 337 and 341 CE and era-appropriate building materials ______ evidence for
diverse cultural landscape of Canada. For her album Orchestral Powwow, Derksen composed new songs in the style of this theory.
traditional powwow music that were accompanied by classical arrangements played by an orchestra. But where an
orchestra would normally follow the directions of a conductor, the musicians on Orchestral Powwow are led by the beat Which choice completes the text with the most logical and precise word or phrase?
of a powwow drum.
A. dismiss

Which choice best states the main purpose of the text? B. provide
A. To examine how Derksen’s musical compositions blend cultures
C. regulate
B. To argue that Derksen should be recognized for creating a new style of music
D. refuse
C. To describe the difficulties Derksen encountered when producing her album

D. To establish a contrast between Derksen’s classical training and her Cree heritage ID: e7b709fc Answer
Correct Answer: B

ID: ae2b3112 Answer Rationale

Correct Answer: A Choice B is the best answer because it most logically completes the text’s discussion of the archaeologists’
Rationale study of the ancient amphitheater in Switzerland. In this context, “provide” means make available or supply.
The text states that the archaeologists believe that the amphitheater dates to the fourth century CE. The text
Choice A is the best answer because it most accurately describes the main purpose of the text, which is to goes on to say that the archaeologists discovered a coin made between 337 and 341 CE (that is, made during
discuss how Derksen’s compositions incorporate elements from both Indigenous and classical music. After the fourth century CE) and building materials appropriate to the era in question. This context suggests that
introducing Derksen, the text describes how the songs Derksen composed for her album Orchestral Powwow these discoveries provide evidence for the archaeologists’ theory about the dating of the amphitheater.
feature aspects of the two musical traditions. Specifically, the text notes that Derksen wrote songs in the style
of traditional powwow music but accompanied them with classical arrangements played by an orchestra that Choice A is incorrect because the archaeologists’ discoveries are presented as supplying evidence in favor of
followed the beat of a powwow drum rather than the directions of a conductor. In this way, Derksen’s their theory about the dating of the amphitheater, not something that would “dismiss,” or reject serious
compositions blend different cultures. consideration of, evidence for that theory. Choice C is incorrect because nothing in the text suggests that the
archaeologists’ discoveries would “regulate,” or govern or bring order to, evidence for the archaeologists’ theory
Choice B is incorrect because although the text suggests that Derksen’s songs contain innovative elements about the dating of the amphitheater. The discoveries are presented as supplying evidence for the
since they blend styles from two different musical traditions, it doesn’t discuss whether her compositions archaeologists’ theory, not as changing how evidence for the theory is controlled or ordered. Choice D is
constitute a new style of music, let alone whether Derksen should be recognized for creating a new style of incorrect because the archaeologists’ discoveries are presented as supplying evidence in favor of their theory
music. Choice C is incorrect because the text doesn’t mention any difficulties Derksen encountered when about the dating of the amphitheater, not something that would “refuse,” or be unwilling to accept, evidence for
producing her album. Rather, the text describes how the songs on the album exemplify how Derksen combines the archaeologists’ theory.
music from two different cultures. Choice D is incorrect because although the text mentions Derksen’s Cree
heritage and suggests that she relies on knowledge of both Indigenous and classical music when she Question Difficulty: Easy
composes her songs, it doesn’t discuss her musical training. Additionally, the text is primarily focused on how
Derksen combines different cultural traditions, not on contrasting Derksen’s training with her heritage.

Question Difficulty: Medium


Question ID 94eb800d Question ID 7b55e895
Assessment Test Domain Skill Difficulty Assessment Test Domain Skill Difficulty

SAT Reading and Writing Craft and Structure Words in Context SAT Reading and Writing Craft and Structure Cross-Text
Connections

ID: 94eb800d
ID: 7b55e895
For a 2020 exhibition, photographer and neurobiologist Okunola Jeyifous ______ a series of new images based on a
series of alphabet posters from the 1970s known as the “Black ABCs,” which featured Black children from Chicago. Text 1
Jeyifous photographed the now-adult models and layered the photos over magnified images of the models’ cells, Some animal species, like the leopard, can be found in many kinds of areas. On the other hand, tropical mountain bird
resulting in what he called “micro and macro portraiture.” species tend to be limited in the types of spaces they can call home. This is because many mountain bird species are
only able to survive at very specific elevations. Over time, these species have likely become used to living at a specific
Which choice completes the text with the most logical and precise word or phrase? temperature. Therefore, these species struggle to survive at elevations that are warmer or colder than they are used to.

A. validated
Text 2
B. created A new study reviewed observations of nearly 3,000 bird species to understand why tropical mountain bird species live
at specific elevations. They noted that when a mountain bird species was found in an area with many other bird
C. challenged
species, it tended to inhabit much smaller geographic areas. It is thus likely that competition for resources with other
D. restored species, not temperature, limits where these birds can live.

Based on the texts, both authors would most likely agree with which statement?
ID: 94eb800d Answer A. Tropical mountain bird species are restricted in where they can live.
Correct Answer: B
B. Scientists have better tools to observe tropical mountain birds than they did in the past.
Rationale
C. Little is known about how tropical mountain birds build their nests.
Choice B is the best answer because it most logically and precisely completes the text’s discussion of
D. Tropical mountain bird species that live at high elevations tend to be genetically similar.
Jeyifous’s series of images for the 2020 exhibition. In this context, “created” means produced. The text
explains that Jeyifous, a photographer and neurobiologist, photographed adults who had appeared as children
in posters from the 1970s, then combined those photographs with magnified images of the adults’ cells—a ID: 7b55e895 Answer
process that resulted in what he called “micro and macro portraiture.” This context suggests that Jeyifous
Correct Answer: A
drew on his dual interests in photography and neurobiology to produce the images for display in the
exhibition. Rationale

Choice A is incorrect because there’s nothing in the text to suggest that Jeyifous “validated,” or corroborated, Choice A is the best answer. Both texts state that tropical mountain bird species have limited ranges or
the series of images. The text describes Jeyifous’s process for composing the images but doesn’t describe habitats, although they disagree on the reason for this. Text 1 claims that temperature is the main factor that
Jeyifous making an effort to evaluate the images for their artistic or scientific legitimacy. Choice C is incorrect determines where these birds can live, while Text 2 claims that competition with other species is the main
because there’s nothing in the text to suggest that Jeyifous “challenged,” or disputed, an aspect of the images; factor. However, both texts agree that these birds are not able to survive in many kinds of areas.
rather, the focus of the text is on the inspiration behind the images and the method Jeyifous used to achieve
them. Choice D is incorrect because the text indicates that Jeyifous made the images himself using a Choice B is incorrect. Neither text mentions the tools or methods that scientists use to observe these birds,
combination of photography and magnified pictures of cells, not that he “restored,” or reconditioned, the either now or in the past. Choice C is incorrect. Neither text mentions anything about how these birds build
images from a deteriorated state. their nests. Choice D is incorrect. Neither text provides any information about the genetic similarity of these
birds, so we have no evidence that either author would agree with this statement.
Question Difficulty: Medium
Question Difficulty: Easy

Question ID 8bc66f89 Question ID c106b9f7


Assessment Test Domain Skill Difficulty Assessment Test Domain Skill Difficulty

SAT Reading and Writing Craft and Structure Text Structure and SAT Reading and Writing Craft and Structure Cross-Text
Purpose Connections

ID: 8bc66f89 ID: c106b9f7


Part of the Atacama Desert in Peru has surprisingly rich plant life despite receiving almost no rainfall. Moisture from Text 1
winter fog sustains plants once they’re growing, but the soil’s tough crust makes it hard for seeds to germinate in the American sculptor Edmonia Lewis is best known for her sculptures that represent figures from history and mythology,
first place. Local birds that dig nests in the ground seem to be of help: they churn the soil, exposing buried seeds to such as The Death of Cleopatra and Hagar. Although Lewis sculpted other subjects, her career as a sculptor is best
moisture and nutrients. Indeed, in 2016 Cristina Rengifo Faiffer found that mounds of soil dug up by birds were far represented by the works in which she depicted these historical and mythical themes.
more fertile and supported more seedlings than soil in undisturbed areas.
Text 2
Which choice best describes the function of the underlined portion in the text as a whole? Art historians have typically ignored the many portrait busts Edmonia Lewis created. Lewis likely carved these busts
A. It elaborates on the idea that the top layer of Atacama Desert soil forms a tough crust. (sculptures of a person’s head) frequently throughout her long career. She is known for her sculptures that represent
historical figures, but Lewis likely supported herself financially by carving portrait busts for acquaintances who paid her
B. It describes the process by which seeds are deposited into Atacama Desert soil. to represent their features. Thus, Lewis’s portrait busts are a central aspect of her career as a sculptor.

C. It identifies the reason particular bird species dig nests in Atacama Desert soil. Based on the texts, both authors would most likely agree with which statement?
D. It explains how certain birds promote seed germination in Atacama Desert soil. A. Lewis’s portrait busts have overshadowed her other work.

B. The Death of Cleopatra is Lewis’s most famous piece.


ID: 8bc66f89 Answer
C. Sculpting representations of historical figures was a short-lived trend.
Correct Answer: D
D. Lewis’s works are varied in the subjects they depict.
Rationale

Choice D is the best answer because it most accurately describes how the underlined portion functions in the
ID: c106b9f7 Answer
text as a whole. The first two sentences establish a natural phenomenon: there is a richness of plant life found
in the Atacama Desert despite the hard soil that makes it challenging for seeds to germinate. The next Correct Answer: D
sentence, which contains the underlined portion, offers a potential explanation for the phenomenon: local Rationale
birds dig ground nests exposing seeds to moisture and materials in the soil necessary for germination. The
last sentence summarizes a study that compared the fertileness of mounds of dirt dug up by birds to mounds Choice D is the best answer. Author 1 acknowledges that Lewis sculpted other subjects besides historical and
that were undisturbed to support the explanation in the underlined portion. Thus, the underlined portion mythical figures, suggesting a variety of subjects depicted. Author 2 mentions that Lewis carved portrait busts
mainly functions to explain how certain birds promote seed germination in the Atacama Desert soil. as well as historical sculptures, which also implies variation among Lewis’s subjects.

Choice A is incorrect because the underlined portion doesn’t address the topic of the soil’s tough crust or its Choice A is incorrect. Neither text suggests that Lewis’s portrait busts have received more attention or
formation. Instead, the text elaborates on the idea that local birds that build ground nests may help seeds appreciation than her other work. Author 1 briefly mentions her “other works,” but mostly focuses on her
germinate in the hard soil. Choice B is incorrect because the underlined portion describes how some birds historical and mythical works. Author 2 states that art historians have typically ignored her portrait busts,
may support seed germination in Atacama Desert soil but doesn’t describe how the seeds are deposited into which suggests that they haven’t overshadowed her other work. Choice B is incorrect. Neither text explicitly
the soil before germination begins. Choice C is incorrect because neither the underlined portion nor the text states that The Death of Cleopatra is Lewis’s most famous piece. Author 1 mentions it as one example of her
as a whole identifies a reason that a particular bird species may choose to dig ground nests in the Atacama historical works, but does not single it out as being more important or influential than Hagar. Author 2 does
Desert soil. not mention it at all, focusing instead on her portrait busts. Choice C is incorrect. This choice isn’t supported
by the texts. Neither text suggests that sculpting historical figures was a trend that faded quickly.
Question Difficulty: Medium
Question Difficulty: Easy
phytoplankton compete for resources with larger nonphytoplankton species. Choice D is incorrect because

Question ID 6977d22b according to Text 2, Behrenfeld and colleagues argue that water density decreases, not increases, competition
between phytoplankton species.

Assessment Test Domain Skill Difficulty Question Difficulty: Hard

SAT Reading and Writing Craft and Structure Cross-Text


Connections

ID: 6977d22b

Text 1
Ecologists have long wondered how thousands of microscopic phytoplankton species can live together near ocean
surfaces competing for the same resources. According to conventional wisdom, one species should emerge after
outcompeting the rest. So why do so many species remain? Ecologists’ many efforts to explain this phenomenon still
haven’t uncovered a satisfactory explanation.

Text 2
Ecologist Michael Behrenfeld and colleagues have connected phytoplankton’s diversity to their microscopic size.
Because these organisms are so tiny, they are spaced relatively far apart from each other in ocean water and, moreover,
experience that water as a relatively dense substance. This in turn makes it hard for them to move around and interact
with one another. Therefore, says Behrenfeld’s team, direct competition among phytoplankton probably happens much
less than previously thought.

Based on the texts, how would Behrenfeld and colleagues (Text 2) most likely respond to the “conventional wisdom”
discussed in Text 1?

A. By arguing that it is based on a misconception about phytoplankton species competing with one another

B. By asserting that it fails to recognize that routine replenishment of ocean nutrients prevents competition between
phytoplankton species

C. By suggesting that their own findings help clarify how phytoplankton species are able to compete with larger
organisms

D. By recommending that more ecologists focus their research on how competition among phytoplankton species is
increased with water density

ID: 6977d22b Answer


Correct Answer: A

Rationale

Choice A is the best answer because based on Text 2, it represents how Behrenfeld and colleagues would most
likely respond to the “conventional wisdom” discussed in Text 1. The conventional wisdom cited holds the
opinion that when there is species diversity within a phytoplankton population, “one species should emerge
after outcompeting the rest”—that is, after being so successful in competing for resources that the other
species vanish from the population. However, Text 2 explains that according to Behrenfeld and colleagues,
phytoplankton are so small and spaced so far apart in the water that there is “much less” direct competition
for resources within phytoplankton populations than scientists had previously thought.

Choice B is incorrect because Text 2 never discusses whether routine replenishment of ocean nutrients affects
competition between phytoplankton species. Choice C is incorrect because the interspecies competition
discussed in both texts is specifically between phytoplankton species, and neither text considers whether

Question ID 213248f7 Question ID 6bc0ba75


Assessment Test Domain Skill Difficulty Assessment Test Domain Skill Difficulty

SAT Reading and Writing Craft and Structure Words in Context SAT Reading and Writing Craft and Structure Text Structure and
Purpose

ID: 213248f7
ID: 6bc0ba75
The following text is adapted from Lewis Carroll’s 1865 novel Alice’s Adventures in Wonderland.
The mimosa tree evolved in East Asia, where the beetle Bruchidius terrenus preys on its seeds. In 1785, mimosa trees
“The second thing is to find my way into that lovely garden. I think that will be the best plan.” It sounded like an were introduced to North America, far from any B. terrenus. But evolutionary links between predators and their prey
excellent plan, no doubt, and very neatly and simply arranged; the only difficulty was, that Alice had not the can persist across centuries and continents. Around 2001, B. terrenus was introduced in southeastern North America
smallest idea how to set about it. near where botanist Shu-Mei Chang and colleagues had been monitoring mimosa trees. Within a year, 93 percent of
the trees had been attacked by the beetles.
As used in the text, what does the word “simply” most nearly mean?

A. Faintly Which choice best describes the function of the third sentence in the overall structure of the text?

A. It states the hypothesis that Chang and colleagues had set out to investigate using mimosa trees and B. terrenus.
B. Hastily
B. It presents a generalization that is exemplified by the discussion of the mimosa trees and B. terrenus.
C. Easily
C. It provides context that clarifies why the species mentioned spread to new locations.
D. Foolishly
D. It offers an alternative explanation for the findings of Chang and colleagues.

ID: 213248f7 Answer


Correct Answer: C ID: 6bc0ba75 Answer
Rationale Correct Answer: B

Rationale
Choice C is the best answer because as used in the text, “simply” most nearly means easily, or involving
minimal difficulty or effort. The text first provides Alice’s reflections on her plan to gain access to a garden and Choice B is the best answer because it most accurately describes the function of the third sentence within the
then offers commentary on her plan by the novel’s narrator. The text indicates that a reason Alice likes her overall structure of the text. The third sentence makes a generalization, asserting that evolutionary links
plan despite not being fully thought through is that she nonetheless believes it can be efficiently arranged. In between predators and prey can persist across great expanses of time and distance. This generalization is
other words, the text indicates that one of the supposed benefits of Alice’s plan is that it can be easily exemplified by the text’s discussion of the relationship between mimosa trees and B. terrenus beetles. When
arranged. mimosa trees were introduced to North America in 1785, no B. terrenus beetles were present, so the
relationship between the trees and the beetles that exists in their native East Asia was disrupted. When the
Choice A is incorrect because the text describes how Alice’s plan can be arranged, and it wouldn’t make sense
beetles were introduced to North America more than 200 years later, however, they quickly attacked mimosa
to say that it can be arranged “faintly,” or with little strength or not strongly. Instead, the text indicates that the
trees, illustrating the generalization that links between predators and prey "can persist across centuries and
plan can be arranged with little difficulty. Choice B is incorrect. Although in some contexts “simply” can mean
continents."
quickly, hastily, or hurriedly, the word “hastily” indicates that something is done too quickly. Although it may
be true that Alice’s plan was made in haste, the text doesn’t focus on this aspect of her plan. Instead, the text Choice A is incorrect because the third sentence doesn’t indicate that Chang and colleagues were
focuses on the plan’s seemingly good qualities, saying that Alice thinks of it as “the best,” and the narrator investigating any hypothesis. According to the text, Chang and colleagues were simply monitoring mimosa
refers to it as “excellent” and “neatly,” or efficiently, arranged. Choice D is incorrect. Although in some contexts trees when the beetles happened to be introduced to the area. Choice C is incorrect because the third sentence
“simply” can mean foolishly, or lacking good sense, it doesn’t have this meaning in this context. Although the doesn’t discuss any particular species, let alone the species mentioned elsewhere in the text, nor does the
text says that Alice doesn’t know how to go about her plan, it begins by presenting her plan in a positive light: sentence explain why species spread to new locations. Choice D is incorrect because the third sentence offers
Alice describes her plan as “the best,” and the narrator refers to the plan as “excellent” and “neatly,” or a generalization about the relationship between predators and prey, not an explanation for the findings by
efficiently, arranged. Chang and colleagues that’s an "alternative" to an explanation presented elsewhere in the text.

Question Difficulty: Easy Question Difficulty: Hard


Question ID b4d29611 Question ID f6352bd3
Assessment Test Domain Skill Difficulty Assessment Test Domain Skill Difficulty

SAT Reading and Writing Craft and Structure Text Structure and SAT Reading and Writing Craft and Structure Text Structure and
Purpose Purpose

ID: b4d29611 ID: f6352bd3


Michelene Pesantubbee, a historian and citizen of the Choctaw Nation, has identified a dilemma inherent to research Many archaeologists assume that large-scale engineering projects in ancient societies required an elite class to plan
on the status of women in her tribe during the 1600s and 1700s: the primary sources from that era, travel narratives and direct the necessary labor. However, recent discoveries, such as the excavation of an ancient canal near the Gulf
and other accounts by male European colonizers, underestimate the degree of power conferred on Choctaw women by Coast of Alabama, have complicated this picture. Using radiocarbon dating, a team of researchers concluded that the
their traditional roles in political, civic, and ceremonial life. Pesantubbee argues that the Choctaw oral tradition and 1.39-kilometer-long canal was most likely constructed between 576 and 650 CE by an Indigenous society that was
findings from archaeological sites in the tribe’s homeland supplement the written record by providing crucial insights relatively free of social classes.
into those roles.
Which choice best describes the overall structure of the text?
Which choice best describes the overall structure of the text?
A. It describes a common view among archaeologists, then discusses a recent finding that challenges that view.
A. It details the shortcomings of certain historical sources, then argues that research should avoid those sources
B. It outlines a method used in some archaeological fieldwork, then explains why an alternative method is superior to
altogether.
it.
B. It describes a problem that arises in research on a particular topic, then sketches a historian’s approach to
C. It presents contradictory conclusions drawn by archaeologists, then evaluates a study that has apparently resolved
addressing that problem.
that contradiction.
C. It lists the advantages of a particular research method, then acknowledges a historian’s criticism of that method.
D. It identifies a gap in scientific research, then presents a strategy used by some archaeologists to remedy that gap.
D. It characterizes a particular topic as especially challenging to research, then suggests a related topic for historians
to pursue instead.
ID: f6352bd3 Answer
Correct Answer: A
ID: b4d29611 Answer
Rationale
Correct Answer: B

Rationale Choice A is the best answer. The text starts by introducing a common view among archaeologists about the
need for an elite class to direct large-scale engineering projects. Then, it discusses the discovery of a large
Choice B is the best answer. The text begins by stating a problem with research on the status of Choctaw canal most likely built by a society without an elite class, which challenges the first view.
women in the 1600s and 1700s: written primary sources underestimate the power they had in their traditional
roles. Then it presents one historian’s solution: looking to oral tradition and archeological findings for more Choice B is incorrect. Although the text discusses carbon dating as an archaeological method, it doesn’t
insight into these roles. compare it to any other alternative methods. Choice C is incorrect. The study doesn’t resolve any
contradictions—rather, it introduces a contradiction to the one view presented at the beginning of the text.
Choice A is incorrect. This isn’t the overall structure. The text never says that research should avoid written Choice D is incorrect. The text never identifies any gaps in scientific research.
primary sources, just that research should also use oral tradition and archeological sites as sources. Choice C
is incorrect. This isn’t the overall structure. The text never mentions the advantages of using written primary Question Difficulty: Medium
sources. Choice D is incorrect. This isn’t the overall structure. The text never says that the status of Choctaw
women during the 1600s and 1700s is too challenging to research. And it doesn’t mention any other topics to
research instead.

Question Difficulty: Medium

Question ID 5fa165f7 Question ID 571cf537


Assessment Test Domain Skill Difficulty Assessment Test Domain Skill Difficulty

SAT Reading and Writing Craft and Structure Words in Context SAT Reading and Writing Craft and Structure Words in Context

ID: 5fa165f7 ID: 571cf537


In the 1960s, Sam Gilliam, a Black painter from the southern United States, became the first artist to drape painted The author’s claim about the relationship between Neanderthals and Homo sapiens is ______, as it fails to account for
canvases into flowing shapes. He later explored a different style, ______ quilt-like paintings inspired by the patchwork several recent archaeological discoveries. To be convincing, his argument would need to address recent finds of
quilting tradition of Black communities in the South. additional hominid fossils, such as the latest Denisovan specimens and Homo longi.

Which choice completes the text with the most logical and precise word or phrase? Which choice completes the text with the most logical and precise word or phrase?

A. predicting A. disorienting

B. refusing B. tenuous

C. hiding C. nuanced

D. creating D. unoriginal

ID: 5fa165f7 Answer ID: 571cf537 Answer


Correct Answer: D Correct Answer: B

Rationale Rationale

Choice D is the best answer because it most logically completes the text’s discussion of Sam Gilliam’s Choice B is the best answer because it most logically completes the text’s discussion of the author’s claim
artworks. As used in this context, “creating” means producing or bringing something into existence. The text about the relationship between Neanderthals and Homo sapiens. As used in this context, “tenuous” means
indicates that Gilliam is an artist who made draped canvases and, later, quilt-like paintings. This context lacking substance. The end of the first sentence states that the author’s claim didn’t consider certain key
supports the idea that Gilliam explored different styles in his art by creating special types of paintings. pieces of evidence—“recent archaeological discoveries”—and is therefore weak.

Choice A is incorrect because the text indicates that Gilliam actually explored and pursued the creation of Choice A is incorrect because it wouldn’t make sense in context to refer to the author’s claim as “disorienting,”
quilt-like paintings; he wasn’t just “predicting,” or declaring in advance, the existence of these paintings. or confusing. The text suggests that the author’s claim is insubstantial, not that it’s difficult to grasp. Choice C
Choice B is incorrect because in this context “refusing” would mean rejecting, and there is nothing in the text is incorrect because referring to the claim as “nuanced,” or subtle, wouldn’t make sense in context. According
to suggest that Gilliam rejected his quilt-like paintings. Instead, the text indicates that he was exploring and to the text, the claim is incomplete because it didn’t consider certain key information about recent
pursuing a new art style in these paintings. Choice C is incorrect because in this context “hiding” would mean archaeological finds; it doesn’t suggest that what’s in the claim lacks precision. Choice D is incorrect because
concealing from view, and there is nothing in the text to suggest that Gilliam attempted to conceal his quilt- saying that the claim is “unoriginal,” or imitative, wouldn’t make sense in context. The text faults the claim
like paintings. Instead, the text indicates that he was exploring and pursuing a new art style in these because it doesn’t consider certain key information about recent archaeological finds; it doesn’t suggest that
paintings. the author’s claim lacks originality.

Question Difficulty: Easy Question Difficulty: Hard


Question ID dba9eaf8 Question ID a756aa95
Assessment Test Domain Skill Difficulty Assessment Test Domain Skill Difficulty

SAT Reading and Writing Craft and Structure Words in Context SAT Reading and Writing Craft and Structure Words in Context

ID: dba9eaf8 ID: a756aa95


Within baleen whale species, some individuals develop an accessory spleen—a seemingly functionless formation of The province of Xoconochco was situated on the Pacific coast, hundreds of kilometers southeast of Tenochtitlan, the
splenetic tissue outside the normal spleen. Given the formation’s greater prevalence among whales known to make capital of the Aztec Empire. Because Xoconochco’s location within the empire was so ______, cacao and other trade
deeper dives, some researchers hypothesize that its role isn’t ______; rather, the accessory spleen may actively support goods produced there could reach the capital only after a long overland journey.
diving mechanisms.
Which choice completes the text with the most logical and precise word or phrase?
Which choice completes the text with the most logical and precise word or phrase?
A. unobtrusive
A. replicable
B. concealed
B. predetermined
C. approximate
C. operative
D. peripheral
D. latent

ID: a756aa95 Answer


ID: dba9eaf8 Answer Correct Answer: D
Correct Answer: D
Rationale
Rationale
Choice D is the best answer because it most logically completes the text’s discussion of the location of the
Choice D is the best answer because it most logically completes the text’s discussion of baleen whale province of Xoconochco within the Aztec Empire. As used in this context, “peripheral” means situated toward
accessory spleens. In this context, “latent” means dormant or functionless. The text sets up a contrast between the outer bounds rather than the center. The text indicates that Xoconochco was located on a coast, hundreds
the idea that baleen whale accessory spleens appear not to have a function and the research indicating that of kilometers away from the capital of the Aztec Empire. The text also states that trade between the province
the accessory spleen may actually have a role in supporting the whales’ diving mechanisms. This context and the capital required “a long overland journey.” This context suggests that Xoconochco was situated toward
therefore conveys the idea that the assumption that baleen whale accessory spleens are latent may be an edge of the empire’s territory rather than near its center.
incorrect.
Choice A is incorrect because it wouldn’t make sense in context to refer to Xoconochco’s location within the
Choice A is incorrect because it wouldn’t make sense to say that the role of the accessory spleen is “replicable,” Aztec Empire as “unobtrusive,” or not blatant or undesirably prominent; it’s not clear how a province’s physical
or capable of being reproduced. The text indicates that the role of the accessory spleen seems to have no location would or wouldn’t be blatant. Instead of focusing on how noticeable Xoconochco’s location was, the
function, but some researchers think it does have a role; the text doesn’t address whether the role of the text emphasizes the province’s distance from the capital of the empire, pointing out that because of this
accessory spleen could or couldn’t be reproduced. Choice B is incorrect because suggesting that the role of the distance trade between the two required “a long overland journey.” Choice B is incorrect because the text
accessory spleen is “predetermined,” or decided in advance, wouldn’t make sense in context. Although the indicates that the province of Xoconochco was located on a coast far from the capital of the Aztec Empire, not
researchers may agree that the role of the accessory spleen or any other organ hasn’t been determined in that it was “concealed,” or kept out of sight or hidden from view. Nothing in the text suggests that Xoconochco
advance, the text focuses on the idea that the accessory spleen was thought to have been functionless but may was actually hidden such that people couldn’t see it, and being hidden wouldn’t necessarily result in trade
in fact serve an active role for baleen whales. Choice C is incorrect because it’s the opposite of what the between the province and the capital requiring “a long overland journey.” Choice C is incorrect because to say
context of the text is conveying. The second sentence of the text indicates that baleen whale accessory that Xoconochco’s location within the Aztec Empire was “approximate” would mean that the location either
spleens may not be useless, not that they aren’t “operative,” or functional. wasn’t precisely correct or was close to some other location. Neither of these meanings would make sense in
context because the text indicates that Xoconochco’s location is known and that it was far from the empire’s
Question Difficulty: Hard capital, so there’s no reason to characterize the location as either not precisely correct or close to another
location.

Question Difficulty: Hard

Question ID 441e2b9e Question ID 88bb0f6f


Assessment Test Domain Skill Difficulty Assessment Test Domain Skill Difficulty

SAT Reading and Writing Craft and Structure Words in Context SAT Reading and Writing Craft and Structure Cross-Text
Connections

ID: 441e2b9e
ID: 88bb0f6f
Researchers and conservationists stress that biodiversity loss due to invasive species is ______. For example, people can
take simple steps such as washing their footwear after travel to avoid introducing potentially invasive organisms into Text 1
new environments. A team led by Bernardo Strassburg has found that rewilding farmland (returning the land to its natural state) could
help preserve biodiversity and offset carbon emissions. The amount of farmland that would need to be restored, they
Which choice completes the text with the most logical and precise word or phrase? found, is remarkably low. Rewilding a mere 15% of the world’s current farmland would prevent 60% of expected
species extinctions and help absorb nearly 299 gigatons of carbon dioxide—a clear win in the fight against the
A. preventable
biodiversity and climate crises.
B. undeniable
Text 2
C. common
While Strassburg’s team’s findings certainly offer encouraging insight into the potential benefits of rewilding, it’s
D. concerning important to consider potential effects on global food supplies. The researchers suggest that to compensate for the
loss of food-producing land, remaining farmland would need to produce even more food. Thus, policies focused on
rewilding farmland must also address strategies for higher-yield farming.
ID: 441e2b9e Answer
Which choice best describes a difference in how the author of Text 1 and the author of Text 2 view Strassburg’s team’s
Correct Answer: A
study?
Rationale
A. The author of Text 2 approaches the study’s findings with some caution, whereas the author of Text 1 is optimistic
Choice A is the best answer because it most logically completes the text’s discussion of how biodiversity loss about the reported potential environmental benefits.
due to invasive species can be avoided. As used in this context, “preventable” means able to be stopped or kept
B. The author of Text 2 claims that the percentage of farmland identified by Strassburg’s team is too low for rewilding
from happening. The text indicates that “people can take simple steps” to avoid bringing possible invasive
to achieve meaningful results, whereas the author of Text 1 thinks the percentage is sufficient.
species into new environments. It presents these steps as an example of how biodiversity loss due to invasive
species is preventable. C. The author of Text 2 believes that the results described by Strassburg’s team are achievable in the near future,
whereas the author of Text 1 argues that they likely aren’t.
Choice B is incorrect because it wouldn’t make sense to say that a simple step like washing your shoes after
traveling is an example of biodiversity loss due to invasive species being “undeniable,” or something that can’t D. The author of Text 2 focuses on rewilding’s effect on carbon emissions, whereas the author of Text 1 focuses on its
be proved to be wrong. Although the text may suggest that biodiversity loss due to invasive species is effect on biodiversity.
something that really happens, the word that completes the text must make the first sentence into an
assertion that is illustrated by the second sentence, and the second sentence illustrates the idea that
ID: 88bb0f6f Answer
biodiversity loss due to invasive species is preventable, not undeniable. Choice C is incorrect because it
wouldn’t make sense to say that a simple step like washing your shoes after traveling is an example of Correct Answer: A
biodiversity loss due to invasive species being “common,” or something that happens regularly. Additionally, Rationale
the text doesn’t provide any information about how frequently invasive species cause biodiversity loss. Choice
D is incorrect because it wouldn’t make sense to say that a simple step like washing your shoes after traveling Choice A is the best answer. Text 1 is extremely positive about Strassburg’s team’s findings, calling the
is an example of biodiversity loss due to invasive species being “concerning,” or something that is troubling or potential results "a clear win in the fight against the biodiversity and climate crises." Text 2 is not as positive,
causes worry. Although the text implies that the phenomenon of biodiversity loss due to invasive species is arguing that while the findings point to "potential benefits," we also need to consider the "potential effects on
itself a concerning phenomenon, the word that completes the text must make the first sentence into an global food supplies."
assertion that is illustrated by the second sentence, and the second sentence illustrates the idea that
biodiversity loss due to invasive species is preventable, not concerning. Choice B is incorrect. This isn’t a difference between the two views. Text 1 does present the 15% number as
enough to achieve meaningful results, but that’s not what Text 2 takes issue with: rather, Text 2 argues that we
Question Difficulty: Easy need to consider the effect that rewilding this much farmland would have on food supplies. Choice C is
incorrect. This isn’t a difference between the two views. Neither text mentions the timeline for achieving the
results described by Strassburg’s team. Choice D is incorrect. This isn’t a difference between the two views.
Text 1 focuses on rewilding’s effects on both carbon emissions and biodiversity. Text 2 doesn’t focus on
rewilding’s effect on carbon emissions at all. Instead, it focuses on a third factor: global food supplies.
Question ID 35e21b06
Question Difficulty: Easy
Assessment Test Domain Skill Difficulty

SAT Reading and Writing Craft and Structure Cross-Text


Connections

ID: 35e21b06
Text 1
Dominique Potvin and colleagues captured five Australian magpies (Gymnorhina tibicen) to test a new design for
attaching tracking devices to birds. As the researchers fitted each magpie with a tracker attached by a small harness,
they noticed some magpies without trackers pecking at another magpie’s tracker until it broke off. The researchers
suggest that this behavior could be evidence of magpies attempting to help another magpie without benefiting
themselves.

Text 2
It can be tempting to think that animals are deliberately providing help when we see them removing trackers and other
equipment from one another, especially when a species is known to exhibit other cooperative behaviors. At the same
time, it can be difficult to exclude the possibility that individuals are simply interested in the equipment because of its
novelty, curiously pawing or pecking at it until it detaches.

Based on the texts, how would the author of Text 2 most likely respond to the researchers’ perspective in Text 1 on the
behavior of the magpies without trackers?

A. That behavior might have been due to the novelty of the magpies’ captive setting rather than to the novelty of the
tracker.

B. That behavior likely indicates that the magpies were deliberately attempting to benefit themselves by obtaining the
tracker.

C. That behavior may not be evidence of selflessness in Gymnorhina tibicen because not all the captured magpies
demonstrated it.

D. That behavior might be adequately explained without suggesting that the magpies were attempting to assist the
other magpie.

ID: 35e21b06 Answer


Correct Answer: D

Rationale

Choice D is the best answer because it reflects how the author of Text 2 would most likely respond to the
researchers’ perspective in Text 1 on the behavior of the magpies without trackers. According to Text 1,
Dominique Potvin and colleagues observed magpies without trackers pecking at a tracker on another magpie
until the device fell off. The researchers suggested that the birds might have been attempting to help the other
bird, with no benefit to themselves. Text 2 generally discusses scenarios in which animals have been observed
removing trackers from each other. The text cautions that it shouldn’t be assumed that these animals are
helping one another deliberately, since they might simply be pecking at trackers out of curiosity, causing them
to fall off eventually. Therefore, the author of Text 2 would most likely respond to Potvin and colleagues’
perspective in Text 1 by saying that the behavior of the magpies without trackers could be adequately
explained without suggesting that they were attempting to assist the other magpie.

Choice A is incorrect because Text 2 never discusses the novelty, or the newness and unusual quality, of the
captive settings in which animals have been observed to remove trackers from other animals, nor does it
suggest that such novelty might account for this behavior. Instead, the text suggests that it’s the novelty of the
Question ID 9b01bcf4
tracking equipment itself that might cause the behavior: interested in the trackers because they’re unusual,
animals might paw or peck at them until they fall off. Choice B is incorrect because Text 2 never suggests that
Assessment Test Domain Skill Difficulty
when animals remove trackers from other animals, they do so because they wish to obtain the trackers for
themselves. Instead, Text 2 argues that animals paw or peck at trackers because they are merely curious about SAT Reading and Writing Craft and Structure Text Structure and
Purpose
them. Choice C is incorrect because Text 2 doesn’t argue that when captured animals are observed removing
trackers from each other, their behavior should be regarded as selfless only if all of them participate in it.
Instead, the text argues that the behavior may not be selfless at all and may instead be attributed to animals’ ID: 9b01bcf4
curiosity about the new and unusual trackers.
The 1967 release of Harold Cruse’s book The Crisis of the Negro Intellectual isolated him from almost all other scholars
and activists of the American Civil Rights Movement—though many of those thinkers disagreed with each other, he
Question Difficulty: Hard
nonetheless found ways to disagree with them all. He thought that activists who believed that Black people such as
himself should culturally assimilate were naïve. But he also sharply criticized Black nationalists such as Marcus Garvey
who wanted to establish independent, self-contained Black economies and societies, even though Cruse himself
identified as a Black nationalist.

Which choice best describes the function of the underlined sentence in the text as a whole?

A. It describes a direction that Cruse felt the Civil Rights Movement ought to take.

B. It indicates that Cruse’s reputation as a persistent antagonist of other scholars is undeserved.

C. It describes a controversy that Cruse’s work caused within the Black nationalist movement.

D. It helps explain Cruse’s position with respect to the community of civil rights thinkers.

ID: 9b01bcf4 Answer


Correct Answer: D

Rationale

Choice D is the best answer. The text as a whole claims that Cruse disagreed with virtually all other Civil
Rights scholars and activists. The underlined sentence describes one way that Cruse both did and didn’t fit in
with those thinkers: he criticized Black nationalists, even though he identified as one.

Choice A is incorrect. The underlined sentence doesn’t do this. It describes Cruse’s criticisms—it never
mentions what Cruse did want the movement to do instead. Choice B is incorrect. This conflicts with the text,
which argues that Cruse did disagree with almost all other scholars of the Civil Rights Movement. Choice C is
incorrect. This is a step too far. The text never says that Cruse’s work caused controversy within the Black
nationalist movement.

Question Difficulty: Hard


Question Difficulty: Medium

Question ID f52cc78c
Assessment Test Domain Skill Difficulty

SAT Reading and Writing Craft and Structure Cross-Text


Connections

ID: f52cc78c
Text 1
Polar bears sustain themselves primarily by hunting seals on the Arctic sea ice, but rising ocean temperatures are
causing the ice to diminish, raising concerns about polar bear population declines as these large predators’ seal-
hunting habitats continue to shrink. A 2020 study examining polar bear populations across the Arctic concluded that
populations affected by sea-ice loss are at great risk of extinction by the end of the twenty-first century.
Text 2
Monitoring carried out by researchers from the Norwegian Polar Institute shows that the polar bear population on the
Arctic archipelago of Svalbard remains stable and well nourished despite rapidly declining sea ice in recent years. The
researchers attribute this population’s resilience in part to a shift in feeding strategies: in addition to hunting seals, the
Svalbard polar bears have begun relying on a diet of reindeer meat and birds’ eggs.

Based on the texts, how would the researchers in Text 2 most likely respond to the conclusion presented in the
underlined portion of Text 1?

A. By noting that it neglects the possibility of some polar bear populations adapting to changes in their environment

B. By suggesting that it is likely incorrect about the rates at which warming ocean temperatures have caused sea ice to
melt in the Arctic

C. By asserting that it overlooks polar bear populations that have not yet been affected by loss of seal-hunting
habitats

D. By arguing that it fails to account for polar bears’ reliance on a single seal-hunting strategy

ID: f52cc78c Answer


Correct Answer: A

Rationale

Choice A is the best answer. Text 2 describes how the Svalbard polar bears have adapted to the loss of sea ice
by diversifying their diet and feeding on reindeer and seabird eggs, resulting in a “stable and well nourished”
population despite environmental challenges. This counters the underlined claim that polar bears facing a
loss of sea ice are at “great risk of extinction” by the end of the century.

Choice B is incorrect. Text 2 does not challenge the fact that sea ice is rapidly declining in the Arctic due to
warming ocean temperatures. In fact, it states that the Svalbard polar bears have faced “rapidly declining sea
ice in recent years.” Choice C is incorrect. The claim in Text 1 is specific to polar bear populations affected by
the loss of seal hunting habitats, so unaffected populations are irrelevant to the claim. Also, Text 2 doesn’t
mention any polar bear populations that haven’t yet been affected by loss of seal hunting habitats. It focuses
on a population that has been affected by sea-ice loss but has managed to survive and thrive nevertheless.
Choice D is incorrect. Text 2 doesn’t imply that polar bears rely on a single seal-hunting strategy. In fact, the
researcher in Text 2 would say that Text 1 fails to account for polar bears’ ability to develop other hunting
strategies and food sources.

Question ID 8634bf4a Question ID e8fb0744


Assessment Test Domain Skill Difficulty Assessment Test Domain Skill Difficulty

SAT Reading and Writing Craft and Structure Words in Context SAT Reading and Writing Craft and Structure Words in Context

ID: 8634bf4a ID: e8fb0744


Diego Velázquez was the leading artist in the court of King Philip IV of Spain during the seventeenth century, but his As an undergraduate researcher in anthropology, Jennifer C. Chen contributed to a groundbreaking study challenging
influence was hardly ______ Spain: realist and impressionist painters around the world employed his techniques and the accepted view that among prehistoric peoples, female participation in hunting was ______. The research team’s
echoed elements of his style. review of data from late Pleistocene and early Holocene burials in the Americas revealed that, in fact, as many as half of
the hunters in those populations were female.
Which choice completes the text with the most logical and precise word or phrase?
Which choice completes the text with the most logical and precise word or phrase?
A. derived from
A. inevitable
B. recognized in
B. satisfactory
C. confined to
C. negligible
D. repressed by
D. commonplace

ID: 8634bf4a Answer


Correct Answer: C ID: e8fb0744 Answer
Correct Answer: C
Rationale
Rationale
Choice C is the best answer because it most logically completes the discussion of the artist Diego Velázquez’s
influence outside Spain. As used in this context, “confined to” means restricted to. The text says that Choice C is the best answer because it most logically completes the text’s discussion of the study of female
Velázquez was the leading artist in the Spanish court during the seventeenth century, but it also notes that participation in hunting among prehistoric peoples. In this context, “negligible” means not significant enough
other painters around the world were influenced by his techniques and style. Thus, Velázquez’s influence was to be worth considering. The text says that the study challenged the accepted view of female participation in
hardly (or almost not) confined to, or restricted to, Spain. hunting among prehistoric peoples. The text goes on to say that the researchers found that “in fact, as many as
half” the hunters in the groups studied were female. The phrase “in fact” establishes a contrast indicating that
Choice A is incorrect because if Velázquez was a leading artist in Spain, it doesn’t make logical sense to claim the finding that as many as half the hunters were female differs from the accepted view. This context
that his influence was hardly (or almost not) derived from, or obtained from, Spain. Moreover, the other suggests, then, that the accepted view is that female participation in hunting was negligible.
painters around the world who employed Velázquez’s techniques would by definition be influenced by Spanish
style. Choice B is incorrect because if Velázquez was a leading artist in the court of King Philip IV of Spain, Choice A is incorrect because the text indicates that the study challenged the accepted view by showing that
then his influence must have been widely recognized, or acknowledged, rather than being hardly (or almost as many as half of hunters among prehistoric peoples were female, which suggests that the accepted view is
not) recognized. Choice D is incorrect because the text gives no indication that deliberately limiting that female participation was low, not that female participation was “inevitable,” or unavoidable. Nothing in
Velázquez’s influence outside Spain was ever considered by anyone. Thus, even if it is true that his influence the text suggests that the accepted view is that prehistoric peoples could not avoid female participation in
was not repressed, or restrained, it doesn’t make logical sense to say so in this context. hunting. Choice B is incorrect because nothing in the text suggests that the accepted view of female
participation in hunting among prehistoric peoples is that such participation was “satisfactory,” or sufficient
Question Difficulty: Medium to meet a requirement or demand. There is no information in the text about any demands or requirements
regarding female participation in hunting, let alone any information about how much female participation in
hunting would be enough to satisfy those demands or requirements. Instead, the text indicates that the study
challenged the accepted view by showing that as many as half the hunters in the groups studied were female,
suggesting that the accepted view is that female participation in hunting was low. Choice D is incorrect
because the text indicates that the study challenged the accepted view by showing that as many as half of
hunters among the prehistoric peoples studied were female, which suggests that the accepted view is that
female participation was low, not that female participation was “commonplace,” or ordinary or unremarkable.
Although the study under discussion suggests that female participation may have been commonplace, that
study is presented as challenging the accepted view, not as reinforcing the accepted view.
Question Difficulty: Hard

Question ID c843d63c
Assessment Test Domain Skill Difficulty

SAT Reading and Writing Craft and Structure Words in Context

ID: c843d63c
The artisans of the Igun Eronmwon guild in Benin City, Nigeria, typically ______ the bronze- and brass-casting
techniques that have been passed down through their families since the thirteenth century, but they don’t strictly
observe every tradition; for example, guild members now use air-conditioning motors instead of handheld bellows to
help heat their forges.

Which choice completes the text with the most logical and precise word or phrase?

A. experiment with

B. adhere to

C. improve on

D. grapple with

ID: c843d63c Answer


Correct Answer: B

Rationale

Choice B is the best answer because it most logically completes the text’s discussion of bronze- and brass-
casting techniques used by the Igun Eronmwon guild. In this context “adhere to” would mean to act in
accordance with. The text states that although members of the Igun Eronmwon guild typically do something
with techniques that have been passed down since the thirteenth century, they “don’t strictly observe every
tradition.” By establishing a contrast with not always following traditions, the context suggests that guild
members do typically adhere to traditional techniques.

Choice A is incorrect because in this context “experiment with” would mean to do something new with.
Although using motors rather than manual bellows is presented as a new approach, the text establishes a
contrast between what the guild members typically do with techniques that have been passed down over
centuries and the idea that the members “don’t strictly observe every tradition.” The phrase “experiment with”
wouldn’t support the contrast because regularly trying new things with the techniques would be an example
of not strictly following all traditions. Choice C is incorrect because in this context “improve on” would mean
to make better. Although using motors rather than manual bellows might be an improved approach, the text
establishes a contrast between what the guild members typically do with techniques that have been passed
down over centuries and the idea that the members “don’t strictly observe every tradition.” The phrase
“improve on” wouldn’t support the contrast because regularly making changes to the techniques would be an
example of not strictly following all traditions. Choice D is incorrect because in this context “grapple with”
would mean to try hard to solve a difficult problem. Although bronze- and brass-casting are likely challenging
tasks, nothing in the text suggests that the guild members have any particular difficulties with the techniques
passed down since the thirteenth century.

Question Difficulty: Medium

You might also like